Giotrnhbitphaicng Hunhnguynanhtunch 230827154116 A1fd3f93

You might also like

You are on page 1of 234

HUỲNH NGUYỄN ANH TUẤN - NGUYỄN VINH TIẾN

HOÀNG MINH HẢO - VÕ THỊ NGÀ - HỒ PHƯƠNG


TRẦN THỊ NHUNG - ĐẶNG ĐÌNH KHÔI
PHAN THỊ ANH ĐÀO - VÕ THỊ THU NHƯ

GIÁO TRÌNH

BÀI TẬP HÓA ĐẠI CƯƠNG


(Giáo trình dùng cho sinh viên các ngành Kỹ thuật)

NHAØ XUAÁT BAÛN


ÑAÏI HOÏC QUOÁC GIA TP. HOÀ CHÍ MINH
HUỲNH NGUYỄN ANH TUẤN, NGUYỄN VINH TIẾN,
HOÀNG MINH HẢO, VÕ THỊ NGÀ, HỒ PHƯƠNG,
TRẦN THỊ NHUNG, ĐẶNG ĐÌNH KHÔI,
PHAN THỊ ANH ĐÀO, VÕ THỊ THU NHƯ

GIÁO TRÌNH

BÀI TẬP HÓA ĐẠI CƯƠNG


(Giáo trình dùng cho sinh viên các ngành Kỹ thuật)

NHÀ XUẤT BẢN ĐẠI HỌC QUỐC GIA


THÀNH PHỐ HỒ CHÍ MINH - 2023
LỜI NÓI ĐẦU
Giáo trình Bài tập Hóa đại cương được Bộ môn Công nghệ Hóa học biên
soạn dựa trên chương trình chuẩn của môn học Hóa Đại cương đang được
giảng dạy cho sinh viên các ngành Công nghệ Kỹ thuật tại Trường Đại học
Sư phạm Kỹ thuật TP. Hồ Chí Minh. Mục đích xây dựng giáo trình này
giúp sinh viên củng cố kiến thức lý thuyết, tiếp cận được phương pháp giải
các bài tập có liên quan và tự rèn luyện. Giáo trình được chia thành 8
chương, trong mỗi chương có 3 phần, bao gồm: tóm tắt lý thuyết, bài tập
có lời giải và bài tập trắc nghiệm không lời giải.
Nhóm tác giả hi vọng quyển giáo trình giúp định hướng phương pháp dạy
và học môn học Hóa Đại cương cho giảng viên và sinh viên Trường Đại
học Sư phạm Kỹ thuật TP. Hồ Chí Minh. Bên cạnh đó, nhóm tác giả cũng
mong muốn đóng góp thêm một tài liệu học tập dành cho sinh viên các
ngành Kỹ thuật nói chung.
Xin chân thành cám ơn các đồng nghiệp tại Bộ môn Công nghệ Hóa học,
Khoa Công nghệ Hóa học – Thực phẩm, Trường Đại học Sư phạm Kỹ
thuật TP. Hồ Chí Minh đã khích lệ, đóng góp ý kiến để chúng tôi hoàn
thành quyển giáo trình này.
Mặc dù đã hết sức cố gắng nhưng chắc chắn quyển giáo trình không thể
tránh khỏi những hạn chế trong lần xuất bản đầu tiên. Nhóm tác giả mong
muốn nhận được sự đóng góp của các độc giả, các đồng nghiệp và các bạn
sinh viên để chúng tôi có thể tiếp tục hoàn thiện trong lần xuất bản tiếp
theo. Mọi đóng góp xin gửi về TS. Huỳnh Nguyễn Anh Tuấn, Bộ môn
Công nghệ Hóa học, phòng A1-802, Trường Đại học Sư phạm Kỹ thuật
TP. Hồ Chí Minh, Số 1, Võ Văn Ngân, TP. Thủ Đức, TP. Hồ Chí Minh.
TP. Hồ Chí Minh, ngày 9 tháng 9 năm 2022
Chủ biên

TS. Huỳnh Nguyễn Anh Tuấn

i
MỤC LỤC
LỜI NÓI ĐẦU ................................................................... .......................i
BẢNG KÝ HIỆU CÁC CHỮ VIẾT TẮT.............................................viii

Chương 1. CẤU TẠO NGUYÊN TỬ VÀ HỆ THỐNG


TUẦN HOÀN CÁC NGUYÊN TỐ HÓA HỌC....................................1
Phần 1. TÓM TẮT LÝ THUYẾT...........................................................2
1.1 KHÁI NIỆM VỀ NGUYÊN TỬ..........................................................2
1.1.1 Nguyên tử và các hạt cơ bản......................................................2
1.1.2 Hiện tượng đồng vị...................................................................3
1.2 CẤU TẠO NGUYÊN TỬ THEO CƠ HỌC LƯỢNG TỬ...................3
1.2.1 Các luận điểm cơ sở................................................................. 3
1.2.2 Đám mây electron (Orbital nguyên tử - AO).............................4
1.2.3 Các số lượng tử và ý nghĩa........................................................4
1.3 HỆ THỐNG TUẦN HOÀN CÁC NGUYÊN TỐ HÓA HỌC.............7
1.3.1 Định luật tuần hoàn....................................................................7
1.3.2 Cấu trúc của bảng hệ thống tuần hoàn.......................................8
1.4 SỰ BIẾN THIÊN TUẦN HOÀN TÍNH CHẤT CỦA CÁC
NGUYÊN TỐ...........................................................................................10
1.4.1 Bán kính nguyên tử và ion......................................................10
1.4.2 Năng lượng ion hóa I.............................................................. 11
1.4.3 Ái lực electron EA....................................................................12
1.4.4 Độ âm điện χ...........................................................................13
1.4.5 Số oxy hóa...............................................................................12
Phần 2. BÀI TẬP CÓ LỜI GIẢI............................................................14
Phần 3. BÀI TẬP TRẮC NGHIỆM......................................................20

Chương 2. LIÊN KẾT HÓA HỌC...................................................... 31


Phần 1. TÓM TẮT LÝ THUYẾT.........................................................32
2.1 BẢN CHẤT LIÊN KẾT HÓA HỌC..................................................32
2.2 CÁC KHÁI NIỆM CƠ BẢN...............................................................32
2.2.1 Năng lượng liên kết.................................................................32
2.2.2 Độ dài liên kết..........................................................................32
2.2.3 Góc hóa trị...............................................................................32
2.2.4 Bậc liên kết...............................................................................32

iii
2.3 LIÊN KẾT CỘNG HÓA TRỊ THEO THUYẾT LIÊN KẾT
HÓA TRỊ-VB.............................................................................................33
2.3.1 Các luận điểm cơ bản..............................................................33
2.3.2 Cơ chế tạo liên kết cộng hóa trị (CHT)....................................33
2.3.3 Các kiểu liên kết cộng hóa trị..................................................33
2.3.4 Tính chất của liên kết cộng hóa trị...........................................34
2.3.5 Lưỡng cực và moment lưỡng cực............................................35
2.3.6 Thuyết lai hóa..........................................................................35
2.3.7 Thuyết đẩy nhau giữa các cặp electron hóa trị (VESPR)..........38
2.3.8 Dự đoán trạng thái lai hóa của nguyên tử trung tâm và
cấu hình phân tử.....................................................................39
2.4. LIÊN KẾT ION...................................................................................39
2.4.1 Bản chất của liên kết ion.........................................................39
2.4.2 Các tính chất của liên kết ion..................................................40
2.4.3 Sự phân cực ion.......................................................................40
2.5. LIÊN KẾT KIM LOẠI......................................................................41
2.5.1 Đặc điểm cấu tạo kim loại.......................................................41
2.5.2 Thuyết miền năng lượng về cấu tạo kim loại..........................41
2.5.3 Các tính chất của kim loại........................................................42
2.6 CÁC LOẠI LIÊN KẾT LIÊN PHÂN TỬ..........................................42
2.6.1 Liên kết Hydro..........................................................................42
2.6.2 Liên kết Van der Waals (VDW)...............................................42
Phần 2. BÀI TẬP CÓ LỜI GIẢI............................................................44
Phần 3. BÀI TẬP TRẮC NGHIỆM......................................................49

Chương 3. NHIỆT ĐỘNG HÓA HỌC.................................................56


Phần 1. TÓM TẮT LÝ THUYẾT.........................................................57
3.1 CÁC KHÁI NIỆM.............................................................................57
3.3.1 Hệ............................................................................................57
3.3.2 Trạng thái nhiệt động..............................................................57
3.3.3 Quá trình.................................................................................58
3.3.4 Năng lượng..............................................................................58
3.2 NGUYÊN LÝ I CỦA NHIỆT ĐỘNG LỰC HỌC.............................59
3.3 HIỆU ỨNG NHIỆT............................................................................60
3.4 PHƯƠNG TRÌNH NHIỆT HÓA HỌC VÀ CHIỀU DIỄN RA
CỦA CÁC QUÁ TRÌNH HÓA HỌC......................................................60
3.3.1 Phương trình nhiệt hóa học......................................................60

iv
3.3.2 Chiều diễn ra của các quá trình hóa học..................................61
3.5 ĐỊNH LUẬT HESS VÀ CÁC HỆ QUẢ.............................................61
3.6 ENTROPY VÀ NGUYÊN LÝ II CỦA NHIỆT ĐỘNG HỌC...........62
3.6.1 Nội dung nguyên lý II..............................................................62
3.6.2 Entropy S.................................................................................62
3.6.3 Tính toán ∆So của phản ứng hóa học......................................62
3.7 THẾ ĐẲNG ÁP VÀ CHIỀU DIỄN RA CỦA CÁC QUÁ
TRÌNH HÓA HỌC..............................................................................63
3.7.1 Thế đẳng áp G..........................................................................63
3.7.2 Thế đẳng áp tạo thành tiêu chuẩn.............................................63
3.7.3 Độ biến đổi thế đẳng áp và điều kiện xảy ra của quá trình
hóa học......................................................................................63
3.7.4 Tính độ biến đổi thế đẳng áp của các phản ứng hóa học.........64
Phần 2. BÀI TẬP CÓ LỜI GIẢI..........................................................65
Phần 3. BÀI TẬP TRẮC NGHIỆM......................................................72

Chương 4. ĐỘNG HÓA HỌC...............................................................87


Phần 1. TÓM TẮT LÝ THUYẾT.........................................................88
4.1 MỘT SỐ KHÁI NIỆM CƠ BẢN.......................................................88
4.2 BIỂU THỨC TỐC ĐỘ PHẢN ỨNG..................................................89
4.2.1 Biểu thức tốc độ phản ứng.......................................................89
4.2.2 Biểu thức hằng số tốc độ phản ứng k......................................89
4.2.3 Thời gian bán hủy t1/2...............................................................90
4.3 CÁC YẾU TỐ ẢNH HƯỞNG ĐẾN TỐC ĐỘ PHẢN ỨNG.............90
4.3.1 Ảnh hưởng của nồng độ..........................................................90
4.3.2 Ảnh hưởng của nhiệt độ..........................................................90
4.3.3 Ảnh hưởng của chất xúc tác.....................................................91
Phần 2. BÀI TẬP CÓ LỜI GIẢI............................................................93
Phần 3. BÀI TẬP TRẮC NGHIỆM......................................................97

Chương 5. CÂN BẰNG HÓA HỌC...................................................106


Phần 1. TÓM TẮT LÝ THUYẾT.......................................................107
5.1 CÂN BẰNG HÓA HỌC..................................................................107
5.5.1 Phản ứng một chiều................................................................107
5.5.2 Phản ứng hai chiều (phản ứng thuận nghịch)........................107
5.5.3 Trạng thái cân bằng hóa học.................................................108

v
5.2 HẰNG SỐ CÂN BẰNG VÀ MỨC ĐỘ XẢY RA PHẢN ỨNG.......108
5.2.1 Hằng số cân bằng..................................................................108
5.2.2 Các lưu ý quan trọng.............................................................109
5.2.3 Ý nghĩa của hằng số cân bằng K........................................... 110
5.3 HẰNG SỐ CÂN BẰNG K VÀ ĐỘ BIẾN ĐỔI THẾ ĐẲNG ÁP
∆G CỦA PHẢN ỨNG (Phương trình đẳng nhiệt Van’t Hoff)............... 111
5.4 DỰ ĐOÁN CHIỀU DIỄN TIẾN CỦA PHẢN ỨNG....................... 111
5.5 SỰ CHUYỂN DỊCH CÂN BẰNG VÀ NGUYÊN LÝ LE
CHATELIER...................................................................................... 112
5.5.1 Ảnh hưởng của nồng độ........................................................ 113
5.5.2 Ảnh hưởng của áp suất, thể tích.............................................113
5.5.3 Ảnh hưởng của nhiệt độ........................................................ 113
Phần 2. BÀI TẬP CÓ LỜI GIẢI..........................................................114
Phần 3. BÀI TẬP TRẮC NGHIỆM....................................................123

Chương 6. DUNG DỊCH – DUNG DỊCH KHÔNG ĐIỆN LY.........131


Phần 1. TÓM TẮT LÝ THUYẾT.......................................................132
6.1 KHÁI NIỆM VỀ DUNG DỊCH........................................................132
6.6.1 Các hệ phân tán và dung dịch................................................132
6.6.2 Dung dịch...............................................................................132
6.2 NỒNG ĐỘ CỦA DUNG DỊCH........................................................132
6.3 PHA TRỘN DUNG DỊCH THEO PHƯƠNG PHÁP ĐƯỜNG
CHÉO.......................................................................................................133
6.4 CƠ CHẾ TẠO THÀNH DUNG DỊCH.............................................134
6.5 ĐỘ TAN VÀ CÁC YẾU TỐ ẢNH HƯỞNG....................................134
6.5.1 Độ tan....................................................................................134
6.5.2 Các yếu tố ảnh hưởng đến độ tan..........................................135
6.6 DUNG DỊCH KHÔNG ĐIỆN LY VÀ CÁC TÍNH CHẤT..............135
6.6.1 Áp suất hơi bão hòa...............................................................135
6.6.2 Nhiệt độ sôi và nhiệt độ đông đặc..........................................136
6.6.3 Áp suất thẩm thấu...................................................................138
Phần 2. BÀI TẬP CÓ LỜI GIẢI..........................................................139
Phần 3. BÀI TẬP TRẮC NGHIỆM....................................................145

Chương 7. DUNG DỊCH ĐIỆN LY....................................................156


Phần 1. TÓM TẮT LÝ THUYẾT.......................................................157
7.1 DUNG DỊCH ACID, BASE, MUỐI TRONG NƯỚC.....................157
7.2 SỰ ĐIỆN LY VÀ THUYẾT ĐIỆN LY............................................157

vi
7.3 ĐỘ ĐIỆN LY....................................................................................158
7.4 CÂN BẰNG TRONG DUNG DỊCH CỦA CHẤT ĐIỆN
LY YẾU...................................................................................................159
7.4.1 Hằng số điện ly........................................................................159
7.4.2 Sự liên hệ giữa hằng số điện ly và độ điện ly – Định luật
pha loãng Ostwald.............................................................................159
7.5 THUYẾT ACID-BASE.....................................................................160
7.5.7.Thuyết acid – base của Bronsted............................................160
7.5.2. Chỉ số Hydro pH...................................................................160
7.7.3 Hằng số acid...........................................................................161
7.7.4 Tính pH của các dung dịch acid, base...................................161
7.6 CÂN BẰNG CỦA CHẤT ĐIỆN LY ÍT TAN..................................163
7.7.1 Tích số tan.............................................................................163
7.7.2 Sự tạo thành và hòa tan một kết tủa của chất điện ly ít tan..........164
Phần 2. BÀI TẬP CÓ LỜI GIẢI..........................................................166
Phần 3. BÀI TẬP TRẮC NGHIỆM....................................................175

Chương 8. ĐIỆN HÓA HỌC...............................................................181


Phần 1. TÓM TẮT LÝ THUYẾT.......................................................182
8.1 PHẢN ỨNG OXY HÓA KHỬ........................................................182
8.1.1 Số oxy hóa.............................................................................182
8.1.2 Phản ứng oxy hóa khử...........................................................182
8.1.3 Phản ứng oxy hóa khử và dòng điện......................................182
8.2 NGUYÊN TỐ GALVANIC...............................................................182
8.2.1 Cấu tạo nguyên tố Galvanic...................................................182
8.2.2 Hoạt động của nguyên tố Galvanic........................................183
8.2.3 Thế điện cực (ϕ) và thế điện cực tiêu chuẩn (ϕo)..................184
8.2.4 Sức điện động của pin và hằng số cân bằng..........................186
8.3. CHIỀU XẢY RA CỦA PHẢN ỨNG OXY HÓA KHỬ.................186
Phần 2. BÀI TẬP CÓ LỜI GIẢI.........................................................187
Phần 3. BÀI TẬP TRẮC NGHIỆM....................................................193

TÀI LIỆU THAM KHẢO......................................................................201


PHỤ LỤC 1............................................................................................202
PHỤ LỤC 2............................................................................................203
PHỤ LỤC 3............................................................................................204
PHỤ LỤC 4............................................................................................ 211
PHỤ LỤC 5............................................................................................214

vii
BẢNG KÝ HIỆU CÁC CHỮ VIẾT TẮT

Ký hiệu Nội dung


 Độ âm điện
 Hệ số nhiệt độ
 Áp suất thẩm thấu
 Thế điện cực của cặp oxy hóa – khử liên hợp
i Nồng độ phần mol
 Độ điện ly
G Độ biến đổi thế đẳng áp
H Hiệu ứng nhiệt
S Độ biến đổi entropy
n Độ biến đổi số mol chất khí trước và sau phản ứng
(aqu) Dung dịch nước
(g) Trạng thái khí
(l) Trạng thái lỏng
(s) Trạng thái rắn
A Thừa số Arrhenius
AO Orbital nguyên tử
C% Nồng độ phần trăm
CHT Liên kết cộng hóa trị
CM Nồng độ mol
Cm Nồng độ molan
CN Nồng độ đương lượng
e Electron
E Sức điện động của pin galvanic
EA Ái lực electron
Ea Năng lượng hoạt hóa
I Năng lượng ion hóa
i Hệ số Van’t Hoff
k Hằng số tốc độ phản ứng
K Hằng số cân bằng
Ka Hằng số acid
Kb Hằng số base

viii
vii
Ký hiệu Nội dung
KC Hằng số cân bằng nồng độ
Kđ Hằng số nghiệm đông
KP Hằng số cân bằng áp suất
Ks Hằng số nghiệm sôi
Ksp Tích số tan
MO Orbital phân tử
MOlk Orbital phân tử liên kết
MOplk Orbital phân tử phản liên kết
n Neutron
p Proton
pH Chỉ số hydro pH
R Hằng số khí
S Độ tan
T Nhiệt độ tuyệt đối
t1/2 Thời gian bán hủy
tđ Nhiệt độ đông đặc
ts Nhiệt độ sôi
VB Liên kết hóa trị
VDW Liên kết Van der Waals
VSEPR Thuyết đẩy nhau giữa các cặp electron hóa trị
vn Tốc độ phản ứng nghịch
vt Tốc độ phản ứng thuận
Z Điện tích hạt nhân

ix
viii
Chương 1. CẤU TẠO NGUYÊN TỬ VÀ HỆ THỐNG
TUẦN HOÀN CÁC NGUYÊN TỐ HÓA HỌC
---oOo---
Mục tiêu chương 1
Sau khi học xong chương này, sinh viên có khả năng:
1. Giải thích được các thành phần cơ bản của nguyên tử, ion.
2. Giải thích được hiện tượng đồng vị và tính toán nguyên tử khối
trung bình.
3. Xác định được bộ bốn số lượng tử của một electron và giải thích
được ý nghĩa của các số lượng tử theo cơ học lượng tử.
4. Trình bày được các nguyên tắc viết cấu hình electron và viết được
cấu hình electron của các nguyên tử bất kỳ.
5. Mô tả được bảng hệ thống tuần hoàn các nguyên tố hóa học.
6. Xác định được vị trí của nguyên tố hóa học trong bảng hệ thống
tuần hoàn bao gồm chu kỳ, nhóm, phân nhóm và số thứ tự của ô.
7. Giải thích được sự biến đổi tuần hoàn các tính chất của nguyên tử
theo chu kỳ và theo nhóm trong bảng hệ thống tuần hoàn.

1
Phần 1. TÓM TẮT LÝ THUYẾT
1.1 KHÁI NIỆM VỀ NGUYÊN TỬ
1.1.1 Nguyên tử và các hạt cơ bản
Nguyên tử được tạo thành từ những tiểu phân
nhỏ hơn: electron và hạt nhân.
Electron (e): Trong nguyên tử, các electron
chuyển động xung quanh hạt nhân tạo nên lớp vỏ
electron.
Hạt nhân: được cấu tạo chủ yếu từ các hạt proton
(p) và nơtron (n).
Hình 1.1. Nguyên tử

Bảng 1.1 Thông số của các hạt cơ bản


Khối lượng Điện tích
Hạt cơ
bản Tương
Tuyệt đối Tương đối Tuyệt đối
đối
Đơn vị Đơn vị
Kg đvC Culong
tĩnh điện e
Electron
9,1.10 -31 0,000549 - 1,6.10 -19 - 4,8.10 -10 -1
(e)
Proton
1,672.10 -27 1,007 + 1,6.10 -19 + 4,8.10 -10 +1
(p)
Nơtron
1,674.10 -27 1,008 0 0 0
(n)

Trong nguyên tử trung hòa:


p=e=Z (1.1)
Trong đó, Z là điện tích hạt nhân
Hạt nhân là thành phần quan trọng nhất quyết định tính chất của nguyên
tử và chiếm phần lớn khối lượng nguyên tử.
A=n+Z (1.2)
Trong đó, A là số khối
Một nguyên tử sẽ được hoàn toàn xác định nếu biết hai thông số là điện
tích hạt nhân (Z) và số khối (A). Thường được ký hiệu:
𝐴
𝑍𝑋 Ví dụ: 35
17𝐶𝑙

2
Cho biết: nguyên tử có A = 35 đvC; p = e = 17 hạt và n = 18 hạt.
1.1.2 Hiện tượng đồng vị
Đồng vị là hiện tượng các nguyên tử của cùng nguyên tố có cùng số proton
nhưng khác số nơtron nên khác nhau về số khối A.
Ví dụ, Clo trong tự nhiên có 2 đồng vị là 35 37
17𝐶𝑙 và 17𝐶𝑙 .
Nguyên tử khối trung bình là đại lượng được sử dụng để đại diện cho
nguyên tử khối của các nguyên tố có nhiều đồng vị.
Xét một nguyên tố X có 2 đồng vị là A1X và A2X. Trong đó, đồng vị A1
chiếm a% và đồng vị A2 chiếm b%. Ta có nguyên tử khối trung bình của
X được tính bằng công thức sau:
𝐴1 . 𝑎% + 𝐴2 . 𝑏% (1.3)
𝐴=
𝑎% + 𝑏%

Ví dụ 1.1. Clo trong tự nhiên là hỗn hợp của hai đồng vị 35Cl chiếm 75,77%
và 37Cl. Tính nguyên tử khối trung bình của Clo.
Giải
Trong tự nhiên 35Cl chiếm 75,77%
→ nên 37Cl chiếm (100 – 75,77) = 24,23%.
35𝑥75,77+37𝑥24,23
→ A= 100
= 35,48 đvC
1.2 CẤU TẠO NGUYÊN TỬ THEO CƠ HỌC LƯỢNG TỬ
1.2.1 Các luận điểm cơ sở
a) Bản chất sóng – hạt của các hạt vi mô
Hạt vi mô như ánh sáng (photon), nguyên tử, phân tử vừa có bản chất hạt
(thể hiện bằng m, r, v) vừa có bản chất sóng (thể hiện bằng bước sóng λ,
tần số ν) thể hiện trong biểu thức:
(1.4)

Trong đó, h là hằng số Plank;


m, v là khối lượng và tốc độ của hạt vi mô.
b) Nguyên lý bất định Heisenberg
Không thể xác định chính xác đồng thời cả vị trí lẫn vận tốc của hạt vi mô.
h (1.5)
∆x. ∆v ≥ 4πm

với Δx, Δv là độ bất định về vị trí và vận tốc của hạt vi mô.

3
c) Phương trình hàm số sóng Schrödinger
(1.6)

Trong đó,
x, y, z: tọa độ;
m: khối lượng hạt vi mô;
h: hằng số Planck;
E: năng lượng toàn phần;
V: thế năng của hạt vi mô phụ thuộc x, y, z;
ψ: hàm sóng (với các biến x, y, z) mô tả sự chuyển động của các hạt vi mô.
Ý nghĩa:
- ψ đặc trưng cho chuyển động của hạt vi mô liên quan đến xác suất có mặt
của hạt vi mô.
- ψ2dv xác định xác suất có mặt của hạt vi mô trong thể tích dv.
1.2.2 Đám mây electron (Orbital nguyên tử - AO)
Trong hệ tọa độ cầu:

x= r.sin.cos
y= r.sin.sin
z= r.cos
x2 + y2 + z2 = r2
Phương trình hàm số sóng Schrödinger có
Hình 1.2. Tọa độ cực
nghiệm:  = [R(r)][()][()]
R(r)] là phần bán kính của hàm sóng ψ, phụ thuộc vào 2 tham số n và l gọi
là số lượng tử chính và số lượng tử phụ.
[Θ(θ)][Φ(φ)] là phần góc của hàm sóng ψ, phụ thuộc vào số lượng tử
từ ml.
Hàm sóng ψ tương ứng với bộ 3 số lượng tử n, l, ml được gọi là một orbital
nguyên tử (Atomic Orbital, AO).
Về hình dạng của một AO ≡ ψ(n, l, ml): là một vùng không gian xung
quanh hạt nhân trong đó các electron chuyển động với mật độ xác suất ψ2
lớn, thường ≥ 90%.
1.2.3 Các số lượng tử và ý nghĩa
a) Số lượng tử chính n
n có giá trị nguyên dương và gián đoạn, n = 1, 2, 3,…, ∞

4
n xác định năng lượng của electron, các electron có cùng giá trị n sẽ tạo
thành một lớp năng lượng:
Z2 Z2
En = −2,818. 10−18 2 , ሺJሻ = −13,6 2 , ሺeVሻ
n n
Các electron có cùng mức năng lượng sẽ có cùng số lượng tử chính n và
hợp thành một lớp electron.
Bảng 1.2. Tên lớp và số lượng tử chính n
n 1 2 3 4 5 6 7…

En E1 E2 E3 E4 E5 E6 E7….

Tên lớp e K L M N O P Q…

b) Số lượng tử phụ ℓ (số lượng tử orbital)


ℓ có giá trị nguyên dương từ 0 đến (n-1). Mỗi giá trị của n có n giá trị của
ℓ.
Đối với nguyên tử nhiều e, ℓ còn xác định trạng thái năng lượng của e gọi
là phân mức năng lượng.
Các e trong mỗi lớp lượng tử có cùng phân mức năng lượng hợp thành
phân lớp e hay phân lớp lượng tử.
ℓ xác định tên và hình dạng của AO. Cụ thể, AO s có dạng hình cầu, AO
p có dạng 2 quả cầu tiếp xúc,…
Bảng 1.3 Tên phân lớp và số lượng tử phụ ℓ
Tên phân lớp e s p d f…

Giá trị ℓ 0 1 2 3…

Orbital s (ℓ = 0) Orbital p (ℓ = 1) Orbital d (ℓ = 2)


Hình 1.3. Hình dạng đám mây electron
c) Số lượng tử từ mℓ
mℓ có giá trị nguyên (âm hay dương) từ 0 đến ±ℓ. Mỗi giá trị của ℓ có
(2ℓ +1) giá trị của mℓ.
5
Bảng 1.4 Các số lượng tử và ý nghĩa
Số lượng Số lượng Phân Các giá trị của số lượng Số lượng orbital
tử chính tử phụ ℓ lớp tử từ mℓ nguyên tử (AO)
n trong phân lớp
1 0 1s 0 1

0 2s 0 1
2
1 2p -1, 0, +1 3

0 3s 0 1

3 1 3p -1, 0, +1 3

2 3d -2, -1, 0, +1, +2 5

0 4s 0 1

1 4p -1, 0, +1 3
4
2 4d -2, -1, 0, +1, +2 5
3 4f -3, -2, -1, 0, +1, +2, +3 7
Số lượng tử mℓ đặc trưng cho sự định hướng trong không gian của AO.
Số lượng tử mℓ đặc trưng cho tương tác của từ trường ngoài (tác dụng lên
nguyên tử) với từ trường của e.
d) Số lượng tử spin ms
Số lượng tử spin ms xác định trạng thái chuyển động riêng của e, là sự tự
quay của e xung quanh trục của mình. ms có 2 giá trị là +1/2 (thuận chiều
kim đồng hồ) hay -1/2 (ngược chiều kim đồng hồ).

 
ms = + 1/2 ms = - 1/2

Hình 1.4. Chiều quay của electron


1.2.4 Nguyên tử nhiều electron
Trong nguyên tử nhiều electron, trạng thái của electron cũng xác định
bằng 4 số lượng tử n, l, ml, và ms.

6
Ngoài lực hút giữa hạt nhân và electron trong nguyên tử còn xuất hiện lực
đẩy giữa các electron với nhau làm xuất hiện hai hiệu ứng: hiệu ứng chắn
và hiệu ứng xâm nhập.
Điện tích hạt nhân thực sự tác dụng với electron là: Z* = Z – S
Z* là điện tích hạt nhân hiệu dụng,
S là hằng số chắn, tính theo công thức Slater.
Lúc này năng lượng của electron được tính theo (1.8) và năng lượng của
electron trong nguyên tử nhiều electron phụ thuộc vào cả n và l.
Z ∗2 Z ∗2 (1.8)
En,l = −2,818. 10−18 , ሺJሻ = −13,6 , ሺeVሻ
n2 n2
a) Nguyên lý vững bền
Trạng thái bền vững nhất của e trong nguyên tử là trạng thái có năng lượng
thấp nhất. Do đó, các electron sắp xếp vào các AO theo trật tự năng lượng
tăng dần.
b) Nguyên lý ngoại trừ Pauli
Trong một nguyên tử không thể có 2 e có cùng 4 số lượng tử. Do đó, trong
một AO chỉ chứa tối đa 2 electron có chiều quay (ms) ngược nhau.
c) Quy tắc Hund
Sự sắp xếp của e vào các AO có cùng phân mức năng lượng sao cho giá
trị tuyệt đối của tổng spin đạt cực đại. Do đó, trong cùng một phân lớp, các
electron sắp xếp sao cho số electron độc thân là tối đa.
d) Quy tắc Kleshkovski
1. Sự sắp xếp e vào các AO theo chiều tăng Z xảy ra theo thứ tự từ những
AO có tổng (n+l) nhỏ đến lớn.
2. Sự sắp xếp của e vào các AO có tổng giá trị (n+l) bằng nhau sẽ xảy ra
theo hướng tăng dần giá trị của n.
1.3 HỆ THỐNG TUẦN HOÀN CÁC NGUYÊN TỐ HÓA HỌC
1.3.1 Định luật tuần hoàn
Tính chất của các đơn chất cũng như dạng và tính chất của các hợp chất
của những nguyên tố hóa học phụ thuộc vào điện tích hạt nhân nguyên tử
của nguyên tố. Điện tích hạt nhân nguyên tử là đại lượng quyết định và
đặc trưng cho tính chất nguyên tố.

7
Hình 1.5. Sơ đồ phân bố năng lượng các AO
1.3.2 Cấu trúc của bảng hệ thống tuần hoàn
Hệ thống tuần hoàn hiện nay gồm 118 nguyên tố chia làm 7 chu kỳ và 8
nhóm.
a) Chu kỳ
Chu kỳ gồm dãy các nguyên tố sắp xếp theo thứ tự tăng dần mà nguyên tử
của chúng có cùng số lớp electron.
Chu kỳ 1 là chu kỳ đặc biệt chỉ có 2 nguyên tố,
Chu kỳ 2, và 3 được gọi là chu kỳ nhỏ, mỗi chu kỳ có 8 nguyên tố.
Bốn chu kỳ còn lại là chu kỳ lớn:
Chu kỳ 4 và chu kỳ 5, mỗi chu kỳ có 18 nguyên tố gồm 8 nguyên tố nhóm
A và 10 nguyên tố nhóm B.
Chu kỳ 6 có 32 nguyên tố gồm 18 nguyên tố tương tự chu kỳ 5 và 14
nguyên tố có tính chất hóa học giống lantan xếp phía dưới bên ngoài bảng
chính (họ lantanit).
Chu kỳ 7 tương tự chu kỳ 6 và cũng có các nguyên tố có tính chất hóa học
giống actini xếp phía dưới bên ngoài bảng chính, dưới họ lantanit (họ
actinit).
Quan hệ giữa cấu tạo nguyên tử của nguyên tố và vị trí của nguyên tố
trong hệ thống tuần hoàn:
Số thứ tự chu kỳ = số lớp electron.
8
b) Nhóm
Nhóm gồm các nguyên tố có tính chất tương tự nhau sắp xếp theo cột dọc.
Nhóm bao gồm các nguyên tố có số electron hóa trị bằng nhau. Trong đó,
Các electron hóa trị là các electron có khả năng tham gia tạo thành các
liên kết hóa học.
Các nguyên tố thuộc các cột 1, 2 và các cột từ 13 đến 18 tạo thành 8 nhóm,
đánh số từ IA đến VIIIA được gọi là các nhóm A (phân nhóm chính).
Mười cột còn lại tạo thành các nhóm B, đánh số từ IIIB đến VIIB (một cột)
và VIIIB (ba cột), sau đó là IB và IIB (một cột). Các nhóm B gọi là phân
nhóm phụ.
Phân nhóm A bao gồm các nguyên tố họ s và họ p. Phân nhóm B bao gồm
các nguyên tố họ d và họ f. Trong đó:
• Nguyên tố s: những nguyên tố có electron ngoài cùng sắp xếp vào
orbital s.
• Nguyên tố p: những nguyên tố có electron ngoài cùng sắp xếp vào
orbital p.
• Nguyên tố d: những nguyên tố có electron xếp vào phân lớp d lớp kề
ngoài cùng, nghĩa là các orbital (n-1)d.
• Nguyên tố f: những nguyên tố có electron sắp xếp vào phân lớp f lớp
thứ 3 kể từ ngoài vào, nghĩa là các orbital (n-2)f.
Quan hệ giữa cấu tạo nguyên tử của nguyên tố và vị trí của nguyên tố
trong hệ thống tuần hoàn:
• Đối với các nguyên tố phân nhóm A:
Số thứ tự nhóm = tổng electron lớp ngoài cùng
• Đối với các nguyên tố phân nhóm B, họ d:
Số thứ tự nhóm phụ thuộc vào số electron lớp ngoài cùng và kề ngoài
cùng (x + y) trong cấu hình tổng quát (n – 1)dx nsy theo bảng sau:

x+y Nhóm

3x+y7 x+y

8  x + y  10 VIII

x + y > 10 x + y - 10

c) Ô
Mỗi nguyên tố chiếm một ô trong bảng hệ thống tuần hoàn.
9
Số thứ tự của ô bằng số điện tích hạt nhân. Số thứ tự ô = Z.
Ví dụ 1.2. Xác định vị trí của các nguyên tố có điện tích hạt nhân lần lượt
là 3, 7, 12, 18, 21, 27, 30 và 35.
Giải
Xác định vị trí nguyên tố trong bảng hệ thống tuần hoàn cần xác định được
4 thông số sau: chu kỳ, nhóm, phân nhóm và số thứ tự của ô chứa nguyên
tố. Kết quả được trình bày như sau:
Chu kỳ Phân nhóm
Ô=
Z Cấu hình e Họ STT chu kỳ = số A: họ s, họ p Nhóm
số Z
lớp e B: họ d, họ f

3 1s2 2s1 s 2 A I 3

7 1s2 2s2 2p3 p 2 A V 7

12 [Ne] 3s2 s 3 A II 12

18 [Ne] 3s2 3p6 p 3 A VIII 18

21 [Ar] 3d1 4s2 d 4 B III 21

27 [Ar] 3d7 4s2 d 4 B VIII 27

30 [Ar] 3d10 4s2 d 4 B II 30

35 [Ar] 3d10 4s2 4p5 p 4 A VII 35

1.4 SỰ BIẾN THIÊN TUẦN HOÀN TÍNH CHẤT CỦA CÁC


NGUYÊN TỐ
1.4.1 Bán kính nguyên tử và ion
a) Bán kính nguyên tử
Người ta phân biệt bán nguyên tử thành bán kính kim loại và bán kính
cộng hóa trị.
Bán kính kim loại của một nguyên tố kim loại bằng nửa khoảng cách giữa
tâm của các nguyên tử kim loại ở gần nhau nhất trong mạng lưới tinh thể
kim loại.
Bán kính cộng hóa trị của một nguyên tử bằng nửa khoảng cách giữa hạt
nhân hai nguyên tử của cùng một nguyên tố tạo thành liên kết đơn cộng
hóa trị.
Đơn vị đo bán kính nguyên tử là picomet (pm) hay angstrom (Å), với:
1 Å = 100 pm = 10-10 m.

10
Quy luật biến thiên
• Trong một chu kỳ, từ trái qua phải, bán kính nguyên tố s, p có khuynh
hướng giảm liên tục. Bán kính của các nguyên tố d giảm chậm và không
đều, bán kính của các nguyên tố f thậm chí còn giảm chậm hơn.
• Trong phân nhóm chính, từ trên xuống, bán kính nguyên tử và bán kính
ion cùng điện tích đều tăng. Đối với các nguyên tố phân nhóm B, khi đi từ
nguyên tố đầu phân nhóm (chu kỳ 4) đến nguyên tố thứ 2 (chu kỳ 5) thì
bán kính nguyên tử có tăng lên. Từ nguyên tố thứ 2 đến nguyên tố thứ 3
(chu kỳ 5 đến chu kỳ 6) thực tế bán kính ít thay đổi (có khi không đổi hoặc
giảm chút ít). Nguyên nhân là do có sự co lantan của 14 nguyên tố f bù trừ
sự tăng lên bán kính phải xảy ra khi chuyển từ chu kỳ 5 sang chu kỳ 6.
b) Bán kính ion:
Nguyên tử nhường hay nhận electron để trở thành cation hay anion có cấu
hình electron lớp ngoài cùng bền vững giống khí trơ.
Cation luôn có kích thước nhỏ hơn so với nguyên tử tạo thành nó.
Anion luôn có kích thước lớn hơn so với nguyên tử tạo thành nó.
c) Các ion và nguyên tử đẳng electron:
Các tiểu phân có cùng số electron và có cùng công thức cấu hình e ở trạng
thái bền vững thì:
• Cation có bán kính nhỏ hơn anion.
• Các cation đẳng e, bán kính giảm khi điện tích (+) tăng.
• Các anion đẳng e, bán kính tăng khi điện tích (-) tăng.
1.4.2 Năng lượng ion hóa I
Là năng lượng tối thiểu cần tiêu tốn để tách electron ra khỏi nguyên tử
không bị kích thích ở trạng thái khí để tạo thành ion dương.
M (g) → M+(g) + e, I1
I đặc trưng cho khả năng nhường electron (tính kim loại) và được đo bằng
đơn vị (kJ.mol-1). I luôn có dấu dương, I càng lớn thì electron càng khó
tách ra khỏi nguyên tử.
Trong cùng một nguyên tử, năng lượng tách electron thứ nhất I1, electron
thứ hai I2… khác nhau
Ví dụ:
M (g) → M+(g) + e, I1
M+ (g) → M2+(g) + e, I2
M2+ (g) → M3+(g) + e, I3
Ta có, I1 < I2 < I3
11
Quy luật biến thiên:
• Trong chu kỳ, từ trái qua phải nói chung I1 có khuynh hướng tăng. Tuy
nhiên, lưu ý là trong cùng 1 chu kỳ ngắn tồn tại các cực trị tại các cặp
Be-B, N-O, Mg-Si, P-S.
• Các cấu hình 1s2 và s2p6 (khí trơ) là bền vững nhất nên năng lượng ion
hóa của chúng là rất lớn. Các cấu hình s2 và p3 là những cấu hình tương
đối bền, có năng lượng ion hóa khá lớn. Đối với các nguyên tố chuyển
tiếp, trong cùng một chu kỳ năng lượng ion hóa ít thay đổi.
• Trong nhóm chính, từ trên xuống dưới I1 có khuynh hướng giảm.
1.4.3 Ái lực electron EA
Là năng lượng thoát ra hay thu vào khi kết hợp thêm electron vào nguyên
tử ở trạng thái khí để tạo thành ion âm.
X + e → X -, EA = - ΔH
EA đặc trưng cho khả năng nhận electron (tính phi kim loại) và cũng được
đo bằng đơn vị giống với I (kJ.mol-1).
Ví dụ:
O (g) + e → O- (g), H = - 141 kJ.mol-1 → EA = 141 kJ.mol-1
Quy luật biến thiên:
• Trong nhóm, từ trên xuống dưới EA giảm.
• Trong chu kỳ, từ trái qua phải EA tăng.
• Các nguyên tố p nhóm VII có EA lớn nhất.
• Các nguyên tố có cấu hình electron ngoài cùng s2, p3, s2p6 có EA nhỏ
nhất.
1.4.4 Độ âm điện χ
Độ âm điện đặc trưng cho khả năng của nguyên tử trong phân tử hút
electron (liên kết) về phía mình.
Theo Mulliken:
1 (1.9)
 = ሺIA + EA ሻ
2
IA, EA là năng lượng ion hóa và ái lực electron.
Linus Pauling dựa trên năng lượng phân ly liên kết:
 −  = 0,102ξ∆
A A
(1.10)
∆ = ∆Elk ሺA − Bሻ = ඥ∆Elk ሺA − Aሻ. ∆Elk ሺB − Bሻ
χA, χB là độ âm điện của các nguyên tố A và B

12
Quy luật biến thiên:
• Trong nhóm phân nhóm chính (A), độ âm điện giảm từ trên xuống dưới.
• Trong chu kỳ, độ âm điện tăng từ trái qua phải.
• F có độ âm điện cao nhất (3,98). Fr có độ âm điện nhỏ nhất (0,70).
• F > O > Cl > N > S > C > P > B > Si…
• Đa số các nguyên tố d có độ âm điện từ 1,2 đến 1,9; các nguyên tố f có
độ âm điện xấp xỉ 1,3.
1.4.5 Số oxy hóa
Số oxy hóa là điện tích dương (+) hay âm (-) của nguyên tố trong hợp chất
được tính với giả thiết hợp chất được tạo thành từ các ion.
Trong hợp chất ion, số oxy hóa của nguyên tố trùng với điện hóa trị. Trong
hợp chất cộng hóa trị, số oxy hóa của nguyên tố là đại lượng quy ước và
có thể bằng hoặc không bằng với cộng hóa trị của nó.
Sự thay đổi tuần hoàn số oxy hóa xảy ra là do các nguyên tố hóa học có
khuynh hướng cho hay nhận electron ở lớp ngoài cùng để đạt cấu hình bền
là s2p6 hoặc d10.
Quy luật biến thiên:
• Trong chu kỳ đi từ trái qua phải, số oxy hóa dương cao nhất tăng dần
và bằng số thứ tự của nhóm.
• Trong một chu kỳ đi từ trái qua phải, số oxy hóa âm thấp nhất giảm dần
và có giá trị bằng 8 trừ đi số thứ tự của nhóm (từ nhóm IV đến nhóm
VII).

13
Phần 2. BÀI TẬP CÓ LỜI GIẢI
---oOo---
Dạng 1. Nguyên tử - Đồng vị
Câu 1.1. Hãy xác định số proton và nơtron trong hạt nhân nguyên tử
226
88Ra.
Giải
Ký hiệu nguyên tử tổng quát:
Z = số hiệu nguyên tử = số proton = số electron = 88
A = số khối = số proton + số nơtron = 226
Số nơtron = A – Z = 226 – 88 = 138
Câu 1.2. Cho các nguyên tử 266 264 277 268
106Sg, 107Bh, 108Hs, 109Mt. Hãy xác định
nguyên tử chứa ít nơtron và nhiều nơtron nhất.
Giải
Nguyên tử 266
106Sg có 266 – 106 = 160 nơtron.
Nguyên tử 264
107Bh có 264 – 107 = 157 nơtron.
Nguyên tử 277
108Hs có 277 – 108 = 169 nơtron.
Nguyên tử 268
109Mt có 268 – 109 = 159 nơtron.
Vậy nguyên tử có ít nơtron nhất là Bh và nhiều nơtron là Hs.
Câu 1.3. Trong tự nhiên, silic tồn tại 3 đồng vị bền với các thông tin như
bảng sau. Hãy tính khối lượng nguyên tử trung bình của silic.
Đồng vị Khối lượng nguyên tử Hàm lượng %
28
Si 27,977 92,93
29
Si 28,976 4,67
30
Si 29,974 3,10
Giải
Khối lượng nguyên tử trung bình của silic:
(27,977 x 0,9293) + (28,976 x 0,0467) + (29,974 x 0,0310) = 28,09 đvC
Câu 1.4. Trong tự nhiên, đồng có 2 đồng vị với các thông tin như bảng
sau. Khối lượng nguyên tử của đồng là 63,546. Tính tỷ lệ phần trăm số
nguyên tử của từng đồng vị?
Đồng vị Khối lượng nguyên tử Hàm lượng %
63
Cu 62,9298 ?
65
Cu 64,9278 ?

14
Giải
Gọi a là tỷ lệ phần trăm số nguyên tử của 65Cu
→ tỷ lệ phần trăm số nguyên tử của 63Cu là (1-a)
→ 62,9298x(1-a) + 64,9278xa = 63,546
→ a = 0,3084
Vậy 65Cu chiếm 30,84% và 63Cu chiếm 69,16%.
Dạng 2. Orbital nguyên tử
Câu 1.5. Có 3 orbital nguyên tử (AO) tương ứng với các bộ số lượng tử
sau. Hãy xác định tên của 3 AO?
(1) n = 5, ℓ = 2; (2) n = 4, ℓ = 3; (3) n = 3, ℓ = 0
Giải
Tên của 3 AO lần lượt là:
(1) n = 5, ℓ = 2 → tên AO là 5d
(2) n = 4, ℓ = 3 → tên AO là 4f
(3) n = 3, ℓ = 0 → tên AO là 3s
Câu 1.6. Hãy xác định tên của các orbital nguyên tử (nếu có) tương ứng
với bộ các số lượng tử sau đây:
(1) n = 3, ℓ = 3;
(2) n = 4, ℓ = 2;
(3) n = 5, ℓ = 3;
(4) n = 5, ℓ = 4.
Giải
Tên của các AO lần lượt là:
(1) n = 3, ℓ = 3 → AO không tồn tại
(2) n = 4, ℓ = 2 → tên AO là 4d
(3) n = 5, ℓ = 3 → tên AO là 5f
(4) n = 5, ℓ = 0 → tên AO là 5s
Câu 1.7. Bộ các số lượng tử nào xác định nên orbital nguyên tử (AO) 2px?
Giải
Orbital nguyên tử 2px cho biết AO này nằm trên lớp thứ 2, phân lớp p và
có định hướng theo trục x. Như vậy, AO này được xác định bởi 3 số lượng
tử lần lượt là n, ℓ, mℓ.
Câu 1.8. Tương ứng với bộ 2 số lượng tử; n = 3, ℓ = 2 có tổng cộng bao
nhiêu AO?
Giải
Số lượng AO phụ thuộc vào tổng số lượng tử từ mℓ có thể có. Vì ℓ = 2 nên
mℓ có thể có các giá trị mℓ = -2, -1, 0, +1, +2. Do đó, có tất cả 5 AO.

15
Dạng 3. Bộ số lượng tử
Câu 1.9. Bộ 4 số lượng tử nào sau đây có thể chấp nhận được?
(1) n = 2, ℓ = 2, mℓ = 1, ms = -1/2
(2) n = 3, ℓ = 0, mℓ = 0, ms = +1/2
(3) n = 4, ℓ = 1, mℓ = 2, ms = -1/2
(4) n = 5, ℓ = -1, mℓ = 0, ms = +1/2
(5) n = 4, ℓ = 2, mℓ = 1, ms = +1/2
(6) n = 1, ℓ = 0, mℓ = 0, ms = +1/2
(7) n = 5, ℓ = 2, mℓ = -2, ms = -1/2
(8) n = 3, ℓ = -1, mℓ = -1, ms = +1/2
Giải
Quy tắc xác định bộ bốn số lượng tử như sau:
n= 1, …, n
ℓ = 0, …, (n-1)

ml = 0,  1, …,  l

ms = -1/2 hoặc +1/2


(1) n = 2, ℓ = 2, mℓ = 1, ms = -1/2 không thể chấp nhận được vì n = 2 thì ℓ
chỉ có 2 giá trị là 0 hoặc 1.
(2) n = 3, ℓ = 0, mℓ = 0, ms = +1/2 có thể chấp nhận được.
(3) n = 4, ℓ = 1, mℓ = 2, ms = -1/2 không thể chấp nhận được vì ℓ = 1 thì
mℓ chỉ có 3 giá trị là -1, 0, +1.
(4) n = 5, ℓ = -1, mℓ = 0, ms = +1/2 không thể chấp nhận được vì ℓ < 0.
(5) n = 4, ℓ = 2, mℓ = 1, ms = +1/2 có thể chấp nhận được
(6) n = 1, ℓ = 0, mℓ = 0, ms = +1/2 có thể chấp nhận được
(7) n = 5, ℓ = 2, mℓ = -2, ms = -1/2 có thể chấp nhận được
(8) n = 3, ℓ = -1, mℓ = -1, ms = +1/2 không thể chấp nhận được vì ℓ < 0.
Dạng 4. Cấu hình electron nguyên tử
Câu 1.10. Viết cấu hình electron của nguyên tử sau
(a) Fe (Z = 26); (b) Cl (Z = 17); (c) Fe2+; (d) Fe3+
Giải
Dựa vào các quy tắc sắp xếp electron vào các AO và sơ đồ trật tự năng
lượng các AO ta có:
(a) Fe: 1s22s22p63s23p63d64s2
(b) Cl-: 1s22s22p63s23p6

16
(c) Fe2+: 1s22s22p63s23p63d6
(d) Fe3+: 1s22s22p63s23p63d5
Lưu ý:
✓ Đối với cấu hình electron của ion âm (anion): điền thêm electron
theo quy tắc Kleshkovski.
✓ Đối với cấu hình electron ion dương (cation): phải viết cấu hình
của nguyên tử trung hòa trước, sau đó sắp xếp lại theo lớp rồi trừ
từ bên ngoài vào.
Câu 1.11. Xác định 4 số lượng tử của electron cuối cùng của Fe (Z = 26).
Giải
Fe: 1s22s22p63s23p63d64s2
Electron cuối cùng của Fe (e thứ 26) nằm trên phân lớp 3d.
Biểu diễn AO nguyên tử
3d6     

mℓ -2 -1 0 +1 +2
Electron cuối cùng của Fe là electron thứ 6 trên 3d nên nằm trên AO có
ml = -2 và ms = -1/2.
Vậy 4 số lượng tử của electron cuối cùng của Fe là:
n = 3, ℓ = 2, mℓ = -2, ms = -1/2.
Bằng cách tương tự ta có thể xác định bộ bốn số lượng tử của 1 electron
bất kỳ trong 26 electron của Fe và mỗi electron sẽ có 1 bộ 4 số lượng tử
khác nhau.
Câu 1.12. Xác định số electron độc thân của Fe (Z = 26) ở trạng thái bình
thường.
Giải
Các electron phân lớp bên trong đã bão hòa nên chứa các electron đã ghép
đôi.
Xét phân lớp ngoài cùng 3d6
Biểu diễn AO nguyên tử
3d6     

mℓ -2 -1 0 +1 +2
Vậy Fe có 4 electron độc thân.

17
Dạng 5. Cấu hình electron nguyên tử và hệ thống tuần hoàn các
nguyên tố hóa học
Câu 1.13. Xác định vị trí của nguyên tố Co (Z = 27) trong hệ thống tuần
hoàn (ô, chu kỳ, nhóm, phân nhóm) và tính chất đặc trưng của Co (kim
loại, phi kim, hay khí hiếm)?
Giải
Cấu hình electron của Co: [Ar] 3d7 4s2
✓ Co có 4 lớp electron nên thuộc chu kỳ 4.
✓ Co có e cuối cùng nằm trên 3d nên là nguyên tố họ d và phân nhóm B.
✓ Co có tổng cộng 7 + 2 = 9 electron hóa trị nên thuộc nhóm VIII.
✓ Co có Z = 27 nên thuộc ô số 27.
✓ Co là nguyên tố chuyển tiếp nên là kim loại.
Như vậy vị trí của Co trong bảng hệ thống tuần hoàn là: chu kỳ 4, nhóm
VIIIB, ô số 27, là kim loại.
Câu 1.14. Cho các tiểu phân sau: C (Z = 6), Cl- (Z = 17), Mn2+ (Z = 25),
B- (Z = 5), Ar (Z = 18), Zn (Z = 30), Fe3+ (Z = 26), Ge2+ (Z = 32). Các tiểu
phân nào là đẳng electron với nhau?
Giải
Các tiểu phân đẳng electron với nhau bao gồm:
(a) C và B-: 1s22s22p2
(b) Cl- và Ar: 1s22s22p63s23p6
(c) Mn2+ và Fe3+: 1s22s22p63s23p63d54s0
(d) Zn và Ge2+: 1s22s22p63s23p64s23d10
Dạng 6. Các tính chất thay đổi tuần hoàn
Câu 1.15. Sắp xếp các nguyên tố dưới đây theo thứ tự bán kính nguyên tử
tăng dần: Fr (Z = 87), F (Z = 9), Na (Z = 23), Cl (Z = 17). Giải thích?
Giải
F và Cl thuộc nhóm VIIA, chu kỳ 2 và 3 nên bán kính tăng theo chiều
F < Cl.
K và Fr thuộc nhóm IA, chu kỳ 3 và 7 nên bán kính tăng theo chiều
Na < Fr.
Na và Cl cùng thuộc chu kỳ 3 nhóm IA và VIIA nên bán kính tăng theo
chiều: Cl < Na.
Do đó, bán kính nguyên tử tăng theo thứ tự: F < Cl < Na < Fr.

18
Câu 1.16. Sắp xếp các ion dưới đây theo thứ tự bán kính tăng dần: O2- (Z
= 8), F- (Z = 9), Na+ (Z = 11), Mg2+ (Z = 12). Giải thích?
Giải
Cả 4 ion trên đều có 10 electron nên chúng là các ion đẳng electron.
Các ion đẳng electron, ion âm có bán kính lớn hơn.
Trong các ion đẳng electron, điện tích dương càng lớn thì bán kính càng
nhỏ; ngược lại điện tích âm càng lớn thì bán kính càng lớn.
Do đó, bán kính ion tăng theo thứ tự: Mg2+ < Na+ < F- < O2-.
Câu 1.17. Sắp xếp các nguyên tố dưới đây theo thứ tự năng lượng ion hóa
I1 tăng dần: Mg (Z = 12), Al (Z = 13), Na (Z = 11), Si (Z = 14). Giải thích?
Giải
Cả 4 nguyên tố trên đều thuộc chu kỳ 3 với electron hóa trị là: Na (3s1),
Mg (3s2), Al (3s23p1), và Si (3s23p2).
Theo chiều từ trái sang phải năng lượng ion hóa I1 sẽ tăng tuy nhiên Mg
có cấu hình electron hóa trị 3s2 bão hòa của phân lớp s nên năng lượng ion
hóa I1 của Mg sẽ cao hơn I1 của Al 3s23p1.
Do đó, năng lượng ion hóa I1 tăng theo thứ tự: Na < Al < Mg < Si.
Câu 1.18. Sắp xếp các nguyên tố dưới đây theo thứ tự ái lực với electron
E tăng dần: Cs (Z = 55), Br (Z =35), K (Z = 19), F (Z = 9). Giải thích?
Giải
K và Cs thuộc nhóm IA, chu kỳ 4 và 6 nên ái lực với electron E tăng theo
chiều Cs < K.
F và Br thuộc nhóm VIIA, chu kỳ 2 và 4 nên ái lực với electron E tăng
theo chiều Br < F.
K và Br cùng thuộc chu kỳ 4 nhóm IA và VIIA nên ái lực với electron E
tăng theo chiều: K < Br.
Do đó, ái lực với electron E tăng theo thứ tự: Cs < K < Br < F.
Câu 1.19. Sắp xếp các nguyên tố dưới đây theo thứ tự độ âm điện tăng
dần: K (Z = 19), Al (Z = 14), Si (Z = 14), S (Z =16). Giải thích?
Giải
K thuộc nhóm IA, chu kỳ 4 nên độ âm điện thấp nhất.
Al, Si và S nhóm IIIA, IVA và VIA; chu kỳ 3 độ âm điện tăng theo chiều
Al < Si < S. Do đó, độ âm điện tăng theo thứ tự: K < Al < Si < S.

19
Phần 3. BÀI TẬP TRẮC NGHIỆM
---oOo---
Câu 1.1. Nguyên tử nào sau đây có số electron = số proton = số nơtron:
4 9 12 16 1 11 23 14 22 40
2He; 4Be; 6C; 8O; 1H; 5B; 11Na; 7N; 10Ne; 20Ca.
A. He, C, O, N, Ca. B. Be, H, B, Na, Ne.
C. He, C, O, N, Ca, H. D. C, O, N, Ca, H, B, Ne.
Câu 1.2. Số proton và nơtron trong hạt nhân nguyên tử 235 92U lần lượt là:
A. 92 – 235 B. 235 – 92
C. 92 – 143 D. 14 – 92
Câu 1.3. Chọn câu đúng:
A. Khối lượng nguyên tử trung bình của một nguyên tử được xem như gần
bằng khối lượng nguyên tử của đồng vị chiếm tỷ lệ % hiện diện nhiều nhất.
B. Khối lượng của các hạt electron, proton, nơtron là xấp xỉ bằng nhau.
C. Trong một nguyên tử hay một ion bất kỳ, số proton luôn luôn bằng số
electron.
D. Hạt nhân nguyên tử có kích thước rất bé hơn kích thước nguyên tử
nhưng lại có khối lượng chiếm gần trọn khối lượng nguyên tử.
Câu 1.4. Tổng số hạt proton, nơtron và electron của một nguyên tử là 48.
Số hạt mang điện nhiều gấp đôi số hạt không mang điện. Vậy số khối A
và điện tích hạt nhân Z của nguyên tố lần lượt là:
A. 32, 16 B. 31, 15
C. 42, 12 D. 39, 19
Câu 1.5. Nguyên tố hóa học là:
A. Tập hợp các nguyên tử có cùng điện tích hạt nhân nguyên tử.
B. Tập hợp các nguyên tử có cùng các tính chất lý, hóa học.
C. Tập hợp các nguyên tử có cùng số khối.
D. Tập hợp các nguyên tử có cùng số nơtron trong nguyên tử
Câu 1.6. Số proton và nơtron trong hạt nhân nguyên tử 209 84Po là:
A. 88 proton, 209 nơtron B. 209 proton, 84 nơtron
C. 84 nơtron, 125 proton D. 84 proton, 125 nơtron
Câu 1.7. Cho các nguyên tử: 92U; 94Pu; 93Np; 243
238 244 237
95Am. Các nguyên tử
chứa ít nơtron và nhiều nơtron nhất là:
A. Pu và Am B. U và Pu
C. Np và Pu D. Np và Am
Câu 1.8. Trong tự nhiên silic gồm có 3 đồng vị:

20
Đồng vị Khối lượng nguyên tử Hàm lượng %
28
14Si 27,977 92,23
29
14Si 28,976 4,67
30
14Si 29,974 3,10
Trong các phát biểu sau đây, có tổng cộng bao nhiêu phát biểu đúng?
(i) Nguyên tử silic có 14 electron.
(ii) Khối lượng nguyên tử trung bình của silic là 28,086.
(iii) Nguyên tử silic có 14 nơtron.
A. 0 B. 1 C. 2 D. 3
Câu 1.9. Trong tự nhiên Mg gồm có 3 đồng vị:
Đồng vị Khối lượng nguyên tử Hàm lượng %
24
12Mg 23,98504 78,99
25
12Mg 24,98584 10,00
26
12Mg 25,98259 11,01
Khối lượng nguyên tử trung bình của Mg được tính như sau:
A. (24 + 25 + 26) : 3 = 25
B. (23,98504 + 24,98584 + 25,98259) : 3 = 24,98449
C. 23,98504x0,7899 + 24,98584x0,1000 + 25,98259x0,1101 = 24,30505
D. 24 x 0,7899 + 25 x 0,1000 + 26 x 0,1101 = 24,3202
Câu 1.10. Nguyên tử của một đồng vị của một nguyên tố X có tổng số hạt
là 52 trong đó số electron ít hơn số nơtron 1 hạt. Đồng vị X có ký hiệu là:
A. 32
16S B. 35
17Cl
28
C. 14Si D. 31
15P
Câu 1.11. Nguyên tử của một đồng vị của một nguyên tố X có tổng số hạt
là 46, có số khối A = 31. Số hạt nơtron có trong hạt nhân nguyên tố X có
giá trị là:
A. N = 16 B. N = 14
C. N = 15 D. N = 17
Câu 1.12. Cho các nguyên tử: 17A ; 17B; 18C; 38
35 37 36
18D. Các cặp nguyên tử
không phải là đồng vị là:
A. A và B
B. C và D
21
C. Chỉ có B và C
D. A và C, A và D, B và C, B và D
Câu 1.13. Cacbon có 2 đồng vị là 126C chiếm 98,89% và 136C chiếm 1,11%.
Khối lượng nguyên tử trung bình của nguyên tố carbon là:
A. 12,5 B. 12,011 C. 12,021 D. 1 2,045
65 63
Câu 1.14. Đồng có 2 đồng vị bền là 29Cu và 29Cu. Nguyên tử khối lượng
nguyên tử trung bình của đồng là 63,54. Thành phần phần trăm của đồng
vị 65
29Cu là:
A. 30%. B. 27%. C. 28%. D. 27,5%.
Câu 1.15. Một nguyên tố R có 2 đồng vị có tỷ lệ số nguyên tử là 27/23.
Hạt nhân của R có 35 hạt proton. Đồng vị 1 có 44 hạt nơtron, đồng vị 2 có
số khối nhiều hơn đồng vị 1 là 2. Khối lượng nguyên tử trung bình của
nguyên tố R là bao nhiêu?
A. 81 B. 80,08 C. 79,92 D. 80,5
Câu 1.16. Khối lượng nguyên tử B là 10,81. B gồm 2 đồng vị: 105B và 115B.
Thành phần % đồngvị 1 115B trong phân tử H3BO3 là bao nhiêu? Biết H = 1
và O = 16 g/mol.
A. 15%. B. 14%. C. 14,51%. D. 14,16%.
52 3+
Câu 1.17. Số proton, nơtron, electron của 24Cr lần lượt là:
A. 24, 28, 24. B. 24, 28, 21.
C. 24, 30, 21. D. 24, 28, 27.
Câu 1.18. Các ion và nguyên tử S2– (Z = 16), Cl- (Z = 17), K+ (Z = 19),
Ca2+ (Z = 20), Ar (Z =18) có:
A. số electron bằng nhau. B. số proton bằng nhau.
C. số nơtron bằng nhau. D. số khối bằng nhau.
Câu 1.19. Có 3 orbital nguyên tử (AO):
(1) n = 5, ℓ = 3; (2) n = 4, ℓ = 1; (3) n = 3, ℓ = 2.
Tên của 3 AO lần lượt là:
A. 5f, 4p, 3d B. 5p, 4s, 3f
C. 5f, 4d, 3d D. 5d, 4p, 3s
Câu 1.20. 1 orbital nguyên tử (AO) 4f tương ứng với bộ số lượng tử nào
sau đây:
A. n = 3, ℓ = 3; B. n = 4, ℓ = 3;
C. n = 5, ℓ = 3; D. n = 4, ℓ = 2;
Câu 1.21. Bộ các số lượng tử nào sau đây xác định nên orbital nguyên tử
(AO) 3dxy:
22
A. n, ℓ B. n, ℓ, mℓ
C. n, ℓ, ms D. n, ℓ, mℓ, ms
Câu 1.22. Tương ứng với bộ 2 số lượng tử; n = 3, ℓ = 1 có tổng cộng:
A. 1 orbital nguyên tử B. 3 orbital nguyên tử
C. 5 orbital nguyên tử D. 7 orbital nguyên tử
Câu 1.23. Có tổng cộng bao nhiêu phát biểu sai trong các phát biểu sau
đây?
(i) Các AO lớp n luôn có năng lượng cao hơn AO lớp (n-1).
(ii) Số lượng tử ℓ xác định hình dạng và số lượng AO trong một phân lớp.
(iii) Công thức 2n2 cho biết số nguyên tố tối đa có thể có trong một chu kỳ
trong hệ thống tuần hoàn.
(iv) Số lượng tử chính n nhận giá trị nguyên dương từ 1 cho đến 7.
A. 1 B. 2 C. 3 D. 4
Câu 1.24. Có tổng cộng bao nhiêu bộ số lượng tử có thể chấp nhận trong
các bộ sau đây?
(i) n = 3, ℓ = 3, mℓ = +1.
(ii) n = 3, ℓ = 2, mℓ = +2.
(iii) n = 3, ℓ = 1, mℓ = -2.
(iv) n = 3, ℓ = 0, mℓ = 0.
A. 4 B. 3 C. 2 D. 1
Câu 1.25. Chọn phát biểu đúng về orbital nguyên tử (AO):
A. Là vùng không gian bên trong đó có xác suất có mặt của electron ≥
90%.
B. Là quỹ đạo chuyển động của electron.
C. Là vùng không gian bên trong đó các electron chuyển động.
D. Là bề mặt có mật độ electron bằng nhau của đám mây electron.
Câu 1.26. Trong các ký hiệu phân lớp lượng tử sau đây ký hiệu nào đúng?
A. 1s, 3d, 4p, 2p, 3f. B. 5p, 3s ,4f, 2d, 1s.
C. 3g, 5f, 2p, 3d, 4s. D. 1s, 2p, 4f, 3p, 4d.
Câu 1.27. Chọn phát biểu đúng trong các phát biểu sau:
1. Trong cùng một nguyên tử, AO np có kích thước lớn hơn AO (n-1)p.
2. Trong cùng một nguyên tử, electron trên AO ns có mức năng lượng lớn
hơn electron trên AO (n-1)s.
3. Trong cùng một nguyên tử, electron trên AO 3dxy có mức năng lượng
lớn hơn electron trên AO 3dyz.
4. Xác suất gặp electron trên orbital 4f ở mọi hướng là như nhau.

23
A. 1, 2, 3, 4. B. 1, 2, 3.
C. 1, 2, 4. D. 1, 2.
Câu 1.28. Orbital 6f có các số lượng tử n, ℓ và số electron tối đa lần lượt
là:
A. n = 6; ℓ = 1; 7 B. n = 6; ℓ = 2; 10
C. n = 6; ℓ = 4; 18 D. n = 6; ℓ = 3; 14
Câu 1.29. Orbital 1s của nguyên tử H có dạng hình cầu, nghĩa là:
A. Xác suất gặp electron 1s của H giống nhau theo mọi hướng trong không
gian.
B. Quỹ đạo chuyển động của e là hình cầu.
C. Khoảng cách của electron 1s đến nhân H luôn luôn không đổi.
D. electron 1s chỉ di chuyển tại vùng không gian bên trong hình cầu ấy.
Câu 1.30. Phát biểu nào sau đây là sai về số lượng tử từ mℓ?
A. Đặc trưng cho sự định hướng của các AO trong không gian.
B. Cho biết số lượng AO trong một lớp lượng tử.
C. Có giá trị bao gồm – ℓ, … , 0 , … , +ℓ.
D. Số giá trị của mℓ phụ thuộc vào giá trị của ℓ.
Câu 1.31. Bộ 4 số lượng tử nào sau đây có thể chấp nhận được?
A. n = 2, ℓ = 3, mℓ = 1, ms = -1/2 B. n = 5, ℓ = 0, mℓ = 0, ms = +1/2
C. n = 4, ℓ = 2, mℓ = -3, ms = -1/2 D. n = 5, ℓ = -3, mℓ = -1, ms = +1/2
Câu 1.32. Bộ 4 số lượng tử nào sau đây không thể chấp nhận được?
A. n = 4, ℓ = 1, mℓ = -1, ms = -1/2 B. n = 2, ℓ = 0, mℓ = 0, ms = +1/2
C. n = 5, ℓ = 3, mℓ = 2, ms = -1/2 D. n = 1, ℓ = 2, mℓ = -2, ms = +1/2
Câu 1.33. Số electron tối đa và số lượng tử chính n của các lớp lượng tử
K và O là:
A. lớp K: 8e, n = 2; lớp O: 72e, n = 6
B. lớp K: 2e, n = 1; lớp O: 25e, n = 5
C. lớp K: 2e, n = 2; lớp O: 32e, n = 4
D. lớp K: 2e, n = 1; lớp O: 50e, n = 5
Câu 1.34. Chọn phương án đúng:
Trạng thái của electron ở lớp ngoài cùng trong nguyên tử có Z = 29 được
đặc trưng bằng các số lượng tử (quy ước electron điền vào các orbital theo
thứ tự mℓ từ –ℓ đến +ℓ):
A. n = 4, ℓ = 0, mℓ = 0, ms = +1/2 và -1/2
B. n = 3, ℓ = 2, mℓ = –2, ms = +1/2
C. n = 4, ℓ = 0, mℓ = 0, ms = +1/2

24
D. n = 3, ℓ = 2, mℓ = +2, ms = –1/2
Câu 1.35. Electron cuối của nguyên tử A có bộ các số lượng tử sau (quy
ước electron điền vào các AO theo thứ tự mℓ từ –ℓ đến +ℓ): n = 3, ℓ = 2,
mℓ = +2, ms = +1/2. Xác định số thứ tự trong bảng hệ thống tuần hoàn và
công thức electron nguyên tử của A ở trạng thái cơ bản:
A. Z = 21, A: 1s22s22p63s23p63d3
B. Z = 25, A: 1s22s22p63s23p63d54s2
C. Z = 24, A: 1s22s22p63s23p63d54s1
D. Z = 22, A: 1s22s22p63s23p63d24s2
Câu 1.36. Trạng thái của electron ở lớp lượng tử ngoài cùng trong các
nguyên tử có Z = 23 được đặc trưng bằng các số lượng tử:
A. n = 4, ℓ = 0, mℓ = 0, ms = +1/2 và -1/2
B. n = 3, ℓ = 2, mℓ = -2, ms = +1/2
C. n = 3, ℓ = 2, mℓ = +2, ms = -1/2
D. n = 4, ℓ = 2, mℓ = +2, ms = -1/2
Câu 1.37. Ion X4+ có cấu hình e phân lớp cuối cùng là 3d3. Vậy giá trị của
4 số lượng tử của e cuối cùng của nguyên tử X là (quy ước mℓ có giá trị từ
–ℓ đến +ℓ):
A. n = 3, ℓ = 1, mℓ = –1, ms = –1/2
B. n = 3, ℓ = 2, mℓ = +2, ms = +1/2
C. n = 3, ℓ = 2, mℓ = +1, ms = +1/2
D. n = 3, ℓ = 2, mℓ = –2, ms = –1/2
Câu 1.38. Electron cuối cùng của ion Cl– có bộ các số lượng tử sau (quy
ước electron điền vào các AO theo thứ tự mℓ từ –ℓ đến +ℓ và Cl có
Z = 17):
A. n = 3, ℓ = 2, mℓ = +1, ms = +1/2
B. n = 3, ℓ = 1, mℓ = −1, ms = +1/2
C. n = 3, ℓ = 1, mℓ = 0, ms = −1/2
D. n = 3, ℓ = 1, mℓ = +1, ms = −1/2
Câu 1.39. Một ion R3+ có phân lớp cuối cùng của cấu hình electron là 3d5.
Cấu hình electron của nguyên tử R là:
A. 1s22s22p63s23p63d54s24p1 B. 1s22s22p63s23p63d64s2
C. 1s22s22p63s23p63d8 D. 1s22s22p63s23p63d54s3
Câu 1.40. Ion S2– có cấu hình electron lớp ngoài cùng là 3s23p6. Ở trạng
thái cơ bản, nguyên tử S có bao nhiêu electron độc thân?
A. 1 B. 2 C. 4 D. 6

25
Câu 1.41. Nguyên tố nào dưới đây có tổng spin trong nguyên tử lớn nhất
theo quy tắc Hund?
A. 24Cr B. 25Mn
C. 29Cu D. 30Zn
Câu 1.42. Nguyên tố nào dưới đây có tổng spin trong nguyên tử nhỏ nhất
theo quy tắc Hund?
A. 25Mn B. 27Co
C. 29Cu D. 30Zn
Câu 1.43. Cho các nguyên tố: 22Ti, 24Cr, 25Mn, 29Cu, 30Zn. Ở trạng thái cơ
bản nguyên tố có nhiều và ít electron độc thân nhất lần lượt là:
A. Mn, Cu B. Cr, Zn
C. Ti, Mn D. Cr, Cu
Câu 1.44. Chu kỳ 3 và chu kỳ 6 có tối đa bao nhiêu nguyên tố?
A. Chu kỳ 3: 8 nguyên tố; Chu kỳ 6: 32 nguyên tố
B. Chu kỳ 3: 8 nguyên tố; Chu kỳ 6: 18 nguyên tố
C. Chu kỳ 3: 18 nguyên tố; Chu kỳ 6: 32 nguyên tố
D. Chu kỳ 3: 8 nguyên tố; Chu kỳ 6: 18 nguyên tố
Câu 1.45. Nguyên tố X ở chu kỳ 4, phân nhóm VIIB. Nguyên tố X có:
A. Z = 26, là kim loại B. Z = 27, là phi kim
C. Z = 18, là khí hiếm D. Z = 11, là kim loại
Câu 1.46. Nguyên tử của nguyên tố X có 7 electron ở lớp ngoài cùng và
thuộc chu kỳ 4. Có các phát biểu sau đây về X:
(1) Cấu hình electron hóa trị của X là 4s23d5.
(2) X có điện tích hạt nhân Z = 35.
(3) X thuộc chu kỳ 4, phân nhóm chính 7B trong bảng hệ thống tuần hoàn.
(4) Số oxy hóa dương cao nhất của X là +7.
Các phát biểu nào là đúng?
A. 2, 3, 4 B. 2, 4
C. 1, 2, 3 D. 1, 3
Câu 1.47. Ion M và ion X có phân lớp cuối cùng lần lượt là 2p6 và 4p5.
3+ -

Phát biểu nào sau đây là đúng?


A. M thuộc chu kỳ 3, phân nhóm 3B, là kim loại; X thuộc chu kỳ 4, phân
nhóm 5A, là phi kim.
B. M thuộc chu kỳ 2, phân nhóm 8A, là khí hiếm; X thuộc chu kỳ 4, phân
nhóm 6A, là phi kim.

26
C. M thuộc chu kỳ 3, phân nhóm 3A, là kim loại; X thuộc chu kỳ 4, phân
nhóm 6A, là phi kim.
D. M thuộc chu kỳ 3, phân nhóm 3A, là kim loại; X thuộc chu kỳ 4, phân
nhóm 6B, là kim loại.
Câu 1.48. Cấu hình electron hóa trị của nguyên tố có số thứ tự 33 trong
bảng hệ thống tuần hoàn các nguyên tố là:
A. 4s14p2 B. 3d104s14p2
C. 4s24p3 D. 3d104s24p1
Câu 1.49. Nguyên tố A có 4 lượng tử của electron cuối cùng là n = 3; ℓ =
2; mℓ = +2; ms =+1/2. Quy ước các orbital được sắp xếp mℓ theo thứ tự từ
-ℓ đến +ℓ. Nguyên tố A có:
A. Z = 23 và là phi kim B. Z = 24 và là phi kim
C. Z = 22 và là kim loại D. Z = 25 và là kim loại
Câu 1.50. Ion A có cấu hình e phân lớp cuối cùng là 3p6. Vị trí của A
5+

trong bảng hệ thống tuần hoàn là:


A. Chu kỳ 4, phân nhóm VIIB, ô 25
B. Chu kỳ 4, phân nhóm VB, ô 23
C. Chu kỳ 3, phân nhóm IVA, ô 14
D. Chu kỳ 4, phân nhóm VIB, ô 24
Câu 1.51. Ion X- có phân lớp electron ngoài cùng là 3p6. Có các phát biểu
sau đây về X:
(i) Cấu hình electron hóa trị của X là 3s23p5.
(ii) X có điện tích hạt nhân Z = 17.
(iii) X thuộc chu kỳ 3, phân nhóm chính VIIA trong bảng hệ thống tuần
hoàn.
(iv) X có số oxy dương lớn nhất là +7.
Có tổng cộng bao nhiêu phát biểu đúng?
A. 1 B. 3 C. 4 D. 2
Câu 1.52. Dãy nguyên tố nào sau đây sắp xếp theo chiều tăng dần của bán
kính nguyên tử?
A. C, N, Si, F. B. Na, Ca, Mg, Al.
C. F, Cl, Br, I. D. S, O, Te, Se
Câu 1.53. Xét các nguyên tố Cl, Al, Na, P, F. Thứ tự tăng dần của bán
kính nguyên tử nào sau đây đúng?
A. Cl < F < P < Al < Na B. F < Cl < P < Al < Na
C. Na < Al < P < Cl < F D. Cl < P < Al < Na < F

27
Câu 1.54. Dãy nguyên tử 6O, 7F, 14Si, 16S có bán kính R tăng dần theo dãy:
A. RS < RSi < RF < RO B. RF < RO < RS < RSi
C. RO < RF < RSi < RS D. RSi < RS < RO < RF
Câu 1.55. Sắp các ion sau: 3Li , 11Na+, 17Cl-, 19K+, 35Br-, 53I- theo chiều
+

tăng dần bán kính.


A. Li+ < Na+ < Cl- < K+ < Br- < I B. Na+ < K+ < Cl- < Br- < I- < Li+
C. K+ < Cl- < Br- < I- < Na+ < Li+ D. Li+ < Na+ < K+ < Cl- < Br- < I-
Câu 1.56. Cho các phát biểu sau đây, có tổng cộng bao nhiêu phát biểu
đúng?
(i) I1 của các nguyên tố phân nhóm chính trong cùng một chu kỳ tăng dần
từ trái sang phải.
(ii) Trong cùng một nguyên tố, có mối liên hệ sau: I3 = I1 + I2.
(iii) Từ trên xuống trong phân nhóm chính nhóm I có I1 giảm dần.
A. 0 B. 1 C. 2 D. 3
Câu 1.57. Cho các nguyên tố 11Na, 12Mg, 13Al, 15P, 16S thuộc chu kỳ 3,
năng lượng ion hóa thứ nhất I1 của các nguyên tố trên tuân theo trật tự nào
sau đây:
A. Na < Al < Mg < S < P B. Na < Mg < Al < P < S
C. Na < Al < Mg < P < S D. S < P < Al < Mg < Na
Câu 1.58. Cho các nguyên tố có cấu hình electron như sau:
(A): 1s22s22p4
(B): 1s22s22p3
(C): 1s22s22p6
(D): 1s22s22p63s1
Năng lượng ion hóa thứ nhất (I1) của các nguyên tố tăng dần theo chiều:
A. D < C < B < A B. A < B < C < D
C. C < B < A < D D. D < A < B < C
Câu 1.59. Ái lực electron của nguyên tố:
A. là năng lượng phát ra (–) hay thu vào (+) khi kết hợp một electron vào
nguyên tử ở thể khí không bị kích thích.
B. tăng đều đặn trong một chu kỳ từ trái qua phải.
C. có trị số bằng năng lượng ion hóa thứ nhất (I1) của nguyên tố đó.
D. là năng lượng cần tiêu tốn để kết hợp thêm một electron vào nguyên tử
trung hòa.
Câu 1.60. Nguyên tố nào sau đây có ái lực với electron lớn nhất?
A. Rb B. C C. I D. Na

28
Câu 1.61. Nguyên tố nào sau đây có ái lực với electron lớn nhất?
A. F B. Cl C. He D. Na
Câu 1.62. Chọn phương án đúng:
Sắp xếp các nguyên tố sau theo thứ tự ái lực với electron tăng dần: 15P, 16S,
17Cl, 25Br
A. P < S < Br < Cl B. P < S < Cl < Br
C. Br < Cl < S < P D. Cl < Br < S < P
Câu 1.63. Nguyên tố nào sau đây có độ âm điện cao nhất?
A. Cl B. F C. Na D. K
Câu 1.64. Dãy nguyên tố nào sau đây sắp xếp theo chiều tăng dần độ âm
điện của nguyên tố?
A. Na, Cl, Mg, C. B. N, C, F, S.
C. Li, H, C, O, F. D. S, Cl, F, P.
Câu 1.65. Chọn phát biểu đúng:
A. Độ âm điện của một kim loại luôn nhỏ hơn độ âm điện của một phi kim.
B. Trong phân nhóm chính, từ trên xuống dưới độ âm điện tăng dần.
C. Trong cùng một chu kỳ, từ trái qua phải độ âm điện tăng.
D. Trong cùng một chu kỳ, các kim loại kiềm có độ âm điện nhỏ nhất.
Câu 1.66. Sắp xếp các nguyên tố sau theo thứ tự độ âm điện tăng dần:
82Pb, 7N, 51Sb, 83Bi.
A. Pb < Bi < Sb < N B. N < Sb < Bi < Pb
C. Pb < N < Sb < Bi D. N < Sb < Pb < Bi
Câu 1.67. Sắp xếp các nguyên tố sau theo thứ tự tính kim loại giảm dần:
55Cs, 20Ca, 12Mg, 19K, 4Be.
A. Be > Mg > Ca > K > Cs B. Cs > K > Ca > Mg > Be
C. K > Cs > Ca > Mg > Be D. Be > Mg > Ca > Cs > K
Câu 1.68. Sắp xếp các hydroxyt của nguyên tố sau theo thứ tự tính bazơ
tăng dần: 55Cs, 20Ca, 12Mg, 19K, 4Be.
A. Be(OH)2 < Mg(OH)2 < Ca(OH)2 < KOH < CsOH
B. CsOH < KOH < Ca(OH)2 < Mg(OH)2 < Be(OH)2
C. KOH < CsOH < Ca(OH)2 < Mg(OH)2 < Be(OH)2
D. Be(OH)2 < Mg(OH)2 < Ca(OH)2 < CsOH < KOH
Câu 1.69. Cho các nguyên tố As, P, S và Cl. Sắp xếp các nguyên tố theo
thứ tự tính phi kim tăng dần là:
A. As < P < S < Cl B. Cl < S < P < As
C. Cl < S < As < P D. P < S < Cl < As

29
Câu 1.70. Cho các hợp chất H2SO4, H3AsO4, HClO4, H3PO4. Sắp xếp các
hợp chất theo thứ tự tính axit giảm dần là:
A. H2SO4 > H3PO4 > HClO4 > H3AsO4
B. HClO4 > H2SO4 > H3PO4 > H3AsO4
C. H3AsO4 > H2SO4 > H3PO4 > HClO4
D. HClO4 > H2SO4 > H3AsO4 > H3PO4
---oOo---

ĐÁP ÁN BÀI TẬP CHƯƠNG 1


1.1- 1.2- 1.3- 1.4- 1.5- 1.6- 1.7- 1.8- 1.9- 1.10-
A C D A A D C C C B
1.11- 1.12- 1.13- 1.14- 1.15- 1.16- 1.17- 1.18- 1.19- 1.20-
A D B B C D B A A B
1.21- 1.22- 1.23- 1.24- 1.25- 1.26- 1.27- 1.28- 1.29- 1.30-
C B C C A D D D A B
1.31- 1.32- 1.33- 1.34- 1.35- 1.36- 1.37- 1.38- 1.39- 1.40-
B D D C B C B D B B
1.41- 1.42- 1.43- 1.44- 1.45- 1.46- 1.47- 1.48- 1.49- 1.50-
A D B A D B C C D B
1.51- 1.52- 1.53- 1.54- 1.55- 1.56- 1.57- 1.58- 1.59- 1.60-
C C B B D D A D A C
1.61- 1.62- 1.63- 1.64- 1.65- 1.66- 1.67- 1.68- 1.69- 1.70-
B A B C D A B A A B

30
Chương 2. LIÊN KẾT HÓA HỌC
---oOo---

Mục tiêu chương 2


Sau khi học xong chương này, sinh viên có khả năng:
1. Giải thích thích được bản chất của liên kết hóa học.
2. Định nghĩa được các đại lượng đặc trưng của liên kết hóa học.
3. Trình bày được cơ chế hình thành và tính chất cơ bản của các loại
liên kết hóa học.
4. Dự đoán được trạng thái lai hóa và cấu trúc hình học phân tử của
các phân tử cộng hóa trị.
5. Dự đoán được độ lớn góc hóa trị và mức độ phân cực của các phân
tử cộng hóa trị.
6. Giải thích được các tính chất hóa lý từ đặc trưng của liên kết trong
phân tử.

31
Phần 1. TÓM TẮT LÝ THUYẾT

2.1 BẢN CHẤT LIÊN KẾT HÓA HỌC


Liên kết hóa học có bản chất điện vì cơ sở tạo thành liên kết là lực tương
tác giữa các electron tích điện âm và hạt nhân tích điện dương.
Trong các tương tác hóa học chỉ có các electron hóa trị (là các electron ở
những lớp hoặc phân lớp ngoài cùng) tham gia thực hiện liên kết.
Các electron hóa trị nằm trong các AO hóa trị.
2.2 CÁC KHÁI NIỆM CƠ BẢN
2.2.1 Năng lượng liên kết là năng lượng cần tiêu tốn để phá hủy liên kết
thành các nguyên tử ở thể khí (hay năng lượng giải phóng ra khi tạo thành
liên kết từ các nguyên tử ở thể khí).
AB (k) + EA-B ↔ A (k) + B (k) → Năng lượng liên kết EA-B = E phân ly AB
2.2.2 Độ dài liên kết là khoảng cách giữa hai hạt nhân của các nguyên tử
tương tác với nhau.
2.2.3 Góc hóa trị là góc tạo thành bởi hai đoạn thẳng tưởng tượng nối hạt
nhân nguyên tử trung tâm với hai hạt nhân nguyên tử liên kết.

107o
Hình 2.1. Góc hóa trị một số phân tử

2.2.4 Bậc liên kết được tính bằng số liên kết giữa hai nguyên tử trong phân
tử. Bậc liên kết càng lớn thì liên kết càng bền và chiều dài liên kết càng
ngắn.
Ví dụ:
Liên kết Độ dài liên kết, dlk (pm) Năng lượng liên kết, ELK (kJ/mol)
C-C 154 346
C=C 134 610
CC 120 835
N-N 145 163
N=N 123 418
NN 110 945

32
2.3 LIÊN KẾT CỘNG HÓA TRỊ THEO THUYẾT LIÊN KẾT HÓA
TRỊ-VB
2.3.1 Các luận điểm cơ bản
• Một cặp nguyên tử trong phân tử được tạo thành từ một hoặc vài cặp
electron dùng chung.
• Bản chất của liên kết cộng hóa trị là sự che phủ lẫn nhau giữa các orbital
nguyên tử hóa trị của các nguyên tử tương tác trong đó có hai electron
có spin trái dấu.
• Liên kết cộng hóa trị càng bền khi mật độ xen phủ của các AO lớn, độ
xen phủ các AO phụ thuộc vào kích thước, hình dạng, hướng xen phủ
và sự chênh lệch năng lượng của các AO.
• Liên kết cộng hóa trị được biểu diễn bằng dấu hai chấm hay gạch nối.
2.3.2 Cơ chế tạo liên kết cộng hóa trị (CHT)
• Liên kết CHT theo cơ chế góp chung: cặp electron liên kết do cả hai
nguyên tử tương tác đóng góp, mỗi nguyên tử đóng góp một electron
hóa trị độc thân.
H H
H C H H C H
H H
Hình 2.2. Liên kết CHT trong phân tử CH4
• Liên kết CHT theo cơ chế cho nhận (liên kết phối trí): cặp electron liên
kết chỉ do một nguyên tử tương tác đóng góp, nguyên tử còn lại chỉ
nhận.
Điều kiện: nguyên tử cho phải có cặp electron hóa trị tự do, nguyên tử nhận
phải có AO hóa trị tự do.
H H
H N + H+ H N H+
H H
Hình 2.3. Liên kết CHT trong ion NH4+
2.3.3 Các kiểu liên kết cộng hóa trị
• Liên kết CHT kiểu  được tạo thành khi sự che phủ giữa các orbital
nguyên tử tương tác xảy ra theo trục nối hai hạt nhân nguyên tử.

33
S S P P p s
Hình 2.4. Sự che phủ trong liên kết CHT kiểu 
• Liên kết CHT kiểu  được tạo thành khi các orbital nguyên tử tương tác
che phủ với nhau về hai bên của trục nối hai hạt nhân.

truïcliên
trục lieâkết
n keát
truïcliên
trục lieânkếtkeát
pz dzx
lieânkết
liên keát 
lieânkết
liên keát 
Hình 2.5. Sự che phủ trong liên kết CHT kiểu 
• Liên kết  không định chỗ (liên kết  nhiều tâm): là loại liên kết  nhưng
được thực hiện từ ba nguyên tử trở lên với electron tham gia tạo liên kết
có thể nhiều hơn hai. Được dùng để giải thích bậc liên kết là số không
nguyên và được ký hiệu bằng các dấu gạch chấm (---).
O O- O- O 2-
C C C C
O- O- O O- O- O O O

Hình 2.6. Liên kết  không định chỗ trong ion CO32-
2.3.4 Tính chất của liên kết cộng hóa trị
a) Tính bão hòa: vì mỗi nguyên tố hóa học chỉ có một số giới hạn AO hóa
trị nên số liên kết cộng hóa trị có thể tạo được cũng có giới hạn. Số liên
kết cộng hóa trị tối đa của mỗi nguyên tố bằng số AO hóa trị của nguyên
tố đó.
b) Tính định hướng: muốn liên kết CHT bền vững thì mức độ che phủ
của các AO phải cực đại. Sự che phủ cực đại xảy ra theo những hướng nhất
định. Do đó liên kết CHT được tạo thành theo những hướng nhất định
trong không gian.
c) Tính phân cực: cặp electron trong liên kết CHT có thể bị lệch về nguyên
tử có độ âm điện lớn hơn dẫn đến sự phân cực trong phân tử CHT. Đám
mây electron bị lệch về phía nguyên tử có độ âm điện lớn hơn làm nguyên
tử này tích điện âm -, nguyên tử kia tích điện dương +.
34
• Nếu phân tử có hai nguyên tử: liên kết CHT có tính phân cực nếu hai
nguyên tử tương tác trong phân tử khác nhau và không phân cực nếu
hai nguyên tử tương tác trong phân tử giống nhau.
• Nếu phân tử có nhiều nguyên tử (lớn hơn hai), sự phân cực được quyết
định bởi moment lưỡng cực.
2.3.5 Lưỡng cực và moment lưỡng cực
Lưỡng cực điện: xem phân tử là một lưỡng điện cực trái dấu nhau (+, -)
cách nhau một khoảng cách L gọi là độ dài lưỡng cực.
Moment lưỡng cực: là đại lượng vectơ có phương dọc theo liên kết, chiều
hướng về nguyên tử âm điện hơn và độ lớn tính bằng công thức (2.1).
 = L.q = L.e. (2.1)
Moment lưỡng cực của phân tử là tổng vectơ moment lưỡng cực của các
liên kết và e hóa trị tự do.
Tính có cực của các phân tử CHT ảnh hưởng đến khả năng hòa tan và hóa
lỏng của chúng: các chất có cực thì tan tốt trong dung môi có cực, các chất
không cực tan tốt trong dung môi không phân cực. Các chất có cực dễ hóa
lỏng hơn các chất không cực.
Ví dụ,
✓ H2, O2, N2 là phân tử gồm hai nguyên tử giống nhau nên là phân tử CHT
không cực.
✓ CO2, BeCl2, CH4, CCl4, C6H6 là các phân tử CHT không cực vì là các
phân tử có tính đối xứng nên  ⃗ = ⃗0.
✓ NO2, SO2, NH3, H2O là các phân tử bất đối xứng nên có 
⃗ ≠ ⃗0 và là các
phân tử có cực.
2.3.6 Thuyết lai hóa
Trong nhiều trường hợp, các nguyên tử trung tâm không sử dụng các AO
hóa trị thuần túy s, p, d, f để tạo liên kết. Trước khi tạo liên kết, chúng trộn
lẫn các AO hóa trị trong nội bộ nguyên tử lại hình thành các AO mới rồi
mới dùng các AO mới này tạo liên kết cộng hóa trị với các nguyên tử khác.
Hiện tượng này gọi là sự lai hóa các AO hóa trị, các AO thu được sau khi
lai hóa gọi là AO lai hóa (hybrid orbital).
• Đặc điểm của sự lai hóa:
- Sự lai hóa chỉ xảy ra trong nội bộ một nguyên tử.
- Số AO lai hóa thu được = số AO tham gia lai hóa.

35
- Các AO lai hóa có năng lượng và hình dạng hoàn toàn giống nhau và
phân bố một cách đối xứng nhất trong không gian.
- Các AO lai hóa chỉ có thể tạo được loại liên kết cộng hóa trị duy nhất là
liên kết sigma (σ).
• Điều kiện để lai hóa bền:
Các AO tham gia lai hóa phải có năng lượng gần bằng nhau, mức độ che
phủ các AO lớn, mật độ electron lớn.
• Các kiểu lai hóa cơ bản
- Lai hóa sp: xảy ra do sự tổ hợp của một AO s và một AO p để hình thành
hai AO lai hóa sp, phân bố đối xứng dưới một góc 180o.

1AO s + 1 AO p 2 AO lai hóa sp


Hình 2.7. Lai hóa sp
Ví dụ, phân tử BeH2.
Be là nguyên tử trung tâm ở điều kiện bình thường không có electron độc
thân. Do đó, Be phải nhận năng lượng chuyển thành trạng thái kích thích
Be* và có 2 electron độc thân trước khi thực hiện lai hóa.

Trước khi thực hiện liên kết, trong nguyên tử Be xảy ra sự lai hóa sp giữa
một AO 1s và 1 AO 2p để tạo hai AO lại hóa sp. Hai AO lai hóa sp của Be
sẽ xen phủ với hai AO hóa trị chứa một e độc thân của H để tạo thành hai
̂ = 180o.
liên kết Be-H với góc liên kết 𝐻𝐵𝑒𝐻

Trạng thái lai Sự xen phủ các orbital Phân tử


hóa sp của Be tạo liên kết Be-H BeH2
Hình 2.9. Lai hóa trong phân tử BeH2
- Lai hóa sp : xảy ra do sự tổ hợp một AO s và hai AO p tạo thành ba AO
2

lai hóa sp2, phân bố đối xứng hướng đến ba đỉnh của một tam giác đều tạo
thành góc 120o.

36
Ví dụ: phân tử BCl3
B sử dụng một orbital 2s và hai orbital
2p tạo thành ba orbital lai hóa sp2. Sự
xen phủ ba orbital lai hóa này với ba
orbital 3p chứa một e hóa trị độc thân Hình 2.10. Lai hóa sp2
của ba nguyên tử Cl tạo ba liên kết
B-Cl, tương đương với góc hoá trị 120o.

3p
Cl
2
sp
o
120
B

Cl Cl
sp 2
3p
3p
Trạng thái lai Sự xen phủ các orbital tạo liên Phân tử BCl3
hóa sp2 của B kết B-Cl
Hình 2.11. Lai hóa trong phân tử BCl3
- Lai hóa sp : xảy ra do sự tổ hợp một AO s và ba AO p tạo thành bốn AO
3

lai hóa sp3 phân bố đối xứng trong không gian hướng đến bốn đỉnh của
một tứ diện đều tạo thành góc 109o28’.
3
sp
p
p
p
sp3
s
sp 3
sp 3
109 o28'
Hình 2.12. Lai hóa sp3
37
Ví dụ: phân tử CCl4

C sử dụng một orbital 2s và ba orbital 2p tạo thành bốn orbital lai hóa sp3.
Sự xen phủ bốn orbital lai hóa này với bốn obitan 3p chứa một e hóa trị
độc thân của bốn nguyên tử Clo tạo bốn liên kết C-Cl, tương đương với
góc hoá trị 109o28’.

Cl

C
Cl

Cl Cl

Sự xen phủ các orbital tạo Phân tử


Trạng thái lai hóa
liên kết C-Cl CCl4
sp3 của C

Hình 2.11. Lai hóa trong phân tử CCl4


2.3.7 Thuyết đẩy nhau giữa các cặp electron hóa trị (VSEPR)
Sự sai lệch góc hóa trị xảy ra khi nguyên tử trung tâm còn các cặp electron
hóa trị tự do còn gọi là cặp electron không liên kết (KLK).
Ví dụ, so sánh góc hóa trị giữa các phân tử sau: CH4, NH3, H2O?
Giải
Góc hóa trị CH4 = 109,28o > NH3 = 107,3o > H2O = 104,5o
Do CH4, NH3, H2O đều có nguyên tử trung tâm lại hóa sp3, CH4 không có
cặp e hóa trị tự do, NH3 có một cặp e hóa trị tự do, H2O có hai cặp e hóa
trị tự do. Độ mạnh giữa các đôi e: KLK-KLK > KLK-LK > LK-LK nên
góc hóa trị CH4 > NH3 > H2O.

38
2.3.8 Dự đoán trạng thái lai hóa của nguyên tử trung tâm và cấu hình
phân tử
• Dự đoán trạng thái lai hóa của nguyên tử trung tâm trong phân tử ABn
Cách 1: Góc liên kết thực nghiệm có giá trị gần với góc của kiểu lai hóa
nào thì nguyên tử trung tâm sẽ lai hóa kiểu đó.
Ví dụ: CO2 (OCO = 180o) → C lai hóa sp.
SO2 (OSO = 119,5o) → S lai hóa sp2.
NH3 (HNH = 107,3o) → N lai hóa sp3.
Cách 2: Tính số đôi e hóa trị theo công thức (2.2):
𝑋−𝑌
𝑇 = 𝜎+
2
Với, : số nguyên tử biên (hoặc số liên kết ) trong phân tử.
X: tổng số electron hóa trị của các nguyên tử trong phân tử.
Y: tổng số electron cần có để bão hòa các nguyên tử biên.
Nếu T = 2, lai hóa sp; T = 3, lai hóa sp2; T = 4, lai hóa sp3.
Sự liên hệ giữa số cặp e hóa trị liên kết, số cặp e hóa trị tự do, kiểu lai
hóa và cấu trúc hình học phân tử được liệt kê trong Bảng 2.1.
Bảng 2.1. Các kiểu lai hóa của các phân tử CHT
Kiểu lai hóa
Số cặp e Số cặp Cấu trúc
nguyên tử Phân tử, ion
hóa trị e tự do hình học
trung tâm
2 0 sp Thẳng hàng CO2, BeCl2,
HCN
3 0 sp 2
Tam giác đều BF3, NO3-, CO32-
3 1 sp2 Dạng góc SO2, NOCl, O3
4 0 sp 3
Tứ diện đều CH4, CCl4, SO42-,
NH4+
4 1 sp3 Tháp đáy tam NH3, NF3
giác
4 2 sp3 Dạng góc H2O, H2S

2.4. LIÊN KẾT ION


2.4.1 Bản chất của liên kết ion: là lực hút tĩnh điện giữa các ion mang
điện tích trái dấu.
Sự hình thành liên kết ion xảy ra gồm hai giai đoạn:
- Các nguyên tử (g) trao đổi e cho nhau tạo thành các ion (g).
- Các ion (g) trái dấu hút nhau bằng lực hút tĩnh điện tạo tinh thể ion (s).
Độ mạnh của liên kết ion phụ thuộc vào những yếu tố sau:
39
• Điện tích ion: điện tích ion càng lớn thì liên kết ion càng mạnh.
• Kích thước ion: bán kính ion càng nhỏ thì liên kết ion càng mạnh.
• Độ chênh lệch độ âm điện giữa các nguyên tử tham gia tạo liên kết càng
lớn thì liên kết tạo thành có độ ion càng lớn.
2.4.2 Các tính chất của liên kết ion
• Không bão hòa
• Không định hướng
• Phân cực mạnh
2.4.3 Sự phân cực ion: là sự dịch chuyển đám mây e đối với hạt nhân của
một ion dưới tác dụng của điện trường một ion khác. Ion có đám mây biến
dạng gọi là ion bị phân cực, còn ion có điện trường tác dụng gọi là ion
phân cực (cation gây phân cực, anion bị phân cực).

+ -

Hình 2.12. Sự phân cực ion


Các yếu tố ảnh hưởng đến độ phân cực và độ bị phân cực của ion: điện
tích, kích thước, cấu hình electron.
✓ Khi điện tích và cấu hình e như nhau thì độ phân cực tăng theo chiều
giảm kích thước ion, độ bị phân cực tăng theo chiều tăng kích thước
ion.
✓ Khi điện tích tăng độ phân cực của ion tăng lên.
✓ Khi điện tích như nhau và kích thước gần bằng nhau thì độ bị phân cực
nhỏ nhất đối với ion có cấu hình khí trơ s2p6, lớn nhất đối với các ion
có cấu hình ns2np6(n-1)d10.
Ảnh hưởng của sự phân cực ion đến tính chất các hợp chất ion:
• Sự phân cực trong liên kết ion làm tăng tính cộng hóa trị, tính ion giảm
dẫn đến độ điện ly giảm.
• Sự phân cực trong liên kết ion làm tăng tính cộng hóa trị, tính ion giảm dẫn
đến điện tích hiệu dụng ion giảm → năng lượng mạng tinh thể giảm → độ
bền tinh thể ion giảm → nhiệt độ nóng chảy, nhiệt độ phân ly giảm.

40
2.5. LIÊN KẾT KIM LOẠI
2.5.1 Đặc điểm cấu tạo kim loại: Mạng tinh thể của kim loại được tạo
thành từ những ion dương ở nút mạng và các electron tự do (là các electron
hóa trị bị bứt ra khỏi nguyên tử) chuyển động hỗn loạn trong toàn bộ tinh
thể kim loại. Chính các electron tự do này tạo nên kiểu liên kết đặc biệt
của kim loại: liên kết nhiều tâm.

Hình 2.13. Mạng tinh thể ion


2.5.2 Thuyết miền năng lượng về cấu tạo kim loại
Giải thích liên kết kim loại theo thuyết miền năng lượng: coi mỗi tinh thể
kim loại là một đại phân tử có khoảng 1023 tâm, mỗi tinh thể gồm nAO
hóa trị của một số lượng rất lớn các nguyên tử kim loại. Sự tổ hợp của nAO
thành nMO (với (n/2) MOlk và (n/2) MOplk) có các mức năng lượng khác
nhau. Do n rất lớn, các MO lân cận nhau chỉ có sự chênh lệch năng lượng
không đáng kể. Tập hợp các MOlk và tập hợp các MOplk tạo thành các miền
năng lượng.
Miền năng lượng chứa các electron hóa trị gọi là miền hóa trị.
Miền năng lượng không chứa các electron, nằm trên miền hóa trị gọi là
miền dẫn.
Nếu miền hóa trị và miền dẫn không che phủ nhau, khoảng cách giữa hai
miền gọi là miền cấm.

Năng lượng Fermi

Hình 2.14. Chất dẫn điện – chất bán dẫn – chất cách điện

41
2.5.3 Các tính chất của kim loại
- Ánh kim;
- Không trong suốt;
- Dẫn điện, dẫn nhiệt tốt;
- Có tính dẻo.
2.6 CÁC LOẠI LIÊN KẾT LIÊN PHÂN TỬ
2.6.1 Liên kết Hydro
Liên kết hydro là liên kết đặc biệt giữa H linh động liên kết với các nguồn
dư điện tử khác của phân tử khác (liên kết H liên phân tử) hay nguyên tử
khác trong chính phân tử đó (liên kết H nội phân tử).
Khi nguyên tử H liên kết với các nguyên tử có độ âm điện lớn như F, O, N
thì các cặp e liên kết bị lệch mạnh về phía F, O, N → H tích điện dương
(H+) gọi là H linh động.
Các nguyên tử của các nguyên tố có độ âm điện lớn, kích thước nhỏ (mật
độ điện tích âm lớn) như N, O, F,… hay các nguồn e (liên kết bội, nhân
thơm…) hoặc các cặp e không liên kết trên các nguyên tử được gọi là các
nguồn giàu điện tử → có thể xem chúng tích điện âm (X-).

Hình 2.15. Một vài liên kết Hydro


• Đặc điểm: Liên kết hydro có độ mạnh trung gian giữa liên kết cộng hóa
trị và liên kết Van der Waals.
• Ảnh hưởng của liên kết hydro đến tính chất của các chất:
- Liên kết hydro làm tăng nhiệt độ sôi và nhiệt độ nóng chảy của chất.
- Giảm độ axit của dung dịch.
- Tăng độ tan trong dung môi.
Ví dụ: Nhiệt độ sôi của H2O cao hơn H2S do H2O có liên kết hydro liên
phân tử còn H2S không có liên kết hydro. CH3OH tan vô hạn trong nước
là do tạo được liên kết hydro với nước.
2.6.2 Liên kết Van der Waals (VDW)
Liên kết VDW là loại liên kết xuất hiện giữa các phân tử hoặc các nguyên
tử đã bão hòa điện tử hóa trị.
Liên kết VDW có thể xuất hiện ở những khoảng cách tương đối lớn, có
năng lượng nhỏ, có tính không chọn lọc và không bão hòa, có tính cộng.
42
• Thành phần: Liên kết Van der Waals xuất hiện do ba loại tương tác:
định hướng, cảm ứng, khuếch tán.
- Tương tác định hướng: xuất hiện giữa các phân tử có cực → tương
tác lưỡng cực - lưỡng cực. Tương tác định hướng ↑ khi moment
lưỡng cực của phân tử ↑ và To↓
- Tương tác cảm ứng: xuất hiện giữa các phân tử có cực và không cực
→ tương tác lưỡng cực – lưỡng cực cảm ứng. Tương tác này chỉ
đáng kể khi moment lưỡng cực của phân tử có cực lớn.
- Tương tác khuếch tán: xuất hiện là nhờ lưỡng cực nhất thời của các
phân tử không cực → tương tác lưỡng cực nhất thời - lưỡng cực nhất
thời ↑ khi moment lưỡng cực ↓ và khối lượng phân tử ↑.
• Ảnh hưởng của liên kết VDW đến tính chất của các chất:
- Phân tử có cực càng lớn, khối lượng phân tử, kích thước của phân tử
càng lớn thì liên kết VDW càng lớn, chất càng dễ hóa lỏng, trạng
thái tập hợp phân tử có mật độ phân tử càng cao, nhiệt độ nóng chảy,
nhiệt độ sôi của phân tử càng lớn.

43
Phần 2. BÀI TẬP CÓ LỜI GIẢI
---oOo---
Dạng 1. Liên kết cộng hóa trị
Câu 2.1 Dự đoán trạng thái lai hóa của nguyên tử trung tâm (được gạch
gạch chân) và cấu trúc hình học của các chất sau:
a) NO3-
b) NOCl,
c) CO2,
d) HCN
Giải
a) NO3-
- =3
- X = 5 + 6.3 + 1 = 24
- Y = 8.3 = 24
- T=3
→ N lai hóa sp2, phân tử NO3- dạng tam giác đều.
b) NOCl
- =2
- X = 5 + 6 + 7 = 18
- Y = 8.1 + 8.1 = 16
- T=3
→ N lai hóa sp2, phân tử NOCl dạng góc.
c) CO2
- =2
- X = 4 + 6.2 = 16
- Y = 8.2 = 16
- T=2
→ C lai hóa sp, phân tử CO2 dạng thẳng hàng.
d) HCN
- =2
- X = 1 + 4 + 5 = 10
- Y = 2.1 + 8.1 = 10
- T=2
→ C lai hóa sp, phân tử HCN dạng thẳng hàng.
Câu 2.2. Xác định trạng thái lai hóa của nguyên tử trung tâm, hình thành
liên kết và cơ cấu hình học của phân tử sau:
44
a) CH4
b) SO2
Giải
a) Xét phân tử CH4
C ở trạng thái kích thích có 4 e hóa trị độc thân: 1s2 2s1 2px1 2py1 2pz1

Tổng số đôi electron hóa trị = 4 → C ở trạng thái lai hóa sp3.
Liên kết trong phân tử CH4 được hình thành như sau: C dùng một AO và
ba AO 2p để tạo bốn AO lai hóa sp3 hướng đến bốn đỉnh của tứ diện đều
(bốn AO lai hóa này đều chứa một e hóa trị độc thân). Bốn AO lai hóa của
C sẽ che phủ cùng trục với bốn AO 1s của bốn H tạo bốn liên kết , góc
hoá trị 109o 28’.
Phân tử CH4 có dạng tứ diện đều:

b) Xét phân tử SO2

O(1) S O(2)
O(1)

O(2)

Tổng số đôi e hóa trị = 3 → S lai hóa sp2.


S sử dụng một AO 3s và hai AO 3px, 3py tạo ba AO lai hóa sp2, hai AO lai
hóa chứa mỗi AO một cặp electron ghép đôi và một AO lai hóa còn lại
chứa một electron độc thân).
Liên kết trong phân tử SO2 được hình thành như sau: Một AO lai hóa chứa
cặp electron ghép đôi của S sẽ xen phủ cùng trục với AO 2pz của O(2). Một
AO lai hóa chứa e độc thân của S sẽ xen phủ cùng trục với AO 2py của O(1)
(tạo liên kết ) và S sử dụng AO 3pz không lai hóa còn lại chứa một

45
electron độc thân xen phủ bên với AO 2pz chứa e độc thân của O(1). Sau
khi hình thành liên kết S còn một cặp electron không liên kết.
Phân tử SO2 có dạng góc, góc hóa trị OSO < 120o vì S còn đôi e KLK sẽ
đẩy mạnh đôi e LK, làm cho góc hóa trị hẹp lại.

Câu 2.3. Cho các liên kết B-Cl, Na-Cl, Ca-Cl, Be-Cl. Hãy sắp xếp các liên
kết này theo trật tự mức độ phân cực tăng dần.
Giải
Với Cl = 3,16, xét độ âm điện của các nguyên tử còn lại và chênh lệch độ
âm điện của các liên kết:
Nguyên tử Độ âm điện 
B 2,04 1,12
Be 1,57 1,59
Ca 1,00 2,16
Na 0,93 2,23
Do đó, độ phân cực của các liên kết tăng dần theo dãy:
B-Cl < Be-Cl < Ca-Cl < Na-Cl
Câu 2.4. Sắp xếp các phân tử sau theo chiều tăng dần của moment lưỡng
cực: BH3, H2S, H2O.
Giải
Phân tử BH3 có kiểu lai hóa sp2, cấu trúc tam giác phẳng, các moment
lưỡng cực của từng liên kết lần lượt triệt tiêu nhau nên có moment lưỡng
cực bằng 0.
H2S và H2O có cùng kiểu lai hóa sp3, cấu trúc góc. Ngoài ra, độ âm điện
của O lớn hơn S nên liên kết H-O phân cực nhiều hơn liên kết H-S. Do đó,
moment tổng trong phân tử H2O lớn hơn H2S.
Như vậy, các phân tử xếp theo chiều tăng dần của moment lưỡng cực là:
BH3 < H2S < H2O.
Câu 2.5. Hãy xác định số electron độc thân có thể có của các nguyên tử
sau:
a) O (Z = 8)
b) F (Z = 9)
c) S (Z = 16)
d) Cl (Z = 17)
46
Giải
a) Cấu hình electron của Oxy là: 1s2 2s2 2p4 nên Oxy chỉ có thể có hai
electron độc thân.
d) Cấu hình electron của Flo là: 1s2 2s2 2p5 nên Flo chỉ có thể có một
electron độc thân.
c) Cấu hình electron của Lưu huỳnh ở trạng thái cơ bản và kích thích như
sau:
• 1s2 2s2 2p6 3s2 3p4
• 1s2 2s2 2p6 3s2 3p3 3d1
• 1s2 2s2 2p6 3s1 3p3 3d2
Nên Lưu huỳnh có thể có 2, 4 hoặc 6 electron độc thân.
d) Cấu hình electron của Clo ở trạng thái cơ bản và kích thích như sau:
• 1s2 2s2 2p6 3s2 3p5
• 1s2 2s2 2p6 3s2 3p4 3d1
• 1s2 2s2 2p6 3s2 3p3 3d2
• 1s2 2s2 2p6 3s1 3p3 3d3
Nên Clo có thể có 1, 3, 5 hoặc 7 electron độc thân.
Dạng 2. Liên kết ion
Câu 2.6. Giải thích vì sao nhiệt độ nóng chảy giảm từ LiF đến LiI?
Chất LiF LiCl LiBr LiI

Tnc (oC) 848 607 550 469


Giải
Từ LiF đến LiI: Bán kính ion X-↑→ Độ bị phân cực của các ion X-↑ →
tính CHT ↑ → tính ion ↓ → Tnc ↓.
Câu 2.7. Cho khoảng cách ngắn nhất (pm) giữa các ion trong các phân tử
NaF, MgO, CaO, SrO và BaO lần lượt là 231; 210,6; 240,5; 258; và 176,2.
Hãy sắp xếp các chất trên theo trật tự tăng dần của độ cứng và nhiệt độ
nóng chảy.
Giải
Khoảng cách giữa các ion càng nhỏ, thì độ cứng và nhiệt độ nóng chảy
càng tăng.
Vì NaF có Z = 1 nên có độ cứng và nhiệt độ nóng chảy nhỏ nhất.

47
Các phân tử còn lại có Z = 2, khi khoảng cách giữa các ion càng nhỏ thì
độ cứng và nhiệt độ nóng chảy càng cao.
Do đó, trật tự tăng dần của độ cứng và nhiệt độ nóng chảy như sau:
NaF < BaO < SrO < CaO < MgO
Dạng 3. Các liên kết liên phân tử
Câu 2.8.
a) Giải thích tại sao SO2 dễ hóa lỏng hơn CO2?
b) Giải thích tại sao ở trạng thái bình thường, các nguyên tố halogen có
trạng thái vật chất lần lượt là F2(k), Cl2(k), Br2(ℓ), I2(r)?
Giải
a) Xét SO2 dễ hóa lỏng hơn CO2 vì: SO2 là phân tử cộng hóa trị có cực,
CO2 là phân tử cộng hóa trị không cực.
b) Xét trạng thái của các chất F2(k), Cl2(k), Br2(ℓ), I2(r) vì: từ F2 đến I2
khối lượng phân tử tăng dần, liên kết VDW tăng dần nên trạng thái tập hợp
phân tử có mật độ phân tử tăng từ F2 đến I2 cho nên F2, Cl2 là chất khí, Br2
là chất lỏng, I2 là chất rắn.
Câu 2.9. So sánh nhiệt độ sôi của:
a) HF, HCl, HBr, HI
b) CH3COOH và HCOOCH3
c) CH3F và CH3I
Giải
a) Nhiệt độ sôi: HF > HI > HBr > HCl do:
HF có liên kết hydro nên nhiệt độ sôi cao nhất.
Khối lượng phân tử từ HCl đến HI tăng nên lực Van der Waals tăng, dẫn
đến nhiệt độ sôi tăng từ HCl đến HI.
b) Nhiệt độ sôi của CH3COOH > HCOOCH3 vì CH3COOH có liên kết
hydro.
c) Nhiệt độ sôi của CH3I > CH3F vì khối lượng phân tử CH3I > CH3F nên
lực VDW của CH3I lớn hơn CH3F.
Câu 2.10. Chất nào có thể tạo được liên kết Hydro trong dãy các chất sau:
H2S, C6H6, C2H5OH, HCOOH, CH3-O-CH3, CH3-CH2-OH.
Giải
Chất có thể tạo được liên kết Hydro bao gồm: C2H5OH, HCOOH,
CH3-CH2-OH.

48
Phần 3. BÀI TẬP TRẮC NGHIỆM
---oOo---
Câu 2.1. Chọn đáp án đúng: liên kết ba giữa hai nguyên tử cacbon trong
phân tử axetilen (HC ≡ HC) gồm:
A. Một liên kết π, hai liên kết σ
B. Một liên kết σ, hai liên kết π
C. Cả ba liên kết đều là liên kết σ
D. Cả ba liên kết đều là liên kết π
Câu 2.2. Chọn đáp án đúng: Trạng thái lai hóa của nguyên tử trung tâm C,
N và O trong các phân tử CH4, NH3 và H2O:
A. Đều có lai hóa sp3
B. C có lai hóa sp3, N có lai hóa sp2, O có lai hóa sp
C. Tất cả đều lai hóa sp2
D. Tất cả đều lai hóa sp
Câu 2.3. Góc liên kết trong các phân tử nào là lớn nhất trong số các phân
tử CH4, NH3, H2O và H2S?
̂
A. 𝐻𝐶𝐻 ̂
B. 𝐻𝑁𝐻
̂
C. 𝐻𝑂𝐻 ̂
D. 𝐻𝑆𝐻
Câu 2.4. Phân tử nào sau đây có cấu trúc thẳng hàng?
A. HOCl B. CO2
C. SO2 D. O3
Câu 2.5. Phân tử nào sau đây có cấu trúc góc?
A. HCN B. NOCl
C. BF3 D. CCl4
Câu 2.6. Phân tử nào sau đây có cấu trúc tam giác đều?
A. H3O+ B. PF3
-
C. BF4 D. SO3
Câu 2.7. Trong số các phân tử sau, phân tử nào có moment lưỡng cực bằng
không?
A. OF2 B. SF2
C. CS2 D. H2S
Câu 2.8. Cách sắp xếp nào dưới đây đúng khi so sánh nhiệt độ sôi của HF,
HCl, HBr, HI:
A. HF < HCl < HBr < HI B. HF > HI > HBr > HCl
C. HF > HCl > HBr > HI D. HF < HI < HBr < HCl
Câu 2.9. Trong phân tử C2H4 có mấy liên kết cộng hóa trị ?
49
A. 2 B. 3 C. 4 D. 5
Câu 2.10. Độ ion của liên kết trong phân tử nào sau đây là lớn nhất?
A. NaF B. CaCl2
C. CaBr2 D. NaCl
Câu 2.11. Bản chất của liên kết ion là:
A. Sự dùng chung cặp electron hóa trị
B. Lực hút tĩnh điện giữa các ion trái dấu
C. Sự di chuyển electron từ nguyên tử này đến nguyên tử khác
D. Sự xen phủ các orbital nguyên tử hóa trị
Câu 2.12. Chọn phương án đúng: Xác định trạng thái lai hóa của các
nguyên tử C trong phân tử sau (từ trái qua phải): CH3─CH2─C≡CH.
A. sp3, sp, sp2, sp3 B. sp, sp2, sp3, sp
C. sp3, sp3, sp, sp D. sp3, sp2, sp, sp2
Câu 2.13. Trong các chất dưới đây, chất nào có cấu trúc thẳng hàng:
A. BeF2 B. NH2-
C. SCl2 D. ClO2-
Câu 2.14. Có các phát biểu sau đây về phân tử CF4 và NF3. Có tổng cộng
bao nhiêu phát biểu đúng?
(i) Góc hóa trị FCF trong phân tử CF4 nhỏ hơn góc hóa trị FNF trong phân
tử NF3.
(ii) CF4 có dạng hình học là tứ diện đều, không cực.
(iii) NF3 có dạng hình học là tam giác phẳng, không cực.
A. 1 B. 12 C. 13 D. 0
Câu 2.15. Chọn phương án đúng.
Theo thuyết VB, ion H3O+ có đặc điểm:
(1) Dạng hình học phân tử là tháp tam giác đều, phân tử có cực.
(2) Oxy ở trạng thái lai hóa sp3, góc HÔH < 109,5o.
(3) Liên kết giữa O và H đều theo cơ chế ghép đôi.
(4) Dạng hình học phân tử là tam giác đều, phân tử không cực.
A. 1,2 đúng B. 2,4 đúng
C. 4,5 đúng D. 3 đúng
Câu 2.16. Liên kết hoá học trong phân tử HCl là:
A. Liên kết ion.
B. Liên kết cộng hoá trị phân cực
C. Liên kết cho - nhận.
D. Liên kết cộng hoá trị không phân cực.

50
Câu 2.17. Chọn phương án đúng. Hãy sắp xếp các ion sau đây theo chiều
tăng dần độ bị phân cực của chúng: 9F- 17Cl
-
34Se
2-
16S
2-

A. F- < Cl- < S2- < Se2- B. Cl- < F- < Se2- < S2-
C. S2- < Se2- < F- < Cl- D. F- < S2- < Cl- < Se2-
Câu 2.18. Chọn phát biểu đúng về nhiệt độ sôi của các chất sau:
(1) HF < HCl < HBr < HI vì khối lượng tăng dần.
(2) I2 < Br2 < Cl2 < F2 vì độ dài liên kết CHT giảm nên năng lượng liên
kết CHT tăng.
(3) Tos (CH3CH2OH) = Tos (CH3OCH3) vì hai phân tử có khối lượng
phân tử bằng nhau.
(4) H2S < H2Se < H2 Te < H2O
A. 4 đúng B. 1, 3 đúng
C. 2, 3 đúng D. 1, 4 đúng
Câu 2.19. Loại liên kết nào chủ yếu trong hợp chất CH3OH:
A. Liên kết ion B. Liên kết cộng hóa trị
C. Liên kết hydro D. Liên kết kim loại
Câu 2.20. Chọn phương án sai:
A. Ion CO32- có dạng tam giác đều, bậc liên kết 1,33; có 1 liên kết  không
định chỗ.
B. Phân tử ClO3- có dạng tam giác đều, bậc liên kết 1; có liên kết .
C. Ion SOCl2 có dạng tháp tam giác, bậc liên kết nguyên; có 1 liên kết .
D. Ion H3O+ có dạng tháp tam giác, bậc liên kết 1; không có liên kết .
Câu 2.21. Chọn phát biểu đúng về thuyết lai hóa của Pauling trong liên
kết cộng hóa trị:
(1) Theo thuyết lai hóa, mỗi nguyên tố hóa học chỉ có thể lai hóa theo một
kiểu xác định.
(2) Các hợp chất H2X đều có trạng thái lai hóa bền sp3 như nhau ở mọi
nguyên tử trung tâm X thuộc nhóm VIA.
(3) Trong các hydro cacbon no, tất cả các nguyên tử cacbon đều có lai hóa
sp3.
(4) Các AO lai hóa có khả năng tham gia tạo liên kết .
A. 3 đúng B. 1, 2, 3 đúng
C. 2, 4 đúng D. 1 đúng
Câu 2.22. Cho phân tử NF3 như hình bên. Có tổng cộng bao nhiêu phát
biểu ĐÚNG về NF3 trong các phát biểu sau đây? (ZN = 7, ZF = 9)
(i) Có kiểu lai hóa sp3
51
(ii) Có moment lưỡng cực bằng không
(iii) Có bậc liên kết bằng 3
(iv) Cấu trúc tứ diện đều
A. 0 B. 2 C. 1 D. 3
Câu 2.23. Toluene có công thức cấu tạo như sau:

Số liên kết  trong phân tử Toluene là bao nhiêu?


A. 7 B. 15 C. 13 D. 10
Câu 2.24. Liên kết chủ yếu có mặt trong hợp chất KBr là:
A. Liên kết ion B. Liên kết kim loại
C. Liên kết hydro D. Liên kết cộng hóa trị
Câu 2.25. Phát biểu nào sau đây là ĐÚNG?
A. Liên kết cộng hóa trị có tính chất không bão hòa và không định hướng
B. Liên kết Van der Waals là liên kết nội phân tử
C. Liên kết hóa học có bản chất điện
D. Liên kết cộng hóa trị và liên kết ion là các loại liên kết yếu
Câu 2.26. Phát biểu nào sau đây về phân tử NH3 là ĐÚNG: (ZN = 7, ZH
= 1)
A. Lai hóa sp3, cấu trúc tam giác phẳng, không phân cực
B. Lai hóa sp3, cấu trúc tháp, phân cực
C. Lai hóa sp2, cấu trúc tam giác phẳng, phân cực
D. Lai hóa sp2, cấu trúc tháp, phân cực
Câu 2.27. Phát biểu nào sau đây là SAI:
A. Phân tử NH3 có bậc liên kết bằng 3
B. Sự định hướng của liên kết cộng hóa trị được quyết định bởi sự lai hóa
của nguyên tử trung tâm tham gia tạo liên kết
C. Khi tạo liên kết thì liên kết σ được hình thành đầu tiên và có vai trò định
hướng phân tử
D. Liên kết đôi kém bền hơn liên kết ba
Câu 2.28. Photphin PH3 là sản phẩm sinh ra từ sự thủy phân của một loại
thuốc diệt chuột. Khi bị nhiễm độc PH3 con người có khả năng tử vong với
các triệu chứng khó hô hấp, đau đầu, chóng mặt, buồn nôn. Hình ảnh nào
sau đây là đúng với cấu tạo của Photphin? (ZP = 15, ZH = 1).
52
A. B.

C. D.
Câu 2.29. Đám mây electron sau đây có mấy cách định hướng trong không
gian?

A. 7 B. 1 C. 3 D. 5
Câu 2.30. Sắp xếp các phân tử sau đây theo chiều tăng dần độ lớn góc hóa
trị: CO2, SO2, NH3, H2O. Cho biết: ZH = 1, ZC = 6, ZN = 7, ZO = 8, ZS =
16.
A. H2O < NH3 < CO2 < SO2 B. NH3 < SO2 < CO2 < H2O
C. H2O < NH3 < SO2 < CO2 D. CO2 < H2O < NH3 < SO2
Câu 2.31. Cho các phát biểu sau:
(a) Liên kết ion luôn là một liên kết phân cực
(b) Liên kết Hydro là một liên kết liên phân tử
(c) Liên kết Hydro làm nước có nhiệt độ sôi cao bất thường so với CH4 (có
khối lượng phân tử gần tương đương)
(d) orbital p che phủ với orbital p luôn tạo thành liên kết cộng hóa trị kiểu
π
(e) liên kết đôi có bậc liên kết bằng 2
Số các phát biểu đúng là:
A. 3 B. 5 C. 2 D. 4
Câu 2.32. Có các phát biểu sau đây về liên kết hóa học:
(a) Liên kết hóa học có bản chất nhiệt
(b) Liên kết π không định chỗ là liên kết hai electron nhiều tâm
(c) Liên kết π không định chỗ có bậc liên kết không là số nguyên
(d) Bậc liên kết càng tăng thì năng lượng liên kết càng tăng

53
(e) Liên kết ion không có tính định hướng và không có tính bão hòa
Có bao nhiêu phát biểu là SAI:
A. 2 B. 3 C. 4 D. 1
Câu 2.33. Cho các nguyên tố với độ âm điện tương ứng như sau: H (ϰ =
2,1); C (ϰ = 2,5); N (ϰ = 3,0) và O (ϰ = 3,5). Trong 4 liên kết cộng hóa trị
sau, liên kết nào bị phân cực ít nhất?
A. N-H B. C-H
C. C-O D. O-H
Câu 2.34. Phát biểu nào sau đây là đúng:
A. Nguyên tử N có 5 electron hóa trị và có thể tạo được 3 liên kết cộng
hóa trị theo cơ chế góp chung
B. Các nguyên tố ở chu kỳ 4 là các nguyên tố chuyển tiếp
C. Các nguyên tố Halogen có thể có 1, 3, 5, 7 electron độc thân
D. Cr (Z = 24) có 4 electron độc thân ở trạng thái bình thường
Câu 2.35. Cho các yếu tố sau:
(1) Độ âm điện của các nguyên tử trong phân tử
(2) Sự lai hóa của nguyên tử trung tâm
(3) Cặp electron hóa trị tự do trong phân tử
(4) Độ dài liên kết
Các yếu tố có ảnh hưởng đến độ lớn góc hóa trị của phân tử là:
A. (1, 4) B. (1, 2)
C. (3, 4) D. (2, 3)
Câu 2.36. Cho các phân tử sau đây: CO2, H2, CH4, NH3, H2O. Số lượng
các phân tử có moment lưỡng cực KHÁC KHÔNG là: (ZF = 9), (ZC = 6),
(ZO = 8), (ZN = 7) và (ZH = 1).
A. 4 B. 2 C. 3 D. 1
Câu 2.37. Số cặp electron hóa trị liên kết và không liên kết của nguyên tử
trung tâm trong phân tử H2O lần lượt là: (Cho biết ZO = 8, ZH = 1)
A. (3, 1) B. (4, 0)
C. (2, 2) D. (2, 1)
Câu 2.38. Số liên kết σ và liên kết π trong phân tử CH3-CH(CH3)-
CH=C=CH-CH3 lần lượt là:
A. 17 – 2 B. 2 – 18
C. 4 – 18 D. 18 – 2

54
Câu 2.39. Khi dự đoán trạng thái lai hóa của NF3, gọi σ là số nguyên tử
biên, X là tổng số electron hóa trị trong phân tử, phát biểu nào sau đây là
đúng: (ZN = 7, ZF = 9).
A. σ = 3, X = 24, lai hóa sp2, cấu trúc góc
B. σ = 3, X = 16, lai hóa sp3, cấu trúc tháp
C. σ = 3, X = 26, lai hóa sp3, cấu trúc tháp
D. σ = 2, X = 18, lai hóa sp2, cấu trúc góc
Câu 2.40 Học thuyết cấu tạo nguyên tử nào ra đời đầu tiên?
A. Thomson B. Rutherford
C. Bohr D. Cơ học lượng tử
---oOo---

ĐÁP ÁN BÀI TẬP CHƯƠNG 2


2.1- 2.2- 2.3- 2.4- 2.5- 2.6- 2.7- 2.8- 2.9- 2.10-
B A A B B D C B D A
2.11- 2.12- 2.13- 2.14- 2.15- 2.16- 2.17 2.18- 2.19- 2.20-
B C A A A B -A A B B
2.21- 2.22- 2.23- 2.24- 2.25- 2.26- 2.27 2.28- 2.29- 2.30-
A C B A C B -A A D C
2.31- 2.32- 2.33- 2.34- 2.35- 2.36- 2.37 2.38- 2.39- 2.40-
D D B A D B -C D C A

55
Chương 3. NHIỆT ĐỘNG HÓA HỌC
---oOo---

Mục tiêu chương 3


Sau khi học xong chương này, sinh viên có khả năng:
1. Định nghĩa được các khái niệm cơ bản của nhiệt động hóa học.
2. Phát biểu và vận dụng được nội dung nguyên lý I và nguyên lý II
của nhiệt động hóa học.
3. Phát biểu và vận dụng được định luật Hess.
4. Tính toán được H, S, G của quá trình hóa học ở điều kiện tiêu
chuẩn.
5. Xác định được các điều kiện để một quá trình hóa học có thể xảy
ra.
6. Đánh giá được sự ảnh hưởng của nhiệt độ đến khả năng xảy ra của
một quá trình hóa học.

56
Phần 1. TÓM TẮT LÝ THUYẾT
3.1 CÁC KHÁI NIỆM
3.1.1 Hệ
Hệ là một vật thể hay nhóm vật thể được nghiên cứu, phần còn lại xung
quanh gọi là môi trường.
Hệ kín là hệ không có sự trao đổi chất nhưng có sự trao đổi năng lượng
với môi trường bên ngoài.
Hệ hở là hệ có sự trao đổi chất và năng lượng đối với môi trường bên
ngoài.
Hệ cô lập là hệ không có sự trao đổi chất và năng lượng đối với môi trường
bên ngoài, do đó thể tích hệ không đổi.
Hệ đoạn nhiệt là hệ không trao đổi chất và nhiệt, song có thể trao đổi công
với môi trường. Hệ cô lập bao giờ cũng đoạn nhiệt.
Hệ cân bằng là hệ có nhiệt độ, áp suất, thành phần giống nhau ở mọi điểm
của hệ và không thay đổi theo thời gian.
Hệ là đồng thể nếu trong toàn bộ thể tích của hệ các tính chất hóa lý đều
giống nhau, nghĩa là không có bề mặt phân chia các phần của hệ thành
những phần có tính chất khác nhau.
Hệ là dị thể nếu có bề mặt phân chia các phần của hệ thành những phần
có tính chất khác nhau.
Pha là phần đồng thể của hệ dị thể có thành phần, cấu tạo, tính chất nhất
định và được phân chia với các phần khác bằng bề mặt phân chia nào đó
gọi là bề mặt phân chia pha. Hệ đồng thể luôn luôn có một pha. Hệ dị thể
có nhiều pha.
3.1.2 Trạng thái nhiệt động
Trạng thái nhiệt động (trạng thái) của hệ là toàn bộ những tính chất hóa,
lý của hệ, được đặc trưng bằng các thông số trạng thái như nhiệt độ (T), áp
suất (P), thể tích (V), nồng độ (C), nội năng (U)…
Hàm trạng thái là đại lượng nhiệt động có giá trị chỉ phụ thuộc vào các
thông số trạng thái của hệ mà không phụ thuộc vào cách biến đổi của hệ.
Mỗi thông số trạng thái cũng là một hàm trạng thái.
Ví dụ, H = U + P.V là một hàm trạng thái của hệ, có tên là hàm enthalpy.
Tính chất của hàm trạng thái f: khi hệ đi từ trạng thái 1 sang trạng thái 2,
độ biến thiên hàm trạng thái được tính bằng Δf = f2 – f1 và không phụ thuộc
vào đường đi của quá trình 1 → 2.

57
Trạng thái cân bằng là trạng thái tương ứng hệ cân bằng nhiệt động, nghĩa
là khi đó giá trị các thông số trạng thái đặc trưng cho hệ ở mọi điểm phải
như nhau và không thay đổi theo thời gian.
Trạng thái tiêu chuẩn cho các chất được quy định như sau:
− Với chất rắn và chất lỏng: nguyên chất tại áp suất 1 atm.
− Chất khí: nguyên chất tại áp suất 1 atm; nếu trong hỗn hợp khí thì áp suất
riêng phần của mỗi khí phải bằng 1atm.
− Với chất tan: dung dịch có nồng độ 1M tại áp suất 1 atm.
− Tại mỗi nhiệt độ, mỗi chất chỉ có một trạng thái tiêu chuẩn. Không có
quy định bắt buộc về nhiệt độ tiêu chuẩn nhưng trên thực tế người ta
thường xét nhiệt độ 25 oC (298 K).
Đại lượng được đo ở điều kiện tiêu chuẩn được ký hiệu như sau:
• ΔHo hoặc ΔHo298
• ΔSo hoặc ΔSo298
• ΔGo hoặc ΔGo298
3.1.3 Quá trình
Quá trình là những sự thay đổi của hệ có kèm theo sự biến đổi của ít nhất
một thông số trạng thái. Nếu sau một biến đổi hệ trở về trạng thái ban đầu,
thì quá trình được gọi là chu trình.
• Quá trình xảy ra ở nhiệt độ không đổi (T = const) gọi là quá trình
đẳng nhiệt.
• Quá trình xảy ra ở áp suất không đổi (P = const) gọi là quá trình
đẳng áp.
• Quá trình xảy ra ở thể tích không đổi (V = const) gọi là quá trình
đẳng tích.
Quá trình thuận nghịch là quá trình có thể xảy ra theo hai chiều ngược
nhau và tương đối chậm sao cho các trạng thái trung gian giống nhau,
không gây nên biến đổi gì trong hệ cũng như môi trường.
Quá trình tự xảy ra tự phát là quá trình có khả năng tự xảy ra mà không
cần tác động từ bên ngoài hệ. Các quá trình tự xảy ra trong tự nhiên đều là
quá trình bất thuận nghịch.
3.1.4 Năng lượng
Năng lượng là thước đo sự chuyển động của các chất.
Năng lượng trao đổi giữa phản ứng hóa học (hệ) và môi trường thường
dưới hai hình thức là nhiệt (Q) và công (W).

58
Nhiệt là thước đo sự chuyển động nhiệt hỗn loạn của các tiểu phân tạo nên
hệ. Công là thước đo sự chuyển động có trật tự, có hướng của các tiểu phân
trong trường lực.
Nội năng (U) của hệ là năng lượng ẩn dấu bên trong của hệ, bao gồm tổng
động năng và thế năng của tất cả các tiểu phân bên trong hệ.
Quy ước về dấu đối với năng lượng trao đổi nói chung, với nhiệt và công
nói riêng:
• Dấu dương (+) nếu hệ thu năng lượng.
• Dấu âm (-) nếu hệ tỏa năng lượng.
Ví dụ, phản ứng tỏa 10 kJ nhiệt thì ghi Q = -10 kJ.
Công của phản ứng hóa học thường là công giãn nở chống lại áp suất bên
ngoài không đổi (áp suất khí quyển): W = - PΔV.
3.2 NGUYÊN LÝ I CỦA NHIỆT ĐỘNG LỰC HỌC
Nội dung nguyên lý I: “Năng lượng không tự sinh ra hoặc tự biến mất
mà chỉ có thể biến đổi từ dạng này sang dạng khác hoặc từ hệ này sang hệ
khác”.
Áp dụng nguyên lý I cho một phản ứng hóa học: Độ biến thiên nội năng
của hệ phản ứng bằng tổng nhiệt lượng (Q) và công (W) mà hệ phản ứng
trao đổi với môi trường.
ΔU = Q + W = Q - PΔV (3.1)
• Trong điều kiện đẳng tích (ΔV = 0)
QV = ΔU (3.2)
có nghĩa là nhiệt phản ứng ở điều kiện đẳng tích bằng biến thiên nội năng
của hệ phản ứng.
• Phản ứng trong điều kiện đẳng áp (P = const)
Qp = ΔH (3.3)
có nghĩa là nhiệt phản ứng ở điều kiện đẳng tích bằng biến thiên enthalpy
của hệ phản ứng.
• Mối liên hệ giữa ΔH và ΔU
ΔH = ΔU + PΔV= ΔU + RTΔn (3.4)
Với n là biến thiên số mol khí trước và sau phản ứng.
Ví dụ:
C2H6 (g) + 7/2 O2 (g) → 2CO2 (g) + 3H2O (g) ∆n = 2 + 3 – 1 – 7/2 = 0,5
N2 (g) + 3H2 (g) → 2NH3 (g) ∆n = 2 – 3 – 1 = – 2
Fe2O3 (s) + 3CO (g) → 2Fe (s) + 3CO2 (g) ∆n = 3 – 3 = 0

59
CaCO3 (s) → CaO (s) + CO2 (g) ∆n = 1
3.3 HIỆU ỨNG NHIỆT
Hiệu ứng nhiệt phản ứng (nhiệt phản ứng), là nhiệt lượng tỏa ra hay thu
vào của một phản ứng hóa học được đo trong những điều kiện xác định.
Hiệu ứng nhiệt tính cho 1 mol chất ở điều kiện chuẩn (1 atm, 25 oC) gọi là
hiệu ứng nhiệt tiêu chuẩn. Ký hiệu: Ho hoặc Ho298.
Nhiệt tạo thành của một chất là hiệu ứng nhiệt của phản ứng tạo thành 1
mol chất đó từ các đơn chất bền ở điều kiện đã cho về nhiệt độ và áp suất.
Khi điều kiện đã cho là điều kiện chuẩn, ta có nhiệt tạo thành tiêu chuẩn.
Ký hiệu: Ho298, tt hoặc Ho298, f.
Các đơn chất bền có Ho298, f = 0.
Ví dụ, ta có ∆Ho298,tt CuO (r) = -157,3 kJ/mol, nghĩa là hiệu ứng nhiệt của
phản ứng giữa 1 mol Cu (s) và 0,5 mol O2 (k) tạo ra 1 mol CuO (s) là
– 157,3 kJ.
Cu (r) + ½ O2 (k) = CuO (r), ∆Ho = -157,3 kJ
Nhiệt đốt cháy của một chất là hiệu ứng nhiệt của phản ứng đốt cháy 1 mol
chất đó bằng khí oxy để tạo thành sản phẩm cháy bền ở điều kiện phản
ứng. Sản phẩm cháy của các nguyên tố C, H, N, S, Cl lần lượt là CO2 (g),
H2O (l), N2 (g), SO2 (g) và HCl (g). Khi điều kiện đã cho là điều kiện
chuẩn, ta có nhiệt đốt cháy tiêu chuẩn. Ký hiệu: Ho298, đc hoặc Ho298, c.
Ví dụ, ta có ∆Ho298,b C3H8 (g) = - 2.220 kJ/mol, nghĩa là hiệu ứng nhiệt của
phản ứng đốt cháy 1 mol C3H8 (g) bằng 5 mol O2 (g) tạo ra 3 mol CO2 (g)
và 4 mol H2O (g) là 2.220 kJ
C3H8 (k) + 5O2→ 3CO2 (k) + 4H2O (k), ∆Ho = - 2.220 kJ
3.4 PHƯƠNG TRÌNH NHIỆT HÓA HỌC VÀ CHIỀU DIỄN RA CỦA
CÁC QUÁ TRÌNH HÓA HỌC
3.4.1 Phương trình nhiệt hóa học
Phương trình nhiệt hóa học là phương trình phản ứng hóa học có ghi kèm
đại lượng hiệu ứng nhiệt và các ký hiệu chỉ trạng thái vật chất. Ví dụ: s
(rắn), l (lỏng), g (khí), aqu (dung dịch nước), gr (graphite) diamond (kim
cương)…
Lưu ý: hiệu ứng nhiệt tỷ lệ thuận với lượng tác chất và sản phẩm.
Ví dụ,
Zn(s) + 2HCl(aqu) → ZnCl2 (aqu) + H2(g), Ho298 = -152,57 kJ
C(gr) + H2O(g) → CO(g) + H2 (g), Ho298 = + 131,25 kJ

60
3.4.2 Chiều diễn ra của các quá trình hóa học
Ở điều kiện nhiệt độ thấp, quá trình tỏa nhiệt (H < 0) là quá trình có khả
năng tự xảy ra, còn quá trình thu nhiệt (H > 0) là quá trình không thể tự
xảy ra được, muốn xảy ra phải tiêu tốn năng lượng.
Ở điều kiện nhiệt độ cao, do có chịu ảnh hưởng của nhiều yếu tố khác mà
quy luật này không còn đúng nữa.
3.5 ĐỊNH LUẬT HESS VÀ CÁC HỆ QUẢ
Định luật Hess: Hiệu ứng nhiệt đẳng áp hay đẳng tích của quá trình hóa
học chỉ phụ thuộc vào bản chất và trạng thái của tác chất và sản phẩm
cuối chứ không phụ thuộc vào đường đi của quá trình, nghĩa là không phụ
thuộc vào số lượng và đặc điểm của các giai đoạn trung gian.
Xét quá trình từ X → Y qua hai con đường khác nhau như sau:

∆H = ∆H1 + ∆H2
= ∆H3 + ∆H4 + ∆H5

Hệ quả 1: Hiệu ứng nhiệt của một phản ứng bằng tổng nhiệt tạo thành của
các sản phẩm trừ đi tổng nhiệt tạo thành của các tác chất (có xét đến các
hệ số tỷ lượng của mỗi chất trong phương trình phản ứng).
∆𝐻 𝑜 = ∑ ∆𝐻 𝑜 − ∑ ∆𝐻 𝑜 (3.5)
𝑓,𝑠ả𝑛 𝑝ℎẩ𝑚 𝑓,𝑡á𝑐 𝑐ℎấ𝑡

Hệ quả 2: Hiệu ứng nhiệt của một phản ứng bằng tổng nhiệt đốt cháy của
tác chất trừ đi tổng nhiệt đốt cháy của các sản phẩm (có xét đến các hệ số
tỷ lượng của mỗi chất trong phương trình hóa học).
∆𝐻 𝑜 = ∑ ∆𝐻 𝑜 − ∑ ∆𝐻 𝑜 (3.6)
𝑐,𝑡á𝑐 𝑐ℎấ𝑡 𝑐,𝑠ả𝑛 𝑝ℎẩ𝑚

Ví dụ 3.1. Xác định hiệu ứng nhiệt của phản ứng:


CaCO3 (s) → CaO (s) + CO2 (g)
∆H f, kJ/mol
o
- 1206,92 - 635,09 - 393,51
Giải
∆Hophản ứng = - 635,09 – 393,51 – (- 1206,92) = 178,32 kJ
Ví dụ 3.2. Xác định hiệu ứng nhiệt của phản ứng:
CH3COOH(l) + C2H5OH(l) → CH3COOC2H5(l) + H2O(l)
∆H c, kJ/mol
o
- 874,58 - 1367,58 - 2238,36 0

61
Giải
∆Hophản ứng = - 874,58 – 1367,58 – (- 2238,36) = - 3,8 kJ
3.6 ENTROPY VÀ NGUYÊN LÝ II CỦA NHIỆT ĐỘNG HỌC
3.6.1 Nội dung nguyên lý II
Nhiệt không thể tự chuyển từ vật thể nguội hơn sang vật thể nóng hơn.
3.6.2 Entropy S
Entropy S là đại lượng đặc trưng cho mức độ mất trật tự của các tiểu
phân bên trong hệ. Trong các bảng tra cứu, entropy của các chất được cho
ở điều kiện tiêu chuẩn và 25 oC. Ký hiệu So hoặc So298 và có đơn vị là
J/(mol.K) hoặc cal/(mol.K).
3.6.3 Tính toán So của phản ứng hóa học
a) Tính định tính
S của phản ứng về phương diện định tính có thể xác định dựa vào độ
biến đổi thể tích V. Việc xác định V của phản ứng dựa trên thể tích các
chất khí tham gia vào phản ứng vì thể tích của chất rắn và lỏng nhỏ hơn rất
nhiều so với chất khí.
• Khi thể tích tăng V > 0 → S > 0
• Khi thể tích giảm V < 0 → S < 0
• Khi thể tích không đổi V = 0 → S = 0
Ví dụ,
C(gr) + CO2(g) = 2CO(g), V = 2V − 1V > 0 → S > 0
N2(g) + 3H2(g) = 2NH3(g), V = 2V − 4V < 0 → S < 0
C(gr) + O2(g) = CO2(g), V = 1V − 1V = 0 → S = 0
b) Tính định lượng
Do entropy cũng là hàm trạng thái, nên biến thiên entropi của một phản
ứng có thể tính toán bằng tổng entropy của các sản phẩm trừ tổng entropy
của các chất đầu.
∆S o = ∑ S o − ∑ So (3.7)
sản phẩm tác chất

Ví dụ 3.3. Tính S của phản ứng sau


o

C(gr) + CO2(k) → 2CO(k)


o -1 -1
S 298 (cal.mol .K ) 1,37 51,06 47,22
Giải
∆So = 2.47,22 – (1,37 + 51,06) = 42,01 cal/K
Entropy và chiều diễn ra của các quá trình trong hệ cô lập

62
Trong hệ cô lập những quá trình tự xảy ra là những quá trình có kèm theo
sự tăng entropy (S > 0). Quá trình sẽ tự xảy ra cho đến khi entropy đạt
giá trị cực đại, không đổi và hệ đạt đến trạng thái cân bằng.
3.7 THẾ ĐẲNG ÁP VÀ CHIỀU DIỄN RA CỦA CÁC QUÁ TRÌNH
HÓA HỌC
3.7.1 Thế đẳng áp G
Năng lượng tự do Gibbs, còn gọi là thế đẳng áp G, là một hàm trạng thái
được định nghĩa:
G = H – TS (3.8)
Đối với quá trình đẳng nhiệt đẳng áp, ta có:
ΔG = ΔH – TΔS (3.9)
đây được gọi là phương trình cơ bản của nhiệt động hóa học.
3.7.2 Thế đẳng áp tạo thành tiêu chuẩn
Thế đẳng áp tạo thành của một chất (kJ/mol hoặc kcal/mol) là biến thiên
thế đẳng áp của phản ứng tạo thành 1 mol chất đó từ các đơn chất bền ở
điều kiện đã cho về nhiệt độ và áp suất. Thế đẳng áp tạo thành tính ở điều
kiện tiêu chuẩn và 25 oC gọi là thế đẳng áp tạo thành tiêu chuẩn, ký hiệu
ΔGof, 298 hoặc ΔGott, 298.
3.7.3 Độ biến đổi thế đẳng áp và điều kiện xảy ra của quá trình hóa học
Điều kiện để phản ứng tự xảy ra ở điều kiện đẳng nhiệt và đẳng áp là:
ΔG = ΔH – TΔS < 0
Thông thường thì ΔH và ΔS của phản ứng không thay đổi đáng kể theo
nhiệt độ, do đó nhiệt độ là đại lượng có ảnh hưởng lớn nhất đến chiều
hướng xảy ra của phản ứng hóa học.
a) Ở nhiệt độ thấp → ∆G  ∆H
• Nếu ∆H < 0 → ∆G < 0 → quá trình có thể xảy ra
• Nếu ∆H > 0 → ∆G > 0 → quá trình không thể xảy ra
b) Ở nhiệt độ cao → ∆G  - T.∆S
• Nếu ∆S < 0 → ∆G > 0 → quá trình không thể xảy ra
• Nếu ∆S > 0 → ∆G < 0 → quá trình có thể xảy ra
c) Ở nhiệt độ không quá cao, không quá thấp → ∆G = ∆H – T.∆S
• Nếu ∆H < 0 và ∆S > 0 → ∆G < 0 với mọi nhiệt độ → quá trình có
thể xảy ra ở bất kỳ nhiệt độ nào.
• Nếu ∆H > 0 và ∆S < 0 → ∆G > 0 với mọi nhiệt độ → quá trình không
thể xảy ra ở bất kỳ nhiệt độ nào.
63
• Nếu ∆H > 0 và ∆S > 0 → quá trình có thể xảy ra ở nhiệt độ T > Tcb.
• Nếu ∆H < 0 và ∆S < 0 → quá trình có thể xảy ra ở nhiệt độ T > Tcb.
∆𝐻
Lưu ý: 𝑇𝑐𝑏 = ∆𝑆
3.7.4 Tính độ biến đổi thế đẳng áp của các phản ứng hóa học
Độ biến đổi thế đẳng áp của một phản ứng có thể tính toán bằng tổng thế
đẳng áp tạo thành của các sản phẩm trừ tổng thế đẳng áp của các chất đầu.
∆Go = ∑ ∆Go − ∑ ∆Go (3.10)
f,sản phẩm f,tác chất

64
Phần 2. BÀI TẬP CÓ LỜI GIẢI
---oOo---
Dạng 1. Nhiệt và công
Câu 3.1. Xác định lượng nhiệt cần để nâng nhiệt độ của 7,35 g nước từ
21 oC tới 98 oC. Biết nhiệt dung riêng của nước là 4,18 J.g-1.oC-1.
Giải
Q = mCT = mC(T2 - T1)
Trong đó: m: khối lượng nước (g)
C: nhiệt dung riêng của nước (J g-1 oC-1)
T1, T2 là nhiệt độ ban đầu và sau
Vậy Q = 7,35  4,18  (98 - 21) = 236.103 J
Câu 3.2. Tính công (J) khi giãn nở 0,225 mol N2 một thể tích 1,50 lít tại
nhiệt độ không đổi (23 oC) chống lại áp suất ngoài 0,750 atm?
Giải
W = - P.V = 0,750  1,5 = - 114 J
Như vậy, hệ thực hiện công (sinh công)
• Lưu ý: 1 bar.L = 100 J; 1 atm.L = 101,325 J
Câu 3.3. Khi giãn nở, một khí hấp thụ lượng nhiệt là 25 J và sinh công
243 J. Xác định độ biến đổi nội năng của hệ.
Giải
Khí hấp thụ (nhận) nhiệt nên Q = + 25 J; Khí sinh công nên W = - 243 J
Theo nguyên lý thứ nhất nhiệt động học: U = Q + W = (+ 25 J) + (- 243
J) = - 218 J
Như vậy, nội năng của hệ giảm.
Câu 3.4. Xác định lượng nhiệt đi kèm khi đốt cháy 1,00 kg đường sucrose
C12H22O11. Biết rằng khi đốt cháy 1 mol sucrose, lượng nhiệt được tạo ra
là H = -5,65.103 kJ/mol. Msucrose = 342,3 g/mol.
Giải
Số mol đường sucrose, n = 1000/342,3 = 2,92 mol
Lượng nhiệt được tạo ra khi đốt cháy 1 mol sucrose:
Q = 2,92 × ሺ−5,65. 103 ሻ = −1,65. 104 kJ
Dạng 2. Hiệu ứng nhiệt phản ứng.
Câu 3.5. Tính Ho của phản ứng dưới đây.
3C (than chì) + 4H2 (g) → C3H8 (g) Ho = ?

65
Biết:
(a) C3H8 (g) + 5O2 (g) → 3CO2 (g) + 4H2O (l) Ho = - 2219,9 kJ
(b) C (gr) + O2 (g) → CO2 (g) Ho = - 393,5 kJ
(c) H2 (g) + 1/2O2 (g) → H2O (l) Ho = - 285,8 kJ
Giải
Phản ứng (b) và (c) đều có C (than chì) và H2 (g). Nhân hai vế phương
trình phản ứng (b) với 3 và phản ứng (c) với 4, ta được:
3(b): 3C (than chì) + 3O2 (g) → 3CO2 (g) 3Ho = 3(-393,5) kJ
4(c) 4H2 (g) + 2O2 (g) → 4H2O (l) 4Ho = 4(-285,8) kJ
Phương trình phản ứng cần tìm Ho có sản phẩm là C3H8. Vì vậy, để có
sản phẩm C3H8 thì ta nhân hai vế phương trình phản ứng (a) với -1:
(-1)(a): C3H8 (g) + 5O2 (g) → 3CO2 (g) + 4H2O (l) Ho = (-1)(-2219,9)
kJ
Tổng 03 phương trình phản ứng và lược giản những chất giống nhau ở 02
vế của các phương trình:
(-a): 3CO2 (g) + 4H2O (l) → C3H8 (g) + 5O2 (g) Ho = + 2219,9 kJ
3(b): 3C (than chì) + 3O2 (g) → 3CO2 (g) Ho = - 1180,5 kJ
4(c) 4H2 (g) + 2O2 (g) → 4H2O (l) Ho = - 1143,2 kJ
3C (than chì) + 4H2 (g) → C3H8 (g) Ho = 103,8 kJ
Câu 3.6. Tính hiệu ứng nhiệt tiêu chuẩn Ho của phản ứng dưới đây.
C2H6 (g) + 7/2O2 (g) → 2CO2 (g) + 3H2O (l) Ho = ?
Hof - 84,7 0 - 393,5 - 285,8
(kJ/mol)
Giải
o o
Áp dụng công thức (3.5): ∆H o = ∑ ∆Hf,sản phẩm − ∑ ∆Hf,tác chất
→ Ho = 2(- 393,5) + 3 (- 285,8) – 1  (- 84,7) = - 1559,7 kJ
Câu 3.7. Tính nhiệt tạo thành tiêu chuẩn x của benzene lỏng, biết:
2C6H6(l) + O2 (g) →12CO2(g) + 6H2O(l) Ho = - 6535 kJ
Hof x 0 - 393,5 - 285,8
(kJ/mol)
Giải
Ho = 12  (-393,5) + 6  (-285,8) – 2  X = -6535 kJ
→ Hott[C6H6 (l)] = x = 49 kJ/mol
Câu 3.8. Tính hiệu ứng nhiệt tiêu chuẩn của phản ứng sau đây:

66
C2H2 (g) + H2 (g) → C2H4 (g) Ho = ?
Hoc (kJ/mol) -1300 -286 -1560
Giải
o o
Áp dụng công thức (3.6): ∆H o = ∑ ∆Hc,tác chất − ∑ ∆Hc,sản phẩm
Ho = 1  (-1300) + 1  (-286) – 1  (-1560) = - 26 kJ
Ví dụ 3.9. Tính hiệu ứng nhiệt tiêu chuẩn của phản ứng kết tủa BaSO4 từ
Ba2+ và SO42-
Ba2+ (aqu) + SO42- (aqu) → BaSO4 (s) Ho = ?
Hof (kJ/mol) - 537,6 - 909,3 - 1473
Giải
Sử dụng nhiệt tạo thành tiêu chuẩn của các ion Ba2+, SO42- và BaSO4. Áp
o o
dụng công thức (3.5): ∆H o = ∑ ∆Hf,sản phẩm − ∑ ∆Hf,tác chất
Ho = 1  (-1473) – 1  (- 537,6) – 1  (- 909,3 kJ/mol) = - 26 kJ
Dạng 3. Độ biến đổi entropy S.
Câu 3.10. Dự đoán dấu của ΔS đối với mỗi quá trình sau:
a) H2O(g) → H2O(l)
b) Nước đá khô thăng hoa.
c) 2 N2O(g) → 2 N2(g) + O2(g)
Giải
Với một quá trình, ∆S o = ∑ ∆S osản phẩm − ∑ ∆Stác
o
chất
a) Một chất ở thể khí có entropy lớn hơn khi ở thể lỏng, nên entropy
giảm và ΔS < 0.
b) Nước đá khô là CO2 ở thể rắn, khi thăng hoa chuyển thành thể khí
có entropy lớn hơn nên entropy tăng và ΔS > 0.
c) Vì số mol khí tăng nên entropy tăng và ΔS > 0.
Câu 3.11. Tính biến thiên entropy của quá trình ngưng tụ 25,0 g hơi nước
thành nước lỏng ở nhiệt độ sôi 100 oC. Biết nhiệt hóa hơi của nước ở nhiệt
độ này là 40,7 kJ/mol.
Giải
Do ngưng tụ và bay hơi là hai quá trình ngược chiều nhau, nên nhiệt của
chúng ngược dấu và bằng nhau về độ lớn.
Nhiệt quá trình ngưng tụ 25,0 g nước là:
25,0
Qbay hơi = −Qngưng tụ = −40700 × = −56528 J
18

67
Vì sự ngưng tụ xảy ra tại nhiệt độ sôi của nước nên nhiệt độ của nước
không đổi trong suốt quá trình. Do đó có thể sử dụng công thức tính ΔS
cho qua trình đẳng nhiệt:
Qngưng tụ −56528
ΔSngưng tụ = = = −151,5 J/K
T 100 + 273
Nhận xét: Đáp số có đơn vị J/K là đúng cho entropy. Đáp số có dấu âm,
phù hợp với quá trình ngưng tụ hơi nước làm giảm entropy.
Câu 3.12. Xét quá trình cháy của khí propane:
C3H8(g) + 5 O2(g) → 3 CO2(g) + 4 H2O(g) ΔHphản ứng = - 2044 kJ
a) Tính biến thiên entropy ΔS của môi trường nếu nhiệt độ môi trường
là 25 oC không đổi.
b) Xác định dấu của ΔS của hệ phản ứng.
c) Xác định dấu của ΔS của vũ trụ.
Giải
a) T = 25 + 273 = 298 K
Do nhiệt độ môi trường không đổi nên ΔS của môi trường được tính theo
công thức:
− ∆H 2044 kJ
∆S = = = 6,859 = 6859 J/K
T 298 K

b) Vì số mol khí tăng lên (5 mol → 7 mol) khi phản ứng xảy ra nên
entropy của hệ tăng, tức ΔS > 0.
c) Biến thiên entropy của vũ trụ bằng tổng biến thiên entropy của hệ
và môi trường:
𝛥𝑆𝑣ũ 𝑡𝑟ụ = 𝛥𝑆𝑝ℎả𝑛 ứ𝑛𝑔 + 𝛥𝑆𝑚ô𝑖 𝑡𝑟ườ𝑛𝑔 > 0 do ΔSphản ứng và ΔSmôi trường đều
dương.
Câu 3.13. Tính ΔSo cho phản ứng sau
4 NH3(g) + 5 O2(g) → 4 NO(g) + 6 H2O(g)
Biết entropy mol tiêu chuẩn của các chất như sau:
NH3(g) O2(g) NO(g) H2O(g)
o -1 -1
S (J.K .mol ) 192,8 205,2 210,8 188,8
Giải
Tính ΔSo của phản ứng theo công thức (3.7):
∆S o = ∑ ∆S osản phẩm − ∑ ∆Stác
o
chất

So = 4 x 210,8 + 6 x 188,8 – 4 x 192,8 – 5 x 205,2 = 178,8 J/K


Nhận xét: ΔSo > 0 là hợp lý vì số mol khí trong phản ứng tăng lên.
68
Dạng 3. Độ biến đổi thế đẳng áp và chiều diễn ra của các quá trình
hóa học
Câu 3.14. Xét phản ứng phân hủy khí carbon tetrachloride:
CCl4 (g) → C (graphite) + 2 Cl2 (g) có ΔH = 95,7 kJ; ΔS = 142,2 J/K
a) Tính ΔG của phản ứng ở 25 oC.
b) Ở 25 oC phản ứng này có tự xảy hay không? Tại sao?
c) Xác định khoảng nhiệt độ mà phản ứng này tự xảy ra.
Giải
a) T = 25 + 273 = 298 K
ΔG = ΔH − TΔS = 95,7 − 298 × 142,2 × 10−3 = 53,3kJ
Lưu ý: Đơn vị nhiệt độ là K, và ΔH, ΔS phải cùng đơn vị.
b) Ở 25 oC phản ứng này không tự xảy ra. Vì có G > 0.
c) Phản ứng tự xảy ra khi ΔG = ΔH − TΔS = 95,7 − T × 142,2 ×
10−3 < 0
95,7
hay 𝑇 > 142,2×10−3 = 673𝐾
Nhận xét: Phản ứng có ΔH và ΔS đều dương nên phản ứng xảy ra ở nhiệt
độ cao (cao hơn 673 K) là hợp lý.
Ví dụ 3.15. Một trong những phản ứng có thể tham gia vào việc tạo mưa
acid là sự oxy hóa SO2 thành SO3 theo phản ứng:
SO2(g) + 1/2 O2(g) → SO3(g)
a) Tính ΔG ở 25 oC của phản ứng trên và cho biết phản ứng này có
o

thể tự xảy ra hay không?


b) Hãy tính ΔGo cho phản ứng ở 125 oC và cho biết ở nhiệt độ cao,
phản ứng này xảy ra tự phát với mức độ lớn hơn hay nhỏ hơn?
SO2(g) O2(g) SO3(g)
o
ΔHf ሺkJ/molሻ - 296,8 0 - 395,7
o . -1. -1
S (J K mol ) 248,1 205,2 256,8
Giải
a) Tính ΔHo của phản ứng theo công thức (3.5):
o o
∆H o = ∑ ∆Hf,sản phẩm − ∑ ∆Hf,tác chất

Ho = – 395,7 – (– 296,8) = – 98,9 kJ


Tính ΔSo của phản ứng theo công thức (3.7):
∆S o = ∑ ∆S osản phẩm − ∑ ∆Stác
o
chất

So = 256,8 – 248,1 – 0,5 x 205, 2 = – 94,9 J/K


69
Tính ΔGo của phản ứng theo công thức (3.9): ΔG = ΔH – TΔS
ΔGo = ΔH o − TΔS o = −98,9 − 298 × ሺ−94,0ሻ × 10−3 = −70,9kJ
Phản ứng này tự xảy ra ở 25 oC vì ΔGo < 0.
b) Tính lại ΔGo của phản ứng ở 125 oC
ΔGo = ΔH o − TΔS o = −98,9 − 398 × ሺ−94,0ሻ × 10−3 = −61,5kJ
Ở nhiệt độ 125 oC, ΔGo tăng lên so với ở 25 oC, nghĩa là phản ứng trở nên
ít tự phát hơn.
Câu 3.16. Ozone ở tầng khí quyển gần mặt đất là một chất gây ô nhiễm,
có thể được tạo thành bởi phản ứng oxy hóa các hydrocarbon:
CH4 (g) + 8O2 (g) → CO2 (g) + 2H2O (g) + 4O3 (g)
Sử dụng các giá trị năng lượng tự do tạo thành tiêu chuẩn bên dưới, hãy
tính ΔGo của phản ứng trên ở 25 oC.
CH4(g) O2(g) CO2(g) H2O(g) O3(g)
𝑜
𝛥𝐺𝑓 ሺ𝑘𝐽/𝑚𝑜𝑙ሻ -50,5 0 -394,4 -228,6 163,2
Giải
o o
Sử dụng công thức (3.10): ∆Go = ∑ ∆Gf,sản phẩm − ∑ ∆Gf,tác chất
Go = – 394,4 + 2 (– 228,6) + 4 (163,2) – (– 50,5) = – 148,3 kJ.
Câu 3.17. Hãy tính ΔGo cho phản ứng: 3 C(s) + 4 H2(g) → C3H8(g)
Cho biết:
C3H8(g) + 5 O2(g) → 3 CO2(g) + 4 H2O(g) ΔGo = – 2074 kJ
C(s) + O2(g) → CO2(g) ΔGo = – 394,4 kJ
2 H2(g) + O2(g) → 2 H2O(g) ΔGo = – 457,1 kJ
Giải
Để giải dạng câu hỏi này, cần phải biến đổi các phản ứng đã biết ΔGo sao
cho khi cộng các phản ứng đã biến đổi này, ta sẽ được phản ứng đang cần
tính ΔGo.
Vì phản ứng thứ nhất có C3H8 là tác chất, và phản ứng đang quan tâm cũng
có C3H8 là tác nhân, ta đảo chiều và đổi dấu ΔGo của phản ứng thứ nhất.
3 CO2(g) + 4 H2O(g) → C3H8(g) + 5 O2(g) ΔGo = +2074 kJ
Phản ứng thứ hai có C là tác chất và CO2 là sản phẩm, giống như phản ứng
đang quan tâm. Tuy nhiên ta cần nhân hệ số 3 vào phản ứng thứ hai để hệ
số của C giống như trong phản ứng quan tâm. Do đó ΔGo của phản ứng
này cũng phải được nhân với 3.
3 C(s) + 3 O2(g) → 3 CO2(g) ΔGo = 3×(-394,4) = -1183 kJ
Trong phản ứng thứ ba, H2(g) là tác chất, cũng giống như trong phản ứng
quan tâm. Tuy nhiên, cần nhân hệ số 2 vào phản ứng này để hệ số của H 2
70
giống như trong phản ứng quan tâm. Phải nhân 2 vào cả ΔGo của phản ứng
này.
4 H2(g) + 2 O2(g) → 4 H2O(g) ΔGo = 2×(- 457,1) = - 914,2 kJ
Cuối cùng, cộng ba phản ứng đã biến đổi ở trên lại với nhau và đảm bảo
rằng tổng thu được chính là phản ứng đang quan tâm. ΔGo của phản ứng
đang quan tâm cũng chính là tổng ΔGo của ba phản ứng đã biến đổi.
3 CO2(g) + 4 H2O(g) → C3H8(g) + 5 O2(g) ΔGo = +074 kJ
3 C(s) + 3 O2(g) → 3 CO2(g) ΔGo = - 1183 kJ
4 H2(g) + 2 O2(g) → 4 H2O(g) ΔGo = - 914,2 kJ
3 C(s) + 4 H2(g) → C3H8(g) ΔGo = - 23 kJ

71
Phần 3. BÀI TẬP TRẮC NGHIỆM
---oOo---
Câu 3.1. Lượng nhiệt (kJ) cần để nâng 9,25 L nước từ nhiệt độ 22 oC lên
29,4 oC là bao nhiêu? Biết nhiệt dung riêng của nước là 4,18 J.g-1.oC-1.
A. 68,45 B. 286,12
C. 16,37 D. 1195,98
Câu 3.2. Lượng nhiệt (kJ) đi kèm khi giảm 33,5 oC đối với thanh nhôm
nặng 5,85 g là bao nhiêu? Biết nhiệt dung riêng của nhôm là
0,903 J.g-1.oC-1.
A. - 177 B. - 5,28
C. - 739,86 D. - 22,08
Câu 3.3. Nhiệt độ cuối cùng ( C) khi 12,6 g nước tại 22,9 oC hấp thụ lượng
o

nhiệt 875 J là bao nhiêu? Biết nhiệt dung riêng của nước là 4,18 J g-1 oC-1.
A. 16,6 B. 69,45 C. 39,5 D. 92,34
Câu 3.4. 1,00 kg mẫu magnesium tại 40 C được nhúng vào 1,00 L nước
o

ở 20,0 oC trong bình cách nhiệt. Nhiệt độ (oC) cuối cùng của hỗn hợp
magnesium-nước là bao nhiêu? Biết nhiệt dung riêng của nước và
magnesium lần lượt 4,18 J.g-1.oC-1 và 1,024 J.g-1.oC-1.
A. 34 B. 54 C. 72 D. 24
Câu 3.5. Nhiệt (kJ) tỏa ra trong quá trình điều chế 283 kg Ca(OH)2 theo
phản ứng hóa học sau là bao nhiêu?
CaO (s) + H2O (l) → Ca(OH)2 (l) Ho = - 65,2 kJ
A. - 2,49.105 B. - 879,85
C. - 184,48 D. - 1,18
Câu 3.6. Nhiệt (kJ) tỏa ra trong quá trình đốt cháy 12,6 L C4H10 (g) tại
23,6 oC là bao nhiêu? Biết phản ứng đốt chảy 1 mol C4H10 (g) diễn ra như
sau:
C4H10 (g) + 13/2O2 (g) → 4CO2 (g) + 5H2O (l) Ho = - 2877 kJ
A. - 115,08 B. - 145.103
C. - 196,48 D. - 6304,34
Câu 3.7. Cho phản ứng đốt cháy CH4 (g) sau:
CH4 (g) + 2O2 (g) → CO2 (g) + 2H2O (l), Ho = - 890,3 kJ
Cần bao nhiêu CH4 (kg) để khi đốt cháy giải phóng ra lượng nhiệt là
2,8.107 kJ? Cho biết: C = 12 và H = 1 g/mol.
A. 503 B. 0,50
C. 31,50 D. 180
72
Câu 3.8. Khi cho 0,205 g KOH vào 55,9 g nước thì nhiệt độ của nước tăng
từ 23,5 đến 24,4 oC. Giả sử nhiệt dung riêng của dung dịch KOH là 4,18
J.g-1.oC-1. Vậy nhiệt giải phóng ra (kJ/mol) khi hòa tan KOH vào nước là
bao nhiêu?
A. - 200 B. - 5
C. - 3,56 D. - 24,39
Câu 3.9. Nhiệt của quá trình hòa tan KI và nước là + 20,3 kJ/mol. Nếu một
lượng KI được hòa tan vào nước ở 23,5 oC để tạo ra 150 mL dung dịch KI
2,5 M thì nhiệt độ cuối của dung dịch là bao nhiêu oC? Biết khối lượng
riêng và nhiệt dung riêng của dung dịch KI 2,5 M là 1,30 g/mL và
2,7 J.g-1.oC-1.
A. 14 B. 10
C. 2,17 D. 2,82
Câu 3.10. Nhiệt hòa tan NaOH vào nước là - 44,5 kJ/mol. Nhiệt độ (oC)
mà nước có thể đạt được là bao nhiêu khi điều chế 500 mL dung dịch
NaOH 7,0 M. Biết nhiệt độ ban đầu của nước là 21 oC. Khối lượng riêng
của dung dịch NaOH là 1,08 g/mL và nhiệt dung riêng là 4,00 J g-1 oC-1.
A. 74 B. 53 C. 80 D. 95
Câu 3.11. Công (J) của quá trình nén một khí từ thể tích 5,62 L xuống 3,37
L bởi áp suất không đổi 1,23 atm là bao nhiêu?
A. 2,76 B. 4,14
C. 280 D. 6,91
Câu 3.12. Hệ sinh công hay nhận công khi thực hiện phản ứng trung hòa
Ba(OH)2 (aqu) bởi HCl (aqu) tại áp suất khí quyển.
A. Hệ sinh công
B. Hệ nhận công
C. Hệ không nhận công cũng không sinh công
D. Không thể kết luận
Câu 3.13. Thực hiện phản ứng chuyển khí NO2 thành N2O4 ở điều kiện áp
suất khí quyển. Quá hình này hệ sinh công hay nhận công?
A. Hệ nhận công
B. Hệ sinh công
C. Hệ không nhận công cũng không sinh công
D. Không thể kết luận
Câu 3.14. Thể tích (L) của hệ thay đổi bao nhiêu khi một hệ thực hiện
công là 325 J tại áp suất 1,0 atm và 298 K?

73
A. 1,01 B. 2,50
C. 3,21 D. 2,13
Câu 3.15. Biến thiên nội năng của hệ là bao nhiêu (J) khi hệ hấp thụ lượng
nhiệt 58 J và sinh công 58 J?
A. 0 B. 116 C. 58 D. 32
Câu 3.16. Biến thiên nội năng của hệ là bao nhiêu (kJ) khi hệ giải phóng
lượng nhiệt 280 cal và nhận công 1,25 kJ?
A. 0,08 B. 1,17
C. 4,18 D. 0,04
Câu 3.17. Chọn phát biểu đúng:
A. Một lượng khí lý tưởng giãn nở đẳng nhiệt sẽ sinh công và giải phóng
nhiệt
B. Một lượng khí lý tưởng giãn nở đẳng nhiệt sẽ sinh công và hấp thụ nhiệt
C. Một lượng khí lý tưởng giãn nở đẳng nhiệt sẽ nhận công và giải phóng
nhiệt
D. Một lượng khí lý tưởng giãn nở đẳng nhiệt sẽ nhận công và hấp thu
nhiệt
Câu 3.18. Nội năng của một lượng khí lý tưởng phụ thuộc vào nhiệt độ.
Chọn phát biểu đúng:
A. Khí lý tưởng giãn nở đẳng nhiệt sẽ sinh công và nội năng tăng
B. Khí lý tưởng giãn nở đẳng nhiệt sẽ sinh công và nội năng không đổi
C. Khí lý tưởng giãn nở đẳng nhiệt sẽ nhận công và nội năng giảm
D. Khí lý tưởng giãn nở đẳng nhiệt sẽ sinh công và nội năng giảm
Câu 3.19. Đại lượng nào phù hợp để chỉ nhiệt phản ứng trong mọi điều
kiện phản ứng:
A. QV B. QP C. U – W D. H
Câu 3.20. So sánh H và U trong phản ứng đốt cháy 1 mol 1-butanol (l).
Giả sử rằng sự thay đổi về số mol khí trong phản ứng ở áp suất không đổi
mới dẫn đến sự thay đổi về thể tích. (Lớn hơn: dương hơn hoặc ít âm hơn;
bé hơn: ít dương hơn hoặc âm hơn)
A. H > U B. H < U
C. H = U D. Không thể kết luận
Câu 3.21. So sánh H và U trong phản ứng đốt cháy 1 mol glucose
C6H12O6 (s). Giả sử rằng sự thay đổi về số mol khí trong phản ứng ở áp
suất không đổi mới dẫn đến sự thay đổi về thể tích. (Lớn hơn: dương hơn
hoặc ít âm hơn; bé hơn: ít dương hơn hoặc âm hơn)
74
A. H > U B. H < U
C. H = U D. Không thể kết luận
Câu 3.22. Xác định hiệu ứng nhiệt H (kJ) của phản ứng CO (g) + 1/2O2
o

(g) → CO2 (g). Biết:


C (gr) +1/2O2 (g) → CO (g) Ho = -110,54 kJ
C (gr) + O2 (g) → CO2 (g) Ho = -393,51 kJ
A. - 504,05 B. - 282,97
C. + 282,97 D. - 307,29
Câu 3.23. Xác định hiệu ứng nhiệt H (kJ) của phản ứng C3H4 (g) + 2H2
o

(g) → C3H8 (g). Biết:


H2 (g) + 1/2O2 (g) → H2O (l) Ho = -285,8 kJ
C3H4 (g) + 4O2 (g) → 3CO2 (g) + 2H2O (l) Ho = -1937 kJ
C3H8 (g) + 5O2 (g) → 3CO2 (g) + 4H2O (l) Ho = -2219,1 kJ
A. - 3248,3 B. - 3870,3
C. - 4441,9 D. - 290
Câu 3.24. Cho các dữ liệu dưới đây:
1/2N2 (g) + 3/2H2 (g) → NH3 (g) Ho1
NH3 (g) + 5/4O2 (g) → NO (g) + 3/2H2O (l) Ho2
H2 (g) + 1/2O2 (g) → H2O (l) Ho3
Xác định Ho của phản ứng N2(g) + O2 (g) → 2NO (g) theo Ho1, Ho2
và Ho3?
A. 2Ho1 - 2Ho2 - 3Ho3 B. 2Ho1 + 2Ho2 - 3Ho3
C. 2Ho1 - 2Ho2 + 3Ho3 D. 2Ho1 + 2Ho2 + 3Ho3
Câu 3.25. Xác định hiệu ứng nhiệt Ho (kJ) của phản ứng:
C2H4 (g) + Cl2 (g) → C2H4Cl2 (l), Ho = ?
Biết:
4HCl (g) + O2 (g) → 2Cl2 (g) + 2H2O (l), Ho = -202,4 kJ
2HCl (g) + C2H4 (g) + 1/2O2 (g) → C2H4Cl2 (l) + H2O (l), Ho = -318,7 kJ
A. - 116,3 B. - 521,1
C. - 260,5 D. - 217,5
Câu 3.26. Xác định hiệu ứng nhiệt H (kJ) của phản ứng:
o

N2H4 (l) + 2H2O2 (l) → N2 (g) + 4H2O (l) Ho = ?


Biết:
N2H4 (l) + O2 (g) → N2 (g) + 2H2O (l) Ho = -622,2 kJ
H2 (g) + 1/2O2 (g) → H2O (l) Ho = -285,8 kJ
75
H2 (g) + O2 (g) → H2O2 (l) Ho = 187,8 kJ
A. - 1095,8 B. - 818,2
C. - 285,8 D. - 1569,4
Câu 3.27. Sử dụng các dữ liệu dưới đây để xác định hiệu ứng nhiệt Ho
(kJ) của phản ứng:
4CO (g) + 8H2 (g) → 3CH4 (g) + CO2 (g) + 2H2O (l) Ho = ?
C (than chì) + 1/2O2 (g) → CO (g) Ho = -110,5 kJ
CO (g) + 1/2O2 (g) → CO2 (g) Ho = -283,0 kJ
H2 (g) + 1/2O2 (g) → H2O (l) Ho = -285,8 kJ
C (gr) + 2H2 (g) → CH4 (g) Ho = -74,81 kJ
CH4 (g) + 2O2 (g) → CO2 (g) + 2H2O (l) Ho = -890,3 kJ
A. - 1410,5 B. - 532,3
C. - 747,5 D. - 754,1
Câu 3.28. Sử dụng các dữ liệu dưới đây để xác định hiệu ứng nhiệt Ho
(kJ) của phản ứng điều chế CCl4 (l):
CS2 (l) + 3Cl2 (g) → CCl4 (l) + S2Cl2 (l) Ho = ?
CS2 (l) + 3O2 (g) → CO2 (g) + 2SO2 (g) Ho = -1077 kJ
2S (s) + Cl2 (g) → S2Cl2 (l) Ho = -58,2 kJ
C (s) + 2Cl2 (g) → CCl4 (l) Ho = -135,4 kJ
S (s) + O2 (g) → SO2 (g) Ho = -296,8 kJ
SO2 (g) + Cl2 (g) → SO2Cl2 (l) Ho = +97,3 kJ
C (s) + O2 (g) → CO2 (g) Ho = -393,5 kJ
CCl4 (l) + O2 (g) → COCl2 (g) + Cl2O (g) Ho = -5,2 kJ
A. - 284 B. - 1960,9
C. - 580,3 D. - 148,1
Câu 3.29. Cho các dữ liệu dưới đây:
CH4 (g) + 2O2 (g) → CO2 (g) + 2H2O (g) Ho = -802 kJ
CH4 (g) + CO2 (g) → 2CO (g) + 2H2 (g) Ho = +247 kJ
CH4 (g) + H2O (g) → CO (g) + 3H2 (g) Ho = +206 kJ
Xác định Ho (kJ) của phản ứng: CH4 (g) + 1/2O2 (g) → CO (g) + 2H2 (g)
A. - 452 B. - 349
C. - 35,8 D. - 143
Câu 3.30. Nhiệt đốt cháy tiêu chuẩn của C4H6 (g), C4H10 (g) và H2 (g) lần
lượt là -2540,2; -2877,6 và -285,8 kJ/mol. Tính hiệu ứng nhiệt (kJ) của
phản ứng dưới đây:
76
C4H6 (g) + 2H2 (g) → C4H10 (g) Ho = ?
A. - 57 B. - 51,6 C. - 909 D. - 234,2
Câu 3.31. Cho các dữ liệu sau:
C6H12O6 (s) + 6O2 (g) → 6CO2 (g) + 6H2O (l), Ho = -2808 kJ
CH3CH(OH)COOH (s) + 3O2 (g) → 3CO2 (g) + 3H2O (l), Ho = -1344 kJ
Xác định hiệu ứng nhiệt (kJ) của phản ứng chuyển đổi 1 mol đường
glucose C6H12O6 (s) thành acid lactic theo phản ứng dưới đây.
C6H12O6 → 2CH3CH(OH)COOH (s) Ho = ?
A. - 120 B. - 1344
C. - 2952 D. - 5496
Câu 3.32. Xác định hiệu ứng nhiệt (kJ) của phản ứng dưới đây:
C3H8 (g) + H2 (g) → C2H6 (g) + CH4 (g), Ho = ?
Hof (kJ/mol) - 103,8 0 0 - 84,68 - 74,81
A. - 55,7 B. - 263,29
C. - 93,93 D. - 113,67
Câu 3.33. Xác định hiệu ứng nhiệt (kJ) của phản ứng dưới đây:
2H2S (g) + 3O2 (g) → 2SO2 (g) + 2H2O (l), Ho = ?
Hof (kJ/mol) - 20,63 0 0 - 296,8 - 285,8
A. - 561,97 B. - 858,77
C. - 233,54 D. – 1123,94
Câu 3.34. Xác định hiệu ứng nhiệt (kJ) của phản ứng dưới đây:
NH4+ (aqu) + HO- (aqu) → H2O (l) + NH3 (g), Ho = ?
Hof (kJ/mol) - 132,5 - 230,0 - 285,8 - 46,11
A. - 234,3 B. 30,6 C. - 30,59 D. 176,41
Câu 3.35. Xác định nhiệt tạo thành tiêu chuẩn (kJ/mol) của ZnS (s):
2ZnS (s) + 3O2 (g) → 2ZnO (s) + 2SO2 (g), Ho = - 878,2 kJ
Hof (kJ/mol) ? 0 - 348,3 - 296,8
A. - 206 B. - 233,1
C. 206 D. 233,1
Câu 3.36. Xác định nhiệt đốt cháy tiêu chuẩn (kJ/mol) của C2H5OH (l)
biết:
C2H5OH (l) + 3O2 (g) → 2CO2 (g) + 3H2O (l), Ho = ?
Hof (kJ/mol) - 277,7 0 - 393,5 - 285,8
A. - 1366,7 B. - 401,6
C. - 1188,6 D. - 973,2

77
Câu 3.37. Xác định nhiệt tạo thành tiêu chuẩn (kJ/mol) của CCl4 (g) biết
CH4 (g) + 4Cl2 (g) → CCl4 (g) + 4HCl (g), Ho = - 397,3 kJ
Hof (kJ/mol) - 74,81 0 ? -92,31
A. - 167,12 B. - 294,4
C. - 102,9 D. - 691,7
Câu 3.38. Xác định nhiệt tạo thành tiêu chuẩn (kJ/mol) của hexane C6H14
(l) biết:
2C6H14(l) +19O2(k) → 12CO2(k) + 14H2O(l), Ho = - 8326 kJ
Hof (kJ/mol) ? 0 -393,5 -285,8
A. - 398 B. - 199
C. - 8524,5 D. - 8723
Câu 3.39. Xác định hiệu ứng nhiệt (kJ) của phản ứng dưới đây:
Al3+(aqu) + 3HO-(aqu) → Al(OH)3(aqu), Ho = ?
Hof (kJ/mol) -531 -230,0 -1276
A. - 1577 B. - 1117
C. - 515 D. - 55
Câu 3.40. Xác định hiệu ứng nhiệt (kJ) của phản ứng dưới đây:
Mg(OH)2(s) + 2NH4+(aqu) → Mg2+(aqu) + 2H2O(l) + 2NH3(g)
Hof (kJ/mol) -924,5 -132,5 -466,9 -285,8 -46,11
A. 58,8 B. - 1590,81
C. 265 D. - 1189,5
Câu 3.41. Phân hủy CaCO3 (s) thành CaO (s) và CO2 (g). Lượng nhiệt
Ho (kJ) cần để phân hủy 1,35.103 kg CaCO3 (s) là bao nhiêu biết Hof
[CaCO3 (s)] = - 1207 kJ/mol, Hof [CaO (s)] = - 635,1 kJ/mol và Hof [CO2
(g)] = - 393,5 kJ/mol
A. 178 B. 2,4.106
C. 13483,14 D. 1,35.106
Câu 3.42. Xác định nhiệt tạo thành tiêu chuẩn của HCOOH (s) biết:
HCOOH (s) + 1/2O2(k) → CO2(k) + H2O(l), Ho = - 255 kJ
Hof (kJ/mol) ? 0 - 393,5 - 285,8
A. - 424 B. - 679,3
C. - 107,7 D. - 934,3
Câu 3.43. Từ các dữ liệu dưới đây, xác định hiệu ứng nhiệt (kJ) của phản
ứng:
C2H4(k) + H2(k) → C2H6(k)

78
Hoc (kJ/mol) - 1411 - 285,8 - 1560
A. - 3256,8 B. - 1696,8
C. - 136,8 D. - 2971
Câu 3.44. Xác định hiệu ứng nhiệt (kJ) của phản ứng dưới đây:
3C(gr) + 4H2(k) → C3H8(k)
H c (kJ/mol) - 394 - 285,8
o
-2219
A. - 1539,2 B. - 679,8
C. - 2613 D. - 106,2
Câu 3.45. Xác định hiệu ứng nhiệt (kJ) của phản ứng dưới đây:
2C(gr) +3H2(k) + 1/2O2(k) → CH3CH2OH(l)
H c (kJ/mol) - 394 - 285,8
o
0 - 1367
A. 687,2 B. 278,4
C. - 1645,4 D. - 509,6
Câu 3.46. Khẳng định nào sau đây là đúng đối với sự thăng hoa của nước
đá khô (CO2 rắn)?
A. ΔH > 0, ΔS > 0 và ΔG dương ở nhiệt độ thấp và âm ở nhiệt độ cao
B. ΔH < 0, ΔS < 0 và ΔG âm ở nhiệt độ thấp và dương ở nhiệt độ cao
C. ΔH < 0, ΔS > 0 và ΔG âm ở mọi nhiệt độ
D. ΔH > 0, ΔS < 0 và ΔG dương ở mọi nhiệt độ
Câu 3.47. Sắp xếp các khí sau theo thứ tự tăng entropy mol tiêu chuẩn:
SO3, Kr, Cl2.
A. Kr < Cl2 < SO3 B. Kr < SO3 < Cl2
C. SO3 < Cl2 < Kr C. Cl2 < Kr < SO3
Câu 3.48. Chất nào sau đây có entropy mol tiêu chuẩn lớn nhất?
A. NH3 (g) B. H2O (l)
C. He (g) D. C (s, graphite)
Câu 3.49. Chọn lượng chất có entropy cao nhất ở 298 K.
A. 0,5 g HCN B. 1 mol HCN C. 2 kg HCN D. 2 mol HCN
Câu 3.50. Tính ΔS cho phản ứng sau từ các dữ kiện entropy mol tiêu
o

chuẩn:
N2O4(g) → 2NO2(g)
o
S (J/mol.K) 304,3 240,45
A. + 176,7 J/K B. – 50,7 J/K
C. – 176,7 J/K D. + 63,8 J/K
Câu 3.51. Chất nào sau đây có entropy mol tiêu chuẩn nhỏ nhất?
A. CH4 (g) B. Na (s)
79
C. He (g) D. H2O (s)
Câu 3.52. Tính ΔS cho phản ứng sau từ các dữ kiện entropy mol tiêu
o

chuẩn:
H2(g) + I2 (g) → 2 HI(g)
o
S (J/mol.K) 130,58 116,73 206,3
A. - 41.10 J/K B. + 165.29 J/K
C. - 165.29 J/K D. + 41.10 J/K
Câu 3.53. Tra bảng số liệu và tính ΔS° (J/K) của phản ứng đốt cháy ethene
bởi oxy dư tạo ra carbon dioxyde và nước theo phản ứng hóa học sau:
C2H4(g) + 3O2(g) → 2CO2(g) + 2H2O(l)
A. - 267,4 B. - 140,9
C. - 347,6 D. + 347,6
Câu 3.54. Tra bảng số liệu và tính ΔS° (J/K) của phản ứng sau:
2SO3 (g) → 2S (s, rhombic) + 3O2 (g)
A. +19,3 B. + 493,1
C. + 166,4 D. - 493,1
Câu 3.55. Tra bảng số liệu và tính ΔS° (J/K ) của phản ứng hydro hóa xúc
tác của acetylene thành ethane: C2H2 (g) + 2H2 (g) → C2H6 (g)
A. - 76,0 B. + 440,9
C. - 232,5 D. + 232,5
Câu 3.56. Khí NO được chuyển hóa thành khí NO2 theo phản ứng sau:
NO (g) + 1/2O2 (g) → NO2 (g)
o
S (J/mol.K) 210,7 205,0 240,0
Tính ΔS khi 0,5 mol khí NO phản ứng với 0,5 mol khí O2 ở điều kiện tiêu
o

chuẩn.
A. - 36,6 J / K B. 83,4 J/K
C. 36,6 J /K D. - 83,4 J / K
Câu 3.57. Tính ΔS khi 3,50 g Ni (M = 58,69 g/mol) phản ứng với oxy dư
o

để tạo thành niken oxyt (NiO) trong điều kiện tiêu chuẩn theo phản ứng:
2Ni (s) + O2 (g) → 2NiO (s)
o
S (J/mol.K) 182,1 205,0 37,99
A. - 49,3 J/K B. + 49,3 J/K
C. - 10,4 J/K D. - 14,7 J/K
Câu 3.58. Tính biến thiên entropy khi 10,0 g methane phản ứng với 10,0
g oxy theo phản ứng sau:
CH4(g) + 2O2(g) → CO2(g) + 2H2O(l)

80
So (J/mol.K) 186,2 205,0 70,0 213,6
A. - 121 J/K B. - 37,9 J/K
C. - 16,8 J/K D. - 242,6 J/K
Câu 3.59. Nhiệt nóng chảy của benzene (C6H6, M = 78,0 g/mol) là 127,40
kJ/kg và nhiệt độ nóng chảy của chất này là 5,5 °C. Biến thiên entropy khi
1 mol benzen nóng chảy ở 5,5 °C là bao nhiêu?
A. 9,95 kJ/K B. 1,81 J/K
C. 35,7 J/K D. 1809 J/K
Câu 3.60. Methanol có nhiệt độ sôi là 64,7 °C và nhiệt hóa hơi tại nhiệt độ
sôi này là 71,8 kJ/mol. Tính giá trị ΔS (J/K) khi 2,15 mol CH3OH (l) sôi ở
64,7°C.
A. 0,457 B. 457
3
C. 2,39.10 D. 2,39
Câu 3.61. Xét một tinh thể chất rắn tinh khiết được làm nóng từ không độ
tuyệt đối đến trên nhiệt độ sôi. Quá trình nào sau đây sẽ kèm theo cho sự
gia tăng entropy lớn nhất của chất?
A. làm tan chảy chất rắn B. làm nóng chất lỏng
C. làm nóng khí D. làm sôi chất lỏng
Câu 3.62. Phản ứng nào sau đây có ∆S dương?
A. AgNO3(aqu) + NaCl(aqu) → AgCl(s) + NaNO3(aqu)
B. H2(g) + I2(g) →2HI(g)
C. C2H2O2 (g) → 2 CO(g) + H2(g)
D. H2O(g) → H2O(l)
Câu 3.63. Phản ứng nào sau đây có ∆S âm?
A. H2O (g) → H2O (s)
B. CaCO3 → CaO + CO2 (g)
C. CuSO4.5H2O (s) → CuSO4 (s) + 5H2O (g)
D. CO2(aqu) → CO2(g)
Câu 3.64. Quá trình nào kèm theo sự tăng entropy của hệ?
A. Tinh thể natri clorua hình thành từ nước muối bay hơi.
B. Khí heli thoát ra khỏi lỗ trên bóng bay.
C. Thạch nhũ hình thành trong hang động.
D. Tất cả những điều này dẫn đến sự tăng etropy của hệ, vì tất cả chúng
đều tự phát.
Câu 3.65. Quá trình nào kèm theo sự giảm entropy của hệ?
A. Tinh thể muối hòa tan trong nước.

81
B. Không khí thoát ra từ một lỗ trên bóng bay.
C. Sắt và oxy phản ứng tạo thành rỉ sét.
D. Không có điều nào trong số này dẫn đến sự giảm entropy của hệ, vì tất
cả chúng đều tự phát.
Câu 3.66. Khi khí argon được đặt trong một thùng chứa khí neon thì argon
tự phân tán khắp khối neon là do:
A. lực hấp dẫn lớn giữa các nguyên tử argon và neon
B. liên kết hydro
C. năng lượng của hệ giảm khi các khí trộn vào với nhau
D. sự phân tán của các nguyên tử argon tạo ra sự gia tăng độ mất trật tự
Câu 3.67. Quá trình nào sau đây dẫn đến sự giảm entropy của hệ?
A. Hòa tan natri clorua trong nước B. Thăng hoa naphthalene
C. Hòa tan oxy trong nước D. Đun sôi rượu
Câu 3.68. Phản ứng nào sau đây có ΔS dương?
A. 2NO2 (g) → N2O4 (g)
B. CO2 (g) → CO2 (s)
C. BaF2 (s) → Ba2+ (aqu) + 2F- (aqu)
D. 2Hg (l) + O2 (g) → 2HgO (s)
Câu 3.69. Trong những chất khí sau đây, chất nào có entropy lớn nhất ở
25 °C và 1 atm?
A. H2 B. C2H6
C. C2H2 D. CH4
Câu 3.70. Đại lượng nhiệt động học nào đặc trưng mức độ mất trật tự trong
một hệ thống?
A. enthalpy B. nội năng
C. entropy D. năng lượng liên kết
Câu 3.71. Tính ∆G cho phản ứng sau:
o

CH4(g) + 2O2(g) → CO2(g) + 2H2O(g)


∆G f (kJ/mol) - 50.75
o
0 - 394.4 - 228.57
A. - 50,8 kJ B. - 115 kJ
C. - 751 kJ D. - 801 kJ
Câu 3.72. Hydro phản ứng với nitơ tạo thành amoniac (NH3) theo phản
ứng: 3H2 (g) + N2 (g) → 2NH3(g). Phản ứng này có ∆H° = - 92,38 kJ và
∆S° = - 198,2 J/K. Xác định ∆G° ở 25°C.
A. + 5.897.104 kJ B. - 16,66 kJ
C. + 297,8 kJ D. - 33,32 kJ

82
Câu 3.73. Axit clohydric (HCl) phản ứng với natri hydroxyt (NaOH) tạo
thành natri clorua (NaCl) và nước. Biết ∆H° = - 56,13 kJ và ∆S° = 79,11
J/K, xác định ∆G° cho phản ứng này ở 20°C.
A. - 79,31 kJ B. 79,31 kJ
C. - 77,73 kJ D. - 1638 kJ
Câu 3.74. Hãy tra cứu số liệu cần thiết và tính ΔG° (kJ) ở 25 oC của phản
ứng sau: 2SO3 (g) → 2S (s, rhombic) + 3O2 (g)
A. + 740,8 B. - 370,4
C. + 370,4 D. + 185,2
Câu 3.75. Hãy tra cứu số liệu cần thiết và tính ΔG° (kJ) ở 25 oC của phản
ứng sau: 2S (s, rhombic) + 3O2 (g) → 2SO3 (g)
A. + 740,8 B. - 370,4
C. + 370,4 D. - 740,8
Câu 3.76. Tính ΔG° (kJ) ở 141 °C của phản ứng sau: P2 (g) + 3Cl2 (g) →
2PCl3 (g). Biết rằng tại 25 oC, phản ứng này có ΔH° = - 720,5 kJ và ΔS° =
- 263,7 J/K. Giả sử rằng ΔH và ΔS không thay đổi đáng kể theo nhiệt độ.
A. - 611,3 B. 3,65.104
C. 1,08.105 D. - 683,3
Câu 3.77. Tính ΔG° (kJ) ở 100 °C của phản ứng sau:
Ca (s) + Cl2 (g) → CaCl2 (s)
Biết rằng tại 25 C, phản ứng này có ΔH° = - 795,8 kJ, ΔG° = - 748,1 kJ
o

và ΔS° = -159,8 J / K. Giả sử rằng ΔH và ΔS không đáng kể thay đổi theo


nhiệt độ.
A. - 855,4 B. - 736,2
. 4
C. 5,88 10 D. - 779,8
Câu 3.78. Chất nào sau đây có năng lượng tự do tạo thành tiêu chuẩn Gof
bằng 0?
(a) H2O (l); (b) O (g); (c) H2 (g)
A. (a) B. (c)
C. (b) và (c) D. (a), (b) và (c)
Câu 3.79. Chất nào sau đây có năng lượng tự do tạo thành tiêu chuẩn Gof
bằng 0?
(a) H2O(l); (b) Na(s); (c) H2(g)
A. (b) B. (c)
C. (b) và (c) D. (a), (b) và (c)

83
Câu 3.80. Biến thiên enthalpy và biến thiên entropy của quá trình bay hơi
một chất lỏng lần lượt là 38,6 kJ/mol và 109,8 J.mol-1.K-1. Tính nhiệt độ
sôi của chất lỏng này, giả sử rằng ΔH và ΔS không thay đổi theo nhiệt độ.
A. 352 °C B. 78,5 °C
C. 2,84 °C D. 624 °C
Câu 3.81. Phản ứng C + H2O (g) → CO (g) + H2 (g) có ΔH° = 131,3 kJ
và ΔS° = 133,6 J/K ở 298 K. Ở điều kiện tiêu chuẩn, phản ứng này là tự
phát ở nhiệt độ lớn hơn bao nhiêu °C?
A. 273 B. 325
C. 552 D. 710
Câu 3.82. Xét phản ứng: NH3 (g) + HCl (g) → NH4Cl (s). Sử dụng bảng
dữ liệu nhiệt động lực học, xác định nhiệt độ (°C) để phản ứng không xảy
ra tự phát.
A. Phản ứng này là tự phát ở mọi nhiệt độ. B. 618,1
C. 432,8 D. 345,1
Câu 3.83. Xét phản ứng: NH3 (g) + HCl (g) → NH4Cl (s). Sử dụng bảng
dữ liệu nhiệt động lực học, xác định nhiệt độ (°C) để phản ứng không xảy
ra tự phát trong điều kiện tiêu chuẩn.
A. 618,1
B. 432,8
C. 345,1
D. Phản ứng này là tự phát ở tất cả các nhiệt độ.
Câu 3.84. Để phản ứng tự phát trong điều kiện tiêu chuẩn ở tất cả các nhiệt
độ thì đại lượng ΔH° và ΔS° có dấu như thế nào?
A. ΔH° > 0 , ΔS° > 0 B. ΔH° > 0 , ΔS° < 0
C. ΔH° < 0 , ΔS° > 0 D. ΔH° < 0 , ΔS° < 0
Câu 3.85. Khi hai chất lỏng hữu cơ không phân cực (ví dụ hexane và
heptane) tự trộn lẫn với nhau thành dung dịch đồng nhất, hiệu ứng nhiệt
của quá trình này là rất nhỏ. Sự tạo thành dung dịch tự xảy ra được chủ
yếu là do:
A. sự hydrate hóa chất tan
B. biến thiên enthalpy của quá trình rất âm
C. sự solvat hóa dung môi
D. sự tăng entropy của hệ
Câu 3.86. Quá trình hòa tan amoni nitrat trong nước là một quá trình thu
nhiệt và tự phát. Quá trình này xảy ra tự phát là do:

84
A. giảm enthalpy của hệ
B. giảm entropy của hệ
C. tăng entropy của hệ
D. tăng năng lượng tự do của hệ
Câu 3.87. Một phản ứng không phải xảy ra tự phát ở nhiệt độ thấp nhưng
trở nên tự phát ở nhiệt độ cao. Dấu của ∆H° và ∆S° lần lượt là gì?
A. ∆H° > 0 và ∆S < 0 B. ∆H° < 0 và ∆S < 0
C. ∆H° < 0 và ∆S > 0 D. ∆H° > 0 và ∆S > 0
Câu 3.88. Một phản ứng xảy ra tự phát là phản ứng:
A. xảy ra rất nhanh
B. sẽ xảy ra mà không cần sự can thiệp từ bên ngoài
C. có hằng số cân bằng rất nhỏ
D. xảy ra rất chậm
Câu 3.89. Phản ứng: 2K (s) + 1/2O2 (g) → K2O (s) là phản ứng tỏa nhiệt
mạnh. Các đại lượng nhiệt động Ho, So, Go của phản ứng này có dấu
như thế nào?
A. Ho < 0; So < 0 ; Go < 0
B. Ho > 0; So > 0 ; Go > 0
C. Ho < 0; So > 0 ; Go > 0
D. Ho > 0; So > 0 ; Go < 0
Câu 3.90. Cho các phản ứng sau:
C2H2 (g) + 2H2 (g) → C2H6 (g) (1)
N2 (g) + O2 (g) → 2NO (g) (2)
KClO4 (s) → KCl (s) + 2O2 (g) (3)
Phản ứng có S lớn nhất và S nhỏ nhất lần lượt là:
A. 1, 2 B. 2, 3
C. 1, 3 D. 3, 1
---oOo---

85
ĐÁP ÁN BÀI TẬP CHƯƠNG 3

3.1- 3.2- 3.3- 3.4- 3.5- 3.6- 3.7- 3.8- 3.9- 3.10-
B A C D A B A A B D
3.11- 3.12- 3.13- 3.14- 3.15- 3.16- 3.17- 3.18- 3.19- 3.20-
C C A C A A B B C B
3.21- 3.22- 3.23- 3.24- 3.25- 3.26- 3.27- 3.28- 3.29- 3.30-
C B D B D B C A C D
3.31- 3.32- 3.33- 3.34- 3.35- 3.36- 3.37- 3.38- 3.39- 3.40-
A A D B A A C B D A
3.41- 3.42- 3.43- 3.44- 3.45- 3.46- 3.47- 3.48- 3.49- 3.50-
B A C D B A A A C D
3.51- 3.52- 3.53- 3.54- 3.55- 3.56- 3.57- 3.58- 3.59- 3.60-
B B A C C A D B C B
3.61- 3.62- 3.63- 3.64- 3.65- 3.66- 3.67- 3.68- 3.69- 3.70-
D C A B C D C C B C
3.71- 3.72- 3.73- 3.74- 3.75- 3.76- 3.77- 3.78- 3.79- 3.80-
D D A A D A B B C B
3.81- 3.82- 3.83- 3.84- 3.85- 3.86- 3.87- 3.88- 3.89- 3.90-
D D C C D C D B A D

86
Chương 4. ĐỘNG HÓA HỌC
---oOo---

Mục tiêu chương 4


Sau khi học xong chương này, sinh viên có khả năng:
1. Định nghĩa được các khái niệm cơ bản của động hóa học.
2. Định nghĩa và viết được biểu thức tính được tốc độ phản ứng trung
bình và tốc độ tức thời.
3. Tính toán được tốc độ phản ứng, bậc phản ứng, hằng số tốc độ phản
ứng, thời gian bán hủy, năng lượng hoạt hóa.
4. Phân tích được các yếu tố ảnh hưởng đến tốc độ phản ứng như nồng
độ, áp suất, nhiệt độ, xúc tác.

87
Phần 1. TÓM TẮT LÝ THUYẾT
4.1 MỘT SỐ KHÁI NIỆM CƠ BẢN
Phản ứng đơn giản là phản ứng xảy ra qua một giai đoạn không có tạo
thành sản phẩm trung gian.
Ví dụ, phản ứng đơn giản NO + O3 → NO2 + O2 chỉ xảy ra một giai đoạn
và không có sản phẩm trung gian.
Phản ứng phức tạp là phản ứng xảy ra qua nhiều giai đoạn. Trong phản
ứng phức tạp, giai đoạn chậm là giai đoạn quyết định vận tốc phản ứng.
Ví dụ, phản ứng N2O5 → 4NO2 + O2 xảy ra qua hai giai đoạn như sau:
• Giai đoạn 1: N2O5 → N2O3 + O2
• Giai đoạn 2: N2O3 + N2O5 → 4NO2
Phản ứng đồng thể là phản ứng xảy ra trong hệ đồng thể (các chất tham
gia và sản phẩm phản ứng cùng pha).
Ví dụ, phản ứng NaOH (aq) + HCl (aq) → NaCl (aq) + H2O (l) là phản
ứng đồng thể.
Phản ứng dị thể là phản ứng xảy ra trong hệ dị thể (các chất tham gia và
sản phẩm phản ứng khác pha).
Ví dụ, phản ứng Fe (s) + HCl (aq) → FeCl2 (aq) + H2(g) là phản ứng
dị thể.
Phân tử số của một giai đoạn phản ứng là số tiểu phân tham gia trong giai
đoạn đó.
Ví dụ:
• I2 → 2I Phân tử số = 1, phản ứng đơn phân tử
• 2HI → H2 + I2 Phân tử số = 2, phản ứng lưỡng phân tử
• NO + O3 → NO2 + O2 Phân tử số = 2, phản ứng lưỡng phân tử
Tốc độ phản ứng là đại lượng đặc trưng cho diễn biến nhanh hay chậm
của một phản ứng hóa học. Tốc độ phản ứng thường được đo bằng biến
thiên nồng độ của chất tham gia hoặc sản phẩm phản ứng trong một đơn
vị thời gian.
Đơn vị của tốc độ phản ứng: mol.L-1.s-1, mol.L-1.phút-1…
Xét phản tổng quát: aA + bB → cC + dD
Tốc độ trung bình của phản ứng:
1 Δ[A] 1 Δ[B] 1 Δ[C] 1 Δ[D] (4.1)
vtb = − =− = =
a Δt b Δt c Δt d Δt
Tốc độ tức thời của phản ứng:

88
1 d[A] 1 d[B] 1 d[C] 1 d[D] (4.2)
v=− =− = =
a dt b dt c dt d dt
Lưu ý: Để đơn giản, thường chọn một tác chất đại diện để khảo sát tốc độ
phản ứng đã cho (ví dụ chất A). Tốc độ tức thời của phản ứng lúc này được
đo bằng vi phân nồng độ A theo thời gian:
1 d[A] (4.3)
v=−
a dt
4.2 BIỂU THỨC TỐC ĐỘ PHẢN ỨNG
4.2.1 Biểu thức tốc độ phản ứng
Xét phản ứng đồng thể tổng quát: aA + bB → cC + dD
v = k.[A]m[B]n (4.4)
Trong đó:
v: vận tốc tức thời của phản ứng tại thời điểm khảo sát
k: hằng số tốc độ phản ứng
m, n: bậc phản ứng theo A, B
• phản ứng đơn giản m = a, n = b
• phản ứng phức tạp m  a, n  b
m + n: bậc tổng cộng hay bậc toàn phần của phản ứng
Lưu ý:
• Bậc phản ứng có thể nhận mọi giá trị ≥ 0 và thường được xác định
bằng thực nghiệm.
• Đối với phản ứng dị thể thì chất rắn không có mặt trong biểu thức
tốc độ phản ứng.
4.2.2 Biểu thức hằng số tốc độ phản ứng k
Hằng số tốc độ phản ứng k là đại lượng phụ thuộc vào bản chất phản ứng
và nhiệt độ phản ứng.
• Đối với phản ứng bậc 0 có dạng: A → sản phẩm
1
k = ሺCo − Cሻ (4.5)
t
• Đối với phản ứng bậc 1 có dạng: A → sản phẩm
1 Co
k = ln (4.6)
t C
• Đối với phản ứng bậc 2 có dạng: A + B → sản phẩm
1 1 1
✓ Nếu [A]o = [B]o → k = t ሺC − C ሻ (4.7)
o
1 1 aሺb−xሻ (4.8)
✓ Nếu [A]o ≠ [B]o → k = t [b−a ln bሺa−xሻ]

89
(với [A]o = a, [B]o = b, b > a)
4.2.3 Thời gian bán hủy t1/2
Thời gian bán hủy t½ là thời gian cần thiết để phản ứng hết 1/2 lượng tác
Co
chất ban đầu. Như vậy lượng tác chất còn lại C = .
2
Co
Khi thay C = vào các công thức trên sẽ tính được thời gian bán hủy của
2
phản ứng.
• Đối với phản ứng bậc 0 có dạng: A → sản phẩm
Co (4.9)
t1/2 =
2k
• Đối với phản ứng bậc 1 có dạng: A → sản phẩm
ln2 (4.10)
t1/2 =
k
• Đối với phản ứng bậc 2 có dạng: A + B → sản phẩm,
với [A]o = [B]o
1 (4.11)
t1/2 =
kCo
4.3 CÁC YẾU TỐ ẢNH HƯỞNG ĐẾN TỐC ĐỘ PHẢN ỨNG
4.3.1 Ảnh hưởng của nồng độ
Hầu hết các phản ứng có tốc độ tăng khi tăng nồng độ tác chất tham gia
phản ứng.
Định luật tác dụng khối lượng: Trong hệ đồng thể, ở nhiệt độ không
đổi, tốc độ phản ứng tỷ lệ với tích số nồng độ các chất tham gia
phản ứng với số mũ bằng hệ số hợp thức của chúng trong phương trình
phản ứng.
Lưu ý: Định luật tác dụng khối lượng chỉ đúng với phản ứng đồng thể
đơn giản.
Ví dụ, xét phản ứng đồng thể đơn giản aA + bB → cC + dD thì biểu thức
tốc độ phản ứng là v = k[A]a [B]b.
4.3.2 Ảnh hưởng của nhiệt độ
Hầu hết các phản ứng có tốc độ tăng khi tăng nhiệt độ.
a) Quy tắc thực nghiệm Van’t Hoff khi tăng nhiệt độ phản ứng lên
10 oC, tốc độ phản ứng tăng từ 2 đến 4 lần, số lần tăng này được gọi là hệ
số nhiệt độ.

90
v2 T2−T1 (4.12)
= γ 10
v1
Trong đó:
 : hệ số nhiệt độ
v1: tốc độ phản ứng ở nhiệt độ T1
v2: tốc độ phản ứng ở nhiệt độ T2
b) Phương trình thực nghiệm Arrhenius
−Ea (4.13)
lnk = + lnA
RT
hoặc
− Ea (4.14)
k = Ae RT

Trong đó:
Ea: năng lượng hoạt hóa (J/mol, cal/mol)
A: thừa số Arrhenius
T: nhiệt độ tuyệt đối, K
R: hằng số khí
✓ Số lần tăng tốc độ phản ứng khi tăng nhiệt độ từ T1 đến T2
Ở nhiệt độ T1:
−Ea (4.15)
ln𝑘1 = + lnA
RT1
Ở nhiệt độ T2:
−Ea (4.16)
ln𝑘2 = + lnA
RT2
Từ (4.15) và (4.16), ta có:
k2 Ea 1 1 (4.17)
= e R ሺT1 − T2ሻ
k1
4.3.3 Ảnh hưởng của chất xúc tác
Chất xúc tác là những chất khi thêm vào phản ứng sẽ làm tăng tốc độ
phản ứng hoặc gây nên phản ứng (nếu về mặt nguyên tắc phản ứng có thể
xảy ra).
Ví dụ, phản ứng tổng hợp H2O từ O2 và H2 ở điều kiện nhiệt độ bình thường
không xảy ra nhưng khi cho thêm một lượng nhỏ Pt thì phản ứng xảy ra
tức thời.
Pt
H2 + O2 → H2O
Ngược lại, có những chất khi thêm vào phản ứng làm cho tốc độ phản ứng
chậm lại được gọi là chất ức chế.
91
Tác dụng chủ yếu của chất xúc tác là làm giảm năng lượng hoạt hóa của
phản ứng bằng cách thay đổi cơ chế phản ứng từ đó làm tăng tốc phản ứng.
Theo (4.14), khi năng lượng hoạt hóa Ea giảm thì k tăng, dẫn đến tốc độ
phản ứng ứng tăng. Ngược lại, khi năng lượng hoạt hóa Ea tăng thì k giảm,
dẫn đến tốc độ phản ứng ứng giảm.
Các đặc tính của xúc tác:
✓ Lượng xúc tác được dùng với một lượng rất nhỏ so với tác chất.
✓ Chất xúc tác được bảo toàn sau phản ứng.
✓ Chất xúc tác có tính chọn lọc cao.

92
Phần 2. BÀI TẬP CÓ LỜI GIẢI
---oOo---
Dạng 1. Biểu thức tốc độ phản ứng và bậc phản ứng
Câu 4.1. Cho phản ứng: 2Fe3+ (aq) + Sn2+ (aq) → 2Fe2+ (aq) + Sn4+ (aq)
Biết nồng độ Fe2+ sau 38,5 giây là 0,0010 M. Xác định tốc độ tạo thành và
mất đi trung bình của sản phẩm và các chất tham gia phản ứng trong
khoảng thời gian trên.
Giải
Tốc độ tạo thành trung bình của Fe2+:
Δ[Fe2+ ] 0,0010
vFe2+ = = = 2,6.10-5 M.s-1
Δt 38,5
Tốc độ tạo thành trung bình của Sn4+:
Δ[Sn4+ ] 0,0005
vSn4+ = = = 1,3.10-5 M.s-1
Δt 38,5
Tốc độ mất đi trung bình của Fe3+:
Δ[Fe3+ ] −0,0010
vFe3+ = − =− = 2,6.10-5 M.s-1
Δt 38,5
Tốc độ mất đi trung bình của Sn2+:
Δ[Sn2+ ] −0,0005
vSn2+ = − =− = 1,3.10-5 M.s-1
Δt 38,5
Tốc độ trung bình của phản ứng:
1 Δ[Fe3+ ] Δ[Sn2+ ] 1 Δ[Fe2+ ] Δ[Sn4+ ]
vtb = − 2 =− =2 = = 1,3.10-5 M.s-1
Δt Δt Δt Δt
Câu 4.2. Trong bình kín dung tích 5 L, ở điều kiện nhiệt độ không đổi,
người ta cho vào 8 mol khí A và 5 mol khí B để xảy ra phản ứng đơn giản
sau: 2A (g) + B (g) → C (g) + 2D (g). Tính vận tốc ban đầu của phản ứng
và tính vận tốc tại thời điểm đã có 40% khí A tham gia phản ứng. Cho biết
hằng số tốc độ phản ứng bằng 0,4 M-2.s-1.
Giải
Biểu thức tốc độ phản ứng v = k.[A]m.[B]n.
Vì phản ứng là đơn giản nên m = 2, n = 1.
→ v = k.[A]2.[B]
Tại thời điểm ban đầu nồng độ A và B lần lượt là:
nA 8
[A]0 = = = 1,6 ሺMሻ
V 5
nB 5
[B]0 = = = 1 ሺMሻ
V 5
Vận tốc ban đầu v0 = k.[A]o2.[B]o = 1,024 (M.s-1)
Tại thời điểm 40% A đã phản ứng:

93
2A(g) + B(g) = C(g) + 2D(g)
Nồng độ ban đầu 1,6 1
Nồng độ phản ứng 0,64 0,32
Nồng độ còn lại 0,96 0,68
Vận tốc vt = k.[A] .[B] = 0,25 (M.s-1)
2

Câu 4.3. Nghiên cứu trên phản ứng 2NO(g) + Cl2(g) → 2NOCl(g) cho
thấy:
(1) Nếu giữ nguyên nồng độ NO, tốc độ phản ứng tăng gấp đôi khi nồng
độ Cl2 tăng gấp đôi.
(2) Nếu giữ nguyên nồng độ Cl2, tốc độ phản ứng tăng gấp 4 lần khi nồng
độ NO tăng gấp đôi.
Hãy viết biểu thức tốc độ và xác định bậc toàn phần của phản ứng.
Giải
Biểu thức tốc độ của phản ứng là: v = k[NO]m[Cl2]n
Từ (1): v’ = k[NO]m(2[Cl2])n = 2v → n = 1
Từ (2): v” = k(2[NO])m[Cl2]n = 4v → m = 2
Vậy biểu thức tốc độ v = k[NO]2[Cl2].
Bậc toàn phần của phản ứng bằng 3.
Câu 4.4.
Viết biểu thức tốc độ, xác định bậc phản ứng và hằng số tốc độ của phản
ứng: 2NO (g) + O2 (g) → 2 NO2 (g) từ những dữ liệu thực nghiệm trong
bảng sau:
Thí nghiệm [NO], M [O2], M V, M.s-1
1 1,0. 10-4 1,0. 10-4 2,8.10-6
2 1,0. 10-4 3,0. 10-4 8,4.10-6
3 2,0. 10-4 3,0. 10-4 3,4.10-5
Giải
Biểu thức tốc độ phản ứng tổng quát có dạng: v = k [NO]m[O2]n
Thay nồng độ và vận tốc phản ứng tương ứng trong từng thí nghiệm vào
phương trình vận tốc tổng quát, ta có:
Thí nghiệm 1: 2,8.10-6 = k (1,0.10-4)m (1,0.10-4)n (1)
Thí nghiệm 2: 8,4.10-6 = k (1,0.10-4)m (3,0.10-4)n (2)
Thí nghiệm 3: 3,4.10-5 = k (2,0.10-4)m (3,0.10-4)n (3)
Lấy (2) chia (1) → n = 1
Lấy (3) chia (2) → m = 2
94
→ v = 2,8.106 [NO]2 [O2]
→ k = 2,8.106 (L2.mol-2.s-1)
Dạng 2. Nồng độ, thời gian và hằng số tốc độ phản ứng
Câu 4.5. Cho phản ứng A → B là phản ứng bậc 1 với dữ liệu thực nghiệm
nồng độ chất A theo thời gian như sau:
Thời gian (phút) Nồng độ A còn lại (M)
0 1,50
5 1,24
Xác định:
a) Hằng số tốc độ phản ứng.
b) Chu kỳ bán hủy của phản ứng.
c) Nồng độ A còn lại sau 15 phút phản ứng.
Giải
a) Đối với phản ứng là bậc một có dạng A → B:
1 Co 1 1,5
k = t ሺln ) = 5 ሺln 1,24) = 0,038 phút-1
C
ln2
b) Chu kỳ bán hủy t1/2 = = 18,2 phút
k
c) Nồng độ A còn lại sau 15 phút phản ứng:
1 Co 1 1,5
k = ሺln )= ሺln ) = 0,83 M
t C 15 C
Dạng 3. Sự ảnh hưởng của nhiệt độ và xúc tác đến tốc độ phản ứng
Câu 4.6. Cho một phản ứng có năng lượng hoạt hóa ban đầu là 167,2
kJ/mol được tiến hành ở 500 K. Khi đưa xúc tác vào hệ và giữ nguyên các
điều kiện phản ứng năng lượng hoạt hóa giảm còn 125,4 kJ/mol. Tốc độ
phản ứng thay đổi như thế nào?
Giải
Từ phương trình Arrhenius:
−Ea1
lnk1 = + lnA
RT
−Ea2
lnk 2 = + lnA
RT
k E −E 167,2 − 125,4
→ ln k2 = a1RT a2 = 8,314.10−3.500
1
𝑘2
Tính toán thu được 𝑘 = 23279.
1
Như vậy tốc độ phản ứng tăng lên 23.279 lần.

95
Câu 4.7. Một phản ứng bậc 2 có năng lượng hoạt hóa bằng 85 kJ/mol tại
500 oC, có hằng số tốc độ 5,0.10-3 M-1.s-1. Tính hằng số tốc độ của phản
ứng tại nhiệt độ 800 oC.
Giải
−Ea1 −Ea2
Từ phương trình Arrhenius lnk1 = + lnA và lnk 2 = + lnA
RT RT
k Ea 1 1
→ ln k2 = ሺT − ሻ
1 R 1 T2
k 85.103 1 1
→ ln 5,0.10
2
−3
= ሺ500+273 − ሻ
8,314 800+273
→ k2 = 0,2 M-1.s-1
Câu 4.8. Cho một phản ứng có vận tốc tăng lên 15,6 lần khi tăng nhiệt độ
thêm 30 oC. Hệ số nhiệt độ của phản ứng này bằng bao nhiêu theo quy tắc
thực nghiệm Van’t Hoff?
Giải
T2−T1
v
Quy tắc thực nghiệm Van’t Hoff: v2 = γ 10
1
30
→ 15,6 = γ 10
→ Hệ số nhiệt độ γ = 2,5.
Câu 4.9. Cho một phản ứng có hệ số nhiệt bằng 3. Theo quy tắc thực
nghiệm Van’t Hoff, cần tăng nhiệt độ từ 25 oC lên bao nhiêu để tốc độ
phản ứng tăng lên 10 lần.
Giải
T2−T1
v
Quy tắc thực nghiệm Van’t Hoff: v2 = γ 10
1
T2−25
→ 10 = γ 10
→ T2 = 45,96 oC

96
Phần 3. BÀI TẬP TRẮC NGHIỆM
---oOo---
Câu 4.1. Cho phản ứng hóa học dưới đây. Chọn phát biểu đúng:
2H2S (g) + O2 (g) → 2S (s) + 2H2O (g)
A. Đây là phản ứng bậc một
B. Đây là phản ứng bậc hai
C. Đây là phản ứng bậc ba
D. Không thể xác định bậc phản ứng từ thông tin đã cho
Câu 4.2. Cho phản ứng: aA + bB → cC+ dD, v = kCAmCBn. Chọn phát
biểu đúng:
A. Bậc phản ứng bằng (a + b)
B. Bậc phản ứng bằng (c+d) - (a + b)
C. Bậc phản ứng bằng (m + n)
D. Không thể xác định bậc phản ứng từ thông tin đã cho
Câu 4.3. Cho một phản ứng đồng thể đơn giản có dạng A → B + C. Chọn
phát biểu sai:
A. Khi bậc phản ứng lớn hơn 1 thì khi nồng độ chất A tăng sẽ làm cho tốc
độ phản ứng tăng.
B. Tốc độ phản ứng tăng khi năng lượng hoạt hóa của phản ứng tăng.
C. Tốc độ phản ứng không phụ thuộc nồng độ sản phẩm.
D. Tốc độ phản ứng thường tăng khi nhiệt độ tăng.
Câu 4.4. Cho phản ứng 4NH3 + 3O2 → 2N2 + 6H2O. Khi tốc độ tạo thành
N2 là 0,27 M.s-1 thì tốc độ mất đi của NH3 là:
A. 0,135 M.s-1 B. 0,27 M.s-1
C. 0,54 M.s-1 D. 0,81 M.s-1
Câu 4.5. Cho biết phản ứng: 2NO + O2 → 2NO2 tuân theo định luật tác
dụng khối lượng. Nếu tăng nồng độ NO lên gấp 3 lần và giữ nguyên nồng
độ O2 thì tốc độ phản ứng sẽ:
A. Tăng 27 lần B. Giảm 27 lần
C. Tăng 9 lần D. Giảm 9 lần
Câu 4.6. Cho biết phản ứng CO + Cl2 → COCl2 tuân theo định luật tác
dụng khối lượng. Nếu nồng độ CO tăng từ 0,3 M lên 1,2 M, đồng thời nồng
độ Cl2 tăng từ 0,2 M lên 0,6 M thì tốc độ phản ứng thay đổi như thế nào?
A. Tăng 3 lần B. Tăng 4 lần
C. Tăng 7 lần D. Tăng 12 lần
Câu 4.7. Cho phản ứng 2NOCl (g) → 2NO (g) + Cl2 (g)
97
Ở 77 oC hằng số tốc độ phản ứng k1 = 8.10-6 M-1s-1 và ở 127 oC hằng số
tốc độ phản ứng k2 = 5,9.10-4 M-1s-1. Xác định năng lượng hoạt hóa Ea
(kJ/mol) của phản ứng.
A. 100,12 B. 50,06
C.138,24 D.125,13
Câu 4.8. Một phản ứng có hệ số nhiệt độ  = 3. Khi tăng nhiệt độ từ 50 oC
đến 70 oC thì tốc độ phản ứng thay đổi như thế nào theo quy tắc thực
nghiệm Van’s Hoff?
A. Tăng 3 lần B. Tăng 4 lần
C. Tăng 9 lần D. Giảm 3 lần
Câu 4.9. Một phản ứng có hệ số nhiệt độ  = 2. Theo quy tắc thực nghiệm
Van’s Hoff, cần tăng nhiệt độ từ 25 oC lên bao nhiêu để tốc độ phản ứng
tăng lên 16 lần?
A. 45 oC B. 55 oC
C. 65 oC D. 75 oC
Câu 4.10. Một phản ứng có hằng số tốc độ là 0,02 s-1 ở 15 oC và bằng 0,38
s-1 ở 52 oC. Tính năng lượng hoạt hóa (kcal/mol) của phản ứng.
A. 14,8 B. 25,32
C. 17,28 D. 36,43
Câu 4.11. Cho phản ứng sau có bậc 2 theo A và cũng là bậc toàn phần của
phản ứng: A + B → C + D. Hằng số tốc độ của phản ứng ở 30 oC là 0,622
M-1phút-1. Hãy xác định thời gian nửa phản ứng của A khi trộn chất A với
nồng độ ban đầu là 4,10.10-2 M với lượng dư chất B.
A. 19,6 phút B. 39,2 phút
C. 27,3 phút D. 15,1 phút
Câu 4.12. Phản ứng bậc nhất A → B xảy ra hoàn thành được 25% trong
42 phút ở 25 oC. Thời gian bán hủy của phản ứng ở nhiệt độ 25 oC bằng
bao nhiêu?
A. 42 phút B. 84 phút
C. 120 phút D. 101 phút
Câu 4.13. Năng lượng hoạt hóa đối với phản ứng bậc nhất sau đây bằng
102 kJ/mol
N2O5(g) → 2NO2(g) + 1/2O2(g)
Giá trị hằng số tốc độ k bằng 1,35.10-4 s-1 ở 35 oC. Tính giá trị của k ở
0 oC.
A. 8,2.10-7 s-1 B. 1,9.10-5 s-1
98
C. 4,2.10-5 s-1 D. 2,2.10-2 s-1
Câu 4.14. Một phản ứng bậc 1 có chu kỳ bán hủy là 45,5 phút tại một nhiệt
độ xác định. Hằng số tốc độ phản ứng tại nhiệt độ trên là:
A. 1,69.10-4 s-1 B. 2,54.10-4 s-1
-4 -1 -1
C. 1,69.10 M s D. 2,54.10-4 M-1s-1
Câu 4.15. Cho phản ứng C2H5Br (aq)+ KOH (aq) → C2H5OH (aq) + KBr
(aq). Biết nồng độ ban đầu của KOH là 0,07 M. Sau 30 phút phản ứng, 10
mL dung dịch hỗn hợp phản ứng trung hòa vừa đủ với 12,84 mL dung dịch
HCl 0,05 M. Tốc độ trung bình phản ứng (mol.L-1.phút-1) trong khoảng
thời gian trên là:
A. 1,93.10-4 B. 3,86.10-4
C. 0,97.10-4 D. 2,90.10-4
Câu 4.16. Chu kỳ bán hủy của đồng vị phóng xạ 95Zr là 60 ngày. Tính thời
gian cần thiết để 1 mol 95Zr phân hủy hết 20%. Giả thiết quá trình phân rã
tuân theo quy luật phản ứng bậc nhất.
A. 38,64 ngày B. 19,32 ngày
C. 27,73 ngày D. 21,27 ngày
Câu 4.17. Cho phản ứng: A + B → C + D. Biết phản ứng bậc 1 theo A
và bậc 1 theo B. Nồng độ ban đầu của A bằng B và bằng 0,05 M. Biết hằng
số tốc độ phản ứng là 0,16 L.mol-1s-1 Tính thời gian để A phản ứng hết
50%.
A. 125 giây B. 100 giây
C. 250 giây D. 200 giây
Câu 4.18. Cho phản ứng: 2 N2O5 → 4 NO2 + O2
Biết hằng số tốc độ phản ứng ở 25 oC và 35 oC lần lượt là 1,72.10-5 s-1 và
6,65.10-5 s-1. Năng lượng hoạt hóa của phản ứng trên (kJ/mol) là:
A. 113 B. 123
C. 103 D. 135
Câu 4.19. Cho một phản ứng xảy ra ở 22 C có năng lượng hoạt hóa 73,3
o

kJ.mol-1. Khi có mặt xúc tác, năng lượng hoạt hóa của phản ứng giảm còn
1,74 kJ.mol-1. Tốc độ phản ứng tăng lên bao nhiêu lần khi có xúc tác ở điều
kiện trên?
A. 5,6.1011 lần B. 3,9.1012 lần
C. 4,7.1012 lần D. 3,3.1011 lần
Câu 4.20. Cho một phản ứng bậc 2 có hằng số tốc độ phản ứng tại 16 oC
và 60 oC lần lượt bằng 5,03.10-2 M-1.s-1 và 6,71 M-1.s-1. Tính năng lượng
hoạt hóa của phản ứng trên (kJ.mol-1).
99
A. 109 B. 99 C. 89 D. 79
Câu 4.21. Cho một phản ứng bậc 2 có hằng số tốc độ phản ứng tại 16 oC
và 60 oC lần lượt bằng 5,03.10-2 M-1.s-1 và 6,71 M-1.s-1. Hằng số tốc độ
phản ứng (M-1.s-1) tại 32 oC là:
A. 0,73 B. 0,68
C. 0,54 D. 0,35
Câu 4.22. Cho phản ứng xảy ra trong hệ đồng thể:
CH3Br + OH- → CH3OH + Br-. Biết các dữ liệu thực nghiệm như sau:
Thí nghiệm [CH3Br], M [OH-], M v, M.s-1
1 0,1 0,1 2,8.10-6
2 0,1 0,17 4,76.10-6
3 0,033 0,2 1,85.10-5
Bậc toàn phần của phản ứng bằng:
A. 1 B. 2 C. 3 D. 4
Câu 4.23. Trộn 2,5 mol H2 với 2,5 mol I2 trong bình kín dung tích 10 L. Sau
20s còn lại 2,4 mol I2. Tính tốc độ trung bình của phản ứng
(mol.L-1.s-1) theo I2 trong khoảng thời gian trên. Biết phản ứng xảy ra như sau:
H2 + I2 → 2 HI
-4
A. 5.10 B. 10-4
C. 10-3 D. 5.10-3
Câu 4.24. Chọn phát biểu sai về đặc tính của chất xúc tác:
A. Không làm thay đổi các đặc trưng nhiệt động của phản ứng.
B. Có tác dụng với một hoặc một vài loại phản ứng nhất định.
C. Làm thay đổi hằng số cân bằng của phản ứng.
D. Làm giảm năng lượng hoạt hóa của phản ứng.
Câu 4.25. Cho phản ứng C2H6 (g) → C2H4 (g) + H2 (g) là phản ứng bậc
một. Tính thời gian (giây) để C2H6 bị phân hủy 50% ở nhiệt độ 527 oC,
biết hằng số tốc độ tại nhiệt độ này là 4,6.10-4 s-1.
A. 1228 B. 853
C. 924 D. 1507
Câu 4.26. Một phản ứng bậc 1 có hằng số tốc độ ở 61 oC và 71 oC lần lượt
bằng 1,57.10-6 phút-1 và 4,45.10-6 phút-1. Tính năng lượng hoạt hóa
(kJ.mol-1) của phản ứng trên.
A. 79,5 B. 89,5
C. 99,5 D. 109,5

100
Câu 4.27. Nghiên cứu trên phản ứng 2NO(g) + Cl2 (g) → 2NOCl (g) cho
thấy khi nồng độ NO không đổi, tốc độ phản ứng tăng gấp đôi khi nồng độ
Cl2 tăng gấp đôi. Khi nồng độ Cl2 không đổi, tốc độ phản ứng tăng gấp 4
lần khi nồng độ NO tăng gấp đôi. Phương trình động học của phản ứng là:
A. v = k.[NO]2.[Cl2] B. v = k.[NO]2.[Cl2] 2
C. v = k.[NO].[Cl2] D. v = k.[NO].[Cl2] 2
Câu 4.28. Cho các phản ứng sau xảy ra trong một giai đoạn. Phản ứng nào
là phản ứng bậc hai?
1) I2 = 2I
2) 2 HI = H2 + I2
3) NO + O3 = NO2 + O2
A. 1 và 2 B. 2 C. 2 và 3 D. 1, 2 và 3
Câu 4.29. Quá trình chuyển hóa cyclopropane thành propene ở pha khí là
một phản ứng bậc 1 có hằng số tốc độ k=6,7.10-4 s-1 tại 500 oC. Tính thời
gian phản ứng để chuyển hóa hết 50% cyclopropane.
A. 23 phút 37 giây B. 16 phút 29 giây
C. 19 phút 5 giây D. 17 phút 14 giây
Câu 4.30. Để thay đổi hằng số tốc tộ của một phản ứng có thể thực hiện
biện pháp nào dưới đây:
A. Thay đổi áp suất khí
B. Thay đổi nhiệt độ phản ứng
C. Thay đổi nồng độ sản phẩm
D. Thay đổi nồng độ chất phản ứng
Câu 4.31. Cho phản ứng đơn giản: 2A (g) + B (g) → C (g). Cần thay đổi
nồng độ chất A như thế nào để tốc độ phản ứng không thay đổi khi giảm
nồng độ chất B xuống 4 lần.
A. Tăng 2 lần B. Giảm 2 lần
C. Tăng 4 lần D. Giảm 4 lần
Câu 4.32. Thời gian phản ứng kết thúc ở 160 oC bằng bao nhiêu nếu hệ số
nhiệt độ của tốc độ phản ứng bằng 3 và ở 200 oC phản ứng hoàn thành sau
17 phút theo quy tắc thực nghiệm Van’t Hoff.
A. 23 giờ B. 3,4 giờ
C. 1,4 ngày D. 12,5 giờ
Câu 4.33. Ở 100 C phản ứng bậc 1 kết thúc sau 1 giờ 30 phút. Vậy ở nhiệt
o

độ nào phản ứng kết thúc sau 10 phút nếu hệ số nhiệt độ của phản ứng
bằng 3 theo quy tắc thực nghiệm Van’t Hoff.

101
A. 120 oC B. 135 oC
C. 117 oC D. 125 oC
Câu 4.34. Cho phản ứng: 2NO (g) + O2 (g) → 2NO2(g). Biểu thức tốc độ
phản ứng là v = k.[NO]2[O2]. Chọn phát biểu đúng:
A. Bậc tổng quát của phản ứng là hai.
B. Khi tăng nồng độ NO lên 3 lần đồng thời giữ nguyên nồng độ O2, tốc
độ phản ứng tăng 3 lần.
C. Khi giảm nồng độ O2 xuống 2 lần đồng thời giữ nguyên nồng độ NO,
tốc độ phản ứng giảm 4 lần.
D. Khi tăng nồng độ NO lên 2 lần đồng thời giữ nguyên nồng độ O2, tốc
độ phản ứng tăng 4 lần.
Câu 4.35. Cho phản ứng: I2 (g) + H2 (g) → 2 HI (g) có biểu thức tốc độ
phản ứng v = k[I2][H2]. Chọn phát biểu đúng:
A. Khi nhiệt độ tăng, tốc độ phản ứng tăng còn hằng số tốc độ phản ứng
không đổi.
B. Khi nhiệt độ không đổi và tăng nồng độ I2, H2; tốc độ phản ứng và hằng
số tốc độ phản ứng đều tăng.
C. Khi nhiệt độ giảm, tốc độ phản ứng và hằng số tốc độ phản ứng đều
giảm nếu các điều kiện phản ứng khác không đổi.
D. Khi nhiệt độ không đổi, nếu giữ nguyên số mol I2, H2 nhưng giảm thể
tích hỗn hợp phản ứng, tốc độ phản ứng và hằng số tốc độ phản ứng đều
giảm.
Câu 4.36. Phản ứng phân hủy khí N2O thành N2 và O2 có hằng số tốc độ
phản ứng k = 5,1011.e-29000/T. Năng lượng hoạt hóa của phản ứng
(kJ.mol-1) là:
A. 2410 B. 241
C. 24,1 D. 2,41
Câu 4.37. Cho phản ứng: A + B + C → D có biểu thức tốc độ phản ứng
v = k[A][B]2. Chọn phát biểu sai:
A. Khi tăng nồng độ của B lên gấp 2 lần, giảm nồng độ của A xuống 4 lần
và giữ nguyên nồng độ của C thì tốc độ phản ứng không đổi.
B. Khi tăng nồng độ của cả 3 chất lên gấp đôi thì tốc độ phản ứng tăng gấp
8 lần.
C. Khi tăng nồng độ của C lên 2 lần và giữ nguyên nồng độ của A và B thì
tốc độ phản ứng không đổi.

102
D. Khi tăng nồng độ của A lên 2 lần và giữ nguyên nồng độ của B và C thì
tốc độ phản ứng tăng gấp 4 lần.
Câu 4.38. Cho phản ứng: 2N2O5(g) → 2N2O4(g) + O2(g) là phản ứng bậc
một với k = 0,0084 s-1 tại nhiệt độ phản ứng. Sau bao lâu thì phản ứng hết
90% lượng N2O5 ban đầu.
A. 120 giây B. 145 giây
C. 137 giây D. 89 giây
Câu 4.39. Cho phản ứng bậc 2: 2I (g) → I2 (g), v = k[I]2
Tại 23 oC phản ứng có hằng số tốc độ k = 7.109 L.mol-1.s-1. Tính nồng độ
(M) của nguyên tử iode sau 2 phút biết nồng độ ban đầu là 0,086 M.
A. 6,72.10-9 B. 5,95.10-13
C. 8,61.10-11 D. 8,25.10-7
Câu 4.40. Cho phản ứng xà phòng hóa ester methyl acetate
CH3COOCH3 + OH- → CH3COO- + CH3OH là phản ứng bậc 2 có biểu
thức tốc độ v = k[CH3COOCH3][ OH-], k = 0,137 L.mol-1.s-1 ở 25 oC. Tính
thời gian (giây) cần thiết để 5% lượng ester bị xà phòng hóa, biết nồng độ
đầu của CH3COOCH3 và OH- bằng nhau và bằng 0,05 M.
A. 7,68 B. 22,54
C. 8,73 D. 12,41
Câu 4.41. Cho phản ứng xà phòng hóa ester ethyl acetate
CH3COOC2H5 + OH- → CH3COO- + C2H5OH là phản ứng bậc 2 có biểu
thức tốc độ v = k[CH3COOCH3][ OH-], k = 0,01 L.mol-1.s-1 ở một nhiệt độ
xác định. Tính nồng độ ester còn lại sau 100 giây, biết nồng độ đầu của
CH3COOC2H5 và OH- bằng nhau và bằng 0,100 M.
A. 0,087 M B. 0,014 M
C. 0,045 M D. 0,091 M
Câu 4.42. Cho phản ứng bậc 0: A → Sản phẩm. Tính hằng số tốc độ phản
ứng tại nhiệt độ khảo sát biết nồng độ A còn lại theo thời gian có giá trị
thực nghiệm như sau:
Thời gian (phút) 0 1 2
Nồng độ A (M) 0,1 0,09 0,08
-1 -1
A. 0,01 M.phút B. 0,02 M.phút
-1
C. 0,005 M.phút D. 0,1 M.phút-1
Câu 4.43. Cho phản ứng A + B → Sản phẩm; có biểu thức tốc độ
v = k[A]2. Ở 25 oC phản ứng có hằng số tốc độ k = 0,622 L.mol-1.phút-1.

103
Tính thời gian nửa phản ứng, biết nồng độ ban đầu của A và B lần lượt là
4,1.10-2 M và 8,2.10-2 M.
A. 17,3 phút B. 39,2 phút
C. 46,7 phút D. 89,5 phút
Câu 4.44. Cho phản ứng 2NO2 (g) → N2O4 (g) có biểu thức tốc độ
v = k[NO2]2 với k = 400 L.mol-1.s-1. Cho nồng độ ban đầu của NO2 là 0,5
M. Tính thời gian (giây) cần thiết để nồng độ NO2 giảm còn 0,01 M.
A. 2,5 B.12,5
C. 25 D. 0,25
Câu 4.45. Cho phản ứng H2O2 (aq) → H2O (l) + O2 (g) có biểu thức tốc
độ v = k[H2O2] với k = 0,56 phút-1. Tính nồng độ H2O2 sau 2 phút biết
nồng độ ban đầu của H2O2 là 0,3 M.
A. 0,2 M B. 0,15 M
C. 0,25 M D. 0,1 M
Câu 4.46. Cho phản ứng (CH3)3COH + HCl → (CH3)3CCl + H2O có bậc
phản ứng là 1 theo (CH3)3COH và cũng là bậc toàn phần của phản ứng.
Chọn phát biểu đúng:
A. Khi tăng nồng độ (CH3)3COH lên 2 lần và giảm nồng độ HCl xuống 2
lần thì tốc độ phản ứng không đổi.
B. Khi tăng nồng độ HCl thì tốc độ phản ứng tăng.
C. Khi tăng nồng độ (CH3)3COH thì tốc độ phản ứng tăng.
D. Khi giảm nồng độ (CH3)3CCl thì tốc độ phản ứng tăng.
Câu 4.47. Cho phản ứng bậc 1: C2H5Cl (g) → C2H4 (g) + HCl (g) có chu
kỳ bán hủy là 84,1 phút tại một nhiệt độ xác định. Tính hằng số tốc độ của
phản ứng tại nhiệt độ trên.
A. 1,37.10-4 s-1 B. 1,47.10-4 s-1
C. 1,57.10- s-1 D. 1,67.10-4 s-1
Câu 4.48. Cho phản ứng N2O5 (g) → 2NO2 (g) +1/2O2 (g). Tại thời điểm
biến thiên nồng độ NO2 tạo thành là 0,1 mol.L-1.s-1 thì biến thiên nồng độ
N2O5 bị tiêu thụ là:
A. - 0,1 mol.L-1.s-1 B. - 0,05 mol.L-1.s-1
C. 0,05 mol.L-1.s-1 D. 0,1 mol.L-1.s-1
Câu 4.49. Cho các phản ứng sau với biểu thức tốc độ tương ứng:
(1) 2 N2O5 → 2 N2O4 + O2, v = k[N2O5]
(2) 2 HI → H2 + I2 , v = k[HI]2
(3) 2 NO + O2 → 2 NO2 , v = k[NO]2[O2]
104
(4) NO + O3 → NO2 + O2, v = k[NO][O3]
Phản ứng nào có bậc là 2:
A. (1) B. (1) và (2)
C. (2) và (4) D. (1), (2) và (4)
Câu 4.50. Cho phản ứng 2HI → H2 + I2 với các giá trị thực nghiệm tại
nhiệt độ khảo sát như bảng sau:

Thí nghiệm [HI], M V, M.s-1


1 0,005 7,5.10-4
2 0,01 3.10-3
Nếu nồng độ ban đầu của HI là 0,02 M thì vận tốc phản ứng (M.s-1) là:
A. 1,2.10-2 B. 2,4.10-2 C. 3,6.10-2 D. 4,8.10-2
---oOo---

ĐÁP ÁN BÀI TẬP CHƯƠNG 4


4.1- 4.2- 4.3- 4.4- 4.5- 4.6- 4.7- 4.8- 4.9- 4.10-
D C B C C D A C C A
4.11- 4.12- 4.13- 4.14- 4.15- 4.16- 4.17- 4.18- 4.19- 4.20-
B D A B A B A C C C
4.21- 4.22- 4.23- 4.24- 4.25- 4.26- 4.27- 4.28- 4.29- 4.30-
D B D C D C A C D B
4.31- 4.32- 4.33- 4.34- 4.35- 4.36- 4.37- 4.38- 4.39- 4.40-
A A A D C B D C B A
4.41- 4.42- 4.43- 4.44- 4.45- 4.46- 4.47- 4.48- 4.49- 4.50-
D A B D D C A B C A

105
Chương 5. CÂN BẰNG HÓA HỌC
---oOo---

Mục tiêu chương 5


Sau khi học xong chương này, sinh viên có khả năng:
1. Định nghĩa được phản ứng một chiều và phản ứng thuận nghịch.
2. Giải thích được tính chất đặc trưng và trạng thái cân bằng của phản
ứng thuận nghịch.
3. Viết được biểu thức hằng số cân bằng KC, KP và mối quan hệ giữa
hai đại lượng này đối với phản ứng thuận nghịch.
4. Tính toán được hằng số cân bằng và hiệu suất phản ứng.
5. Phát biểu được nguyên lý chuyển dịch cân bằng và giải thích được
sự tác động của các yếu tố nhiệt độ, nồng độ, áp suất đến khả năng
chuyển dịch cân bằng của một phản ứng thuận nghịch.

106
Phần 1. TÓM TẮT LÝ THUYẾT
5.1 CÂN BẰNG HÓA HỌC
5.1.1 Phản ứng một chiều
Phản ứng một chiều là phản ứng chỉ xảy ra theo một chiều trong bất kỳ
điều kiện nào hoặc ít nhất là tại điều kiện thực hiện phản ứng.
Ví dụ, phản ứng NaOH + HCl → NaCl + H2O là phản ứng một chiều.
Phản ứng một chiều xảy ra đến khi hết một trong các nguyên liệu hoặc tất
cả các nguyên liệu đều chuyển hóa hết thành sản phẩm. Như vậy, phản ứng
một chiều có hiệu suất 100%.
5.1.2 Phản ứng thuận nghịch
Phản ứng thuận nghịch là phản ứng xảy ra theo cả hai chiều thuận và
nghịch ở cùng điều kiện.
Ví dụ, phản ứng ester hóa sau đây là phản ứng thuận nghịch.
CH3COOH +C2H5OH ⇌ CH3COOC2H5 + H2O
Trong cùng một điều kiện, khi cho CH3COOH và C2H5OH vào bình phản
ứng ở giai đoạn đầu, phản ứng sinh ra CH3COOH và H2O. Tuy nhiên, khi
CH3COOH tương tác với H2O thì xảy ra phản ứng ngược lại và tạo thành
CH3COOH và C2H5OH.
Xét phản ứng thuận nghịch tổng quát đồng thể và đơn giản sau:
Thuận
Aa +bB ⇌ cC +dD
Nghịch

Phản ứng aA + bB → cC + dD gọi là phản ứng thuận. Biểu thức tốc độ


phản ứng thuận là vt = kt [A]a [B]b. Ở thời điểm ban đầu, khi nồng độ A, B
lớn thì vt lớn. Theo thời gian thực hiện phản ứng, nồng độ A, B ngày càng
giảm dẫn đến vt giảm.
Phản ứng cC + dD → aA + bB gọi là phản ứng nghịch. Biểu thức tốc độ
phản ứng nghịch là vn = kn [C]c [D]d. Ở thời điểm ban đầu, khi nồng độ C,
D thấp thì vt nhỏ. Theo thời gian thực hiện phản ứng, nồng độ C, D ngày
càng tăng dẫn đến vn tăng. Như vậy, trong hệ hóa học có hai phản ứng xảy
ra đồng thời: phản ứng thuận có tốc độ ngày càng giảm, phản ứng nghịch
có tốc độ ngày càng tăng. Do đó, đến một thời điểm nhất định, tốc độ hai
phản ứng này bằng nhau, đó là thời điểm phản ứng đạt trạng thái cân bằng.
Tại trạng thái cân bằng, có bao nhiêu A và B mất đi để tạo thành C và D
thì cũng sẽ có bấy nhiêu C và D mất đi tạo thành A và B. Trong bình phản

107
ứng luôn luôn có đầy đủ các nguyên liệu và sản phẩm là A, B, C và D. Như
vậy, phản ứng thuận nghịch có hiệu suất nhỏ hơn 100%.
Xu hướng của tốc độ phản ứng thuận và phản ứng nghịch được minh họa
trong hình sau:
Tốc độ
Vt

Cân bằng Vt = Vn

Vn
Thời gian
t
Hình 5.1. Tốc độ phản ứng thuận và nghịch theo thời gian
5.1.3 Trạng thái cân bằng hóa học
Trạng thái cân bằng hóa học là trạng thái của phản ứng hóa học có vt = vn
và tỷ lệ lượng chất giữa các chất phản ứng với sản phẩm phản ứng không
thay đổi ở những điều kiện bên ngoài nhất định.
Đặc điểm:
✓ Tốc độ phản ứng thuận bằng tốc độ phản ứng nghịch
✓ Tỷ lệ lượng chất giữa các chất không thay đổi ở những điều kiện
bên ngoài nhất định
✓ Cân bằng hóa học là cân bằng động
✓ Không thay đổi theo thời gian nếu những điều kiện bên ngoài
quyết định trạng thái cân bằng không thay đổi.
✓ Trạng thái cân bằng hóa học được đặc trưng bằng một hằng số
cân bằng K.
5.2 HẰNG SỐ CÂN BẰNG VÀ MỨC ĐỘ XẢY RA PHẢN ỨNG
5.2.1 Hằng số cân bằng
Xét phản ứng thuận nghịch đồng thể đơn giản tổng quát:
aA + bB ⇌ cC + dD

108
Ta có:
• Hằng số cân bằng nồng độ KC
Trong đó:
[C]c [D]d ✓ KC: hằng số cân bằng nồng độ.
KC = a
[A] [B] b ✓ [A], [B], [C] và [D] là nồng độ (mol/l)
của chất A, B, C và D trong hệ lúc phản
ứng đạt trạng thái cân bằng.
• Hằng số cân bằng áp suất Kp
Trong đó:
c d
PC PD ✓ KP: hằng số cân bằng áp suất.
KP = a b
PA PB ✓ PA, PB, PC và PD là áp suất riêng phần
(atm) của chất A, B, C và D trong hệ lúc
phản ứng đạt trạng thái cân bằng.
• Mối quan hệ giữa Kp và KC
Ở cùng điều kiện phản ứng, ta có:
KP = KC (R T)n (5.1)
Trong đó: R = 0,082 L.atm.mol-1.K-1
T: nhiệt độ tuyệt đối, K
n: độ biến đổi số mol chất khí trước và sau phản ứng.
5.2.2 Các lưu ý quan trọng
a) Nếu tham gia vào phản ứng có chất khí, lỏng và rắn (phản ứng dị thể)
thì khi viết biểu thức hằng số cân bằng chỉ chú ý đến chất khí. Vì nồng độ
hay áp suất hơi bão hòa của chất rắn hoặc lỏng là đại lượng không đổi ở
nhiệt độ nhất định.
Ví dụ 5.1.
[CO][H2 O]
CO2 (g) + H2 (g) ⇌ CO (g) + H2O (g) KC =
[CO2 ][H2 ]
[NH3 ]2
N2 (g) + 3H2 (g) ⇌ 2NH3 (g) KC =
[N2 ][H2 ]3
[CO2 ]3
Fe2O3 (s) + 2CO (g) ⇌ 2Fe (s) + 3CO2 (g) KC =
[CO]3
b) Hằng số cân bằng gắn liền với phương trình cân bằng cụ thể, tức là phụ
thuộc chiều và hệ số cân bằng của phương trình.

109
Ví dụ 5.2.
2
PCO 2
2CO (g) + O2 (g) ⇌ 2CO2 (g) (1) KP = 2
PCO . PO2
PCO2
CO (g) + 1/2O2 (g) ⇌ CO2 (g) (2) K′P = 1/2 = ඥK p
PCO .PCO
2
1/2
PCO .PO 1
2
CO2 (g) ⇌ CO (g) + 1/2O2 (g) (3) K′′P = = K′
PCO2 P

c) Xét phản ứng (1) có hằng số cân bằng K1 và phản ứng (2) có hằng số
cân bằng K2:
• Nếu phản ứng (3) = (1) + (2) → hằng số cân bằng K3 = K1.K2
• Nếu phản ứng (3) = (1) – (2) → hằng số cân bằng K3 = K1.K −1
2
Ví dụ 5.3. Cho các phản ứng xảy ra ở cùng điều kiện:
2HCl (g) ⇌ H2 (g) + Cl2 (g), K1 = 4,90.10-11 (1)
-2
2HI (g) ⇌ H2 (g) + I2 (g), K2 = 3,80.10 (2)
Hãy tính hằng số cân bằng K3 của phản ứng:
2HI (g) + Cl2 (g) ⇌ 2HCl (g) + I2 (g) (3)
Giải
K 3,80.10−2
Phản ứng (3) = (2) – (1) → K 3 = K2 = 4,90.10−11 = 7,76.108
1

5.2.3 Ý nghĩa của hằng số cân bằng K


• Đối với mỗi phản ứng hóa học K là đại lượng không đổi tại một
nhiệt độ xác định (hằng số cân bằng chỉ phụ thuộc vào nhiệt độ và
bản chất chất phản ứng, không phụ thuộc vào nồng độ hay áp suất
riêng phần các chất tham gia phản ứng).
• K cho biết mức độ xảy ra phản ứng về định tính (nông hay sâu) và
về định lượng (tính được hiệu suất phản ứng). K càng lớn thì phản
ứng diễn tiến càng sâu và có hiệu suất càng lớn.
• Hằng số cân bằng là đại lượng không có thứ nguyên.
Ví dụ 5.4. Xét hai phản ứng đồng thể sau:
A + B ⇌ C + D, KC = 2 (1)
E + F ⇌ G + H, KC = 6 (2)
Phản ứng (2) sẽ diễn tiến sâu hơn và có hiệu suất cao hơn phản ứng (1).
Ví dụ 5.5. Thực hiện phản ứng H2 (g) + I2 (g) ⇌ 2HI (g) trong bình kín tại
700 oC với áp suất riêng phần ban đầu của H2 và I2 tương ứng là 1,5 và 1,2
atm. Hãy tính hiệu suất của phản ứng theo H2 và I2 tại thời điểm cân bằng,
biết phản ứng có Kp = 54.
110
Giải
Phản ứng H2 (g) + I2 (g) ⇌ 2HI (g)
PBan đầu 1,5 1,2 -
PPhản ứng x x 2x x < 1,2
PCân bằng 1,5 – x 1,2 – x 2x
P2HI
Lúc cân bằng, ta có: K P = P
H2 .PI
2
ሺ2xሻ2
Suy ra: ሺ1,5−xሻ.ሺ1,2−xሻ = 54
Giải phương trình, ta có x = 1,03 hoặc x = 1,88.
So sánh điều kiện, nhận nghiệm x = 1,03.
Vậy:
1,03
✓ Hiệu suất của phản ứng tính theo H2 là: %H2 = . 100 = 68,6%
1,5
1,03
✓ Hiệu suất của phản ứng tính theo I2 là: %I2 = . 100 = 85,8%
1,2

5.3 HẰNG SỐ CÂN BẰNG K VÀ ĐỘ BIẾN ĐỔI THẾ ĐẲNG ÁP G


CỦA PHẢN ỨNG (Phương trình đẳng nhiệt Van’t Hoff)
Giữa hằng số cân bằng (đặc trưng cho trạng thái cân bằng của quá trình
hóa học) và độ biến đổi thế đẳng áp Go có mối liên quan định lượng chặt
chẽ.
• Trường hợp phản ứng xảy ra giữa các chất khí ở điều kiện tiêu chuẩn:
GoT = -RTlnKP (5.2)
• Trường hợp phản ứng xảy ra trong dung dịch ở điều kiện tiêu chuẩn:
GoT = -RTlnKC (5.3)
5.4 DỰ ĐOÁN CHIỀU DIỄN TIẾN CỦA PHẢN ỨNG
Xét phản ứng thuận nghịch đồng thể đơn giản tổng quát sau:
aA (g) + bB (g) ⇌ cC (g) + dD (g)
Ở nhiệt độ và áp suất không đổi ta có:
PCc . PDd (5.4)
∆GT = ∆GT + RTln a b = ∆GTo + RTlnQP
o
PA . PB
Thay (5.2) vào (5.4), ta có:
𝑄𝑃 (5.5)
∆𝐺𝑇 = RTln
𝐾𝑃

PCc . PDd (5.6)


𝑄P =
PAa . PBb
111
Trong đó:
QP: thương số phản ứng hay chỉ số phản ứng.
PA, PB, PC và PD là áp suất riêng phần của các khí A, B, C và D lúc
tính G của phản ứng.
• Nếu QP > Kp → GT > 0 → Chiều nghịch của phản ứng diễn tiến ưu
thế.
• Nếu QP < Kp → GT < 0 → Chiều thuận của phản ứng diễn tiến ưu
thế.
• Nếu QP = Kp → GT = 0 → Hệ phản ứng đang ở trạng thái cân bằng.
Ví dụ 5.6. Cho phản ứng 2NO2 (g) ⇌ N2O4 (g), KP = 9,2 ở 25 oC. Hỏi ở
cùng nhiệt độ, phản ứng trên đi theo chiều nào ứng với các trường hợp sau:
a) PN2O4 = 0,90 atm; PNO2 = 0,10 atm
b) PN2O4 = 0,72 atm; PNO2 = 0,28 atm
c) PN2O4 = 0,10 atm; PNO2 = 0,90 atm
Giải
0,90
a) QP = 0,102 = 90 > KP → phản ứng đi theo chiều nghịch (chiều nghịch
diễn tiến ưu thế).
0,72
b) QP = 0,282 = 9,2 = KP → phản ứng ở trạng thái cân bằng.
0,10
a) QP = 0,902 = 0,12 < KP → phản ứng đi theo chiều thuận (chiều thuận
diễn tiến ưu thế).
5.5 SỰ CHUYỂN DỊCH CÂN BẰNG VÀ NGUYÊN LÝ LE
CHATELIER
Như đã biết, trạng thái cân bằng hóa học sẽ không thay đổi nếu các điều
kiện bên ngoài tác động lên hệ phản ứng vẫn giữ nguyên. Khi thay đổi một
trong các yếu tố bên ngoài thì trạng thái cân bằng này sẽ bị phá vỡ, các
phản ứng thuận và nghịch trong hệ phản ứng sẽ tiếp tục thay đổi để hướng
tới một trạng thái cân bằng mới. Sự thay đổi trạng thái cân bằng khi thay
đổi điều kiện bên ngoài gọi là sự dịch chuyển cân bằng.
Nguyên lý chuyển dịch cân bằng (nguyên lý Le Chatelier – 1884): khi
tác dụng từ ngoài vào hệ đang cân bằng bằng cách thay đổi một điều kiện
nào đó quyết định vị trí cân bằng (nồng độ, nhiệt độ, áp suất) thì vị trí cân
bằng của hệ sẽ dịch chuyển theo chiều chống lại sự thay đổi đó.

112
5.5.1 Ảnh hưởng của nồng độ
Đối với hệ phản ứng đang cân bằng, nếu tăng (hay giảm) nồng độ của một
chất thì cân bằng sẽ chuyển dịch về phía làm giảm (hay tăng) nồng độ chất
đó.
Ví dụ 5.7. Xét phản ứng đồng thể sau đây đang ở trạng thái cân bằng:
2A (g) + B (g) ⇌ C (g) + 3D (g)
Sau khi phản ứng đạt cân bằng, nếu:
Giải pháp Ứng xử của hệ Chiều dịch chuyển
Tăng A Giảm A Thuận
Tăng C Giảm C Nghịch
Giảm B Tăng B Nghịch
Giảm D Tăng D Thuận
5.5.2 Ảnh hưởng của áp suất
Đối với hệ phản ứng của các chất khí đang cân bằng, nếu tăng (hay giảm)
áp suất của hệ thì cân bằng sẽ dịch chuyển về phía giảm (hay tăng) số phân
tử khí của phản ứng.
Ví dụ 5.8. Xét phản ứng đồng thể sau đây đang ở trạng thái cân bằng:
2A (g) + B (g) ⇌ C (g) + 3D (g)
Sau khi phản ứng đạt cân bằng, nếu:
Giải pháp Ứng xử của hệ Chiều dịch chuyển
Tăng áp Giảm áp (giảm số phân tử khí) Nghịch
Giảm áp Tăng áp (tăng số phân tử khí) Thuận
5.5.3 Ảnh hưởng của nhiệt độ
Đối với hệ phản ứng đang cân bằng, nếu tăng (hay giảm) nhiệt độ của hệ
thì cân bằng sẽ chuyển dịch về phía thu nhiệt (hay tỏa nhiệt).
Ví dụ 5.9. Xét phản ứng đồng thể sau đây đang ở trạng thái cân bằng:
2NO2 (g) ⇌ N2O4 (g), Ho = - 57,2 kJ
Sau khi phản ứng đạt cân bằng, nếu:
Giải pháp Ứng xử của hệ Chiều dịch chuyển
Tăng nhiệt độ Giảm nhiệt độ (thu nhiệt) Nghịch
Giảm nhiệt độ Tăng nhiệt độ (tỏa nhiệt) Thuận

113
Phần 2. BÀI TẬP CÓ LỜI GIẢI
---oOo---
Dạng 1. Hằng số cân bằng Kp và Kc
Câu 5.1. Viết biểu thức hằng số cân bằng KP cho từng phản ứng sau:
a) NO2 (g) + SO2 (g) ⇌ NO (g) + SO3 (g)
b) 2SO2 (g) + O2 (g) ⇌ 2SO3 (g)
c) Fe3O4 (s) + 4H2 (g) ⇌ 3Fe (s) + 4H2O (g)
d) Ca(HCO3) (s) ⇌ CaO (s) + 2CO2 (g) + H2O (g)
Giải
a) Đây là phản ứng đồng thể.
PNO . PSO3
KP =
PNO2. PSO2
b) Đây là phản ứng đồng thể.
2
PSO
KP = 2 3
PSO2 . PO2
c) Đây là phản ứng dị thể nên các chất có trạng thái rắn (s) không xuất hiện
trong biểu thức tính hằng số cân bằng.
PH42O
KP = 4
PH2
d) Đây là phản ứng dị thể
2
K P = PCO 2
. PH2O
Câu 5.2. Tại một điều kiện xác định, phản ứng thuận nghịch
N2(g) + 2O2(g) ⇌ 2NO2(g) có KC = 50.
Hãy tính hằng số cân bằng KC của các phản ứng thuận nghịch sau ở cùng
điều kiện.
a) 2NO2(g) ⇌ N2(g) + 2O2(g)
b) NO2(g) ⇌ 1/2 N2(g) + O2(g)
Giải
[NO2 ]2
Ta có: K C = [N 2
= 50
2 ].[O2 ]
[N2 ][O2 ]2 1
a) K′C = = 50 = 0,02
[NO2 ]2
1
[N2 ]2 [O2 ]
b) K′′C = = ඥK′C = ξ0,02 = 0,14
[NO2 ]
Câu 5.3. Cho 1,0 mol khí PCl3 và 2,0 mol khí Cl2 có thể tích không đổi là
2 lít tại một nhiệt độ xác định để thực hiện phản ứng:
114
PCl3 (g) + Cl2 (g) PCl5 (g).
Sau khi phản ứng đạt cân bằng thì trong bình có 0,4 mol PCl3. Tính giá trị
hằng số cân bằng KC của phản ứng trên.
Giải
Ta có, [PCl3]ban đầu = 0,50 M; [Cl2]ban đầu = 1,0 M và [PCl3]cân bằng = 0,20 M.
PCl3 (g) + Cl2 (g) ⇌ PCl5 (g).
Nồng độ ban đầu 0,50 1,0 0
Nồng độ phản ứng x x x (x < 0,50 M)
Nồng độ cân bằng 0,50 – x 1,0 – x x
Theo giả thiết: 0,5 – x = 0,20 → x = 0,30
[PCl5 ] 0,30
→ K C = [PCl = 0,20 x 0,70 = 2,14
3 ].[Cl2 ]
Dạng 2. Mối quan hệ giữa KP và KC
Câu 5.4. Tại 150oC, phản ứng thuận nghịch A(g) ⇌ B(g) + C(g) có hằng
số cân bằng KC = 50. Tính giá trị hằng số cân bằng KP của phản ứng ở
cùng điều kiện.
Giải
Theo công thức KP = KC (RT)n
Với R = 0,082 L.atm.mol-1K-1, T = 150 + 273 = 423 K và n = 1 + 1 – 1 =
1.
→ KP = 50 (0,082 . 423)1 = 1734,3
Lưu ý: n là độ biến đổi số mol chất khí của phản ứng.
Câu 5.5. Ở 375 oC, phản ứng thuận nghịch N2(g) + 3H2(g) ⇌ 2NH3(g) có
giá trị hằng số cân bằng KP = 4,3.10-4. Hãy tính giá trị hằng số cân bằng
KC ở cùng điều kện.
Giải
Theo công thức (5.1) KP = KC (RT)n
Với R = 0,082 L.atm.mol-1K-1, T = 370 + 273 = 648 K và n = 2 – 3 – 1
=–2
→ KC = KP (RT) –n = 4,3.10-4 (0,082 . 648)2 = 1,214
Câu 5.6. Trộn 1,0 mol khí CO và 2,0 mol hơi nước ở 800 oC trong một
bình phản ứng có thể tích 1 lít để thực hiện phản ứng:
CO (g) + H2O (g) CO2 (g) + H2 (g).
Khi phản ứng đạt cân bằng thì trong bình có 0,7 mol khí CO2. Tính giá trị
hằng số cân bằng KP và KC của phản ứng ở 800oC.

115
Giải
CO (g) + H2O (g) ⇌ CO2 (g) + H2 (g)
Nồng độ ban đầu 1,0 2,0 0 0
Nồng độ phản ứng 0,7 0,7 0,7 0,7
Nồng độ cân bằng 0,3 1,3 0,7 0,7
0,7 x 0,7
Vì n = 0 nên K C = K P = = 1,26
0,3x 1,3
Câu 5.7. Nén 10,0 mol khí A và 4,0 mol khí B vào một bình kín có thể
tích không đổi là 8 lít ở một nhiệt độ không đổi 25oC để thực hiện phản
ứng sau:
2A (g) + B (g) ⇌ C (g) + D (g)
Ở trạng thái cân bằng có 60% lượng chất B tham gia phản ứng. Tính giá
trị hằng số cân bằng KP và KC của phản ứng.
Giải
Từ dữ kiện đề ra: [A]ban đầu = 1,25 M; [B]ban đầu = 0,50 M
2A (g) + B (g) ⇌ C (g) + D (g)
Nồng độ ban đầu 1,25 0,5 0 0
Nồng độ phản ứng 0,6 0,3 0,3 0,3
Nồng độ cân bằng 0,65 0,2 0,3 0,3
0,3 x 0,3
Do đó, K C = = 1,07
ሺ0,65ሻ2 x0,2
KP = 1,07 (0,082 . 298)-1 = 0,04
Dạng 3. Phương trình đẳng nhiệt Van’t Hoff
Câu 5.8. Cho phản ứng sau ở 370 oC: N2 (g) + 3H2 (g) ⇌ 2NH3 (g), KC =
1,2. Trộn 0,40 mol N2, 0,20 mol H2 và 0,10 mol NH3 trong bình kín có thể
tích 2,0 lít ở 370 oC.
a) Ở thời điểm pha trộn lúc đầu, hệ phản ứng có đạt cân bằng hay không?
Tại sao?
b) Sau khi pha trộn xong, chiều thuận hay chiều nghịch của phản ứng diễn
tiến ưu thế hơn? Tại sao?
Giải
a) Ta có, [N2]ban đầu = 0,20 M, [H2]ban đầu = 0,10 M; [NH3]ban đầu = 0,05 M.
Lập biểu thức QC tại thời điểm ban đầu:
[NH3 ]2 0,052
QC = [N 3
= 0,20 x 0,13 = 12,5
2 ][H2 ]
Do QC ≠ KC nên ở thời điểm lúc đầu sau khi pha trộn, hệ phản ứng chưa
đạt trạng thái cân bằng.

116
b) Phản ứng đạt cân bằng khi QC = KC. Như vậy giá trị QC phải giảm dần
từ 12,5 về 1,2. Do đó, nồng độ NH3 trong hệ phản ứng giảm dần và chiều
nghịch của phản ứng sẽ diễn tiến ưu thế hơn.
Câu 5.9. Ở một điều kiện xác định, phản ứng thuận nghịch:
H2 (g) + I2 (g) 2HI (g) có KC = 40.
Hãy xác định trạng thái của hệ phản ứng tại từng thời điểm có nồng độ của
H2, I2 và HI trong hệ phản ứng lần lượt là:
a) 2M, 5M và 10M
b) 0,25M, 0,25M và 2M
c) 0,8M, 2M và 8M
Giải
Lập biểu thức tính QC theo cả 3 trường hợp và so sánh với KC ta có:
a) QC = 10 < KC. Vậy QC có xu hướng tăng dần nên chiều thuận sẽ diễn
tiến ưu thế.
b) QC = 64 > KC. Vậy QC có xu hướng giảm dần nên chiều nghịch sẽ diễn
tiến ưu thế.
c) QC = 64 = KC. Vậy hệ phản ứng đạt trạng thái cân bằng.
Dạng 4. Xác định nồng độ và áp suất của hệ lúc cân bằng
Câu 5.10. Trong một bình kín thể tích không đổi, phản ứng phân hủy khí
NO2 ở 1000K (trạng thái ban đầu chỉ có khí NO2) được diễn ra như sau:
2NO2 (g) ⇌ 2NO (g) + O2 (g), Kp = 158
Ở trạng thái cân bằng, áp suất riêng phần của khí NO là 0,80 atm. Hãy xác
định áp suất riêng phần ban đầu và khi cân bằng của khí NO2 cũng như áp
suất chung của hệ tại trạng thái cân bằng.
Giải
2NO2 (g) ⇌ 2NO (g) + O2 (g), Kp = 158
Áp suất ban đầu 2x 0 0 (x > 0,40)
Áp suất phản ứng 0,80 0,80 0,40
Áp suất cân bằng 2x – 0,80 0,80 0,40
P2NO . PO2 ሺ0,80ሻ2 . 0,40
Ta có, K P = = = 158
P2NO2 ሺ2x−0,80ሻ2

→ x = 0,42
Lúc ban đầu áp suất riêng phần của khí NO2 = 2x = 2 . 0,42 = 0,84 atm.
Lúc cân bằng áp suất riêng phần khí NO2 = 2x – 0,8 = 0,04 atm
Áp suất chung của hệ lúc cân bằng Pchung = 0,04 + 0,80 + 0,40 = 1,24 atm.

117
Câu 5.11. Phản ứng C (gr) + CO2 (g) 2CO (g) ở 815o C có hằng số
cân bằng KP = 10. Tại trạng thái cân bằng, áp suất chung của hệ là P = 2,5
atm. Hãy tính áp suất riêng phần của CO tại cân bằng.
Giải
Gọi x atm (x < 2,5) là áp suất riêng phần của CO lúc cân bằng
→ Áp suất riêng phần của của CO2 lúc cân bằng là (2,5 – x ) atm
P2
K P = P CO
CO2
x2
→ 10 = 2,5−x
→ x = 2,07
Câu 5.12. Ở 800 oC, phản ứng CO (g) + H2O (g) ⇌ CO2 (g) + H2 (g) có
KC = 1. Ở thời điểm ban đầu, nồng độ của CO và H2O tương ứng là 0,2 và
0,6 M. Xác định nồng độ từng chất trong bình ở trạng thái cân bằng.
Giải
CO (g) + H2O (g) ⇌ CO2 (g) + H2 (g)
Nồng độ ban đầu 0,2 0,6 0 0
Nồng độ phản ứng x x x x (x < 0,2)
Nồng độ cân bằng 0,2 – x 0,6 – x x x
2 [CO ][H2 ]
Ta có K C = [CO][H
2 O]
x. x
→ 1 = ሺ0,2−xሻሺ0,6−xሻ
→ x = 0,15
Vậy lúc cân bằng [CO] = 0,2 – 0,15 = 0,05 M; [H2O] = 0,6 – 0,15 = 0,45
M
[CO2] = [H2] = 0,15 M
Câu 5.13. Ở 800 C phản ứng CO (g) + H2O (g) ⇌ CO2 (g) + H2 (g) có
o

KC = 1. Ở thời điểm ban đầu, nồng độ của CO, H2O và H2 tương ứng là
0,2; 0,6 và 0,1 M. Xác định nồng độ từng chất trong bình ở trạng thái cân
bằng.
Giải
CO (g) + H2O (g) ⇌ CO2 (g) + H2 (g)
Nồng độ ban đầu 0,2 0,6 0 0,2
Nồng độ phản ứng x x x x (x < 0,2)
Nồng độ cân bằng 0,2 – x 0,6 – x x 0,2 + x
2 [CO ][H2 ]
Ta có K C = [CO][H
2 O]

118
x . ሺ0,2 + xሻ
→ 1 = ሺ0,2−xሻሺ0,6−xሻ
→ x = 0,12
Vậy lúc cân bằng:
[CO] = 0,2 – 0,12 = 0,08 M; [H2O] = 0,6 – 0,12 = 0,48 M
[CO2] = 0,12 M; [H2] = 0,2 + 0,12 = 0,32 M
Câu 5.14. Phản ứng thuận nghịch PCl5 (g) ⇌ PCl3 (g) + Cl2 (g) có KC =
30 tại 750 K. Tính hiệu suất phản ứng phân hủy của khí PCl5 tại nhiệt độ
750 K.
Giải
Giả sử ở thời điểm ban đầu, nồng độ khí PCl5 là 1 M. Ta có:
PCl5 (g) ⇌ PCl3 (g) + Cl2 (g)
Nồng độ ban đầu 1 0 0 (x < 1)
Nồng độ phản ứng x x x
Nồng độ cân bằng 1–x x x
[PCl3 ][Cl2 ] x .x
Ta có, K C = = 1−x = 30
[PCl5 ]
→ x = 0,97
→ H = 97%
Câu 5.15. Phản ứng thuận nghịch H2 (g) + S (s) ⇌ H2S (g), KC = 6,8.10-2.
Trộn 0,4 mol H2 và 1,0 mol bột lưu huỳnh đun nóng lên 90oC trong bình
có thể tích 1 lít. Tính áp suất riêng phần của H2S tại trạng thái cân bằng.
Giải
Giả sử ở thời điểm ban đầu, nồng độ khí PCl5 là 1 M. Ta có:
H2 (g) + S (s) ⇌ H2S (g)
Nồng độ ban đầu 0,4 0 (x < 0,4)
Nồng độ phản ứng x x
Nồng độ cân bằng 0,4 – x x
[H2 S] x
Ta có, K C = = = 6,8.10−2
[H2 ] 0,4−x
→ x = 2,55.10-2
nRT 2,55.10−2 .0,082 ሺ90+273ሻ
Áp suất riêng phần của H2S = = = 0,76 (atm)
V 1
Dạng 5. Hằng số cân bằng và thế đẳng áp.
Câu 5.16. Cho phản ứng thuận nghịch H2 (g) + I2 (g) ⇌ 2HI (g) có giá trị
hằng số cân bằng Kp = 45,9 ở 490 oC. Hãy tính Go của phản ứng ở điều
kiện nhiệt độ này.

119
Giải
Go = – RT lnKp = - 8,314.10-3.763.ln45,9 = -24,273 (kJ)
Câu 5.17. Cho phản ứng với các dữ kiện sau:
H2 (g) + CO2 (g) ⇌ H2O (g) + CO (g)
H f (kJ/mol) 0
o
- 393,5 - 241,8 - 110,5
o -1 -1
S (J.mol .K ) 130,6 213,6 188,7 197,6
Tính hằng số cân bằng Kp của phản ứng ở 25 C.
o

Giải
Ta có:
Ho = – 241,8 – 110,5 + 393,5 = 41,2 (kJ)
So = 188,7 + 197,6 – 130,6 – 213,6 = 42,1 (J.K-1)
Go = Ho – T.So = 41,2 – 298.42,1.10-3 = 28,65 (kJ)
28,65
lnK P = − 8,314.10−3.298 → KP = 9,50.10-6
Câu 5.18. Cho phản ứng thuận nghịch với các dữ kiện sau:
2NO2 (g) ⇌ N2O4 (g)
G f (kJ/mol)
o
51,30 97,82
a) Tính KP, KC của phản ứng ở 25 C.
o

b) Từ đó, hãy tính KP và KC của phản ứng thuận nghịch N2O4 (g) ⇌ 2NO2
(g) ở 25oC.
Giải
a) Go = 97,82 – 2 . 51,30 = – 4,78 kJ
Go = – RT lnKp
→ – 4,78 = – 8,314.10-3.298 . lnKp
→ KP = 6,88
→ Ngoài ra, Kp = KC (RT)n
Với n = 1 – 2 = – 1, tính được KC = 168,12
b) Gọi KP’ là hằng số cân bằng của phản ứng.
PN2 O4 P2NO2 1 1
Ta có K P = và K′P = = Kp = 6,88 = 0,15
P2NO2 PN2O4

Tương tự, K’C = 5,95.10 -3

Dạng 6. Nguyên lý chuyển dịch cân bằng (Le Chatelier)


Câu 5.19. Một hệ phản ứng tổng hợp NH3 từ N2 và H2 theo phương trình
phản ứng thuận nghịch sau và đang ở trạng thái cân bằng:
N2 (g) + 3H2 (g) 2NH3 (g), Ho < 0.

120
Cân bằng phản ứng sẽ dịch chuyển theo chiều nào trong các trường hợp
sau đây:
a) Bơm thêm khí H2 vào bình (thể tích bình không đổi).
b) Nén piston để giảm thể tích của bình.
c) Đun nóng bình (thể tích bình không đổi).
Giải
Phân tích phản ứng thuận nghịch như sơ đồ sau:

Theo phương trình phản ứng trên, tính chất của phản ứng khi diễn tiến như
sau:
• Khi phản ứng diễn tiến theo chiều thuận thì: nồng độ N2 và H2 giảm;
nồng độ NH3 tăng; áp suất hệ phản ứng giảm và nhiệt độ hệ phản ứng
tăng.
• Khi phản ứng diễn tiến theo chiều nghịch thì: nồng độ N2 và H2 tăng;
nồng độ NH3 giảm; áp suất hệ phản ứng tăng và nhiệt độ hệ phản ứng
giảm.
Do đó, theo nguyên lý chuyển dịch cân bằng ta có các kết luận:
a) Khi bơm thêm H2 vào bình (tăng H2) thì phản ứng dịch chuyển theo
chiều giảm nồng độ H2 là chiều thuận.
b) Khi nén piston để giảm thể tích của bình (tăng áp suất) thì phản ứng
dịch chuyển theo chiều giảm áp suất là chiều thuận.
c) Khi đun nóng bình (nhiệt độ tăng) thì phản ứng dịch chuyển theo chiều
giảm nhiệt độ là chiều nghịch.
Câu 5.20. Một hệ phản ứng tổng hợp NH3 từ N2 và H2 theo phương trình
phản ứng thuận nghịch sau và đang ở trạng thái cân bằng:
N2 (g) + 3H2 (g) ⇌ 2NH3 (g), Ho < 0.
Muốn tăng hiệu suất của phản ứng thì giải pháp kỹ thuật nào cần được thực
hiện đối với các yếu tố nồng độ, nhiệt độ và áp suất.
Giải
Muốn tăng hiệu suất của phản ứng thì phản ứng phải dịch chuyển theo
chiều thuận. Như vậy, các giải pháp kỹ thuật có thể sử dụng là:
• Nén thêm N2 và H2 vào bình phản ứng.

121
• Rút NH3 ra khỏi bình phản ứng.
• Giảm nhiệt độ bình phản ứng.
• Tăng áp suất bình phản ứng.
Câu 5.21. Một hệ phản ứng tổng hợp SO3 từ SO2 và O2 theo phương trình
phản ứng thuận nghịch sau và đang ở trạng thái cân bằng:
2SO2 (g) + O2 (g) ⇌ 2SO3 (g), Ho < 0.
Nồng độ SO3 sẽ tăng hay giảm khi thực hiện các giải pháp kỹ thuật sau:
a) Tăng nhiệt độ
b) Tăng áp suất
c) Nén SO2 vào bình
d) Thêm xúc tác thích hợp
e) Nén khí Heli vào bình
Giải
a) Tăng nhiệt độ phản ứng dịch chuyển theo chiều nghịch nên nồng độ SO3
giảm.
b) Tăng áp suất phản ứng dịch chuyển theo chiều thuận nên nồng độ SO3
tăng.
c) Nén SO2 vào bình phản ứng dịch chuyển theo chiều thuận nên nồng độ
SO3 tăng.
d) Cân bằng không dịch chuyển khi thêm xúc tác nên nồng độ SO3 không
thay đổi.
e) Việc thêm một khí trơ cũng không làm dịch chuyển cân bằng nên nồng
độ SO3 không thay đổi.

122
Phần 3. BÀI TẬP TRẮC NGHIỆM
---oOo---
Câu 5.1. Phản ứng thuận nghịch là:
A. Phản ứng có thể xảy ra đồng thời theo hai chiều ngược nhau trong cùng
một điều kiện.
B. Phản ứng xảy ra theo hai chiều ngược nhau tùy điều kiện phản ứng.
C. Phản ứng tự xảy ra cho đến khi hết các chất phản ứng.
D. Phản ứng có hiệu suất 100%.
Câu 5.2. Biểu thức tính hằng số cân bằng KP của phản ứng N2(g) + 3H2(g)
↔ 2NH3(g) nào sau đây là đúng?
P2NH3 𝑃𝑁𝐻3
A. K P = P 3 B. K P = 𝑃
N2 PH2 𝑁 2 𝑃𝐻
2
PN2 P3H2 PN2 𝑃𝐻2
C. K P = D. K P =
P2NH3 PNH3

Câu 5.3. Chọn biểu thức KC đúng:


[CO][H2 O]
A. CO2 (g) + H2 (g) ⇌ CO (g) + H2O (g) KC =
[CO2 ][H2 ]
[NH3 ]
B. N2 (g) + 3H2 (g) ⇌ 2NH3 (g) KC =
[N2 ][H2 ]
[CO2 ]3 [Fe]
C. Fe2O3 (s) + 2CO (g) ⇌ 2Fe (s) + 3CO2 (g) KC =
[CO]3
[NO2 ]2
D. 2NO (g) + O2 (g) ⇌ 2NO2 (g) KC =
[CO] + [O2 ]
Câu 5.4. Biểu thức nào sau đây biểu diễn mối quan hệ giữa hằng số cân
bằng và thế đẳng áp đẳng nhiệt một cách tổng quát nhất (ở đktc):
A. Go = - 4,576.T.lnKP B. Go = - R.T.lnKp
∆Ho ∆S0

C. Go = - R.T.lnKC D. k p = 10 4,567T 104,567
Câu 5.5. Với một phản ứng thuận nghịch có G < 0. Phát biểu nào dưới
đây là phù hợp với hệ cân bằng?
A. Độ lớn của hằng số cân bằng < 1
B. Độ lớn của hằng số cân bằng > 1
C. Độ lớn của hằng số cân bằng = 1
D. Độ lớn của hằng số cân bằng = 0
Câu 5.6. Ở một nhiệt độ, phản ứng thuận nghịch dưới đây có hằng số cân
bằng KC = 4

123
A (g) + B (g) ⇌ C (g) + D (g)
Tại một thời điểm nào đó, nồng độ mol của từng chất như sau: [A] = 0,2
M, [B] = 0,2 M, [C] = 0,2 M, [D] = 0,4 M.
Phát biểu nào dưới đây là đúng ứng với thời điểm này:
A. Hệ đang ở trạng thái cân bằng.
B. Chiều thuận của phản ứng đang diễn tiến ưu thế
C. Chiều nghịch của phản ứng đang diễn tiến ưu thế
D. Không đủ thông tin để kết luận về diễn biến của phản ứng
Câu 5.7. Tại một nhiệt độ xác định, phản ứng thuận nghịch CO2(g) + H2(g)
⇌ CO(g) + H2O(g) có Kc = 9/4. Giả sử lúc đầu, trong bình phản ứng có
1mol CO2, 1 mol H2, 1 mol CO và 1 mol H2O. Tại trạng thái cân bằng, số
mol CO là bao nhiêu?
A. 0,12 B. 0,24
C. 1,20 D. 2,40
Câu 5.8. Tại 300 C, trộn 1,0 mol A; 1,4 mol B và 0,5 mol C vào bình kín
o

thể tích 1,0 lít. Phản ứng A(g) + B(g) ⇌ 2C(g) xảy ra. Khi cân bằng nồng
độ của C trong bình là 0,75M. Hãy xác định hằng số cân bằng Kc của phản
ứng.
A. 0,05 B. 0,5 C. 5 D. 50
Câu 5.9. Trộn 1,0 mol khí CO với 3 mol hơi nước ở 850 C trong một bình
o

phản ứng dung tích 1 lít để thực hiện phản ứng thuận nghịch CO(g) +
H2O(g) ⇌ CO2(g) + H2(g). Tại trạng thái cân bằng, số mol cacbonic thu
được là 0,75 mol. Phát biểu nào dưới đây là đúng?
A. Kc = 1, Kp = 2 B. Kc = 1, Kp = 1
C. Kc = 2, Kp = 1 D. Kc = 2, Kp = 2
Câu 5.10. Phản ứng phân hủy Hydro iodua 2HI(g) ⇌ H2(g) + I2(g) tại một
nhiệt độ xác định có Kc= 1/64. Hãy xác định tỷ lệ % HI phân hủy tại nhiệt
độ phản ứng.
A. 10 B. 20 C. 30 D. 40
Câu 5.11. Phát biểu nào dưới đây là đúng?
A. Một hệ ở trạng thái cân bằng, nếu ta thay đổi bất kỳ một yêu tố nào xác
định điều kiện cân bằng (áp suất khí, nồng độ, nhiệt độ) thì cân bằng sẽ
chuyển dịch theo chiều chống lại sự thay đổi đó.
B. Khi nhiệt độ của một hệ cân bằng tăng, cân bằng sẽ chuyển dịch theo
chiều tỏa nhiệt, khi nhiệt độ của hệ giảm cân bằng sẽ chuyển dịch theo
chiều hấp thụ nhiệt.

124
C. Khi áp suất của hệ cân bằng giảm, cân bằng sẽ chuyển dịch theo chiều
giảm số phân tử khí.
D. Khi thêm một lượng tác chất hoặc sản phẩm vào hệ cân bằng, cân bằng
sẽ chuyển dịch theo hướng gia tăng thêm lượng chất đó.
Câu 5.12. Phản ứng thuận nghịch CO2(g) + H2(g) ⇌ CO(g) + H2O(g) được
thực nghiệm trong một bình kín thể tích 1 lít. Ở trạng thái cân bằng, trong
bình phản ứng có 0,4 mol CO2, 0,4 mol H2, 0,8 mol CO và 0,8 mol hơi
nước. Phát biểu nào dưới đây là phù hợp với hệ cân bằng và sau khi tăng
áp suất chung của hệ?
A. KC = 8, cân bằng chuyển dịch theo chiều thuận
B. KC = 8, cân bằng chuyển dịch theo chiều nghịch
C. KC = 4, cân bằng chuyển dịch theo chiều thuận
D. KC = 4, cân bằng không chuyển dịch
Câu 5.13. Có một phản ứng thuận nghịch sau H2O(g) ⇌ H2(g) + 1/2 O2(g),
Ho. Khi tăng nhiệt độ, giá trị hằng số cân bằng của phản ứng tăng. Phát
biểu nào dưới đây là phù hợp với đặc điểm của đại lượng Ho của phản
ứng?
A. Ho > 0 B. Ho = 0
C. Ho < 0 D. Không thể kết luận
Câu 5.14. Phản ứng thuận nghịch sau có chiều thuận là chiều thu nhiệt:
A(g) + B (g) ⇌ 2C(g).
Để cân bằng chuyển dịch theo chiều thuận, biện pháp nào dưới đây cần
làm?
A. Tăng áp suất B. Tăng nhiệt độ
C. Giảm áp suất D. Giảm nhiệt độ
Câu 5.15. Cho các phản ứng thuận nghịch sau ở 820 oC:
CaCO3 (s) ⇌ CaO (s) + CO2 (g), K1 = 0,200
C (gr) + CO2 (g) ⇌ 2CO (g), K2 = 2,00
Xác định hằng số cân bằng K3 của phản ứng CaCO3 (s) + C (gr) ⇌ CaO
(s) + 2CO (g) ở 820 oC.
A. 0,100 B. 1,00
C. 0,400 D. 0,300
Câu 5.16. Có cân bằng sau: 2SO3(g) ⇌ 2SO2(g) + O2(g), Ho > 0.
Phát biểu nào sau đây là đúng?
A. Khí thêm khí SO2 vào hệ, cân bằng chuyển dịch theo chiều thuận
B. Khi giảm nhiệt độ, cân bằng chuyển dịch theo chiều thuận
125
C. Khi giảm áp suất, cân bằng chuyển dịch theo chiều nghịch
D. Khi tăng áp suất kèm theo giảm nhiệt độ, cân bằng chuyển dịch theo
chiều nghịch.
Câu 5.17. Tác động nào sẽ làm tăng hiệu suất đối với phản ứng thuận
nghịch sau:
CaCO3 (s) ⇌ CaO (s) + CO2 (g), Ho > 0
A. Tăng nhiệt độ B. Hạ nhiệt độ
C. Tăng áp suất D. Tăng nồng độ CO2
Câu 5.18. Cho phản ứng thuận nghịch 4HCl (g) + O2 (g)  2Cl2 (g) +
2H2O (g). Trong điều kiện nào cân bằng chuyển dịch về bên trái?
A. Giảm áp suất hệ phản ứng
B. Tăng nồng độ oxy
C. Giảm thể tích của hệ phản ứng xuống 2 lần
D. Giảm nồng độ clo
Câu 5.19. Xét cân bằng hóa học sau:
2NO2 (g) ⇌ N2O4 (g), Ho298 = - 14kcal/mol
(nâu) (không màu)
Màu nâu của NO2 sẽ đậm nhất khi:
A. Đun nóng lên 373oK B. Làm lạnh đến 0oC
C. Tăng áp suất D. Giữ ở 298oK
Câu 5.20. Phản ứng thuận nghịch 2N2O5 (g) ⇌ O2 (g) + 4NO2 (g) đang
cân bằng. Nếu giảm thể tích của bình phản ứng xuống 2 lần thì:
A. Cân bằng dịch chuyển sang trái
B. Tốc độ phản ứng thuận và nghịch thay đổi như nhau
C. Cân bằng không chuyển dịch
D. Cân bằng chuyển dịch sang phải
Câu 5.21. Hằng số cân bằng của phản ứng CO(g) + H2O(g) ⇌ H2(g) +
CO2(g) ở 858oC bằng 1. Thực hiện phản ứng trên tại 858oC với nồng độ
ban đầu của CO là 1M và H2O là 3M. Nồng độ của CO và H2 lúc cân bằng
lần lượt là:
A. 0,75 M và 0,25 M B. 0,25 M và 0,75M
C. 2,25 M và 0,75 M D. 0.75 M và 2,25 M
Câu 5.22. Cho phản ứng thuận nghịch sau: NO2(g) ⇌ NO(g) + 1/2O2(g)
có hằng số cân bằng K1 Biểu thức hằng số cân bằng K2 nào sau đây là đúng
cho phản ứng NO(g) + 1/2O2(g) ⇌ NO2(g) ở cùng điều kiện?
1
A. K 2 = K B. K2 = K1
1
126
C. K2 = K12 D. K 2 = ඥK1
Câu 5.23. Cho phản ứng sau:
4HCl(g) + O2(g) ⇌ 2H2O(g) + 2Cl2(g), H < 0
Nhiệt độ và áp suất có ảnh hưởng đến cân bằng của phản ứng trên là:
A. Giảm nhiệt độ, tăng áp suất thì cân bằng phản ứng dịch chuyển theo
chiều nghịch.
B. Tăng nhiệt độ, giảm áp suất thì cân bằng phản ứng dịch chuyển theo
chiều thuận.
C. Tăng nhiệt độ, tăng áp suất thì cân bằng phản ứng dịch chuyển theo
chiều nghịch.
D. Giảm nhiệt độ, giảm áp suất thì cân bằng phản ứng dịch chuyển theo
chiều nghịch.
Câu 5.24. Cho phản ứng thuận nghịch 2CuO(s) ⇌Cu(s) + O2(g). Hằng số
cân bằng của phản ứng được viết như thế nào là đúng?
1
A. K = [O B. K = [O2 ]
2]
1
C. K = [O2 ]1/2 D. K = [O 1/2
2]
Câu 5.25. Cho phản ứng H2(g) + Br2(g) ⇌ 2HBr(g) có Kp = 3,5.104 ở 1495
K. Hãy xác định Kp của phản ứng 1/2H2(g) + 1/2Br2(g) ⇌ HBr(g) ở 1495K.
A. 1,50.1010 B. 1,22.109
C. 3,50.102 D. 1,87.102
Câu 5.26. Khí N2 (áp suất riêng phần ban đầu 1 atm) phản ứng với khí H2
(áp suất riêng phần ban đầu 2 atm) tạo thành NH3 trong bình kín ở 2000oC
theo phản ứng:
N2 (g) + 3H2 (g) ⇌ 2NH3 (g)
Tại thời điểm cân bằng, áp suất tổng trong bình là 2 atm. Xác định áp suất
riêng phần của H2 tại thời điểm cân bằng.
A. 4 atm B. 1 atm
C. 0,5 atm D. 2 atm
Câu 5.27. Cho phản ứng thuận nghịch 2NO(g) + Cl2(g) ⇌ 2NOCl(g).
Nồng độ ban đầu của NO và Cl2 lần lượt là 0,5M và 0,2M. Ở 25oC khi
phản ứng đạt cân bằng có 20% NO đã tham gia phản ứng. KP của phản ứng
ở nhiệt độ 25oC là bao nhiêu?
A. 0,017 B. – 0,017
C. 0,416 D. – 0,416

127
Câu 5.28. Cho phản ứng: 3Fe(s) + 4H2O(g) ⇌ Fe3O4(s) + 4H2(g). Tại
1200K phản ứng đạt cân bằng, áp suất riêng phần của hơi nước là 0,02
atm, áp suất tổng của hệ là 0,05 atm. Xác định Kp của phản ứng tại 1200K.
A. 1,5 B. 10,12
C. 3,0 D. 5,06
Câu 5.29. Ở 820 C hằng số cân bằng Kp của các phản ứng như sau:
o

CaCO3 (s) ⇌ CaO (s) + CO2 (g), K1 = 0,200


C (gr) + CO2 (g) ⇌ 2CO (g), K2 = 2,00
Cho 1,00 mol CaCO3 (s) và 1,00 mol C (gr) vào bình chân không dung tích
22,4 lít ở 820 oC thực hiện phản ứng CaCO3 (s) + C (gr) ⇌ CaO (s) + 2CO
(g). Tính số mol CO trong bình phản ứng lúc cân bằng.
A. 0,158 B. 0,129
C. 0,871 D. 0,921
Câu 5.30. Cho các phát biểu sau đây:
(i) Một hệ ở trạng thái cân bằng, nếu thay đổi bất kỳ một yếu tố nào xác
định điều kiện cân bằng (áp suất khí, nồng độ, nhiệt độ) thì cân bằng sẽ
chuyển dịch theo chiều chống lại sự thay đổi đó.
(ii) Khi nhiệt độ của một hệ cân bằng tăng, cân bằng sẽ chuyển dịch theo
chiều thu nhiệt, khi nhiệt độ của hệ giảm cân bằng sẽ chuyển dịch theo
chiều tỏa nhiệt.
(iii) Khi áp suất của hệ cân bằng giảm, cân bằng sẽ chuyển dịch theo chiều
giảm số phân tử khí.
Có bao nhiêu phát biểu đúng:
A. 2 B. 0 C. 1 D. 3
Câu 5.31. Cho phản ứng N2(g) + O2(g) ⇌ 2NO(g) có Kp = 0,05 tại nhiệt
độ 2200oC. Áp suất riêng phần ban đầu của N2 bằng 0,8 atm và của O2
bằng 0,2 atm. Hãy xác định áp suất riêng phần của NO (atm) lúc cân bằng
được thiết lập ở 2200oC.
A.0,052 atm B. 0,078 atm
C. 0,039 atm D. 0,025 atm
Câu 5.32. Cho phản ứng thuận nghịch sau C (gr) + H2O (g) ⇌ CO (g) +
H2(g) có ∆Go298 = – 79,854 kJ. Hãy xác định Kp của phản ứng ở điều kiện
tiêu chuẩn.
A. 2.105 B. 9,95.1013
C. 2.1014 D. 5,3.1010

128
Câu 5.33. Tại một nhiệt độ xác định, cho 12 mol khí SO3 vào bình kín có
thể tích 3 lít thì xảy ra phản ứng 2SO3(g) ⇌ 2SO2(g) + O2(g). Khi phản
ứng đạt cân bằng, số mol SO2 là 3 mol. Hãy xác định hằng số cân bằng KC
của phản ứng trên.
A. 0,055 B. 0,5 C. 1,0 D. 0,11
Câu 5.34. Trạng thái cân bằng hóa học của một phản ứng thuận nghịch có
tính chất:
A. Là cân bằng như cân bằng cơ học.
B. Là cân bằng tĩnh.
C. Là cân bằng tuyệt đối.
D. Là cân bằng động.
Câu 5.35. Biểu thức tính hằng số cân bằng KC sau đây biểu diễn cho phản
ứng hóa học nào?
[NH3 ]2
KC =
[N2 ][H2 ]3
A. N2 (g) + 3H2 (g) ⇌ 2 NH3 (g)
B. 2 NH3 (g) ⇌ N2 (g) + 3H2 (g)
C. 1/2 N2 (g) + 3/2 H2 (g) ⇌ NH3 (g)
D. NH3 (g) ⇌ 1/2 N2 (g) + 3/2 H2 (g)
Câu 5.36. Xét phản ứng: CO (g) + 2H2 (g) ⇌ CH3OH (g) xảy ra ở 780 oC
là phản ứng thu nhiệt mạnh theo chiều thuận. Sau khi phản ứng đạt cân
bằng, muốn thu được nhiều CH3OH (g) thì giải pháp nào sau đây được
thực hiện?
A. Giảm nhiệt độ, tăng áp suất
B. Tăng nhiệt độ, giảm áp suất
C. Tăng nhiệt độ, tăng áp suất
D. Giảm nhiệt độ, giảm áp suất
Câu 5.37. Xét phản ứng thuận nghịch 2NO (g) + Cl2 (g) ⇌ 2NOCl (g) xảy
ra tại nhiệt độ xác định. Một hỗn hợp phản ứng ban đầu chứa [NO] = 0,10
M và [Cl2] = 0,05 M. Sau khi, phản ứng đạt cân bằng, nồng độ của NOCl
là 0,04 M. Giá trị của hằng số cân bằng KC của phản ứng tại nhiệt độ trên
là bao nhiêu?
A. 14,0 B. 14,8
C. 15,0 D. 15,8
Câu 5.38. Xét phản ứng FeO (s) + H2(g) ⇌ Fe(s) + H2O(g). Cho 1 mol
FeO phản ứng với 1,5 mol H2 ở nhiệt độ 1000K. Hãy tính % FeO đã phản
129
ứng khi hệ đạt trạng thái cân bằng, biết rằng ở nhiệt độ này phản ứng có
Kp = 0,5.
A. 40% B. 60% C. 50% D. 70%
Câu 5.39. Cho biết có bao nhiêu phát biểu đúng đối với trạng thái cân bằng
hóa học:
(i) Là cân bằng động.
(ii) Là thời điểm có tốc độ phản ứng thuận bằng tốc độ phản ứng nghịch.
(iii) Không thay đổi khi các yếu tố bên ngoài cố định.
(iv) Được đặc trưng bằng một hằng số cân bằng.
A. 3 B. 4 C. 1 D. 2
Câu 5.40. Có tổng cộng bao nhiêu phát biểu đúng về đặc điểm của phản
ứng thuận nghịch trong số các phát biểu sau?
(i) Có hiệu suất bằng 100% khi thời gian phản ứng đủ lớn.
(ii) Có hiệu suất càng cao khi hằng số cân bằng có giá trị càng lớn.
(iii) Các phản ứng trong hệ phản ứng thuận nghịch không bao giờ dừng lại
dù đã đạt trạng thái cân bằng.
A. 0 B. 1 C. 2 D. 3
----oOo----

ĐÁP ÁN BÀI TẬP CHƯƠNG 5


5.1- 5.2- 5.3- 5.4- 5.5- 5.6- 5.7- 5.8- 5.9- 5.10-
A A A B B B C B B B
5.11- 5.12- 5.13- 5.14- 5.15- 5.16- 5.17- 5.18- 5.19- 5.20-
A D A B D C A D A A
5.21- 5.22- 5.23- 5.24- 5.25- 5.26- 5.27- 5.28- 5.29- 5.30-
B A D B D C A D A A
5.31- 5.32- 5.33- 5.34- 5.35- 5.36- 5.37- 5.38- 5.39- 5.40-
B B A D A C B C B C

130
Chương 6. DUNG DỊCH –
DUNG DỊCH KHÔNG ĐIỆN LY
---oOo---

Mục tiêu chương 6


Sau khi học xong chương này, sinh viên có khả năng:
1. Định nghĩa được dung dịch và các hệ phân tán.
2. Tính toán và chuyển đổi được các loại nồng độ dung dịch.
3. Giải thích được cơ chế tạo thành dung dịch.
4. Định nghĩa được độ tan và giải thích các yếu tố ảnh hưởng đến độ
tan.
5. Định nghĩa được áp suất hơi bão hòa, nhiệt độ sôi, nhiệt độ đông
đặc và áp suất thẩm thấu của dung dịch lỏng.
6. Phát biểu được các định luật Raoult 1 và Raoult 2 từ đó tính được
độ giảm áp suất hơi bão hòa, nhiệt độ sôi, nhiệt độ đông đặc và áp
suất thẩm thấu của chất lỏng.

131
Phần 1. TÓM TẮT LÝ THUYẾT
6.1 KHÁI NIỆM VỀ DUNG DỊCH
6.1.1 Các hệ phân tán và dung dịch
Hệ phân tán là những hệ trong đó có ít nhất một chất phân bố (chất phân
tán) vào một chất khác (môi trường phân tán) dưới dạng những hạt có kích
thước nhỏ. Các hệ phân tán bao gồm:
• Hệ phân tán thô là những hệ vi dị thể có đường kính hạt lớn hơn
10-5 cm.
• Hệ phân tán cao là những hệ siêu vi dị thể có đường kính hạt trong
khoảng 10-5 – 10-7 cm.
• Hệ dung dịch thực là những hệ có có kích thước hạt nhỏ hơn 10-7 cm
(cỡ phân tử hay ion).
6.1.2 Dung dịch
Dung dịch là hệ đồng thể gồm hai hay nhiều chất mà thành phần của nó có
thể thay đổi trong một giới hạn rộng.
Dung dịch gồm dung môi (môi trường phân tán) và chất tan (chất phân
tán).
mdd = mct + mdm = a + b (6.1)
Trong đó,
• mdd = a + b: khối lượng dung dịch
• mct = a: khối lượng chất tan
• mdm = b: khối lượng dung môi
Có ba loại dung dịch: dung dịch khí, dung dịch lỏng và dung dịch rắn.
Trong đó dung dịch lỏng với dung môi là nước có vai trò quan trọng nhất
trong đời sống, khoa học và kỹ thuật.
6.2 NỒNG ĐỘ CỦA DUNG DỊCH
Nồng độ dung dịch là lượng chất tan trong một lượng hay một thể tích nhất
định của dung dịch hoặc dung môi.
Các loại nồng độ dung dịch được tóm tắt trong Bảng 6.1.
Bảng 6.1. Các loại nồng độ dung dịch
Liên hệ giữa
Đại Ký Đơn
Diễn giải Công thức các công
lượng hiệu vị
thức
số gam chất tan có
Nồng a CM . M
C% % trong 100 g dung C% = 100 C% =
độ a+b
10. d
dịch
132
phần
trăm
Nồng số mol chất tan có
CM a C%. d. 10
độ M trong 1 L dung dịch CM = CM =
M. V M
mol
Nồng tỷ số giữa số mol nA
A =
độ của một chất với nA + nB
i - nB
phần tổng số mol của các B =
mol chất trong hỗn hợp nA + nB
Nồng số mol chất tan có
Cm a. 1000
độ mol/kg trong 1 kg dung môi Cm =
M. b
molan
Nồng số đương lượng gam
độ chất tan có trong 1 L a
CN N CN = CN = n.CM
đương dung dịch Đ. V
lượng

Trong đó,
• a (g): khối lượng chất tan
• b (g): khối lượng dung môi
• V (L): thể tích dung dịch
• M (g/mol): khối lượng phân tử chất tan
• nA, nB: số mol chất A và chất B
• Đ: đương lượng chất tan
• n: số đơn vị đương lượng, phụ thuộc vào phản ứng hóa học
• d (g/mL): khối lượng riêng dung dịch
6.3 PHA TRỘN DUNG DỊCH THEO PHƯƠNG PHÁP ĐƯỜNG
CHÉO
Pha trộn dung dịch theo nồng độ phần trăm (C%)
(6.2)

Pha trộn dung dịch theo nồng độ mol (CM)


(6.3)

Pha trộn dung dịch theo tỷ trọng (d)


133
(6.4)

Lưu ý:
- Chất rắn xem như dung dịch có nồng độ 100%.
- Dung môi xem như dung dịch có nồng độ 0%.
6.4 CƠ CHẾ TẠO THÀNH DUNG DỊCH
Cơ chế tạo thành dung dịch theo quan niệm hiện đại bao gồm hai quá trình:
• Quá trình vật lý (quá trình chuyển pha): là quá trình phá vỡ mạng tinh
thể chất tan và phân bố các tiểu phân chất tan tạo thành trong dung môi.
• Quá trình hóa học (quá trình solvat hóa): là quá trình tương tác của
các tiểu phân chất tan với dung môi tạo thành hợp chất solvat. Nếu dung
môi là nước gọi là quá trình hydrate hóa.
Cơ chế tạo thành dung dịch được minh họa trong Hình 6.1.

Hình 6.1: Quá trình hòa tan NaCl và nước


6.5 ĐỘ TAN VÀ CÁC YẾU TỐ ẢNH HƯỞNG
6.5.1 Độ tan
Độ tan (S) của một chất là nồng độ của dung dịch bão hòa của chất đó ở
điều kiện xác định.
• Đối với chất rắn: S thường được tính bằng số gam chất tan tan được
trong 100 gam dung môi.
• Đối với chất khí: S thường được tính bằng số mL chất tan tan được
trong 100 g dung môi.
• Đối với chất điện ly ít tan: S thường được tính bằng số mol chất tan tan
được trong 1 L dung dịch (nồng độ mol/L hay M).
Đánh giá mức độ hòa tan theo độ tan như sau:
Gần như không tan Khó tan Dễ tan

S (g/100g dung môi) 0,01 1 10

134
6.5.2 Các yếu tố ảnh hưởng đến độ tan
a) Ảnh hưởng của bản chất chất tan và dung môi
Quy tắc kinh nghiệm: “Các chất có cấu tạo phân tử giống nhau thì dễ hòa
tan vào nhau”. Quy tắc này có nghĩa là chất phân cực tan trong dung môi
phân cực, chất kém phân cực tan trong dung môi kém phân cực.
b) Ảnh hưởng của nhiệt độ đến độ tan
• Quá trình hòa tan khí trong lỏng: A (g) + D (l) ⇌ A (aqu), H < 0 nên
khi nhiệt độ giảm thì độ tan tăng, và ngược lại.
• Quá trình hòa tan lỏng trong lỏng (trường hợp hai chất hòa tan hữu hạn)
thì thông thường có H > 0 nên khi nhiệt độ giảm thì độ tan giảm, và
ngược lại.
• Quá trình hòa tan rắn trong lỏng thì tùy thuộc vào quá trình hòa tan là
thu nhiệt hay tỏa nhiệt. Nếu quá trình là thu nhiệt H > 0 thì khi nhiệt
độ giảm thì độ tan giảm, và ngược lại. Nếu quá trình là tỏa nhiệt
H < 0 thì khi nhiệt độ giảm thì độ tan tăng, và ngược lại.
c) Ảnh hưởng của áp suất đến độ tan
Áp suất không ảnh hưởng nhiều đến độ tan của chất rắn và chất lỏng nhưng
ảnh hưởng đáng kể đến chất khí.
Đối với quá trình hòa tan khí trong lỏng, khi nhiệt độ không thay đổi, áp
suất tăng thì độ tan chất khí tăng.
d) Ảnh hưởng của ion lạ
Tăng nồng độ các ion cùng tên sẽ làm chuyển dịch cân bằng điện ly của
chất tan về phía dạng phân tử ít tan cho nên làm giảm độ tan của chất. Vì
vậy trong kỹ thuật hòa tan, cần chọn thứ tự hòa tan thích hợp, hòa tan chất
ít tan trước rồi mới hòa tan chất dễ tan.
6.6 DUNG DỊCH KHÔNG ĐIỆN LY VÀ CÁC TÍNH CHẤT
Tính chất tập hợp của dung dịch chỉ phụ thuộc vào số lượng phân tử chất tan
trong một thể tích dung môi và không liên quan đến tính chất riêng của phân tử
chất tan. Tính chất tập hợp liên quan đến các hiện tượng: độ giảm áp suất hơi; độ
tăng nhiệt độ sôi; độ hạ nhiệt độ đông đặc; và áp suất thẩm thấu, được thể hiện
thông qua định luật Raoult 1, Raoult 2 và định luật Van’t Hoff. Lưu ý rằng, chỉ
áp dụng đúng đối với dung dịch loãng chứa chất tan không bay hơi, không điện
ly và không tạo kết tủa rắn trong dung môi.
6.6.1 Áp suất hơi bão hòa
a) Áp suất hơi bão hòa của chất lỏng nguyên chất
Bay hơi (H>0)
Chất lỏng Chất hơi
Ngưng tụ (H<0)
135
Hơi tạo thành trên bề mặt chất lỏng gây ra áp suất hơi và khi quá trình
thuận nghịch đạt trạng thái cân bằng (Gbh = 0) thì áp suất hơi được gọi là
áp suất hơi bão hòa của chất lỏng hay dung môi nguyên chất.
Áp suất hơi bão hòa đặc trưng cho sự bay hơi của các chất lỏng. Nó là đại lượng
phụ thuộc vào nhiệt độ (tăng khi nhiệt độ tăng) và bản chất của chất lỏng.
b) Độ giảm áp suất hơi bão hòa của dung dịch - Định luật Raoult 1
Bay hơi (H>0)
Dung môi (l) + Chất tan Chất hơi
Ngưng tụ (H<0)
Dung dịch
Khi thêm chất tan vào dung môi lỏng thì nồng độ dung môi lỏng sẽ giảm,
làm cân bằng chuyển dịch về phía chiều tạo thêm dung môi lỏng, nghĩa là
giảm sự tạo thành chất hơi. Do đó, áp suất hơi bão hòa trên bề mặt giảm.
• Định luật Raoult 1. Độ giảm tương đối áp suất hơi bão hòa của dung
môi trong dung dịch loãng với chất tan không điện ly, không bay hơi,
bằng phân số mol chất tan trong dung dịch.
P0 −P ∆P
= = ct (xct: phân số mol của chất tan) (6.5)
P0 P0
P
hay = dm (xdm: phân số mol của dung môi)
P0
Đơn vị áp suất: atm, mmHg.
6.6.2 Nhiệt độ sôi và nhiệt độ đông đặc
a) Nhiệt độ sôi của chất lỏng

Bay hơi (H>0)


Chất lỏng Chất hơi
Ngưng tụ (H<0)
Nhiệt độ sôi của một chất lỏng là nhiệt độ tại đó áp suất hơi bão hòa của
nó bằng áp suất môi trường xung quanh.
Bay hơi (H > 0)
Dung môi (l) + Chất tan Chất hơi
Ngưng tụ (H < 0)
Dung dịch
Nồng độ chất tan càng lớn, áp suất hơi bão hòa của dung dịch càng giảm
xuống, do đó nhiệt độ sôi của dung dịch càng cao.
• Định luật Raoult 2 (đối với nhiệt độ sôi): Độ tăng nhiệt độ sôi của
dung môi trong dung dịch loãng với chất tan không bay hơi và không
điện ly, tỷ lệ thuận với nồng độ molan của chất tan trong dung dịch.
136
ts = ts – tso = Ks x Cm (6.6)
Trong đó,
• ts: nhiệt độ sôi của dung dịch
• tso: nhiệt độ sôi của dung môi nguyên chất
• Ks hằng số nghiệm sôi của dung môi, phụ thuộc vào bản chất
của dung môi.
Đối với chất lỏng nguyên chất, khi sôi vẫn tiếp tục giữ nhiệt độ sôi không
thay đổi. Đối với dung dịch có chứa chất tan không bay hơi và không điện
ly, trong quá trình sôi, nồng độ dung dịch tăng lên và nhiệt độ dung dịch
tiếp tục tăng. Khi dung môi bay hơi đi làm nồng độ chất tan tăng dần đến
khi dung dịch đạt bão hòa, chất tan kết tinh tách khỏi dung dịch và bắt đầu
xuất hiện tinh thể chất tan → hệ đạt cân bằng hòa tan - kết tủa → nồng độ
dung dịch không thay đổi nữa, dung dịch tiếp tục giữ nhiệt độ sôi này.
b) Nhiệt độ đông đặc của chất lỏng

Đông đặc (H<0)


Chất lỏng Chất rắn
Tan chảy (H>0)

Nhiệt độ đông đặc của một chất lỏng là nhiệt độ tại đó áp suất hơi trên bề
mặt pha lỏng bằng áp suất hơi trên bề mặt pha rắn.
Đông đặc (H < 0)
Dung môi (l) + Chất tan Chất rắn
Tan chảy (H > 0)
Dung dịch
Dung dịch có nhiệt độ đông đặc thấp hơn dung môi nguyên chất.
Nồng độ dung dịch càng lớn, nhiệt độ đông đặc của dung dịch càng thấp.
• Định luật Raoult 2 (đối với nhiệt độ đông đặc): Độ giảm nhiệt độ đông đặc
của dung môi trong dung dịch loãng với chất tan không bay hơi và không
điện ly, tỷ lệ thuận với nồng độ molan của chất tan trong dung dịch.
tđ = tđo– tđ = Kđ x Cm (6.7)
Trong đó:
• tđ: nhiệt độ đông đặc của dung dịch
• tđo: nhiệt độ đông đặc của dung môi nguyên chất
• Kđ hằng số nghiệm đông của dung môi, phụ thuộc vào bản
chất của dung môi.

137
Đối với chất lỏng nguyên chất, khi đông đặc vẫn tiếp tục giữ nhiệt độ đông
đặc không thay đổi. Đối với dung dịch có chứa chất tan không bay hơi và
không điện ly, trong quá trình đông đặc, dung môi tách khỏi dung dịch,
nồng độ chất tan dung dịch tăng lên và nhiệt độ dung dịch tiếp tục giảm.
Đến khi bắt đầu xuất hiện tinh thể chất tan (khi dung môi tách ra khỏi dung
dịch làm nồng độ chất tan tăng dần đến khi dung dịch đạt bão hòa, chất tan
kết tinh tách khỏi dung dịch) làm cho thành phần dung dịch không thay
đổi nữa (hệ đạt cân bằng), dung dịch tiếp tục giữ nhiệt độ đông đặc này
đến khi toàn bộ dung dịch đông đặc thành một hỗn hợp rắn.
Lưu ý: ở áp suất 1 atm, nước sôi ở 100 oC và đông đặc ở 0 oC.
6.6.3 Áp suất thẩm thấu
Khi cho hai dung dịch có nồng độ khác nhau tiếp xúc với nhau, quan sát
thấy chất tan khuếch tán từ dung dịch có nồng độ đậm đặc sang dung dịch
có nồng độ loãng hơn, kết quả dẫn đến sự phân bố đồng đều chất tan trong
toàn bộ thể tích của hệ, đó là sự san bằng nồng độ.
Nếu đặt một màng bán thẩm (chỉ cho phép phân tử dung môi đi qua, còn
phân tử chất tan không thể đi qua) thì sự khuếch tán xảy ra một chiều. Gọi
là sự thẩm thấu.
Áp suất thẩm thấu của dung dịch được định nghĩa bằng áp suất bên ngoài
tác dụng lên dung dịch để cho hiện tượng thẩm thấu không xảy ra.
• Áp suất thẩm thấu của dung dịch không phụ thuộc bản chất chất tan và
dung môi mà chỉ phụ thuộc vào số lượng tiểu phân chất tan.
• Áp suất thẩm thấu của dung dịch tỷ lệ thuận với nồng độ chất tan và
nhiệt độ tuyệt đối của dung dịch.
 = CM R T (6.8)
Trong đó,
• : áp suất thẩm thấu (atm)
• T : nhiệt độ tuyệt đối (K)
• CM: nồng độ mol chất tan
• R = 0,082 L.atm.mol-1K-1
Khi thay CM = n/V sẽ có V = nRT, có dạng tương tự phương trình trạng
thái khí lý tưởng.
Từ đây, định luật Van’t Hoff được phát biểu: Áp suất thẩm thấu của dung
dịch có độ lớn bằng áp suất gây bởi chất tan, nếu như ở cùng nhiệt độ đó,
nó ở trạng thái khí và chiếm thể tích bằng thể tích dung dịch.

138
Phần 2. BÀI TẬP CÓ LỜI GIẢI
---oOo---
Dạng 1. Tính toán nồng độ dung dịch
Câu 6.1. Một dung dịch được điều chế bằng cách trộn 1,00 g ethanol
(C2H5OH) với nước để đạt được dung dịch có thể tích 100 mL. Hãy tính
nồng độ mol, nồng độ phần trăm, nồng độ phần mol và nồng độ molan của
dung dịch ethanol. Biết ở 20 oC khối lượng riêng của dung dịch này là
0,996 g/mL.
Giải
Ta có:
Chất tan: ethanol (C2H5OH). Dung môi: nước (H2O)
a = mct =1,00 g
M = Mct = 46,07 g.mol-1 (C2H5OH)
d = 0,996 g/mL
V = Vdd = 100 mL
→ mdd = d x Vdd = 99,6 g
→ b = mdm = mdd – mct = 99,6 -1,00 = 98,6 g
- Nồng độ mol:
𝐧𝐜𝐭 𝐦𝐜𝐭 𝟏, 𝟎𝟎
𝐂𝐌 = = =
𝐕𝐝𝐝 𝐌𝐜𝐭 × 𝐕𝐝𝐝 𝟒𝟔, 𝟎𝟕 × 𝟏𝟎𝟎 × 𝟏𝟎−𝟑
= 𝟎, 𝟐𝟏𝟕 ሺ𝐌ሻ
- Nồng độ phần trăm
𝐦𝐜𝐭 𝟏, 𝟎𝟎
𝐂% = 𝟏𝟎𝟎% = × 𝟏𝟎𝟎% = 𝟏, 𝟎𝟎𝟒%
𝐦𝐝𝐦 + 𝐦𝐜𝐭 𝟗𝟗, 𝟔
- Nồng độ phần mol
𝟏, 𝟎𝟎
𝐧𝐜𝐭 𝟒𝟔, 𝟎𝟕
𝐜𝐭 = = = 𝟎, 𝟎𝟎𝟑𝟗
𝐧𝐜𝐭 + 𝐧𝐝𝐦 𝟏, 𝟎𝟎 𝟗𝟖, 𝟔
+
𝟒𝟔, 𝟎𝟕 𝟏𝟖, 𝟎
- Nồng độ molan
𝐦𝐜𝐭 × 𝟏𝟎𝟎𝟎 𝟏, 𝟎𝟎 × 𝟏𝟎𝟎𝟎
𝐂𝐦 = = = 𝟎, 𝟐𝟐𝟎 ሺ𝐦𝐨𝐥/𝐤𝐠ሻ
𝐌𝐜𝐭 × 𝐦𝐝𝐦 𝟒𝟔, 𝟎𝟕 × 𝟗𝟖, 𝟔
Câu 6.2. Một dung dịch acid sulfuric điện giải trong aqui chì có nồng độ
H2SO4 3,75 M và khối lượng riêng 1,230 g/mL. Hãy tính nồng độ phần
trăm và nồng độ molan của dung dịch acid sulfuric.

139
Giải
Ta có:
Chất tan: acid sulfuric (H2SO4) (Mct = 98,1 g/mol). Dung môi: nước
(H2O).
CM = 3,75 mol/L
→ nghĩa là trong 1 L dung dịch có chứa 3,75 mol H2SO4.
d = 1,230 g/mL
→ nghĩa là 1 mL dung dịch nặng 1,230 g
Xét một dung dịch có Vdd = 1000 mL = 1 L thì:
→ mdd = 1230 g
→ nct = 3,75 mol
- Nồng độ phần trăm:
𝐦𝐜𝐭 𝐧𝐜𝐭 . 𝑴𝒄𝒕 𝟑, 𝟕𝟓 × 𝟗𝟖, 𝟏
𝐂% = 𝟏𝟎𝟎% = × 𝟏𝟎𝟎% = × 𝟏𝟎𝟎%
𝐦𝐝𝐝 𝐦𝐝𝐝 𝟏𝟐𝟑𝟎
= 𝟐𝟗, 𝟗𝟏%
- Nồng độ molan:
𝐧𝐜𝐭 𝐧𝐜𝐭 𝟑, 𝟕𝟓
𝐂𝐦 = = =
𝐦𝐝𝐦 𝐦𝐝𝐝− 𝐦𝐜𝐭 𝟏, 𝟐𝟑 − ሺ𝟗𝟖, 𝟏 × 𝟑, 𝟕𝟓 × 𝟏𝟎−𝟑ሻ
= 𝟒, 𝟑𝟓 ሺ𝐦𝐨𝐥/𝐤𝐠ሻ
Dạng 2. Xác định khối lượng phân tử
Câu 6.3. Một nhà hóa học tìm cách xác định một hormone kiểm soát quá
trình chuyển hóa bằng cách tìm khối lượng phân tử của nó. Thí nghiệm
được thực hiện bằng cách hòa tan 0,546 g mẫu trong 15,0 g benzene, xác
định độ giảm nhiệt độ đông đặc của dung dịch là 0,240 oC. Hãy tính toán
khối lượng phân tử của hormone. Biết hằng số nghiệm đông của benzene
là 5,12 oC.kg/mol.
Giải
Ta có:
Chất tan: hormone. Dung môi: benzene (C6H6)
mct = 0,546 g; mdm = 15,0 g ; ∆𝑡đ = 0,240 o C
Kđ, benzene = 5,12 oC.kg.mol-1
Nồng độ molan của dung dịch là:
∆t đ 0,240
Cm = = = 4,69 × 10−2 ሺmol/kgሻ
Kđ 5,12
Cm có thể được tính toán bằng công thức:

140
𝐦𝐜𝐭 × 𝟏𝟎𝟎𝟎
𝐂𝐦 =
𝐌𝐜𝐭 × 𝐦𝐝𝐦
Khối lượng phân tử của hormone là:
mct × 1000 0,546 × 1000
Mct = = = 776,12 ሺg/molሻ
Cm × mdm 4,69 × 10−2 × 15,0
Câu 6.4. Áp suất hơi bão hòa của nước ở 70 oC là 233,80 mmHg. Khi hòa
tan 12,2 g một chất tan không điện ly vào 270 g nước, dung dịch thu được
có áp suất hơi bão hòa 230,68 mmHg. Xác định khối lượng mol phân tử
của chất tan trên.
Giải
Ta có:
270
P ndm P 230,68
dm = P → =P → 18
270 12,2 = 233,80 → M = 60 g/mol
0 ndm +nct o +
18 M
Dạng 3. Áp suất hơi của dung dịch.
Câu 6.5. Hãy tính áp suất hơi ở 25 oC của dung dịch chứa 99,5 g sucrose
(C12H22O11) và 300,0 mL nước. Áp suất hơi của nước nguyên chất ở 25 oC
là 23,8 mmHg. Giả sử khối lượng riêng của nước là 1,00 g/mL.
Giải
Chất tan: sucrose (C12H22O11). Dung môi: nước (H2O)
mct = 99,5 g; Po = 23,8 mmHg (ở 25oC)
Vdm = 300,0 mL; dH2O = 1,00 g/mL
→ mdm = 300,0 g
Tính toán số mol của mỗi thành phần trong dung dịch:
99,5
nC12 H22 O11 = = 0,2907 (mol)
342,30
300,0
nH2 O = = 16,65 (mol)
18,02
Sử dụng số mol của mỗi thành phần để tính toán phân số mol của dung
môi (H2O):
n H2 O 16,65 mol
H =n = 0,2907 mol + 16,65 mol = 0,9828
2O C12 H22 O11 + nH2 O

Sử dụng phân số mol của nước và áp suất hơi của nước nguyên chất để
tính toán áp suất hơi của dung dịch.
Pdd = H O PoH O = 0,9828 x 23,8 = 23,4 (mmHg)
2
2
Câu 6.6. Hòa tan 90 gam Glucose vào 500g nước được dung dịch A ở
25oC. Biết ở nhiệt độ này nước có áp suất hơi bão hòa bằng 23,76 mmHg.
a) Tính áp suất hơi bão hòa của dung dịch A.

141
b) Tính độ giảm áp tuyệt đối và tương đối của dung dịch A.
c) Tính nồng độ molan và nồng độ % của dung dịch A.
Giải
a) Áp suất hơi bão hòa của dung dịch
500
nH 2O
P1 = Po .N H 2O = Po . = 23,76. 18 = 23,34 mmHg
nH 2O + nGlucose 500 90
+
18 18
b) Độ giảm áp tuyệt đối:
P = Po − P1 = 0,42 mmHg
Độ giảm áp tương đối:
∆𝐏 𝐏𝐨 − 𝐏𝟏
= . 𝟏𝟎𝟎 = 𝟏, 𝟕𝟕 %
𝐏𝐨 𝐏𝐨
90.1000
c) Nồng độ molan của dung dịch: Cm = = 1mol / kg
180 .500
90
Nồng độ phần trăm của dung dịch: C % = .100 = 15,25%
90 + 500
Dạng 4. Nhiệt độ sôi và nhiệt độ đông đặc của dung dịch.
Câu 6.7. Tính nhiệt độ sôi ts và nhiệt độ đông đặc tđ của dung dịch chứa 9
gam glucose trong 100 gam nước. Biết ks = 0,51 oC.kg/mol và kđ = 1,86
o
C.kg/mol.
Giải
𝐦𝐜𝐭 .𝟏𝟎𝟎𝟎 𝟗 . 𝟏𝟎𝟎𝟎
Ta có: 𝐂𝐦 = → Cm = 𝟏𝟖𝟎 .𝟏𝟎𝟎 = 0,5 m
𝐌𝐜𝐭 .𝐦𝐝𝐦
ts = ks. Cm= 0,51 . 0,5 = 0,26 oC
→ ts = 100 + 0,255 = 100,26 oC
tđ = kđ. Cm= 1,86 . 0,5 = 0,93 oC
→ tđ = 0 – 0,93 = - 0,93 oC
Câu 6.8. Hãy tính khối lượng của ethylene glycol (gam) cần phải thêm vào
1,0 kg nước để tạo được dung dịch sôi ở 105,0 oC. Biết Ks = 0,51 oC.kg/mol
Giải
Ta có độ tăng nhiệt độ sôi: ∆t s = t s,dd − t s,dm = Cm . K s
∆ts 5,0
→ nồng độ molan dung dịch: Cm = = = 9,77 ሺmol/kgሻ
Ks 0,512
Ngoài ra, ta có:
𝐦𝐜𝐭 . 𝟏𝟎𝟎𝟎
𝐂𝐦 =
𝐌𝐜𝐭 . 𝐦𝐝𝐦
142
→ khối lượng chất tan:
9,77 . 62,07 . 1000
𝑚𝑐𝑡 = = 6,1 . 102 ሺgሻ
1000
Câu 6.9. Hãy tính nhiệt độ đông đặc của ethylene glycol trong dung dịch
có nồng độ molan là 1,7 mol/kg. Cho biết Kđ = 1,86 oC.kg/mol.
Giải
Ta có: ∆t đ = t đ,dm − t đ,dd = Cm . K đ = 1,7 . 1,86 = 3,2 o C
→ tđ, dd = 0 - 3,2 = - 3,2 oC
Câu 6.10. Tính nhiệt độ sôi và nhiệt độ đông đặc của dung dịch Glixerol
C3H5(OH)3 15%. Biết Ks = 0,51 oC.kg/mol và Kđ = 1,86 oC.kg/mol.
Giải
Xét một dung dịch Glixerol 15% có khối lượng 100 gam sẽ chứa hai thành
phần:
• 15 gam Glicerol
• 85 gam dung môi
Ta có nồng độ molan của dung dịch:
𝐦𝐜𝐭 .𝟏𝟎𝟎𝟎 𝟏𝟓 . 𝟏𝟎𝟎𝟎
𝐂𝐦 = = = 1,92 mol/kg
𝐌𝐜𝐭 .𝐦𝐝𝐦 𝟗𝟐 . 𝟖𝟓
ts = ks. Cm= 0,51 . 1,92 = 0,96 oC
→ ts = 100 + 0,255 = 100,96 oC
tđ = kđ. Cm= 1,86 . 1,92 = 3,57 oC
→ tđ = 0 – 3,57 = - 3,57 oC
Dạng 5. Áp suất thẩm thấu của dung dịch
Câu 6.11. Tính áp suất thẩm thấu ở 25 oC của dung dịch chứa 23,0 gam
Glixerol C3H5(OH)3 trong 500 mL dung dịch.
Giải
23
Ta có: π = C . R . T = . 0,082 . 298 = 12,2 atm
92 x 0,5
Câu 6.11. Manit là một chất tan không điện ly và không bay hơi. Hòa tan
9,0 gam manit vào nước được 250 mL dung dịch X. Ở 273 K, dung dịch
X có áp suất thẩm thấu là 4,5 atm. Xác định khối lượng phân tử của manit.
Giải
𝑚
Ta có: 𝜋 = 𝐶 . 𝑅 . 𝑇 = 𝑀 .𝑐𝑡𝑉 . 𝑅 . 𝑇
9,0
→ 4,5 = M . . 0,082 . 273
0,25
→ M = 179 gam/mol

143
Câu 6.11. Áp suất thẩm thấu của máu là 7,65 atm. Cần hòa tan bao nhiêu
gam Glucose vào nước được 2 L dung dịch và khi tiêm vào cơ thể, dung
dịch Glucose này có áp suất thẩm thấu như máu?
Giải
𝑚
Ta có: 𝜋 = 𝐶 . 𝑅 . 𝑇 = 𝑀 𝑥𝑐𝑡𝑉 . 𝑅 . 𝑇
𝑚
→ 7,65 = 180𝑐𝑡. 2 . 0,082 . 310
→ mct = 108 gam

144
Phần 3. BÀI TẬP TRẮC NGHIỆM
---oOo---
Câu 6.1. Chọn phát biểu đúng trong các phát biểu sau:
A. Độ tan của đa số chất ít tan giảm khi nhiệt độ của dung dịch tăng.
B. Độ tan của chất ít tan chỉ phụ thuộc vào bản chất chất tan và nhiệt độ.
C. Độ tan của chất ít tan sẽ tăng khi cho vào dung dịch một trong các ion
của chất tan đó.
D. Độ tan của chất tan chỉ phụ thuộc vào bản chất dung môi, chất tan, nhiệt
độ và sự có mặt của một trong các ion của chất tan đó.
Câu 6.2. Chọn phát biểu đúng:
A. Nhiệt độ sôi của chất lỏng là nhiệt độ tại đó áp suất hơi bão hòa của
chất lỏng bằng áp suất môi trường.
B. Tại cùng một nhiệt độ, áp suất hơi bão hòa của dung dịch loãng chứa
chất tan không điện ly, không bay hơi lớn hơn áp suất hơi bão hòa của
dung môi nguyên chất.
C. Nước sôi ở 100 oC.
D. Nước muối sôi ở nhiệt độ thấp hơn nước nguyên chất xét cùng áp suất
bên ngoài.
Câu 6.3. Dựa trên công thức của định luật Raoult 2: ∆t = KCm, phát biểu
nào sau đây là đúng về K?
A. K là hằng số phụ thuộc vào nhiệt độ và bản chất của dung môi.
B. K là hằng số phụ thuộc vào nồng độ chất tan, nhiệt độ và bản chất
dung môi.
C. K là hằng số chỉ phụ thuộc vào bản chất dung môi.
D. K là hằng số phụ thuộc vào bàn chất chất tan và dung môi.
Câu 6.4. Chọn câu trả lời chính xác. Nhiệt độ sôi của chất lỏng là nhiệt
độ mà tại đó:
A. Áp suất hơi bão hòa của chất lỏng < 760 mmHg.
B. Áp suất hơi bão hòa của chất lỏng > 760 mmHg.
C. Áp suất hơi bão hòa của chất lỏng = 760 mmHg.
D. Áp suất hơi bão hòa của chất lỏng bằng áp suất môi trường.
Câu 6.5. Trong 200 g dung môi chứa a gam đường glucose có khối lượng
phân tử M, hằng số nghiệm đông của dung môi là Kđ. Biểu thức nào sau
đây đúng?
A. ∆tđ = 5Kđ.(a/M) B. ∆tđ = 1/5Kđ.(A/M)
C. ∆tđ = Kđ.(A/M) D. ∆tđ = Kđ.A

145
Câu 6.6. Chọn phát biểu đúng:
A. Nồng độ phần trăm khối lượng cho biết số mol chất tan có trong một
L dung dịch.
B. Nồng độ phần trăm khối lượng cho biết số gam chất tan có trong một
L dung dịch.
C. Nồng độ phần trăm khối lượng cho biết số gam chất tan có trong 100
gam dung môi.
D. Nồng độ phần trăm khối lượng cho biết số gam chất tan có trong 100
gam dung dịch.
Câu 6.7. Dung dịch NaNO3 40,0% có khối lượng riêng là 1,32 g/mL, khối
lượng mol của NaNO3 là 85 g/mol. Thể tích của dung dịch NaNO3 40,0%
có chứa 0,15 mol NaNO3 là bao nhiêu?
A. 42,0 mL B. 24,1 mL
C. 3,86 mL D. 38,2 mL
Câu 6.8. Tính thể tích dung dịch (L) HCl 4M cần thiết để có thể pha thành
1 L dung dịch HCl 0,5M.
A. 0,125 L B. 0,0125 L
C. 0,875 L D. 12,5 L
Câu 6.9. Chọn đáp án đúng. Ở áp suất và nhiệt độ không đổi, nồng độ
dung dịch loãng (có chất tan không bay hơi và không tạo dung dịch rắn
với dung môi) càng tăng thì:
A. Nhiệt độ sôi tăng B. Nhiệt độ sôi giảm
C. Nhiệt độ đông đặc tăng D. Áp suất hơi bão hòa tăng
Câu 6.10. Hòa tan 5 gam mỗi chất C6H12O6, C12H22O11 và C3H5(OH)3
trong 500 gam nước. Trong các dãy sau, dãy nào xếp các chất trên theo
nhiệt độ sôi của dung dịch tăng dần? Biết rằng các chất trên không bay hơi;
C = 12, O = 16 và H= 1.
A. C12H22O11 < C6H12O6 < C3H5(OH)3
B. C6H12O6 < C3H5(OH)3 < C12H22O11
C. C3H5(OH)3 < C6H12O6 < C12H22O11
D. C12H22O11 < C3H5(OH)3 < C6H12O6
Câu 6.11. Tính áp suất hơi bão hòa của nước trong dung dịch chứa 5 gam
chất tan trong 100 gam nước ở nhiệt độ 25 oC. Cho biết ở nhiệt độ này,
nước tinh khiết có áp suất hơi bão hòa bằng 23,76 mmHg và khối lượng
phân tử chất tan bằng 62,5 gam/mol.
A. 23,4 mmHg B. 0,34 mmHg

146
C. 22,6 mmHg D. 19,0 mmHg
Câu 6.12. Ở 25 C, áp suất hơi bão hòa của nước nguyên chất là 23,76
o

mmHg. Khi hòa tan 2,7 mol glycerol vào 100 mol nước ở nhiệt độ trên thì
độ giảm áp suất hơi bão hòa của dung dịch bằng bao nhiêu?
A. 23,13 mmHg B. 0,64 mmHg
C. 0,62 mmHg D. 23,10 mmHg
Câu 6.13. Dung dịch nước của một chất tan không điện ly sôi ở 373,52 K.
Nồng độ molan của dung dịch này là bao nhiêu? Cho biết Ks = 0,52
o
C.kg.mol-1.
A. 0,01 B. 1,0
C. 10 D. 0,1
Câu 6.14. Chọn phát biểu đúng trong các phát biểu sau:
A. Độ tan của đa số chất ít tan giảm khi nhiệt độ của dung dịch tăng.
B. Độ tan của chất ít tan chỉ phụ thuộc vào bản chất chất ít tan đó và nhiệt
độ.
C. Độ tan của chất ít tan sẽ tăng khi cho vào dung dịch ion cùng loại với
một trong các ion của chất ít tan đó.
D. Không có phát biểu nào đúng.
Câu 6.15. Chọn các phát biểu sai:
1. Dung dịch loãng là dung dịch chưa bão hòa vì nồng độ chất tan nhỏ.
2. Dung dịch là một hệ đồng thể.
3. Thành phần của một hợp chất là xác định còn thành phần của dung dịch
có thể thay đổi.
4. Dung dịch bão hòa là dung dịch đậm đặc.
A. 1,3 B. 2,4
C. 2,3 D. 1,4
Câu 6.16. So sánh nhiệt độ sôi của các dung dịch C6H12O6 (t1), C3H5(OH)3
(t2) và C12H22O11 (t3) cùng chứa a gam chất tan trong 1000 gam nước nào
sau đây là đúng? Biết rằng các chất này là chất không điện ly và không
bay hơi.
A. t3 > t2 > t1 B. t1 > t2 > t3
C. t2 > t3> t1 D. t2 > t1 > t3
Câu 6.17. Chọn phát biểu sai:
A. Nhiệt độ sôi của dung dịch loãng với chất tan không bay hơi nghịch
biến với nồng độ chất tan.

147
B. Độ giảm tương đối áp suất hơi bão hòa của dung môi trong dung dịch
bằng nồng độ phần mol của chất tan.
C. Nhiệt độ kết tinh của dung môi trong dung dịch loãng với chất tan không
bay hơi đồng biến với nồng độ chất tan.
D. Áp suất hơi bão hòa của dung dịch loãng với chất tan không bay hơi
đồng biến với nồng độ phần mol của chất tan.
Câu 6.18. Chọn phát biểu sai:
A. Nhiệt độ sôi của chất lỏng là nhiệt độ ở đó áp suất hơi bão hòa của nó
bằng với áp suất môi trường.
B. Nhiệt độ sôi của dung dịch chứa chất tan không bay hơi luôn luôn cao
hơn nhiệt độ sôi của dung môi nguyên chất ở cùng điều kiện áp suất ngoài.
C. Nhiệt độ đông đặc của dung môi nguyên chất luôn thấp hơn nhiệt độ
đông đặc của dung môi trong dung dịch chứa chất tan không bay hơi.
D. Áp suất hơi bão hòa của dung môi trong dung dịch chứa chất không bay
hơi luôn nhỏ hơn áp suất hơi bão hòa của dung môi tinh khiết.
Câu 6.19. Chọn câu đúng:
A. Độ giảm tương đối áp suất hơi bão hòa của dung môi trong dung dịch
chứa chất tan không bay hơi bằng phần mol của dung môi trong dung dịch.
B. Áp suất hơi bão hòa của dung môi trong dung dịch chứa chất tan không
bay hơi luôn nhỏ hơn áp suất hơi bão hòa của dung môi tinh khiết ở cùng
giá trị nhiệt độ.
C. Áp suất hơi bão hòa của dung môi trong dung dịch chứa chất tan không
bay hơi tỷ lệ thuận với phần mol của chất tan trong dung dịch.
D. Áp suất hơi bão hòa của dung dịch loãng không phụ thuộc vào bản chất
của chất tan.
Câu 6.20. Dung dịch nước của một chất tan bay hơi không điện ly sôi ở
105,2 oC, hằng số nghiệm sôi của nước Ks = 0,52 oC.kg.mol-1 ở nhiệt độ
đang khảo sát. Nồng độ molan (mol/kg) của dung dịch này là bao nhiêu?
A. 10 B. 1
C. 5 D. Không đủ dữ liệu để tính
Câu 6.21. Xác định khối lượng KOH và nước để điều chế 75 L dung dịch
KOH 12% có khối lượng riêng d = 1100 kg/m3.
A. 9,9 kg KOH; 72,6 kg H2O B. 72,6 kg KOH; 72,6 kg H2O
C. 4,95 kg KOH; 36,3 kg H2O D. 36,3 kg KOH; 4,95 kg H2O
Câu 6.22. Xác định khối lượng KOH và nước để điều chế 75 L dung dịch
KOH 2,35 M có khối lượng riêng d = 1100 kg/m3.

148
A. 9,9 kg KOH; 72,6 kg H2O B. 72,6 kg KOH; 72,6 kg H2O
C. 4,95 kg KOH; 36,3 kg H2O D. 36,3 kg KOH; 4,95 kg H2O
Câu 6.23. Xác định nồng độ mol (M) dung dịch KOH 12% có khối lượng
riêng d = 1100 kg/m3?
A. 2,35 M B. 3,25 M
C. 3,52 M D. 2,53 M
Câu 6.24. Xác định nồng độ phần trăm (C%) dung dịch KOH 2,35 M có
khối lượng riêng d = 1100 kg/m3?
A. 12% B. 24%
C. 21% D. 42%
Câu 6.25. Xác định thể tích (mL) dung dịch KOH 40% (d = 1460 kg/m3)
để pha thành 800 mL dung dịch KOH 12% (d = 1100 kg/m3).
A. 108,8 mL B. 180,8 mL
C. 361,6 mL D. 217,6 mL
Câu 6.26. Xác định thể tích (mL) dung dịch KOH 12% (d = 1100 kg/m3)
thu được khi pha loãng 180,8 mL dung dịch KOH 40% (d = 1460 kg/m3).
A. 600 mL B. 700 mL
C. 800 mL D. 900 mL
Câu 6.27. Xác định nồng độ phần trăm (C%) của 800 mL dung dịch (d =
1100 kg/m3) thu được khi pha loãng 180,8 mL dung dịch KOH 40% (d =
1460 kg/m3).
A. 12% B. 24%
C. 21% D. 42%
Câu 6.28. Xác định thể tích (mL) dung dịch NaOH cần để trung hoà 20
mL dung dịch acid nồng độ 0,1 N. Biết trong 1 L dung dịch có 10 g NaOH.
A. 2 mL B. 8 mL
C. 10 mL D. 20 mL
Câu 6.29. Ở 20 C áp suất hơi nước bão hòa là 17,5 mmHg. Xác định độ
o

giảm áp suất hơi nước bão hoà khi hoà tan 2,94 g glycerine C3H5(OH)3 vào
100 g nước.
A. 0,1 mmHg B. 0,05 mmHg
C. 0,15 mmHg D. 0,12 mmHg
Câu 6.30. Ở 20 C áp suất hơi nước bão hòa là 17,5 mmHg. Xác định áp
o

suất hơi nước bão hoà khi hoà tan 2,94 g glycerine C3H5(OH)3 vào 100 g
nước.
A. 17,2 mmHg B. 17,4 mmHg

149
C. 17,8 mmHg D. 17,6 mmHg
Câu 6.31. Xác định nhiệt độ sôi ( C) của dung dịch đường saccharose
o

(C12H22O11) 5% trong nước. Cho biết nước có Ks = 0,52 oC.kg.mol-1.


A. 100,02 oC B. 100,05 oC
C. 100,08 oC D. 100,10 oC
Câu 6.32. Xác định nhiệt độ đông đặc (oC) của dung dịch đường
saccharose (C12H22O11) 5% trong nước. Cho biết nước có Kđ = 1,86
o
C.kg.mol-1.
A. -0,285 oC B. -0,286 oC
C. -0,287 oC D. -0,288 oC
Câu 6.33. Xác định áp suất hơi của dung dịch đường saccharose
(C12H22O11) 5% trong nước ở nhiệt độ 65 oC, biết áp suất hơi nước bão hòa
ở nhiệt độ này là 187,5 mmHg.
A. 186,57 mmHg B. 186,98 mmHg
C. 187,24 mmHg D. 187,55 mmHg
Câu 6.34. Xác định áp suất hơi của dung dịch chứa 13,68 g đường
C12H22O11 trong 90 gam nước ở nhiệt độ 65 oC, biết áp suất hơi nước bão
hòa ở nhiệt độ này là 187,5 mmHg.
A. 186,2 mmHg B. 186,0 mmHg
C. 187,3 mmHg D. 187,6 mmHg
Câu 6.35. Xác định khối lượng (g) đường C12H22O11 chứa trong 90 gam
nước để áp suất hơi của dung dịch này đạt 186 mmHg ở nhiệt độ 65 oC.
Biết áp suất hơi nước bão hòa ở nhiệt độ này là 187,5 mmHg.
A. 6,84 g B. 10,26 g
C. 11,97 g D. 13,68 g
Câu 6.36. Áp suất hơi nước bão hòa ở 65 oC bằng 187,5 mmHg. Ở cùng
nhiệt độ này, áp suất hơi của dung dịch chứa 13,68 g chất hòa tan trong 90
g nước bằng 186 mmHg. Xác định khối lượng phân tử chất tan.
A. 228 g.mol-1 B. 342 g.mol-1
C. 380 g.mol-1 D. 456 g.mol-1
Câu 6.37. Áp suất hơi nước bão hòa ở 70 oC bằng 233,8 mmHg. Ở cùng
nhiệt độ này, áp suất hơi của dung dịch chứa 12 g chất hòa tan trong 270 g
nước bằng 230,68 mmHg. Xác định khối lượng phân tử chất tan.
A. 48 g.mol-1 B. 60 g.mol-1
C. 80 g.mol-1 D. 100 g.mol-1

150
Câu 6.38. Xác định áp suất hơi của dung dịch chứa 12 g chất hòa tan trong
270 g nước ở nhiệt độ 70 oC. Biết chất hoà tan có khối lượng phân tử là 60
g.mol-1, áp suất hơi nước bão hòa ở 70 oC bằng 233,8 mmHg.
A. 230,68 mmHg B. 231,46 mmHg
C. 231,87 mmHg D. 232,50 mmHg
Câu 6.39. Xác định khối lượng (g) chất hoà tan chứa trong 270 g nước để
áp suất hơi của dung dịch này đạt 230,68 mmHg ở nhiệt độ 70 oC. Biết áp
suất hơi nước bão hòa ở nhiệt độ này là 233,8 mmHg và khối lượng phân
tử chất hoà tan là 60 g.mol-1.
A. 10,8 g B. 12 g
C. 14,4 g D. 15 g
Câu 6.40. Xác định nhiệt độ sôi của dung dịch chứa 9 gam glucose
(C6H12O6) trong 100 g nước. Biết nước có Ks = 0,51 oC.kg.mol-1.
A. 100,26 oC B. 100,42 oC
C. 100,53 oC D. 100,68 oC
Câu 6.41. Xác định nhiệt độ đông đặc của dung dịch chứa 9 gam glucose
(C6H12O6) trong 100 g nước. Biết nước có Kđ = 1,86 oC.kg.mol-1.
A. - 0,78 oC B. - 0,85 oC
C. - 0,93 oC D. - 0,97 oC
Câu 6.42. Khi hòa tan lưu huỳnh vào 40 gam benzene thì nhiệt độ sôi của
dung dịch tăng lên 0,81 oC. Xác định khối lượng lưu huỳnh đã hoà tan. Giả
sử trong dung dịch này, một phân tử lưu huỳnh gồm 8 nguyên tử. Biết
benzene có Ks = 2,53 oC.kg.mol-1.
A. 2,68 g B. 2,86 g
C. 3,24 g D. 3,42 g
Câu 6.43. Xác định độ tăng nhiệt độ sôi của dung dịch khi hoà tan lưu
huỳnh vào 40 gam benzene. Giả sử trong dung dịch này, một phân tử lưu
huỳnh gồm 8 nguyên tử. Biết benzene có Ks = 2,53 oC.kg.mol-1.
A. 0,78 oC B. 0,81 oC
C. 0,86 oC D. 0,90 oC
Câu 6.44. Xác định khối lượng (gam) glucose (C6H12O6) phải có trong 1
L dung dịch để áp suất thẩm thấu của nó bằng áp suất thẩm thấu của dung
dịch chứa 3 gam formaldehyde HCHO trong 1 L dung dịch ở cùng nhiệt
độ đó.
A. 12 gam B. 16 gam
C. 18 gam D. 24 gam

151
Câu 6.45. Xác định khối lượng (gam) formaldehyde HCHO phải có trong
1 L dung dịch để áp suất thẩm thấu của nó bằng áp suất thẩm thấu của dung
dịch chứa 18 g glucose (C6H12O6) trong 1 L dung dịch ở cùng nhiệt độ đó.
A. 3 gam B. 4 gam
C. 6 gam D. 8 gam
Câu 6.46. Xác định khối lượng phân tử của một chất biết rằng 18 gam chất
này trong 1 L dung dịch có áp suất thẩm thấu bằng áp suất thẩm thấu của
dung dịch chứa 3 gam formaldehyde HCHO trong 1 L dung dịch ở cùng
nhiệt độ đó.
A. 150 g.mol-1 B. 180 g.mol-1
-1
C. 200 g.mol D. 225 g.mol-1
Câu 6.47. Xác định khối lượng phân tử của một chất biết rằng 3 g chất này
trong 1 L dung dịch có áp suất thẩm thấu bằng áp suất thẩm thấu của dung
dịch chứa 18 g glucose (C6H12O6) trong 1 L dung dịch ở cùng nhiệt độ đó.
A. 30 g.mol-1 B. 50 g.mol-1
C. 60 g.mol-1 D. 75 g.mol-1
Câu 6.48. Hòa tan 350 g KNO3 trong 500 gam nước ở 60 oC. Để nguội
xuống 20 oC. Hỏi có bao nhiêu gam KNO3 kết tinh lại? Biết độ tan của
KNO3 ở 60 oC và 20 oC tương ứng là 100 gam/100 gam nước và 31,6
gam/100 gam nước.
A. 168 g B. 176 g
C. 192 g D. 210 g
Câu 6.49. Cần hoà tan bao nhiêu gam KNO3 trong 500 g nước 60 oC sau
đó làm nguội xuống 20 oC để thu được 192 gam KNO3 kết tinh lại? Biết
độ tan của KNO3 ở 60 oC và 20 oC tương ứng là 100 gam/100 gam nước
và 31,6 gam/100 gam nước.
A. 300 g B. 350 g
C. 450 g D. 500 g
Câu 6.50. Tính nồng độ dung dịch acid thu được khi hòa tan 40 gam SO3
vào 450 gam nước.
A. 8% B. 10%
C. 15% D. 18%
Câu 6.51. Xác định khối lượng (gam) SO3 cần hoà tan vào 450 g nước để
thu được dung dịch có nồng độ 10%.
A. 10 gam B. 20 gam
C. 40 gam D. 50 gam

152
Câu 6.52. Tìm khối lượng nước cần hòa tan 40 gam SO3 để thu được dung
dịch có nồng độ 5,6%.
A. 450 gam B. 475 gam
C. 500 gam D. 525 gam
Câu 6.53. Tìm khối lượng nước cần hòa tan 188 gam K2O để thu được
dung dịch KOH 5,6%.
A. 2815 gam B. 3216 gam
C. 3654 gam D. 3812 gam
Câu 6.54. Tính nồng độ dung dịch KOH thu được khi hòa tan 188 gam
K2O vào 3812 gam nước.
A. 5,6% B. 2,8%
C. 11,2% D. 4,2%
Câu 6.55. Xác định khối lượng (gam) K2O cần hoà tan vào 3812 gam nước
để thu được dung dịch KOH 5,6%.
A. 160 g B. 174 g
C. 188 g D. 202 g
Câu 6.56. Cho hai dung dịch loãng với dung môi là nước:
- Dung dịch 1: dung dịch glucose có nồng độ molan m1.
- Dung dịch 2: dung dịch saccharose có nồng độ molan m2.
Gọi t1 và t2 lần lượt là nhiệt độ sôi của hai dung dịch 1 và dung dịch 2. Biết
t1 - t2 = 1 oC ở một áp suất xác định. Tìm mối liên hệ giữa m1 và m2 biết
hằng số nghiệm sôi của nước Ks = 0,51 oC.kg.mol-1.
A. m1 - m2 = 1,96 B. m1 - m2 = 2,05
C. m1 - m2 = 2,12 D. m1 - m2 = 2,18
Câu 6.57. Cho hai dung dịch loãng với dung môi là nước:
- Dung dịch 1: dung dịch glucose có nồng độ molan m1.
- Dung dịch 2: dung dịch saccharose có nồng độ molan m2.
Gọi t1 và t2 lần lượt là nhiệt độ sôi của hai dung dịch 1 và dung dịch 2. Biết
t1 - t2 = 1oC ở một áp suất xác định. Biết phân tử lượng của glucose,
saccharose và nước lần lượt là 180, 342 và 18 g.mol-1. So sánh độ giảm
tương đối áp suất hơi bão hòa của hai dung dịch này ở một nhiệt độ xác
định nào sau đây là đúng?
A. Độ giảm tương đối áp suất hơi bão hòa của dung dịch 1 gấp 2 lần dung
dịch 2.
B. Độ giảm tương đối áp suất hơi bão hòa của dung dịch 2 gấp 2 lần dung
dịch 1.

153
C. Độ giảm tương đối áp suất hơi bão hòa của dung dịch 1 lớn hơn dung
dịch 2.
D. Độ giảm tương đối áp suất hơi bão hòa của dung dịch 2 lớn hơn dung
dịch 1.
Câu 6.58. Công thức phân tử của glucose và saccharose lần lượt là
C6H12O6 và C12H22O11. Hằng số nghiệm sôi của nước ở điều kiện đang xét
là 0,52 oC.kg/mol. Cho 4 dung dịch loãng sau đây với dung môi là nước:
- Dung dịch 1: dung dịch glucose có nồng độ a%.
- Dung dịch 2: dung dịch glucose có nồng độ molan m2.
- Dung dịch 3: dung dịch glucose có nồng độ molan m3.
- Dung dịch 4: dung dịch saccharose có nồng độ a%.
Tìm mối liên hệ giữa a và m2 sao cho nhiệt độ sôi của dung dịch 1 và dung
dịch 2 bằng nhau.
50𝑎 40𝑎
A. 𝑚2 = [9ሺ100−aሻ] B. 𝑚2 = [9ሺ100−aሻ]
40𝑎 50𝑎
C. 𝑚2 = [9ሺ120−aሻ] D. 𝑚2 = [9ሺ120−aሻ]
Câu 6.59. Công thức phân tử của glucose và saccharose lần lượt là
C6H12O6 và C12H22O11. Hằng số nghiệm sôi của nước ở điều kiện đang xét
là 0,52 oC.kg.mol-1. Cho 4 dung dịch loãng sau đây với dung môi là nước:
- Dung dịch 1: dung dịch glucose có nồng độ a%.
- Dung dịch 2: dung dịch glucose có nồng độ molan m2.
- Dung dịch 3: dung dịch glucose có nồng độ molan m3.
- Dung dịch 4: dung dịch saccharose có nồng độ a%.
Tìm mối liên hệ giữa m2 và m3 sao cho nhiệt độ sôi của hai dung dịch 2 và
3 kém nhau 2 oC.
2 2
A. m2 - m3 = 0,52 B. m2 - m3 = 0,48
3 3
C. m2 - m3 = 0,52 D. m2 - m3 = 0,48
Câu 6.60. Công thức phân tử của glucose và saccharose lần lượt là
C6H12O6 và C12H22O11. Hằng số nghiệm sôi của nước ở điều kiện đang xét
là 0,52 oC.kg.mol-1. Cho 4 dung dịch loãng sau đây với dung môi là nước:
- Dung dịch 1: dung dịch glucose có nồng độ a%.
- Dung dịch 2: dung dịch glucose có nồng độ molan m2.
- Dung dịch 3: dung dịch glucose có nồng độ molan m3.
- Dung dịch 4: dung dịch saccharose có nồng độ a%.
So sánh nhiệt độ sôi của dung dịch 1 (t1) và 4 (t4) nào sau đây là đúng?
154
A. t1 < t4 B. t4 ˃ t1
C. t4 < t1 D. t1 ˃ t4
---oOo---

ĐÁP ÁN BÀI TẬP CHƯƠNG 6


6.1- 6.2- 6.3- 6.4- 6.5- 6.6- 6.7- 6.8- 6.9- 6.10
D A C D A D B A A -A
6.11 6.12 6.13 6.14 6.15 6.16 6.17 6.18 6.19 6.20
-A -C -B -D -D -D -D -C -B -D
6.21 6.22 6.23 6.24 6.25 6.26 6.27 6.28 6.29 6.30
-A -A -A -A -B -C -A -B -A -B
6.31 6.32 6.33 6.34 6.35 6.36 6.37 6.38 6.39 6.40
-C -B -B -B -D -B -B -A -B -A
6.41 6.42 6.43 6.44 6.45 6.46 6.47 6.48 6.49 6.50
-C -C -B -C -A -B -A -C -B -B
6.51 6.52 6.53 6.54 6.55 6.56 6.57 6.58 6.59 6.60
-C -A -D -A -C -A -C -A -A -C

155
Chương 7. DUNG DỊCH ĐIỆN LY
---oOo---
Mục tiêu chương 7
Sau khi học xong chương này, sinh viên có khả năng:
1. Định nghĩa được các acid, base, muối, chất điện ly yếu, điện ly
mạnh và chất điện ly khó tan.
2. Giải thích được sự khác nhau về các tính chất cơ bản giữa dung
dịch chất không điện ly và dung dịch chất điện ly.
3. Phát biểu được thuyết điện ly Arrhenius.
4. Tính toán và xác định được mối quan hệ giữa độ điện ly, hằng số
điện ly và nồng độ dung dịch.
5. Trình bày được sự điện ly của nước, tích số ion của nước, từ đó
tính được giá trị pH của các dung dịch acid, base, muối.
6. Trình bày được sự điện ly của chất điện ly khó tan, tích số tan, từ
đó tính toán được độ tan của các chất điện ly khó tan trong nước
nguyên chất và trong dung dịch có chứa ion.
7. Xác định được điều kiện hình thành và hòa tan kết tủa.

156
Phần 1. TÓM TẮT LÝ THUYẾT

7.1 DUNG DỊCH ACID, BASE, MUỐI TRONG NƯỚC


Thực nghiệm cho thấy có hai sự khác biệt giữa dung dịch acid, base và
muối trong nước với dung dịch các chất tan tồn tại dưới dạng phân tử (như
đường, rượu,…):
• Các dung dịch acid, base, muối trong nước không tuân theo các định
luật Raoult và định luật Van’t Hoff về độ giảm áp suất hơi bão hòa (P),
độ tăng nhiệt độ sôi (ts) và độ hạ nhiệt độ đông đặc (tđ), áp suất thẩm
thấu (). Khi sử dụng các công thức của các định luật trên thì phải hiệu
chỉnh bằng hệ số Van’t Hoff như sau:
P ' P (7.1)
=i = ix2
P0 P0
' = i CRT (7.2)
t's = its = i x Ks x Cm (7.3)
t'đ = itđ = i x Kđ x Cm (7.4)
 ' P ' t' (7.5)
i= = =
  P t
Trong đó:
', P’, t' là những đại lượng được đo từ thực nghiệm.
, P, t là những đại lượng được tính theo các định luật Raoult
và Van’t Hoff.
i là hệ số Van’t Hoff được xác định bằng thực nghiệm (i >1).
• Dung dịch acid, base và muối trong nước có tính dẫn điện dù các chất
nguyên chất không dẫn điện.
7.2 SỰ ĐIỆN LY VÀ THUYẾT ĐIỆN LY
Thuyết điện ly (Arrhenius – 1887): Khi hòa tan vào nước, các chất acid,
base và muối phân ly thành các ion dương (cation) và ion âm (anion).
Trong đó, sự phân ly thành ion của các chất tan trong dung dịch (hay khi
nóng chảy) gọi là sự điện ly; còn các chất phân ly thành ion gọi là chất
điện ly.

157
n
Hệ số Van’t Hoff i=n (7.6)
0
Trong đó: n là số tiểu phân thực tế có trong dung dịch.
no là số tiểu phân phân ly.
Do sự điện ly mà số tiểu phân thực tế trong dung dịch tăng lên so với số
phân tử hòa tan (i > 1), do đó làm giảm áp suất hơi bão hòa nhiều hơn, làm
tăng điểm sôi và làm hạ điểm đông đặc nhiều hơn, đồng thời làm tăng áp
suất thẩm thấu. Tuy nhiên, thuyết điện ly của Arrhenius không tính tới sự
tương tác giữa các tiểu phân trong dung dịch, làm cho thuyết Arrhenius bị
hạn chế, không giải thích được nhiều hiện tượng xảy ra trong dung dịch
điện ly. Sau này thuyết điện ly hiện đại của Kablucov đã khắc phục điều
đó và định nghĩa: sự điện ly là sự phân ly của các chất tan dưới tác dụng
của các tiểu phân dung môi thành những ion solvat hóa.
7.3 ĐỘ ĐIỆN LY
Cân bằng điện ly:

Độ điện ly  là đại lượng đặc trưng cho mức độ điện ly trong dung dịch.
n
α=n 01
0
Trong đó:
n: nồng độ chất đã điện ly hay số phân tử đã phân ly thành ion
n0: nồng độ chất hòa tan hay tổng số phân tử đã hòa tan trong dung dịch
Độ điện ly  của chất tan phụ thuộc vào: bản chất chất tan, dung môi,
nồng độ và nhiệt độ.
Mối liên hệ giữa độ điện ly  và hệ số Van’t Hoff i:
Giả sử hòa tan n0 phân tử chất điện ly có độ điện ly , v là số ion mà một
phân tử điện ly ra.

vn0 + n0 − n0
i= =  (v − 1) + 1
n0
𝑖−1 (7.7)
Vậy 𝛼 = 𝑣−1

158
7.4 CÂN BẰNG TRONG DUNG DỊCH CỦA CHẤT ĐIỆN LY YẾU
7.4.1 Hằng số điện ly
Khi hòa tan chất điện ly yếu AB vào nước, xảy ra sự cân bằng:

[ A+ ][ B − ] (7.8)
K=
[ AB]
Hằng số cân bằng K được gọi là hằng số điện ly, đặc trưng cho khả năng
điện ly của một chất, phụ thuộc nhiệt độ, bản chất chất điện ly và bản chất
dung môi. Đối với cùng dung môi, ở một nhiệt độ xác định, chất điện ly có
K càng lớn thì điện ly càng mạnh và ngược lại.
7.4.2 Sự liên hệ giữa hằng số điện ly và độ điện ly – Định luật pha loãng
Ostwald

Co α. Co α
K=
Co − Co α
α2 (7.9)
Suy ra: K = Co 1−α

Nếu  << 1 (  5%), một cách gần đúng có thể coi 1-   1. Do đó:
K (7.10)
K = C2 hay α = √ C
Do vậy, đối với một chất điện ly, tại một nhiệt độ xác định, pha loãng dung
dịch làm giảm nồng độ dung dịch và do đó làm tăng độ điện ly. Đó là nội
dung của định luật pha loãng Ostwald.
Lưu ý quan trọng: Đối với chất điện ly yếu đa bậc, quá trình điện ly diễn
ra theo từng bậc. Mỗi bậc điện ly sẽ được đặc trưng bằng một hằng số điện
ly K riêng. Tuy nhiên, trong đa số trường hợp chỉ xét bậc điện ly thứ nhất
do hằng số điện ly bậc thứ 2, thứ 3 luôn luôn nhỏ hơn hằng số bậc thứ nhất
nhiều (lớn hơn 104 lần).

159
Ví dụ. Tính nồng độ ion H+ trong dung dịch H2CO3 0,01 M ở 25 oC. Biết
hằng số điện ly K1 = 4,7.10-3 và K2 = 4,3.10-7.
Giải
[H+ ][HCO−
3]
H2CO3 ⇌ H + + HCO3- K1 = = 4,7.10−3
]H2 CO3 ]
[H+ ][CO2−
3 ]
HCO3- ⇌ H + + CO32- , K 2 = = 4,3.10−7
[HCO−
3]
Do K1 > K2 khoảng 104 nên chỉ tính bậc điện ly thứ nhất:
H2CO3 ⇌ H + + HCO3-
Nồng độ ban đầu C
Nồng độ phân ly C𝛼 C𝛼 C𝛼
Nồng độ cân bằng C - C𝛼 C𝛼 C𝛼
K
→ [H + ] = Cα = C√C = ξKC = ඥ4,7.10−3 . 0,01 = 6,9.10−3 M

7.5. THUYẾT ACID-BASE


7.5.1. Thuyết acid – base của Bronsted
Acid là tất cả những tiểu phân có khả năng cho proton H+, base là tất cả
những tiểu phân có khả năng nhận proton H+. Khi cho proton, acid tạo
thành base liên hợp (HA/A-), khi nhận proton, base tạo thành acid liên hợp
với nó (BH+/B).

H–A + B ⇌ A– + H–B
Acid Base Base liên hợp Acid liên hợp
7.5.2. Chỉ số Hydro pH
Cân bằng điện ly của nước được biểu diễn theo phương trình sau:
H2O + H2O ⇌ H3O+ + OH-
Có thể viết đơn giản như sau: H2O ⇌ H+ + OH-
Hằng số cân bằng của nước hay tích số ion của nước: Kw = [H+][OH-]
Kw phụ thuộc vào nhiệt độ, tại 25 oC (298 K), Kw = [H+][OH-] ≈ 10-14
Nước là hợp chất lưỡng tính vì vừa có khả năng cho và nhận proton.
• Chỉ số Hydro pH:
pH = - lg[H+] (7.11)
Một cách tương tự, người ta định nghĩa chỉ số OH:
pOH = - lg[OH-] (7.12)

160
Tại T  25oC → [H+].[OH-] = 10-14
→ pH + pOH = 14 (7.13)
pH là đại lượng đơn giản đặc trưng cho tính acid-base của dung dịch.
Trong nước nguyên chất:
• Dung dịch trung tính: [H+] = [OH-] = 10-7 M pH = 7.
• Trong dung dịch acid: [H ] > [OH ]
+ -
pH < 7
• Trong dung dịch base: [H ] < [OH ]
+ -
pH > 7
7.5.3 Hằng số acid
Xét một cặp acid-base liên hợp trong dung môi nước:
HA + H2O ⇌ A−+ H3O+ A− + H2O ⇌ HA + OH−
Hằng số điện ly (hằng số acid) Hằng số điện ly (hằng số base)
của acid HA: của base A−
+ −
[H3 O ]. [A ] [HA]. [OH − ]
Ka = ሺ7.14ሻ Kb = ሺ7.15ሻ
[HA] [A− ]
Ka.Kb= [H3O+].[OH-]= Kw
Đối với một cặp acid–base liên hợp tại 25 oC: pKa + pKb= pKw = 14
Hằng số acid Ka và hằng số base Kb chỉ phụ thuộc vào bản chất của acid,
base và phụ thuộc vào nhiệt độ.
Đối với một acid: Ka càng lớn (hay pKa càng nhỏ) thì acid càng mạnh và
base liên hợp của nó càng yếu, và ngược lại. Acid mạnh khi Ka  10-1, acid
trung bình khi 10-5 < Ka < 10-1, acid yếu với Ka < 10-5.
Đối với một base: Kb càng lớn (hay pKb càng nhỏ) thì base càng mạnh và
acid liên hợp của nó càng yếu, và ngược lại.
7.5.4 Tính pH của các dung dịch acid, base
Để tính pH của dung dịch cần phải tính được nồng độ [H3O+] (để đơn giản
thường thể hiện [H+]) trong dung dịch. Lưu ý H+ có thể tham gia vào quá
trình điện ly acid, base và cũng như vào quá trình điện ly nước. Tùy theo
độ mạnh của acid-base (biết được thông qua giá trị Ka), nồng độ của acid
hoặc base mà quá trình điện ly của nước có ảnh hưởng đến pH của dung
dịch hay không.
a) Tính pH của dung dịch acid mạnh
Trong dung dịch tồn tại các cân bằng phân ly:
HnA → An- + nH+
H2O ⇌ OH- + H+
• Bỏ qua sự điện ly của H2O khi dung dịch không quá loãng

161
HnA → An- + nH+
Nồng độ ban đầu CA 0 0
Nồng độ điện ly CA CA nCA
Nồng độ cân bằng 0 CA nCA
+
[H ] = nCA
→ pH = −lg ሺnCA ሻ (7.16)
• Tính thêm [H ] do sự điện ly của nước đóng góp trong trường hợp dung
+

dịch quá loãng (CA < 10-6 M)


Kw
[H + ] = nCA + [OH − ] = nCA +
[H + ]
+ 2 +
[H ] − nCA [H ] − K w = 0 (7.17)
Giải phương trình, chọn nghiệm dương là nồng độ [H ], từ đó tính pH.
+

b) Tính pH của dung dịch base mạnh


Trong dung dịch tồn tại các cân bằng phân ly:
B(OH)n → Bn+ + nOH-
H2O ⇌ OH- + H+
• Bỏ qua sự điện ly của H2O khi dung dịch không quá loãng
Nồng độ ban đầu CB 0 0
Nồng độ điện ly CB CB nCB
Nồng độ cân bằng 0 CB nCB

[OH ] = nCB (7.18)
• Tính thêm [OH ] do sự điện ly của nước đóng góp trong trường hợp
-

dung dịch quá loãng (CB <10-6 M)


Kw
[OH − ] = nCB + [H + ] = nCB +
[OH − ]
[OH − ]2 − nCB [OH − ] − K w = 0 (7.19)
Giải phương trình, chọn nghiệm dương là nồng độ [OH ], từ đó tính pH.
-

c) Tính pH dung dịch acid yếu: bỏ qua sự điện ly của H2O nếu acid
không quá yếu.
HA ⇌ A- + H+
Nồng độ ban đầu CA 0 0
Nồng độ điện ly x x x
Nồng độ cân bằng CA - x x x
2
x2  H + 
Do đó: Ka = = (7.20)
C A − x C A −  H + 
162
Giải phương trình, chọn nghiệm dương là nồng độ [H+], từ đó tính pH.
Nếu nồng độ acid không quá bé, điều kiện gần đúng CA >> [H+]
2
 H + 
Ka =
CA
[ H + ] = K a .C A
1 (7.21)
hay pH = ሺpK a − lgCA ሻ
2
Lưu ý: Khi sử dụng công thức này, phải kiểm tra lại điều kiện CA >> [H+].
d) Tính pH dung dịch base yếu
Bỏ qua sự điện ly của H2O nếu base không quá yếu.
BOH ⇌ B+ + OH-
Nồng độ ban đầu CB 0 0
Nồng độ điện ly x x x
Nồng độ cân bằng CB - x x x
Do đó:
OH − 
2 (7.22)
x2
Kb = =
CB − x CB − OH − 
Giải phương trình, chọn nghiệm dương là nồng độ [OH-], từ đó tính pH.
Nếu nồng độ base không quá bé, điều kiện gần đúng CB >> [OH-]:
2
OH − 
Kb =
CB
[OH − ] = Kb .CB
Suy ra:
1
pH = 14 − ሺpK b − lgCB ሻ
2
1 (7.23)
hoặc pH = 7 + 2 ሺpK a + lgCB ሻ
Lưu ý: Khi sử dụng công thức này, phải kiểm tra lại điều kiện CB >> [OH-].
7.6. CÂN BẰNG CỦA CHẤT ĐIỆN LY ÍT TAN
7.6.1 Tích số tan
Xét sự hòa tan của chất điện ly ít tan như CaF2. Trong dung dịch bão hòa
của CaF2, có cân bằng sau:
CaF2 (s) ⇌ Ca2+ (aq) + 2F- (aq)
Hằng số cân bằng của quá trình này:

163
K sp = [Ca2+ ][F − ]2 tại 25 oC, 𝐾sp =1,46.10-10
𝐾𝑠𝑝 đặc trưng cho tính tan của các chất điện ly do đó được gọi là tích
số hòa tan. Một cách ký hiệu khác của tích số tan trong các tài liệu
khác là T.
Trong trường hợp tổng quát, đối với chất điện ly ít tan AmBn, ta có:
AmBn (s) ⇌ mAn+ (aq) + nBm– (aq)
K sp = [An+ ]m [B m− ]n (7.24)
Tại nhiệt độ không đổi, trong dung dịch bão hòa chất điện ly ít tan, tích
nồng độ các ion với số mũ theo hệ số cân bằng trên phương trình phản
ứng là một hằng số, có tên là tích số tan 𝐾sp . 𝐾sp phụ thuộc bản chất chất
điện ly ít tan và nhiệt độ.
a) Mối liên hệ giữa tích số tan Ksp và độ tan mol (S) của chất điện ly ít
tan
Độ tan mol S (mol/L) biểu thị bằng số mol chất rắn tan tối đa trong 1 lít
dung dịch hay chính là nồng độ dung dịch bão hòa của dung dịch chất điện
ly ít tan.
AmBn (s) ⇌ mAn+ (aq) + nBm– (aq)
Nồng độ ban đầu 0 0
Nồng độ điện ly mS nS
Nồng độ cân bằng mS nS
m n m+n
K sp = m n S
(7.25)
m+n K sp
S= √ m n
m n
Như vậy, khi biết tích số tan thì sẽ tính được độ tan mol của chất điện ly ít
tan và ngược lại. Tích số tan càng nhỏ thì chất rắn sẽ càng ít tan do S nhỏ.
b) Hiệu ứng ion cùng loại
Khi có mặt ion chung, ion giống như thành phần của chất điện ly ít
tan, cân bằng hòa tan sẽ chuyển dịch theo chiều nghịch hay nói cách
khác độ tan mol của chất điện ly ít tan sẽ nhỏ hơn trong dung môi
nguyên chất.
7.6.2 Sự tạo thành và hòa tan một kết tủa của chất điện ly ít tan
AmBn (s) ⇌ mAn+ (aq) + nBm– (aq)
Chỉ số phản ứng: Q = [An+ ]m [Bm− ]n (7.26)
[A ] và [B ] là nồng độ của các ion tại một thời điểm xác định.
n+ m-

164
• Nếu Q > K sp , G > 0, cân bằng chuyển dịch theo chiều nghịch, hay kết
tủa được tạo thành.
• Nếu Q < K sp , G < 0, cân bằng chuyển dịch theo chiều thuận, hay kết
tủa tan.
• Nếu Q = K sp , G = 0, hệ đạt cân bằng, thời điểm kết tủa bắt đầu tạo
thành và cũng là thời điểm kết tủa bắt đầu tan.

165
Phần 2. BÀI TẬP CÓ LỜI GIẢI
---oOo---
Dạng 1. Cân bằng của dung dịch điện ly yếu.
Câu 7.1. Tìm hằng số điện ly của acid acetic biết rằng trong dung dịch
0,01M độ điện ly của nó là 4,3%.
Giải
α2 0,0432
Theo (7.9) K a = Co 1−α2 = 0.01 1−0,043 = 1,9.10−5
Câu 7.2. Tìm độ điện ly acid HCN 0,05M, biết nó có pKa = 9,15.
Giải
K a = 10−pKa = 10−9,15
α2 α2
Theo (7.9) K a = Co 1−α2 → 10−9,15 = 0,05 1−α2 → α = 1,19. 10−4
Câu 7.3. Dung dịch HNO2 có K = 5.10−4. Hỏi nồng độ dung dịch của nó
là bao nhiêu để độ điện ly của nó bằng 20%?
Giải
α2 0,22
Theo (7.9) K a = Co 1−α2 → 5 × 10−4 = Co 1−0,2 → Co = 0,01 M
Câu 7.4. Tại một nhiệt độ xác định, độ điện ly của dung dịch acid acetic
0,100 M bằng 1,32%. Hãy cho biết, tại nồng độ nào thì độ điện ly của dung
dịch acid acetic bằng 90,0%?
Giải
Gọi C1, α1 và C2, α2 là nồng độ, độ điện ly của dung dịch acid acetic.
Tại nhiệt độ xác định, hằng số acid là hằng số:
α2 α2
Theo (7.9) K a = C1 ሺ1−α1 = C2 ሺ1−α2
1 ሻ 2 ሻ
Thay số: C1 = 0,10 M; α1 = 1,32 %; α2 = 90 % → C2 = 2,18.10-6 M
Dạng 2. Tính pH dung dịch.
Câu 7.5. Tính pH của các dung dịch sau tại nhiệt độ khảo sát là 25 oC:
a) HNO3 0,10 M
b) HNO3 1,0.10-7 M
c) KOH 0,50 M
d) NaOH 1,0x10-8M
e) H2SO4 1,0.10-4 M
Giải
a) HNO3 → H+ + NO3-
Nồng độ ban đầu (M) 0,10
Nồng độ phân ly (M) 0,10 0,10 0,10

166
→ pH = - lg[H+] = - lg (0,10) = 1,00
b) Tính thêm [H+] do nước điện ly (CA <10-6 M)
[H+]2 - CA[H+] - Kw = 0
Giải phương trình bậc hai → [H+] = 1,618.10-7 M → pH = 6,79
c) KOH → K+ + OH-
Nồng độ ban đầu (M): 0,50
Nồng độ phân ly (M): 0,50 0,50 0,50
→ pH = 14 + lg[OH ] = 14 + lg0,50 = 13,69
-

d) Tính thêm [OH-] do nước điện ly (CB <10-6 M)


[OH-]2 - CB [OH-] - Kw = 0
Giải phương trình bậc hai:
→ [OH-] = 1,051.10-7 (M)
→ pH = 14 + lg[OH-] = 7,02
e) Giải tương tự câu a, lưu ý H2SO4 acid mạnh 2 bậc
→ [H+] = 2.1,0.10-4 = 2,0.10-4 (M) → pH = - lg (2,0.10-4) = 3,70
Câu 7.6. Tính pH của các dung dịch sau tại 25 oC:
a) CH3COOH 0,10 M, biết rằng Ka = 1,76.10-5
b) NH4OH 0,10, biết rằng pKa = 9,23
c) NH4Cl 0,50, biết rằng Ka = 1,76.10-5
d) HCOONa 0,10 M, biết rằng pKa = 3,75
Giải
a) CH3COOH ⇌ H+ + CH3COO-
Nồng độ ban đầu (M) 0,10
Nồng độ phân ly (M) x x x
Nồng độ cân bằng (M) 0,10 − x x x
[H+ ].[CH3 COO]− x2
Ta có: Ka = = 0,10−x = 1,76.10-5 → x = 1,32. 10−3 M
[CH3 OOH]
→ pH = 2,88.
b) pKb = - lg(Kb) = 14,0 - 9,23 = 4,77 → Kb = 10-4,77
NH4OH ⇌ NH4+ + OH-
Nồng độ ban đầu (M) 0,10
Nồng độ phân ly (M) x x x
Nồng độ cân bằng (M) 0,10 – x x x
[NH4 + ][OH− ] x2
→ Kb = = = 10−4,77 → [OH-] = 1,29.10-3 M
[NH4 OH] 0,10 − x
→ pH = 14 + lg[OH-] = 14 + lg (1,25.10-3) = 11,11

167
c) NH4+ + H2O ⇌ H3O + + NH3
Giải tương tự câu a, pH = 2,53.
d) HCOO- + H2O ⇌ HCOOH + OH-
Giải tương tự câu b, pH = 8,38.
Câu 7.7. Dung dịch acid C6H5COOH có pH = 4,00. Hãy tính nồng độ
mol/L của C6H5COOH biết hằng số điện ly của C6H5COOH là 6,28.10-5,
tại 25 oC.
Giải
pH = 4,00 → [H+] = 1,0.10-4; pKa = 4,20
Áp dụng phương trình (7.21): pH = 1/2 (pKa - lgCA)
Thay số vào phương trình → CA = 1,6.10-4 M
Kiểm tra điều kiện CA >> [H+], không thỏa mãn nên:
2
x2  H + 
Áp dụng phương trình (7.20): K a = =
C A − x C A −  H + 
Thay số vào phương trình → CA = 2,6.10-4 M
Lưu ý: Có thể giải theo phương trình (7.20) ngay từ đầu.
Câu 7.8. Dung dịch muối HCOONa có pH = 9,00. Hãy tính nồng độ mol/L
của HCOONa biết hằng số acid bằng 1,80.10-4, tại 25 oC.
Giải
pH = 9,00 → pOH = 5; Kb = 5,55.10-11
1
Cách 1: Áp dụng phương trình (7.23): pH = 7 + ሺpK a + lgCB ሻ
2
Thay số vào phương trình → CB = 1,8 M
Kiểm tra điều kiện CB >> [OH-], thỏa mãn.
Cách 2: Áp dụng phương trình (7.22):
2
x2 OH − 
Kb = =
CB − x CB − OH − 
Thay số vào phương trình → CB = 1,8 M
Câu 7.9. Tính nồng độ OH-, pH và độ điện ly của dung dịch NH3 0,20 M.
Biết rằng hằng số base của NH3 ở 25 oC là Kb = 1,76.10-5.
Giải
NH3 + H2O ⇌ NH4+ + OH-
Nồng độ ban đầu (M) 0,20
Nồng độ phân ly (M) x x x

168
Nồng độ cân bằng (M) 0,20-x x x
+
[NH4 ][OH− ] x2
Kb = = = 1,76.10−5 → [OH-] = 1,87.10-3 M
[NH3 ] 0,20 − x
→ pH = 14 + lg[OH-] = 14+ lg (1,25.10-3) = 11,27
1.87.10−3
= 100 % = 0,94 %
0.20
Câu 7.10. Cho dung dịch acid HClO 0,050 M (Ka = 3,0.10-8 ở 25 oC). Hãy
viết cân bằng điện ly, xác định độ điện ly α, nồng độ H+ và pH của dung
dịch ở 25 oC.
Giải
HClO ⇌ H+ + ClO-
Nồng độ ban đầu (M) 0,050
Nồng độ phân ly (M) x x x
Nồng độ cân bằng (M) 0,050 − x x x
[H+ ].[ClO]− x2
Ta có: Ka = = 0,050 − x = 3,0.10-8
[HClO]
→ x = 3,87. 10 −5
(M) → pH = 4,41
3,87.10−5
Độ điện ly:  = 100% = 0,077%
0.050
Dạng 3. Tính pH khi trộn các dung dịch
Câu 7.11. Tính pH của các dung dịch sau tại 25 oC:
a) CH3COOH 0,10 M, độ điện ly  = 1,33%
b) Trộn lẫn 1,0 lít dung dịch H2SO4 1,0.10-4 M với 4,0 lít dung dịch KOH
1,0.10-4M
c) Trộn lẫn 1,0 lít dung dịch H2SO4 0,10 M với 1,0 lít dung dịch
CH3COOH 0,10 M có độ điện ly α = 0,10%
Giải
a) CH3COOH ⇌ H+ + CH3COO-
Nồng độ ban đầu (M) 0,10
Nồng độ phân ly (M) α0,1 α0,1 α0,1
Nồng độ cân bằng (M) 0,1(1- αሻ α0,1 α0,1
Thay số: α = 1,33%
→ [H+] = 1,33.10-3 (M)
→ pH = -lg(1,33.10-3) = 2,88.
b) nH2 SO4 = 1,0.10-4 (mol); nKOH = 4,0.10-4 (mol)
H2SO4 + 2KOH → K2SO4 + H2O
→ nKOH(dư) = 2,0.10-4(mol)

169
2,0.10−4
→ [KOH] = ሺ4,0 + 1,0ሻ = 4,0. 10−5 M
→ pH = 14+ lg[KOH] = 14+ lg (4,0.10-5) = 9,60
c) Nồng độ các acid sau khi trộn lẫn:
[CH3COOH] = 0,050 M; [H2SO4] = 0,050 M
H2SO4 → 2H+ + SO42-
Nồng độ ban đầu (M): 0,050 0,10 0,050
Nồng độ phân ly (M): 0,050 0,10 0,050
CH3COOH ⇌ H+ + CH3COO-
Nồng độ ban đầu (M): 0,050
Nồng độ phân ly (M): 0,10%. 0,050 0,10%. 0,050 0,10%.0,050
Nồng độ cân bằng (M): 0,04995 5,0.10 -5
5,0.10-5
→ [H+] = 5,0.10-5 + 0,10 = 5.10-5 + 2. 0,050 = 0,10
→ pH = 1,00
Câu 7.12.
a) Cần thêm vào 1,0 lít dung dịch acid HNO3 2,0 M bao nhiêu lít dung dịch
NaOH 1,8 M để thu được dung dịch có pH = 1,00?
b) Cần thêm vào 1,0 lít dung dịch acid acid HNO3 2,0 M bao nhiêu lít dung
dịch NaOH 1,8 M để thu được dung dịch có pH = 13,00?
Giải
Phản ứng: H+ + OH- → H2O
a) Dung dịch thu được có pH = 1,00 cho thấy H+ dư; [H+] = 1,0.10-1 M
1,0.2,0 − V.1,8
→ 1,0.10-1 = 1,0 + V
→ V = 1,0 L
b) Dung dịch thu được có pH = 13,00 cho thấy OH- dư;
[OH-] = 1,0.10-1 M
V.1,8 − 1,0.2,0
→ 1,0.10-1 = 1,0 + V
→ V = 1,24 L
Dạng 4. Cân bằng của chất điện ly ít tan
Câu 7.13. Tại 25 oC, tích số tan của Ag2SO4 bằng 7.10-5. Tính độ hòa tan
của bạc sulfat biểu diễn bằng mol/L và g/L. Biết rằng khối lượng mol của
Ag2SO4 là 312 g/mol.
Giải
Xét cân bằng hòa tan:
Ag2SO4 (s) ⇌ 2Ag+ (aq) + SO42– (aq)

170
Nồng độ ban đầu (M): 0 0
Nồng độ điện ly (M): S 2S S
Nồng độ cân bằng (M): 2S S
3 Ksp
K sp = 22. 11. S3 → S = √ 4

→ S = 2,6.10 mol/L = 312 . 2,6.10-2 = 8,1 g/L.


-2

Câu 7.14 Ở cùng một nhiệt độ khảo sát, độ hòa tan của canxi oxalate
CaC2O4 trong dung dịch muối amoni oxalate (NH4)2C2O4 0,05M nhỏ hơn
trong nước nguyên chất bao nhiêu lần, biết tích số tan của canxi oxalate
bằng 3,8.10-9?
Giải
Đặt S, S’ là độ hòa tan của CaC2O4 trong nước và trong dung dịch
(NH4)2C2O4 0,050 M
- Trong nước nguyên chất:
CaC2O4 (s) ⇌ Ca2+ (aq) + C2O42– (aq)
Nồng độ ban đầu (M): 0 0
Nồng độ điện ly (M): S S S
Nồng độ cân bằng (M): S S
Ksp= S → S = ඥK sp = 6,12.10 (M)
2 -5

Trong dung dịch (NH4)2C2O4 0,050 M:


(NH4)2C2O4 → 2NH4+ + C2O42-
0,050 0,050
CaC2O4 (s) ⇌ Ca (aq) + C2O42– (aq)
2+

Nồng độ ban đầu 0 0,050


Nồng độ điện ly S’ S’ S’ + 0,050
Nồng độ cân bằng S’ S’ + 0,050
→ Ksp = S’(S’ + 0,050) → S’ = 7,60.10 M
-8

S 6,12.10−5
Độ giảm độ tan = S′ = 7,60.10−8 = 805 lần
Câu 7.15. Độ hòa tan của PbI2 ở 18 oC bằng 1,5.10-3 M. Hãy tính:
a) Nồng độ của ion Pb2+ và I- trong dung dịch bão hòa PbI2 ở 18 oC.
b) Tích số hòa tan của PbI2 ở 18 oC.
c) Khi thêm KI vào thì độ hòa tan của PbI2 tăng hay giảm? Vì sao?
d) Tại 18 oC, muốn giảm độ hòa tan của PbI2 đi 15 lần, thì phải thêm bao
nhiêu mol KI vào trong 0,50 lít dung dịch bão hòa PbI2.

171
Giải
a) Độ tan mol của PbI2 chính là nồng độ của dung dịch bão hòa PbI2.
→ S = 1,5.10-3 M
PbI2 (s) ⇌ Pb2+ (aq) + 2I– (aq)
Nồng độ ban đầu (M) 0 0
Nồng độ điện ly (M) S S 2S
Nồng độ cân bằng (M) S 2S
→ [Pb ] = S = 1,5 . 10 (M); [I ] = 2 . S = 3.10 M
2+ -3 - -3

b) K sp = [Pb2+]. [I-]2 = 1,35 . 10-8 tại 18 oC.


c) Thêm KI vào nồng độ I- tăng → cân bằng dịch chuyển theo chiều nghịch
→ độ tan giảm.
d) Độ tan giảm 15 lần → S’ = [Pb2+] = 10-4 M
Mà: K sp = [Pb2+] . [I-]2 = 1,35.10-8 → [I-] = 0,0116 M
Nồng độ I- trong dung dịch bằng tổng nồng độ do KI phân ly và PbI2 phân
ly
→ 0,0116 = [KI] + 2S’ → [KI] = 1,14.10-2 M
Số mol KI cần thêm vào là: n = 0,50.1,14.10-2 = 5,7.10-3 mol
Câu 7.16. Kết tủa BaSO4 có tạo thành hay không nếu trộn lẫn hai thể tích
bằng nhau của dung dịch BaCl2 0,010 M với dung dịch CaSO4 bão hòa.
Cho biết tích số tan của BaSO4 và CaSO4 lần lượt bằng 1,08.10-10 và
6,1.10-5 tại 25 oC.
Giải
Xét cân bằng hòa tan: BaSO4 (s) ⇌ Ba2+ (aq) + SO42– (aq)
Kết tủa có tạo thành hay không, dựa trên so sánh Q và Ksp
Q = [Ba2+][SO42-]
0,010
[Ba2+] = = 0,0050 ሺMሻ;
2
S 1 Ksp
[SO42-] = = . √ = 3,90.10-3 M
2 2 2

Thay vào: Q = 1,95.10-5 > Ksp → kết tủa BaSO4 được tạo thành.
Câu 7.17. Kết tủa Ag3PO4 tạo thành hay không trong các trường hợp sau:
a) Trộn lẫn 1 thể tích dung dịch Na3PO4 0,005 M với 4 thể tích AgNO3
0,005.
b) Trộn lẫn 4 thể tích dung dịch Na3PO4 0,001 M với 1 thể tích AgNO3
0,02 M.
Cho biết K sp = 1,8.10-18 tại nhiệt độ khảo sát.
172
Giải
Xét cân bằng hòa tan: Ag3PO4 (s) ⇌ 3Ag+ (aq) + PO43– (aq)
Q = [Ag+]3 [PO43-]
a) Nồng độ sau trộn lẫn:
4 . 0,0050 1 . 0,0050
[Ag+] = = 0,0040 M; [PO3−
4 ]= = 0,0010 M
ሺ4+1ሻ ሺ4+1ሻ
Q = 6,4.10-11 > Ksp → tạo thành kết tủa.
b) Tương tự, Q = 5,12.10-11 > Ksp → có tạo thành kết tủa Ag3PO4.
Câu 7.18. Xác định nồng độ Ba2+ cần thiết để bắt đầu kết tủa BaSO4 khi
cho muối dễ tan BaCl2 dạng tinh thể vào dung dịch Na2SO4 1,5.10-3 M. Giả
sử việc thêm muối BaCl2 không làm thay đổi thể tích dung dịch. Cho biết
ở nhiệt độ khảo sát, K sp = 1,1.10-10.
Giải
Xét cân bằng hòa tan BaSO4:
BaSO4 (s) ⇌ Ba2+ (aq) + SO42– (aq)
Thời điểm bắt đầu tạo kết tủa BaSO4 khi: Q = K sp
Ksp
→ [Ba2+ ] = = 7,3x10-8 M
[SO2−
4 ]
Câu 7.19. Trộn lẫn dung dịch Pb(NO3)2 0,010 M với dung dịch H2SO4
0,010 M với thể tích bằng nhau. Chứng minh rằng dung dịch sau khi trộn
lẫn có kết tủa PbSO4. Tìm nồng độ mol/L của ion Pb2+ và SO42- trong dung
dịch bão hòa. Biết rằng, tất cả quá trình đều khảo sát tại cùng một nhiệt độ
và tích số tan của PbSO4 là 1.38.10-8.
Giải
Nồng độ sau trộn lẫn: [Pb2+] = 0,0050 M; [SO42–] = 0,0050 M
Xét cân bằng hòa tan PbSO4:
PbSO4 (s) ⇌ Pb2+ (aq) + SO42– (aq)
Q = [Pb2+] [SO42-] = 0,0050 . 0,0050 = 2,5.10-5 > K sp = 1,38.10-8
→ tạo thành kết tủa PbSO4
Dung dịch bão hòa → K sp = [Pb2+].[SO42-] → [Pb2+] = [SO42-]
= ඥ1,38.10−8 = 1,17.10-4 M.
Câu 7.20. Tính độ hòa tan, pH, pOH của dung dịch bão hòa Zn(OH)2. Biết
tích số tan của Zn(OH)2 ở 25 oC là 4,3.10-17.
Giải
Xét cân bằng hòa tan:
Zn(OH)2 (s) ⇌ Zn2+ (aq) + 2OH– (aq)
173
Nồng độ điện ly (M) S 2S
Nồng độ cân bằng (M) S 2S
→ K sp = 4.S3 → S = 2,2.10-6 (M)
→ pOH = -lg[OH-] = -lg (2.S) = 5,36 → pH = 14-pOH = 8,64
Câu 7.21. Thêm từ từ NaOH rắn vào dung dịch Mg(NO3)2 0,010 M (giả
sử không có sự thay đổi thể tích). Tính pH tại đó Mg(OH)2 bắt đầu kết tủa.
Cho biết tích số tan của Mg(OH)2 tại nhiệt độ khảo sát là 1,2.10-11.
Giải
Xét cân bằng hòa tan:
Mg(OH)2 (s) ⇌ Mg2+ (aq) + 2OH– (aq)
Kết tủa Mg(OH)2 bắt đầu tạo thành: Q = K sp = [Mg2+][OH-]2
Ksp
→ [OH-] =√[Mg2+ ] = 3,46.10-5 (M) → pH = 14 + lg[OH-] = 9,54.
Câu 7.22. Tích số tan của Zn(OH)2 ở 25 oC được ký hiệu là K sp . Hãy thiết
lập biểu thức tính pH của dung dịch bão hòa Zn(OH)2 ở nhiệt độ đó và
biểu thức tính Go của phản ứng hòa tan Zn(OH)2 trong dung dịch bão hòa
ở nhiệt độ đã cho theo K sp .
Giải: Xét cân bằng hòa tan:
Zn(OH)2 (s) ⇌ Zn2+ (aq) + 2OH– (aq)
Nồng độ điện ly (M) S S 2S
Nồng độ cân bằng (M) S 2S
3 Ksp
→ K sp = 4.S3 → S = √ 4
3 Ksp
pH = 14+ lg[OH-] = 14 + lg(2.S) = 14 + lg 2 √ 4
Biểu thức ∆G = -RTlnKP = -RTlnKC (dung dịch lỏng nên Kp = KC)
o

∆Go = -RTln[Zn2+].[OH-]2 = -RTlnK sp

174
Phần 3. BÀI TẬP TRẮC NGHIỆM
---oOo---
Câu 7.1. Tại 25 C, chỉ số acid pKa của HF, HCN, HClO lần lượt là 3,46;
o

9,18 và 7,53. Hãy cho biết thứ tự độ mạnh giảm dần của acid là:
A. HF > HCN > HClO B. HF > HClO > HCN
C. HClO > HCN > HF D. HCN > HClO > HF
Câu 7.2. Chỉ ra cặp acid-base liên hợp trong cân bằng điện ly sau:
CN-(aq) + H2O (l) ⇌ HCN (aq) + OH- (aq)
A. CN-/HCN; H2O/OH- B. HCN/CN-; OH-/H2O
C. CN-/HCN; OH-/H2O D. HCN/CN-; H2O/OH-
Câu 7.3. Chỉ ra cặp acid-base liên hợp trong cân bằng điện ly sau:
HCO3-(aq) + H3O+ (aq) ⇌ H2CO3 (aq) + H2O (l)
A. HCO3-/H2CO3; H2O/H3O+
B. HCO3-/H2CO3-; H3O+/H2O
C. H2CO3/ HCO3-; H3O+/H2O
D. H2CO3/ HCO3-; H2O/H3O+
Câu 7.4. Tại 25 oC, chỉ số acid (pKa) của HF, HCN, HClO lần lượt là 3,46;
9,18 và 7,53. Thứ tự độ mạnh giảm dần của base liên hợp của các acid trên
là:
A. F- > CN- > ClO- B. F- > ClO- > CN-
C. ClO- > CN- > F- D. CN- > HClO > F-
Câu 7.5. Đại lượng nào đặc trưng cho độ mạnh của một acid tại một nhiệt
độ xác định?
A. pH B. Hằng số acid Ka
C. Độ điện ly α D. pOH
Câu 7.6. Tại 25 C, acid formic có hằng số acid là 1,80.10-4. Hằng số base
o

và chỉ số base của ClO- tại 25 oC lần lượt là:


A. 5,56.10-11; 3,74 B. 5,56.10-11; 10,26
C. 1,80.10-4; 10,26 D. 1,80.10-4; 3,74
Câu 7.7. Tại 25 oC, NH3 có hằng số base là 1,76.10-5 tại 25 oC. Hằng số
acid và chỉ số acid của NH4+ tại 25 oC lần lượt là:
A. 5,68.10-10; 4,76 B. 1,76.10-5; 9,24
C. 5,68.10-10; 9,24 D. 1,76.10-5; 4,76
Câu 7.8. Tại 25 oC, pH của dung dịch acid HNO3 1,0.10-10 M và pOH của
dung dịch HCl 0,10 M lần lượt là:
A. 10,00; 1,00 B. 10,00; 13,00

175
C. 7,00; 13,00 D. 8,00; 1,00
Câu 7.9. Tại 25 C, nồng độ H3O và pH của dung dịch acid propanoic
o +

CH3CH2COOH 0,050 M là bao nhiêu? Biết rằng Ka = 1,34.10–5.


A. 7,23.10-4; 3,14 B. 2,43.10-5; 4,61
C. 4,23.10-3; 2,37 D. 8,18.10-4; 3,09
Câu 7.10. Tại 25 oC, pH của dung dịch acid H2SO4 0,050 M là:
A. 1,30 B. 1,00
C. 7,00 D. 5,00
Câu 7.11. Tại 25 C, pOH của dung dịch acid NaOH được tạo thành bằng
o

cách hòa tan 4,0 g NaOH (M = 40 g/mol) vào nước được 1,0 L dung dịch
là bao nhiêu?
A. 1,00 B. 1,30
C. 13,00 D. 12,70
Câu 7.12. Tại 25 C, pH của dung dịch acid H2SO4 được tạo thành bằng
o

cách cho 2,5 mL dung dịch acid sunfuric đậm đặc (96%, d = 1,84 g/mL)
vào 1,0 lít nước cất là bao nhiêu?
A. 1,00 B. 1,35
C. 1,04 D. 2,04
Câu 7.13. Chỉ số hydro pH của dung dịch KOH 0,1 M là bao nhiêu biết
độ điện ly của KOH trong điều kiện này là 30%?
A. 1,00 B. 1,52
C. 12,48 D. 13,00
Câu 7.14. Chỉ số hydro pH của dung dịch thu được sau khi trộn lẫn 0,50
lít dung dịch H2SO4 1,0.10-4 M vào 1,0 lít dung dịch KOH 1,0.10-4 M là
bao nhiêu?
A. 4,00 B. 7,00
C. 8,00 D. 2,00
Câu 7.15. Tại nồng độ mol nào thì dung dịch acid acetic CH3COOH có độ
điện ly là 20% biết hằng số acid ở nhiệt độ đang xét có giá trị 1,76.10-5?
A. 6,42.10-4 B. 4,4.10-4
C. 1,76.10-4 D. 3,52.10-4
Câu 7.16. Tại 25 oC, nồng độ OH- và pH của dung dịch ethylamin
(CH3CH2NH2) 0,010 M lần lượt là bao nhiêu, biết rằng Kb=4,28.10–4?
A. 2,07.10-3; 11,32 B. 2,43.10-5; 4,61
C. 7,23.10-4; 10,86 D. 4,23.10-6; 8,63

176
Câu 7.17. Dung dịch acid HCN có nồng độ 0,20 M có hằng số
Ka = 6,2.10-10. Độ điện ly  và pH của dung dịch lần lượt là:
A.  = 4,9.10-5; pH = 3,20 B.  = 5,6.10-3; pH = 4,95
C.  = 1,1.10-5; pH = 5,43 D.  = 3,9.10-5; pH = 2,67
Câu 7.18. Dung dịch acid C6H5COONa có pH = 8,38. Hãy tính nồng độ
mol/L của dung dịch C6H5COOH biết hằng số điện ly của nó ở nhiệt độ
khảo sát là 6,28.10-5.
A. 0,014 B. 0,036
C. 0,024 D. 0,056
Câu 7.19. Cần thêm bao nhiêu lít dung dịnh KOH 1,8M vào 1,0 lít dung
dịch HCl 2,0 M để được dung dịch có pH là 2,0?
A. 1,1 B. 2,0 C. 1,5 D. 3,1
Câu 7.20. Phát biểu nào sau đây là đúng về các dung dịch H2SO4, HCl,
NaOH, Ba(OH)2 có cùng nồng độ, tại cùng nhiệt độ?
A. Dung dịch HCl có pH thấp nhất và NaOH có pH cao nhất.
B. Dung dịch H2SO4 có pH thấp nhất và NaOH có pH cao nhất.
C. Dung dịch HCl có pH thấp nhất và Ba(OH)2 có pH cao nhất.
D. Dung dịch H2SO4 có pH thấp nhất và Ba(OH)2 có pH cao nhất.
Câu 7.21. Có các dung dịch sau ở cùng nhiệt độ: HCl 1,0x10-10 M;
CH3COOH 0,10 M (Ka = 1,76.10-5); KOH 1,0.10-5 M; NaCN 0,10 M (Ka
= 6,2.10-10), sắp xếp nào sau đây về giá trị pH của các dung dịch là đúng?
A. NaCN > KOH > HCl > CH3COOH
B. KOH > NaCN > CH3COOH > HCl
C. HCl > CH3COOH > NaCN > KOH
D. CH3COOH > HCl > NaCN > KOH
Câu 7.22. pH của dung dịch nước thay đổi bao nhiêu đơn vị khi thêm 6,0
g acid CH3COOH vào 1,0 lít nước, biết rằng tại nhiệt độ khảo sát
CH3COOH có Ka = 1,76.10-5?
A. 4,12 B. 6,00
C. 2,88 D. 6,75
Câu 7.23. Cho các dung dịch sau đây ở cùng một nhiệt độ: HCl 1,0.10-10
M; CH3COOH 0,10 M (Ka = 1,76.10-5); KOH 1,0.10-5 M; NaCN 0,10 M
(Ka = 6,2.10-10), sắp xếp nào sau đây về giá trị pH của các dung dịch là
đúng?
A. NaCN > KOH > HCl > CH3COOH
B. KOH > NaCN > CH3COOH > HCl
177
C. HCl > CH3COOH > NaCN > KOH
D. CH3COOH > HCl > KOH >NaCN
Câu 7.24. pOH của dung dịch nước thay đổi bao nhiêu đơn vị khi thêm
3,0 mL dung dịch NH3 đậm đặc (C = 25%, d = 0,91 g/cm3) vào 1,0 lít
nước, biết rằng tại nhiệt độ khảo sát NH3 có Kb = 1,76.10-5.
A. 4,12 B. 3,08
C. 3,92 D. 2,75
Câu 7.25. Trong dung môi nước, ở cùng một nhiệt độ và nồng độ, hãy cho
biết dung dịch amin nào sau đây có độ điện ly lớn nhất: NH3, CH3NH2,
CH3CH2NH2, C6H5NH2? Biết hằng số base của các amin trên lần lượt là:
1,76.10-5; 4,8.10-4; 4,28.10-4; 3,94.10-10.
A. NH3 B. CH3NH2 C. CH3CH2NH2 D. C6H5NH2
Câu 7.26. Trong dung môi nước, ở cùng một nhiệt độ và cùng nồng độ,
hãy cho biết các dung dịch amin nào sau đây lần lượt có pOH lớn nhất và
nhỏ nhất: NH3, CH3NH2, CH3CH2NH2, C6H5NH2. Biết thứ tự hằng số base
của các amin trên lần lượt là: 1,76.10-5; 4,8.10-4; 4,28.10-4; 3,94.10-10.
A. NH3 và CH3NH2
B. CH3NH2 và NH3
C. CH3CH2NH2 và C6H5NH2
D. C6H5NH2 và CH3CH2NH2
Câu 7.27. Cho các dung dịch sau ở cùng nhiệt độ khảo sát,: HCl 1,0.10-5
M; HCl 1,0.10-10 M; NaCl 0,10 M; KOH 2,0.10-9 M; NH4Cl 0,050 M và
KCN 0,10 M. Hãy cho biết có bao nhiêu dung dịch có pH = 7. Biết hằng
số acid của NH4+, HCN lần lượt là 5,68.10-10 và 6,2.10-10.
A. 2 B. 3 C. 4 D. 5
Câu 7.28. Cho các dung dịch sau ở cùng nhiệt độ khảo sát: HCl 1,0x10-5
M; HCl 1,0x10-10 M; NaCl 0,10M; KOH 2,0.10-9 M; NH4Cl 0,050 M và
KCN 0,10 M. Hãy cho biết có bao nhiêu dung dịch pH lớn hơn 7. Biết
hằng số acid của NH4+, HCN lần lượt là 5,68.10-10; 6,2.10-10.
A. 1 B. 2 C. 3 D. 4
Câu 7.29. Cho các dung dịch sau ở cùng nhiệt độ khảo sát: HCl 1,0.10-5
M; HCl 1,0.10-10 M; NaCl 0,10 M; KOH 2,0.10-9 M; NH4Cl 0,050 M và
KCN 0,10 M. Hãy cho biết có bao nhiêu dung dịch có pH nhỏ hơn 7. Biết
hằng số acid của NH4+, HCN lần lượt là 5,68.10-10; 6,2.10-10.
A. 1 B. 2 C. 3 D. 4

178
Câu 7.30. Trộn hai thể tích bằng nhau của hai dung dịch BaCl2 và Na2SO4
0,0010 M. Nồng độ của dung dịch BaCl2 tối thiểu là bao nhiêu để có kết
tủa BaSO4 hình thành? Biết rằng, tại nhiệt độ đang xét, tích số tan của
BaSO4 là 1,07.10-10.
A. 1,08.10-7M B. 4.28.10-7M
C. 2,16.10-7M D. Không đủ dữ kiện để tính
Câu 7.31. Tích số tan của Ag2CrO4 ở 25 C là 1,12.10-12. Độ tan (mol/L)
o

của Ag2CrO4 tại nhiệt độ đó là:


A. 5,62.10-6 B. 6,5.10-5 C. 4,8.10-4 D. 2,4.10-4
Câu 7.32. Tính độ tan mol (M) của Fe(OH)3 trong dung dịch có pOH =
8,0. Biết tích số tan của Fe(OH)3 tại nhiệt độ đang xét là 1,0.10-36.
A. 1,0.10-16 B. 1,0.10-12
C. 2.10-12 D. 2.10-16
Câu 7.33. Hãy cho biết thứ tự tạo thành kết tủa khi cho từ từ dung dịch
AgNO3 0,10 M vào 1,0 lít dung dịch AgCl, AgBr, AgI có cùng nồng độ
0,10 M. Biết tích số tan của AgCl, AgBr, AgI ở 25 oC lần lượt là:
1,77.10-10; 5,35.10-13; 8,51.10-17.
A. AgCl – AgBr – AgI B. AgI – AgBr – AgCl
C. AgC – AgBr – AgI D. Không dự đoán được
Câu 7.34. Tích số tan của CaF2 ở 25 C là 1,46. 10-10. Nồng độ bão hòa
o

(g/L) của dung dịch CaF2 tại nhiệt độ trên là:


A. 3.32.10-4 B. 4,22.10-3
C. 1,47.10-5 D. 5,15.10-5
Câu 7.35. Độ tan của PbSO4 (M = 303 g/mol) trong nước ở nhiệt độ thí
nghiệm là 0,038 g trong 1000 mL dung dịch. Tìm tích số tan của PbSO4 ở
nhiệt độ trên.
A. 8,0.10-9 B. 1,6.10-8
C. 8.0.10-10 D. 1.6.10-9
Câu 7.36. Thêm từ từ KOH rắn vào dung dịch Mg(NO3)2 0,010 M (giả sử
không có sự thay đổi thể tích). Tính pH tại đó Mg(OH)2 bắt đầu kết tủa.
Cho biết tích số tan của Mg(OH)2 tại nhiệt độ thực nghiệm là 1,2.10-11.
A. 9,54 B. 12,42 C. 11,51 D. 13.04
Câu 7.37. Cần thêm bao nhiêu gam NaCl vào 100 ml dung dịch bão hòa
AgCl để độ tan AgCl giảm còn 1,2.0-5 M. Biết tích số tan của AgCl tại
nhiệt độ trên là 1,6.10-10 (giả sử không có sự thay đổi thể tích).
A. 3,9.10-6 B. 8.3.10-6

179
C. 5,8.10-6 D. 9,8.10-6
Câu 7.38. Cho 0,5 lít dung dịch CaCl2 3.10-2 M vào 1,0 lít dung dịch K2SO4
3.10-2 M. Tính thương số phản ứng Q và dự đoán kết tủa CaSO4 có tạo
thành hay không? Biết CaSO4 có Ksp = 7,10.10-5 ở nhiệt độ khảo sát.
A. Q = 2,0.10-4; kết tủa không tạo thành
B. Q = 2,0.10-4; kết tủa tạo thành
C. Q = 2,0.10-8; kết tủa không tạo thành
D. Q = 2,0.10-8; kết tủa tạo thành
Câu 7.39. Cho 5,0 mL dung dịch Ca(NO3)2 0,02 M vào ống nghiệm 1 và
5,0 mL dung dịch AgNO3 4.10-3 M vào ống nghiệm 2. Sau đó đồng thời
cho vào 2 ống nghiệm trên mỗi ống 5 mL dung dịch Na3PO4 0,1 M. Biết
tích số tan của Ca3(PO4)2 và Ag3PO4 ở nhiệt độ khảo sát lần lượt là 1,2.10-
26
và 1,8.10-18. Kết luận nào sau đây đúng?
A. Ống nghiệm 1 xuất hiện kết tủa; ống nghiệm 2 không xuất hiện kết tủa
B. Ống nghiệm 1 không xuất hiện kết tủa; ống nghiệm 2 xuất hiện kết tủa
C. Cả hai ống nghiệm đều không xuất hiện kết tủa
D. Cả hai ống nghiệm đều xuất hiện kết tủa
Câu 7.40. Tính độ tan mol/L của CaF2 trong dung dịch NaF 0,100 M. Biết
tích số tan của CaF2 ở 25 oC là 1,46. 10-10.
A. 1.46.10-8 B. 1,46.10-4
C. 5.15.10-8 D. 5,15.10-4
---oOo---

ĐÁP ÁN BÀI TẬP CHƯƠNG 7


7.1- 7.2- 7.3- 7.4- 7.5- 7.6- 7.7- 7.8- 7.9- 7.10
B D C D B B C C C -B
7.11 7.12 7.13 7.14 7.15 7.16 7.17 7.18 7.19 7.20
-A -B -C -B -D -A -B -B -A -D
7.21 7.22 7.23 7.24 7.25 7.26 7.27 7.28 7.29 7.30
-A -A -D -C -C -C -B -A -B -A
7.31 7.32 7.33 7.34 7.35 7.36 7.37 7.38 7.39 7.40
-B -B -B -A -B -A -B -B -D -A

180
Chương 8. ĐIỆN HÓA HỌC
---oOo---
Mục tiêu chương 8
Sau khi học xong chương này, sinh viên có khả năng:
1. Định nghĩa được phản ứng oxy hóa khử, quá trình oxy hóa và quá
trình khử.
2. Phân biệt được điều kiện để hóa năng chuyển thành điện năng hay
nhiệt năng.
3. Trình bày được cấu tạo, nguyên tắc hoạt động và tính toán được
các đại lượng đặc trưng của nguyên tố Galvanic.
4. Trình bày được dãy điện hóa và nguyên tắc sắp xếp các cặp oxy
hóa khử vào dãy điện hóa.
5. Xác định được điều kiện để phản ứng oxy hóa khử có thể xảy ra.

181
Phần 1. TÓM TẮT LÝ THUYẾT
8.1 PHẢN ỨNG OXY HÓA KHỬ
8.1.1 Số oxy hóa
Số oxy hóa là điện tích dương (+) hay âm (-) của nguyên tố trong hợp chất
được tính với giả thiết hợp chất được tạo thành từ các ion.
Quy tắc xác định số oxy hóa:
• Số oxy hóa các nguyên tố tự do bằng 0.
• Số oxy hóa của ion một nguyên tử bằng điện tích của ion đó.
• Số oxy hóa của kim loại kiềm (Na, K,...) luôn bằng +1.
• Số oxy hóa của O và H trong đa số trường hợp là -2 và +1.
• Số oxy hóa của mỗi nguyên tố trong hợp chất cộng hóa trị bằng điện
tích của nguyên tố đó khi xem cặp electron chuyển hẳn về nguyên tố có
độ âm điện lớn hơn.
• Tổng số oxy hóa của một phân tử trung hòa về điện luôn bằng 0.
8.1.2 Phản ứng oxy hóa khử
Phản ứng oxy hóa khử là phản ứng có thay đổi số oxy hóa hoặc có sự cho
và nhận electron. Trong đó chất cho electron được gọi là chất khử, chất
nhận electron được gọi là chất oxy hóa.
Quá trình oxy hóa: là quá trình cho electron ứng với số oxy hóa tăng.
Ví dụ, M - ne → Mn+
Quá trình khử: là quá trình nhận electron ứng với số oxy hóa giảm.
Ví dụ, Mn+ + ne → M
Trong đó: Mn+ được gọi là chất oxy hóa và M được gọi là chất khử. Cặp
Mn+/ M được gọi là cặp oxy hóa-khử liên hợp.
8.1.3 Phản ứng oxy hóa khử và dòng điện
- Nếu quá trình cho nhận electron của phản ứng oxy hóa khử diễn ra trực
tiếp trong dung dịch thì hóa năng sẽ chuyển thành nhiệt năng.
- Nếu quá trình cho nhận electron của phản ứng oxy hóa khử diễn ra ở
mạch ngoài thì hóa năng sẽ chuyển thành điện năng.
- Dụng cụ đặc biệt giúp chuyển hóa hóa năng thành điện năng gọi là nguyên
tố Galvanic hay pin điện hóa học.
8.2 NGUYÊN TỐ GALVANIC
8.2.1 Cấu tạo nguyên tố Galvanic
Nguyên tố Galvanic là thiết bị dùng để chuyển hóa năng lượng của một
phản ứng oxy hóa khử tự xảy ra thành điện năng bằng cách tách riêng quá
trình oxy hóa và quá trình khử.
182
Cấu tạo: Pin Galvanic tiêu chuẩn có cấu tạo gồm hai điện cực bằng thanh
kim loại được nhúng trong dung dịch điện ly có chứa ion kim loại làm điện
cực tương ứng. Hai điện cực này sẽ được nối với mạch ngoài thông qua
dây dẫn và các thiết bị sử dụng điện. Một cầu muối được dùng để nối hai
dung dịch điện ly có vai trò tạo kênh dẫn cho phép ion di chuyển giữa hai
dung dịch điện ly nhằm đảm bảo sự trung hòa điện tích trong mỗi dung
dịch điện ly, và qua đó đóng kín mạch điện. Thông thường, cầu muối được
làm bằng ống thủy tinh chứa dung dịch muối bão hòa và được bít chặt hai
đầu bằng màng xốp cho phép ion di chuyển. Tuy nhiên, trong các thiết kế
đơn giản cầu muối có thể được thay thế bằng các màng xốp nhằm ngăn
không cho các dung dịch điện ly hòa tan nhanh chóng vào nhau nhưng vẫn
đảm bảo khả năng dẫn ion.
8.2.2 Hoạt động của nguyên tố Galvanic
Khi pin Galvanic hoạt động, tại anode diễn ra quá trình oxy hóa và tại
cathode diễn ra quá trình khử. Các electron sẽ di chuyển từ anode sang
cathode thông qua dây dẫn. Chiều dòng điện ngược chiều với dòng
electron. Anode đóng vai trò cực âm (cho electron) và cathode đóng vai
trò cực dương (nhận electron). Hình 8.1 minh họa cấu tạo của pin Galvanic
Cu-Zn.
1,10 V

Cực đồg Cực kẽm


Cầu muối
(Cathode) (Anode)

Cu2+ + 2e → Cu, o = + 0,34 V Zn – 2e → Zn2+, o = - 0,76 V


Zn + Cu → Zn2+ + Cu
2+

Hình 8.1 Nguyên tố Galvanic Cu - Zn


Ví dụ, cho pin Galvanic hoạt động dựa trên phản ứng oxy hóa khử:
Zn + Cu2+ → Zn2+ + Cu
Tại anode (cực âm): xảy ra quá trình oxy hóa (quá trình cho electron)
Zn – 2e → Zn2+
Tại cathode (cực dương): xảy ra quá trình khử (quá trình nhận electron).

183
Cu2+ + 2e → Cu
8.2.3 Ký hiệu nguyên tố Galvanic
Ký hiệu pin Galvanic được viết theo chiều từ anode sang cathode (trái sang
phải). Nếu chất oxy hóa và chất khử liên hợp ở cùng một pha dùng dấu “,”
để phân tách, nếu ở hai pha khác nhau dùng dấu “|” để phân tách. Giữa hai
điện cực dùng dấu “||” để ngăn cách. Lưu ý một số tài liệu ký hiệu “/” được
sử dụng thay thế cho “|”.
Ví dụ, cho pin Galvanic tạo thành từ thanh Zn nhúng trong dung dịch
ZnSO4 (anode) và thanh Cu nhúng trong dung dịch CuSO4 (cathode), ký
hiệu pin trên là:
(-) Zn | ZnSO4 || CuSO4 | Cu (+)
Lưu ý: Nếu cả hai thành phần cấu tạo nên điện cực chỉ có dung dịch hoặc
khí (thiếu kim loại đóng vai trò dẫn điện) thì thường dùng kim loại trơ (Pt)
làm trung gian.
Ví dụ, nguyên tố Galvanic hoạt động trên cơ sở phản ứng:
Cu + FeCl3 → CuCl2 + FeCl2 có ký hiệu như sau:
(-) Cu | CuCl2 || FeCl3 | FeCl2, Pt (+)
8.2.4 Thế điện cực () và thế điện cực tiêu chuẩn (o)
Thế điện cực là đại lượng điện thế đặc trưng của một điện cực đo được tại
bề mặt điện cực khi pin hoạt động. Nó đặc trưng cho khả năng trao đổi
electron của điện cực. Nếu thế khử càng lớn thì khuynh hướng nhận
electron càng mạnh và ngược lại.
Xét bán phản ứng: [ox] + n e- → [kh]
RT [ox] (8.1)
Ta có phương trình Nerst: φ = φo + ln
[ox]/[kh] nF [kh]
Trong đó,
φ: thế điện cực của cặp oxy hóa – khử liên hợp tại điều kiện đang xét, còn
gọi là thế khử. Nếu phản ứng trên viết theo chiều ngược lại ta có thế oxy
hóa.
𝜑 𝑜 [𝑜𝑥]/[𝑘ℎ] : thế điện cực của cặp oxy hóa – khử liên hợp tại điều kiện tiêu
chuẩn (chất tan nồng độ là 1M, chất khí áp suất là 1 atm, chất rắn ở dạng
tinh thể nguyên chất ở áp suất 1 atm, chất lỏng nguyên chất ở áp suất 1
atm). Thế điện cực chuẩn của rất nhiều điện cực đã được xác định và được
liệt kê dưới dạng các bảng tra.
Tuy nhiên, ta chỉ có khả năng đo được điện thế của điện cực khi so sánh
nó với một điện cực khác. Thông thường, điện cực hydro tiêu chuẩn SHE

184
(standard hydrogen electrode) với thế điện cực quy ước là 0 volt được sử
dụng làm điện cực so sánh. Ngoài ra, ta cũng có thể sử dụng các điện cực
so sánh khác như điện cực AgCl/Ag hay điện cực calomel Hg2Cl2/Hg.
[ox] và [kh]: nồng độ của dạng oxy hóa và dạng khử trong dung dịch với
số mũ bằng hệ số tỷ lượng tương ứng trong nửa phản ứng.
n: số mol electron trao đổi
F: hằng số Faraday (= 96.500 culong/mol)
T: nhiệt độ (K)
Nếu xét tại 298 K (25 ℃), phương trình trên chuyển thành:
0,059 [ox] (8.2)
φ = φo[ox]/[kh] + lg
n [kh]
Ví dụ, cho bán phản ứng: MnO4 + 8H + 5e → Mn2+ + 4H2O
- +

Khi đó thế điện cực của bán phản ứng trên tại 25 ℃ là:
0,059 [MnO4 ]. [H + ]8
φ = φo[MnO4]/[Mn2+] + lg
5 [Mn2+ ]
• Áp dụng cho phản ứng hóa học: a[ox]1 + b[kh]2 → c[ox]2 + d[kh]1
RT [ox]1a . [kh]b2
E = φcathode − φanode = E o + ln
nF [ox]c2 . [kh]1d
Trong đó,
E: sức điện động của pin Galvanic tại điều kiện đang xét.
Sức điện động của pin Galvanic: là sự chênh lệch điện thế giữa anode và
cathode khi pin hoạt động.
E o = φocathode − φoanode là chênh lệch thế điện cực (hiệu điện thế) giữa
cathode và anode tại điều kiện tiêu chuẩn.
[ox]1 và [kh]1, [ox]2 và [kh]2 là nồng độ của các dạng oxy hóa và dạng khử
liên hợp tại điều kiện phản ứng.
Lưu ý: Khi pin hoạt động thì sức điện động sẽ thay đổi theo thời gian.
Ví dụ. Cho pin Galvanic hoạt động dựa trên phản ứng:
Cu + 2Fe3+ → Cu2+ + 2Fe2+
Khi đó phương trình Nerst có dạng:
E = φAg+/Ag − φZn2+/Zn
RT [Fe3+ ]2
= φoFe3+/Fe2+ − φoCu2+ /Cu + ln
nF [Cu2+ ]1 . [Fe2+ ]2
Trong đó, thanh Fe3+/Fe2+ đóng vai trò cathode và Cu2+/Cu anode, nồng độ
của Cu được bỏ qua trong công thức trên do chúng ở trạng thái rắn.

185
8.2.5 Sức điện động của pin
Sức điện động của pin là giá trị (trị số tuyệt đối) của hiệu số điện thế lớn
nhất giữa hai điện cực của pin.
Ở điều kiện tiêu chuẩn:
E o = φocathode − φoanode (8.3)
Ở điều kiện bất kỳ:
E = φcathode − φanode (8.4)
8.2.6 Sức điện động của pin và hằng số cân bằng
Ở điều kiện tiêu chuẩn:
∆Go = −n. F. E o (8.5)
∆Go = −R. T. lnK cb (8.6)
RT 0,059 (8.7)
Eo = lnK cb , xét tại 298K → E o = lg K cb
nF n
Trong đó: Kcb là hằng số cân bằng của phản ứng xảy ra trong pin.
n: số electron trao đổi của phản ứng xảy ra trong pin
8.3. CHIỀU XẢY RA CỦA PHẢN ỨNG OXY HÓA KHỬ
Quy tắc: Phản ứng oxy hóa khử xảy ra theo chiều dạng oxy hóa của cặp
oxy hóa khử có thế điện cực lớn hơn sẽ oxy hóa dạng khử của cặp oxy hóa
khử có thế điện cực nhỏ hơn.
Nếu giá trị thế điện cực tiêu chuẩn của hai cặp oxy hóa khử tham gia phản
ứng có giá trị cách xa nhau thì sử dụng thế điện cực tiêu chuẩn để xét chiều
xảy ra của phản ứng.
Nếu giá trị thế điện cực tiêu chuẩn của hai cặp oxy hóa khử tham gia phản
ứng có giá trị gần nhau thì sử dụng giá trị thế điện cực ứng với điều kiện
thực tế để xét chiều xảy ra của phản ứng.

186
Phần 2. BÀI TẬP CÓ LỜI GIẢI
Dạng 1. Cân bằng phản ứng oxy hóa khử và xác định chất oxy hóa,
chất khử
Phương pháp:
Bước 1: Xác định số oxy hóa của từng nguyên tố
Bước 2: Xác định chất oxy hóa, chất khử
Bước 3: Thực hiện cân bằng số electron trao đổi trong quá trình oxy hóa
và quá trình khử và từ đó xác định hệ số tỷ lượng của phương trình phản
ứng
Câu 8.1. Cân bằng các phương trình phản ứng bên dưới và xác định chất
oxy hóa, chất khử của phản phản ứng sau:
a) H2O2 + KI + H2SO4 → K2SO4 + I2 + H2O
b) KMnO4 + KNO2 + H2SO4 → MnSO4 + KNO3 + K2SO4 + H2O
c) KClO3 + CrCl3 + KOH → K2CrO4 + KCl + H2O
Giải
a) H2O2 + KI + H2SO4 → K2SO4 + I2 + H2O
Sau khi xác định số oxy hóa của từng chất và chất oxy hóa, chất khử của
phản ứng trên ta thực hiện cân bằng số electron trao đổi:
1 × 2I − − 2e → I2
{
1 × 2O− + 2e → 2O−2
Kết quả: H2O2 + 2KI + H2SO4 → K2SO4 + I2 + H2O
Trong đó, H2O2 đóng vai trò chất oxy hóa, KI đóng vai trò chất khử
c) KMnO4 + KNO2 + H2SO4 → MnSO4 + KNO3 + K2SO4 + H2O
Sau khi xác định số oxy hóa của từng chất và chất oxy hóa, chất khử của
phản ứng trên ta thực hiện cân bằng số electron trao đổi:
2 × Mn+7 + 5e → Mn+2
{
5 × N+3 − 2e → N+5
Kết quả: 2KMnO4 + 5KNO2 + 3H2SO4 → 2MnSO4 + 5KNO3 + K2SO4 +
3H2O
Trong đó, KMnO4 đóng vai trò chất oxy hóa (nhận e), KNO2 đóng vai trò
chất khử (cho e)
c) KClO3 + CrCl3 + KOH → K2CrO4 + KCl + H2O
Sau khi xác định số oxy hóa của từng chất và chất oxy hóa, chất khử của
phản ứng trên ta thực hiện cân bằng số electron trao đổi:
1 × Cl+5 + 6e → Cl−
{
2 × Cr +3 − 3e → Cr +6

187
Kết quả: KClO3 + 2CrCl3 + 10KOH → 2K2CrO4 + 7KCl + 5H2O
Trong đó, KClO3 đóng vài trò chất oxy hóa, CrCl3 đóng vai trò chất khử
Dạng 2. Viết ký hiệu pin Galvanic dựa trên phương trình phản ứng
oxy hóa khử và xác định anode, cathode
Câu 8.2. Cho các phản ứng sau xảy ra trong một pin Galvanic. Hãy xác
định các nửa phản ứng xảy ra tại cathode, anode và viết ký hiệu của pin
Galvanic.
a) 2Cr + 3Fe2+ → 3Fe + 2Cr3+
b) H2 + 2AgCl → 2H+ + 2Ag + 2Cl-
Giải
a) 2Cr + 3Fe2+ → 3Fe + 2Cr3+
Anode (cực âm): Cr – 3e → Cr3+
Cathode (cực dương): Fe2+ + 2e → Fe
Ký hiệu pin: (-) Cr | Cr3+ || Fe2+ | Fe (+)
Khi pin hoạt động có dòng electron chạy từ thanh crôm (Cr) sang thanh sắt
(Fe) thông qua dây dẫn còn dòng điện theo chiều ngược lại.
b) H2 + 2AgCl → 2H+ + 2Ag + 2Cl-
Anode (cực âm): H2 – 2e → 2H+
Cathode (cực dương): AgCl + e → Ag + Cl-
Ký hiệu pin: (-) Pt| H2 | H+ || Cl- | AgCl, Ag (+)
Vì H2 ở thể khí nên được kết hợp điện cực trơ Pt làm anode. Còn tại cathode
AgCl là chất rắn khó tan sẽ được phủ lên bề mặt thanh Ag đóng vai trò
điện cực, Cl- đóng vai trò dung dịch điện ly. Khi pin hoạt động, dòng
electron chạy từ thanh bạch kim (Pt) sang thanh bạc (Ag) thông qua dây
dẫn còn dòng điện theo chiều ngược lại.
Câu 8.3. Cho các ký hiệu pin Galvanic. Xác định các nửa phản ứng xảy ra
tại anode, cathode, và viết phương trình phản ứng oxy hóa khử tương ứng.
a) (-) Zn | Zn2+ || Pb2+ | Pb (+)
b) (-) Cu | CuSO4 || Fe3+, Fe2+ | Pt (+)
c) (-) Fe | Fe2+ || MnO4−, Mn2+, H+ | Pt (+)
Giải
a) (-) Zn | Zn2+ || Pb2+ | Pb (+)
Anode (cực âm): Zn – 2e → Zn2+
Cathode (cực dương): Pb2+ + 2e → Pb
Phản ứng tổng quát: Zn + Pb2+ → Zn2+ + Pb

188
Khi pin hoạt động, có dòng electron chuyển từ cực kẽm sang cực chì thông
qua dây dẫn còn dòng điện theo chiều ngược lại.
b) (-) Cu | CuSO4 || Fe3+, Fe2+ | Pt (+)
Anode (cực âm): Cu – 2e → Cu2+
Cathode (cực dương): 2Fe3+ + 2e → 2Fe2+
Phản ứng tổng quát: Cu + 2Fe3+ → Cu2+ + 2Fe2+
Khi pin hoạt động, có dòng electron chuyển từ cực đồng (Cu) sang cực
bạch kim (Pt) thông qua dây dẫn còn dòng điện theo chiều ngược lại. Lưu
ý, đối với pin này do cặp oxy hóa/khử liên hợp tại cathode Fe3+/Fe2+ đều ở
dạng chất tan trong dung dịch nên thanh Pt (kim loại trơ) được kết hợp làm
điện cực.
c) (-) Fe | Fe2+ || MnO4−, Mn2+, H+ | Pt (+)
Anode (cực âm): Fe – 2e → Fe2+
Cathode (cực dương): MnO4- + 8H+ + 5e → Mn2+ + 4H2O
Phản ứng tổng quát: 5Fe + 2MnO4- + 16H+ → 5Fe2+ + 2Mn2+ + 8H2O
Khi pin hoạt động, có dòng electron chuyển từ sắt (Fe) sang cực bạch kim
(Pt) thông qua dây dẫn còn dòng điện theo chiều ngược lại. Tại cathode,
phản ứng diễn ra trong môi trường axit nên có sự hiện diện của ion H+.
Tương tự, thanh Pt được dùng làm điện cực do MnO4− và Mn2+ đều ở dạng
chất tan trong dung dịch.
Dạng 3. So sánh tính oxy hóa, khử và xác định chiều phản ứng.
Phương pháp: Thế điện cực càng lớn (o càng dương) thì dạng oxy hóa
càng mạnh và dạng khử liên hợp của nó càng yếu và ngược lại. Phản ứng
luôn diễn ra theo chiều dạng oxy mạnh tác dụng với dạng khử mạnh sinh
ra dạng oxy hóa yếu và dạng khử yếu hơn.
Câu 8.4. Cho thế điện cực tiêu chuẩn của các cặp oxy hóa/khử liên hợp
sau:
o(Fe3+/Fe2+) = 0,77 V
o(I2/I-) = 0,54 V
o(Ni2+/Ni) = - 0,23 V
o(Zn2+/Zn) = - 0,76 V
o(Br2/Br-) = 1,09 V
a) Sắp xếp theo tính oxy hóa tăng dần?
b) Sắp xếp theo tính khử tăng dần?
c) Chất nào có thể oxy hóa Ni?
Giải
189
a) o càng dương thì dạng oxy hóa càng mạnh, xu hướng nhận thêm e càng
mạnh. Do đó, ta sắp xếp dạng oxy hóa theo chiều tăng dần của o:
Zn2+ < Ni2+ < I2 < Fe3+ < Br2
b) o càng nhỏ (càng âm) thì dạng khử càng mạnh, xu hướng cho đi e càng
mạnh. Do đó, ta sắp xếp dạng khử theo chiều giảm dần của o:
Br− < Fe2+ < I- < Ni < Zn
c) Ni đóng vai trò chất khử với thế o(Ni2+/Ni) = -0,23 V, như vậy ta cần
tìm chất đóng vai trò chất oxy hóa với giá trị o lớn hơn. Vậy, ta có Fe3+,
I2, Br2 đáp ứng điều kiện này.
Dạng 4. Tính sức điện động của pin, hằng số cân bằng và biến thiên
thế đẳng áp tiêu chuẩn của phản ứng xảy ra trong pin.
Câu 8.5. Cho pin sau: (-) Zn | Zn2+ (1M) || Cu2+ (1M) | Cu (+)
Biết rằng o(Cu2+/Cu) = 0,337 V; o(Zn2+/Zn) = -0,763 V
a) Viết phương trình phản ứng oxy hóa khử của phản ứng xảy ra khi pin
hoạt động.
b) Tính hằng số cân bằng và biến thiên thế đẳng áp tiêu chuẩn của phản
ứng trong pin.
Giải
a) Anode (cực âm): Zn – 2e → Zn2+
Cathode (cực dương): Cu2+ + 2e → Cu
Phản ứng tổng quát: Zn + Cu2+ → Zn2+ + Cu
b) Xét tại điều kiện tiêu chuẩn:
E o = o(Cu2+/Cu) - o(Zn2+/Zn) = 0,337 - (-0,763) = 1,1 V
∆Go = −n. F. E o = −2 × 96500 × 1,1 = −212300 J = −212,3 kJ
∆Go = −R. T. lnK cb
↔ −212399 = −8,314 × 298 × lnK cb → Kcb = 1,704 × 1037
Câu 8.6. Ở 25°C, cho pin Galvanic gồm điện cực Cu nhúng trong dung
dịch CuSO4 0,5M (điện cực 1) và điện cực Pt nhúng vào dung dịch chứa
ion Fe3+, Fe2+, (điện cực 2), cho biết [Fe3+]= 2[Fe2+].
a) Viết sơ đồ pin và phản ứng xảy ra trong pin khi pin hoạt động.
b) Tính sức điện động của pin.
c) Biết rằng lượng CuSO4 sử dụng lớn hơn rất nhiều so với lượng Fe3+ và
Fe2+ trong dung dịch nên nồng độ của Cu2+ xem như không đổi trong suốt
[𝐹𝑒 3+ ]
quá trình phản ứng, tìm tỷ số khi pin ngừng hoạt động. Cho biết
[𝐹𝑒 2+ ]
φo Cu2+ / Cu = 0,34V và φo Fe3+ / Fe2+ = 0,77V
190
Giải
a) Cu2+ + 2e ⇄ Cu
0,059
φሺ1ሻ = φo Cu2+/ Cu + lg[Cu2+ ]
2
0,059
= 0, 34 + lg 0,5 = 0,331 ሺVሻ
2
Fe3+ + e ⇄ Fe2+
0,059 [Fe3+ ]
φሺ2ሻ = φo Fe3+/ Fe2+ + lg ( 2+ )
1 [Fe ]
0,059
= 0,77 + lg 2 = 0,788 ሺVሻ
1
Ta có: φሺ2ሻ > φሺ1ሻ ⇒ Cu đóng vai trò anode (cực âm), Pt đóng vai trò
cathode (cực dương)
Ký hiệu pin: (-) Cu | CuSO4 || Fe3+, Fe2+ | Pt (+)
Cực (-) : Cu – 2e → Cu2+
Cực (+) : 2Fe3+ + 2e → 2Fe2+
Phản ứng trong pin: Cu + 2Fe3+ → Cu2+ + 2Fe2+
b) Sức điện động của pin:
E = φcathode − φanode = φሺ2ሻ − φሺ1ሻ = 0,788 – 0,331 = 0,457 (V)
c) Pin ngừng hoạt động khi E = 0 hay φcathode = φanode
0,059 [Fe3+ ] Fe3+
→ 0,77 + lg ([Fe2+ ]) = 0,331 ⟹ Fe2+ = 4,8.10-8
1
Câu 8.7. Cho pin Galvanic gồm 2 nửa pin sau: Zn | Zn(NO3)2 (0,1M) và
Ag | AgNO3 (0,1M). Cho biết φo Zn2+/ Zn= -0,76 V và φo Ag+/ Ag = 0,8 V
a) Thiết lập sơ đồ pin.
b) Viết phương trình phản ứng khi pin làm việc.
c) Tính sức điện động của pin ở 25 ℃.
d) Tính nồng độ Zn2+ và Ag+ khi pin không có khả năng phát điện (pin
đã dùng hết).
Giải
a) Tính thế điện cực:
0,059 0,059
φZn2+/Zn = φo Zn2+/Zn + lg[Zn2+ ] = −0,76 + lg 0,1 = −
2 2
0,785 V
0,059 0,059
φAg+/Ag = φo Ag+/Ag + lg[Ag + ] = 0,8 + lg 0,1 = −0,859 V
1 1

191
Vì φZn2+/Zn < φAg+/Ag nên Zn đóng vai trò anode, Ag đóng vai trò
cathode. Từ đó, ta có ký hiệu pin: (-) Zn | Zn(NO3)2 (0,1M) || AgNO3
(0,1M) | Ag (+)
b) Khi pin làm việc:
Anode (cực âm) có sự oxy hóa Zn : Zn – 2e → Zn2+
Cathode (cực dương) có sự khử Ag+: Ag+ + e → Ag
Phản ứng tổng quát khi pin làm việc: Zn + 2Ag+ → Zn2+ + 2Ag
c) Sức điện động của pin:
Epin = φAg+/Ag − φZn2+/Zn ≈ 1,53V
d) Khi pin dừng hoạt động: Epin = 0
Gọi x (M) là nồng độ của ion Ag+ giảm đi trong phản ứng khi pin dừng
hoạt động. Ta có:
Zn + 2Ag+ → Zn2+ + 2Ag
Ban đầu: - 0,1 0,1 -
Phản ứng: - x x/2 -
Cân bằng: - 0,1 – x 0,1 + x/2 -
Epin = φAg+/Ag − φZn2+/Zn
0,059 0,059
= (φo Ag+/Ag + lg[Ag + ]) − (φo Zn2+ /Zn + lg[Zn2+ ])
1 2
2 2
0,059 ([Ag]+ ) 0,059 ([Ag]+ )
= (φo Ag+ − φ0 Zn2+ ) + lg [Zn2+ ]
= 1,56 + lg [Zn2+ ]
2 2
Ag Zn
2
0,059 ([Ag]+ )
Epin = 0 ⇒ lg [Zn2+ ]
= −1,56
2
0,059 ሺ0,1−xሻ2
→ lg x = −1,56
2 0,1+
2
ሺ0,1−xሻ2
→ x ≈ 0,1M ⟹ x ≈ 10−51,86 ≈ 0 (*)
0,1+
2
Vậy nồng độ Zn2+ khi pin dừng hoạt động là:
x
[Zn2+ ] = 0,1 + ≈ 0,15M
2
Xét phương trình (*), nồng độ Ag+ khi pin dừng hoạt động là:
x
[Ag + ] = 0,1 − x = √(0,1 + ) . 10−51,86 ≈ 4,55. 10−27 M
2

192
Phần 3. BÀI TẬP TRẮC NGHIỆM
Câu 8.1. Cho phản ứng oxy hóa khử:
K2Cr2O7 + FeSO4 + H2SO4 → Cr2(SO4)3 + Fe2(SO4)3 + K2SO4 + H2O
Cân bằng phản ứng trên. Nếu hệ số trước K2Cr2O7 là 1 thì hệ số đứng
trước H2SO4 và Fe2(SO4)3 lần lượt là:
A. 7,3 B. 7,6 C. 5,3 D. 4,5
Câu 8.2. Chọn câu đúng:
Trong phản ứng: 3Cl2 + I- + 6OH- → 6Cl- + IO3 + 3H2O
A. Chất oxy hóa là Cl2, chất bị oxy hóa là I-
B. Chất khử là Cl2, chất oxy hóa là I -
C. Chất bị oxy hóa là Cl2, chất bị khử là I-
D. Cl2 bị khử, I- là chất oxy hóa
Câu 8.3. Cho các số liệu sau:
𝜑 𝑜 (Ca2+/Ca) = - 2,79 V
𝜑 𝑜 (Zn2+/Zn) = - 0,764 V
𝜑 𝑜 (Fe2+/Fe) = - 0,437 V
𝜑 𝑜 (Fe3+ /Fe2+) = + 0,771 V
Các chất được sắp xếp theo thứ tự tính oxy hóa tăng dần như sau:
A. Fe3+ < Fe2+ < Zn2+ < Ca2+
B. Ca2+ < Zn2+ < Fe2+ < Fe3+
C. Zn2+ < Fe3+ < Ca2+ < Fe2+
D. Ca2+ < Zn2+ < Fe3+ < Fe2+
Câu 8.4. Tại 25 ºC, thế của điện cực đồng thay đổi như thế nào khi pha
loãng dung dịch muối Cu2+ của điện cực xuống 10 lần:
A. Giảm 29,5 mV B. Giảm 59 mV
C. Tăng 29,5 mV D. Không thay đổi
Câu 8.5. Một điện cực Ag nhúng vào dung dịch AgNO3, thế của điện cực
này sẽ thay đổi như thế nào khi:
1) Thêm HCl (có kết tủa AgCl)
2) Thêm NaOH (có kết tủa Ag2O)
3) Thêm nước (pha loãng)
A. Tăng cho cả 3 trường hợp.
B. Giảm cho cả 3 trường hợp.
C. Không thay đổi cho cả 3 trường hợp.
D. Chỉ giảm cho 2 trường hợp đầu.
Câu 8.6. Tính hằng số cân bằng của phản ứng bên dưới tại 25 ℃
193
3 Mg2+ + 2 Al → 3 Mg + 2 Al3+
A. ~10-72 B. ~1069
C. ~1023 D. ~10-24
Câu 8.7. Chọn cách viết đúng:
Xác định sơ đồ các pin hoạt động dựa trên cơ sở phản ứng oxy hóa khử:
Sn + Pb(NO3)2 ⇌ Sn(NO3)2 + Pb
A. (-) Sn | Sn(NO)2 || Pb(NO3)2 | Pb (+)
B. (-) Pb | Pb(NO3)2 || Sn(NO3)2 | Sn (+)
C. (-) Sn | Sn(NO3)2|| Pb(NO3)2 | Pb (+)
D. (-) Sn(NO3)2 | Sn || Pb | Pb(NO3)2 (+)
Câu 8.8. Đối với điện cực hydro (Hình 8.2) khi thay đổi nồng độ H+ thì
tính oxy hóa của điện cực thay đổi. Vậy khi giảm nồng độ H+ và giữ
nguyên áp suất khí H2 thì:
A. Tính oxy hóa của H+ tăng do 𝜑 tăng.
B. Tính oxy hóa của H+ tăng do 𝜑 giảm.
C. Tính khử của H2 tăng do φ giảm.
D. Tính khử của H2 tăng do 𝜑 tăng.

Hình 8.2. Điện cực Hydro


Câu 8.9. Trong các phát biểu sau, phát biểu sai là:
A. Kim loại làm điện cực có thế điện cực càng dương thì càng có tính oxy
hóa.
B. Kim loại làm điện cực có thế điện cực càng dương thì càng có tính khử
mạnh.
C. Sức điện động của pin phụ thuộc vào nồng độ chất oxy hóa và chất khử.
D. Sức điện động của pin phụ thuộc vào nhiệt độ pin.
Câu 8.10. Chọn phát biểu sai. Cho pin Galvanic gồm điện cực hydro tiêu
chuẩn (điện cực 1) nối với một điện cực tạo thành bằng cách nhúng điện
cực H2 (Pt) vào trong dung dịch HCl 0,1M (điện cực 2). Ở nhiệt độ nhất
định pin này có:
A. Thế điện cực của điện cực (2) giảm khi nồng độ của dung dịch HCl
giảm
B. Sức điện động của pin giảm khi pha loãng dung dịch ở điện cực (2)
C. Điện cực (1) làm điện cực dương
D. Quá trình oxy hóa xảy ra trên điện cực (2)

194
Câu 8.11. Chọn trường hợp đúng:
Cho quá trình điện cực: MnO4- + 8H+ + 5e ⇌ Mn2+ +4H2O
Phương trình Nerst đối với quá trình đã cho ở 25 oC có dạng:
0,059 [MnO− + 8
4 ] .[H ]
A. φ = φo + . lg( )
5 [Mn2+ ]
[MnO− + 8
4 ] .[H ]
B. φ = φo + 0,059. lg( )
[Mn2+ ]
0,059 [Mn2+ ]
C. φ = φo + . lg([MnO−] .[H+]8)
5 4
0,059 [MnO− + 8
4 ] .[H ]
D. φ = φo + . lg([Mn2+ ][H 4
)
5 2 O]
Câu 8.12. Chọn cách viết đúng:
Sơ đồ các pin hoạt động trên cơ sở các phản ứng oxy hóa khử :
HCl (dd) + Zn(r) ⇌ ZnCl2 (dd) + H2(k) là:
A. (-) H2(Pt) | HCl || ZnCl2 | Zn (+)
B. (-) Zn | ZnCl2 || HCl | H2(Pt) (+)
C. (-) H2(Pt) | HCl || Zn | ZnCl2 (+)
D. (-) ZnCl2| Zn || HCl | H2(Pt) (+)
Câu 8.13. Chọn đáp án đúng.
Cho pin Galvanic tạo bởi điện cực (1) (gồm một thanh Ag nhúng trong
dung dịch AgNO3 (0,001M) và điện cực (2) (gồm thanh Ag nhúng trong
dung dịch AgNO3 0,1M). Đối với nguyên tố này:
A. Quá trình khử xảy ra trên cực (1).
B. Cực (1) là cưc dương.
C. Điện cực (2) bị tan ra.
D. Ở mạch ngoài electron chuyển từ điện cực (1) sang điện cực (2).
Câu 8.14. Cho biết thế oxy hóa khử của Zn2+/Zn và Ag+/Ag lần lượt bằng
–0,763V và 0,799V. Pin (-) Zn | Zn2+ (1M) || Ag+ (1M) | Ag (+) có sức điện
động thay đổi như thế nào khi tăng nồng độ Zn2+ và Ag+ lên 2 lần
A. Không đổi B. Tăng lên
C. Giảm xuống D. Không xác định được
Câu 8.15. Chọn phát biểu đúng.
A. Pin Galvanic chuyển hóa năng lượng của phản ứng oxy hóa khử có ∆G
> 0 thành điện năng.
B. Khi pin Galvanic hoạt động, sức điện động của pin giữ nguyên theo thời
gian.
C. Sức điện động của một pin Galvanic tại điều kiện bất kỳ luôn nhỏ hơn
sức điện động của nó tại điều kiện tiêu chuẩn.
195
D. Sức điện động của pin Galvanic phụ thuộc nồng độ muối của kim loại
làm điện cực và nhiệt độ phản ứng.
Câu 8.16. Cho dãy hoạt động các cặp oxy hóa – khử sắp theo thứ tự 𝜑 𝑜
tăng dần, ta có thứ tự sau: Zn2+/Zn < 2H+/H2 < Cu2+/Cu < Ag+/Ag
Có tổng cộng bao nhiêu phản ứng trong các phản ứng sau có thể xảy ra
tự phát:
(i) Zn + 2H+ → Zn2+ + H2
(ii) Cu + 2H+ → Cu2+ + H2
(iii) Zn + 2Ag+ → 2Ag + Zn2+
A. 0 B. 1 C. 3 D. 2
Câu 8.17. Cho các thế khử tiêu chuẩn:
Fe3+ + e ⇌ Fe2+ φo = +0,77V
Ti4+ + e ⇌ Ti3+ φo = - 0,01 V
Ce4+ + e ⇌ Ce3+ φo = + 1,14 V
1) Fe3+ + Ti3+ ⇌ Fe2+ + Ti4+
2) Ce4+ + Ti3+ ⇌ Ce3+ + Ti4+
3) Ce3+ + Fe3+ ⇌ Ce4+ + Fe2+
Phản ứng có thể xảy ra tự phát là:
A. 1 B. 1 & 2
C. 2 D. 1, 2 & 3
Câu 8.18. Chọn đáp án đúng nhất.
Thế khử tiêu chuẩn của các cặp Br2/2Br – là 1,07 V; Fe3+/Fe2+ là 0,77V;
Cu2+/Cu là 0,34V; MnO4-/Mn2+ là 1,52V; Sn4+/Sn2+ là 0,15V. Brom Br2 có
thể oxy hóa được:
A. Fe2+ thành Fe3+
B. Fe2+ thành Fe3+ và Sn2+ thành Sn4+
C. Sn2+ thành Sn4+
D. Fe2+ thành Fe3+ , Sn2+ thành Sn4+ và Cu thành Cu2+
Câu 8.19. Cho hai pin có ký hiệu và sức điện động tương ứng:
(-) Zn | Zn2+ || Pb2+ | Pb (+) E1 = 0,63V
2+ 2+
(-) Pb | Pb || Cu | Cu (+) E2 = 0,47V
Vậy sức điện động của pin (-) Zn | Zn || Cu2+ | Cu (+) sẽ là:
2+

A. –1,1V B. 1,1V
C. 0,16V D. –0,16V
Câu 8.20. Chọn đáp án đúng:
Cho thế khử tiêu chuẩn của các bán phản ứng sau:
196
Fe3+ + e = Fe2+ φo = 0,77 V
I2 + 2e = 2I- φo = 0,54 V
Phản ứng: 2Fe2+ + I2 → 2Fe3+ + 2I- có đặc điểm:
A. Eo = -0,23 V; phản ứng không thể xảy ra tự phát.
B. Eo = -1,00 V; phản ứng không thể xảy ra tự phát.
C. Eo = 1,00 V; phản ứng có thể xảy ra tự phát.
D. Eo = 0,23 V; phản ứng có thể xảy ra tự phát.
Câu 8.21. Biết sức điện động của hai pin Galvanic sau đây ở điều kiện
tiêu chuẩn:
(-) Zn (r) | Zn2+ (dd) || Pb2+ (dd) | Pb (r) (+) Eo = 0,637V
(-) Pb (r) | Pb2+ (dd) || Ag+ (dd) | Ag (r) (+) Eo = 0,925V
Trong các giá trị dưới đây, giá trị nào ứng với sức điện động của pin
Galvanic sau ở điều kiện tiêu chuẩn: (-) Zn (r) | Zn2+ (dd) || Ag+ (dd) | Ag
(r) (+) Eo = ?
A. 1,562V B. -1,562V
C. -0,288V D. 0,288V
Câu 8.22. Trong các chất bên dưới, chất nào có tính oxy hóa mạnh nhất?
A. Pb2+ B. Ag+
C. I2 D. Pb
Câu 8.23. Cho pin Galvanic bên dưới:
Mg | Mg 2+ || Zn2+ | Zn, E˚ = 1,61 V
Biết φo (Zn2+/Zn) = – 0,76 V, tính φo (Mg2+/Mg)
A. –2,37 V B. +0,85 V
C. +2,37 V D. –0,85 V
o
Câu 8.24. Cho φ (Sn /Sn ) = 0,15 V. Xác định giá trị của tỷ lệ
4+ 2+

[Sn4+]/ [Sn2+] để thế của điện cực này bằng 0,169 V.


A. 4,41 B. 2,00 C. 2,49 D. 3,5
Câu 8.25. Nhúng một thanh Zn vào dung dịch CuBr2 thì có phản ứng xảy
ra, trong khi nhúng thanh Cu vào dung dịch ZnBr2 thì không thấy phản
ứng xảy ra, phát biểu nào sau đây sai?
Zn + CuBr2 → Cu + ZnBr2
Cu + ZnBr2 → không có phản ứng
A. Zn có tính khử mạnh hơn Cu
B. Cu+2 có tính oxy hóa mạnh hơn Zn2+
C. Cu là chất khử mạnh hơn Zn
D. Cu2+ dễ bị khử hơn Zn2+
197
Câu 8.26. Cho pin Galvanic bên dưới, tính biến thiên thế đẳng áp tiêu
chuẩn ΔG° ở 25 ℃của phản ứng biết sức điện động của pin: Mg | Mg2+ ||
Zn2+ | Zn E˚ = +1,61 V
A. - 311 kJ B. +311 kJ
C. - 155 kJ D. +155 kJ
Câu 8.27. Tính hằng số cân bằng K của phản ứng dưới đây, biết sức điện
động tiêu chuẩn của pin tạo thành từ phản ứng này ở 25 ℃ là 0,0218 V.
3Zn(s) + 2Cr3+(aq) → 3Zn+2(aq) + Cr(s) Eopin = +0,0218 V
A. 1,3.10-3 B. 12,7
C. 165 D. 5,46
Câu 8.28. Cho pin Galvanic: Pb | Pb(NO3)2 (1,0 M) || AgNO3 (1,0 M) | Ag
Xác định nửa phản ứng xảy ra ở cực âm.
A. Pb2+ + 2 e- → Pb B. Pb → Pb2+ + 2 e-
C. Ag+ + e- → Ag D. Ag → Ag+ + e-
Câu 8.29. Cho sơ đồ pin sau: (-) Fe| Fe2+ (1M) || Cu2+ (1M) | Cu (+)
Tính E0 và xác định cathode khi pin Galvanic hoạt động.
A. Fe, -0.41 V B. Cu, 0,57 V
C. Cu, -0,25 V D. Fe, -0,57 V
Câu 8.30. Chọn phát biểu đúng. Vai trò của cầu muối trong pin Galvanic là:
A. Giúp vận chuyển electron từ cực này qua cực khác
B. Giúp trộn lẫn 2 dung dịch ở các điện cực
C. Đảm bảo sự cân bằng điện tích trong pin
D. Ngăn cản quá trình ion di chuyển giữa các dung dịch
Câu 8.31. Trong ký hiệu pin điện Galvanic, ký hiệu || thể hiện:
A. Sự phân chia các pha B. Điện cực khí
C. Dây dẫn điện D. Cầu muối
Câu 8.32. Chọn phát biểu đúng cho pin Galvanic:
(I) Quá trình khử xảy ra ở cathode.
(II) Khối lượng anode sẽ tăng dần khi pin hoạt động
(III) Sức điện động của pin nhỏ hơn hoặc bằng 0.
A. Chỉ phát biểu III B. Chỉ phát biểu II
C. Chỉ phát biểu I D. II và III
Câu 8.33. Trong một pin Galvanic, electron sẽ di chuyển theo chiều:
A. Từ anode sang cathode trong dung dịch
B. Từ cathode sang anode trong dung dịch
C. Từ anode sang cathode thông qua dây dẫn bên ngoài

198
D. Từ cathode sang anode thông qua dây dẫn bên ngoài
Câu 8.34. Cho pin Galvanic sau: (-) M | M+ || X+ | X (+)
Trong đó φo (M+/M) = 0,44V và φo (X+/X) = 0,63V.
Phát biểu nào sau đây là đúng:
A. M + X → M+ + X+ là phản ứng tự xảy ra
B. M + X+ → M+ + X là phản ứng tự xảy ra
C. Epin = 1,07 V
D. Epin = -1,07 V
Câu 8.35. Thế điện cực của một điện cực hydro với pH = 10 tại 25 ℃ và
áp suất P(H2) = 1 atm là?
A. 0,59 V B. -0,59 V
C. 0,059 V D. -0,059 V
Câu 8.36. Cho pin hoạt động dựa trên phản ứng oxy hóa khử của Ag+ và
Cu có sức điện động +0,46 V tại 25ºC, tính ΔG° của phản ứng 2Ag+ + Cu
→ 2Ag + Cu2+ xảy ra trong pin trên?
A. -44,5 kJ B. -98 kJ
C. -89 kJ D. -89 J
Câu 8.37. Tính sức điện động của pin Galvanic được tạo thành bằng cách
nhúng điện cực đồng vào hai dung dịch Cu2+ 0,002M và 1M.
A. 0,02 V B. 0,08 V
C. 0,03 V D. 1,00 V
Câu 8.38. Cho pin Galvanic dựa trên phản ứng 2Ag+ + Cu → Cu2+ + 2Ag.
Tính hằng số cân bằng của phản ứng trong pin trên tại 25 ℃?
A. 3,92.1015 B. 3,92.10-15
C. -3,92.1015 D. Không thể xác định được
Câu 8.39. Tính thế điện cực ứng với bán phản ứng sau:
MnO4- (0,010 M) + 8H+ (0,20 M) + 5e- → Mn2+ (0,020 M) + 4H2O
A. 1,50 V B. 1,86 V
C. 1,44 V D. 1,58 V
Câu 8.40. Tính thế điện cực của bán phản ứng Al3+ + 3e- → Al ở 25 ℃.
Biết [Al3+] = 0,10 M và φoAl3+/Al = −1,66 V.
A. -1,84 V B. -1,60 V
C. -1,68 V D. -1,66 V
---oOo---

199
ĐÁP ÁN BÀI TẬP CHƯƠNG 8
8.1- 8.2- 8.3- 8.4- 8.5- 8.6- 8.7- 8.8- 8.9- 8.10-
A A B A B A C C B B
8.11- 8.12- 8.13- 8.14- 8.15- 8.16- 8.17- 8.18- 8.19- 8.20-
A B D B D D B D B A
8.21- 8.22- 8.23- 8.24- 8.25- 8.26- 8.27- 8.28- 8.29- 8.30-
A B A A C A C B B C
8.31- 8.32- 8.33- 8.34- 8.35- 8.36- 8.37- 8.38- 8.39- 8.40-
D C C B B C B A C C

200
TÀI LIỆU THAM KHẢO
Tiếng Việt

1. Nguyễn Đình Soa (2017), Hóa Đại cương, NXB ĐH Quốc gia
TP. Hồ Chí Minh.
2. Nguyễn Đức Chung (2014), Hóa Đại cương, NXB ĐH Quốc gia
TP. Hồ Chí Minh.
3. Nguyễn Đức Chung (2015), Bài tập Hóa Đại cương, NXB ĐH
Quốc gia TP. Hồ Chí Minh.
Tiếng Anh

4. Thomas R. Gilbert, Rein V. Kirss, Natalie Foster, Geoffrey


Davies (2014), Chemistry an atoms-focused approach, W. W.
Norton & Company New York, London.
5. John W. Hill, Ralph H. Petrucci, Terry W. McCreary, Scott S.
Perry (2005), General Chemistry, Pearson Education
International.

201
Phụ lục 1. MỘT VÀI ĐƠN VỊ ĐO HỆ SI
---oOo---

Đại lượng Tên gọi Ký hiệu


Độ dài mét m
Khối lượng kilogam kg
Thời gian giây s
Nhiệt độ kevin K
Lượng chất mol mol
Diện tích mét vuông m2
Thể tích mét khối m3
Tần số dao động hec Hz
Khối lượng riêng kilogam trên mét khối kg/m3
Thể tích riêng mét khối trên kilogam m3/kg
Lực niuton N
Áp suất pascal Pa
Công, năng lượng jun J
Công suất wat W
Điện tích culong C
Moment lưỡng cực culong-met C.m
Điện thế vôn V
Điện trở ôm Ω
Nhiệt dung Jun trên kevin J.K-1
Nhiệt dung riêng Jun trên kilogam độ K J.kg-1.K-1

202
Phụ lục 2. MỘT VÀI HẰNG SỐ VẬT LÝ QUAN TRỌNG
---oOo---
Tên đại lượng Ký Giá trị
hiệu
Tốc độ ánh sáng trong chân C 2,99792458.108 m.s-1
không
Hằng số Plank h 6,626076.10-34 J.s
Không độ tuyệt đối T - 273,15 oC
Điện tích electron e 1,602177.10-19 C
Đơn vị khối lượng nguyên tử u 1,660540.10-27 kg
Khối lượng nghỉ của electron me 9,109390.10-31 kg
Khối lượng nghỉ của proton Mp 1,672623.10-27 kg
Khối lượng nghỉ của neutron Mn 1,674929.10-27 kg
Hằng số điện môi của chân không εo 8,854188.10-12 C2.J-1.m-1
Hằng số Avogadro No 6,0221271.1023 mol-1
Hằng số Faraday F 9,648431.104 C.mol-1
Hằng số khí lý tưởng R 8,31451 J.mol-1.K-1
1,98722 cal.mol-1.K-1
0,082058 l.atm. mol-1.K-1

203
Phụ lục 3. CÁC TÍNH CHẤT NHIỆT ĐỘNG CỦA MỘT
SỐ CHẤT Ở 25 oC
---oOo---
Khối
ΔHof So ΔGof
Chất lượng mol
(kJ/mol) [J/(mol.K)] (kJ/mol)
(g/mol)
NGUYÊN TỬ VÀ ION
Ag+ (aq) 107.87 105.6 72.7 77.1
Ag (g) 107.87 284.9 173.0 246.0
Ag (s) 107.87 0.0 42.6 0.0
Al3+ (aq) 26.982 -531 -321.7 -485
Al (g) 26.982 330.0 164.6 289.4
Al (s) 26.982 0.0 28.3 0.0
Al (l) 26.982 10.6 39.6 -1.2
Ar (g) 39.948 0.0 154.8 0.0
Au (g) 196.97 366.1 180.5 326.3
Au (s) 196.97 0.0 47.4 0.0
B (g) 10.811 565.0 153.4 521.0
B (s) 10.811 0.0 5.9 0.0
Ba2+ (aq) 137.33 -537.6 9.6 -560.8
Ba (g) 137.33 180.0 170.2 146.0
Ba (s) 137.33 0.0 62.8 0.0
Be (g) 9.0122 324.0 136.3 286.6
Be (s) 9.0122 0.0 9.5 0.0
Br – (aq) 79.904 -121.6 82.4 -104.0
Br (g) 79.904 111.9 175.0 82.4
Br2 (g) 159.808 30.9 245.5 3.1
Br2 (l) 159.808 0.0 152.2 0.0
C (g) 12.011 716.7 158.1 671.3
C (s, diamond) 12.011 1.9 2.4 2.9
C (s, graphite) 12.011 0.0 5.7 0.0
Ca2+ (aq) 40.087 -542.8 -55.3 -553.6
Ca (g) 40.087 177.8 154.9 144.0
Ca (s) 40.087 0.0 41.6 0.0
Cl – (aq) 35.453 -167.2 56.5 -131.2
Cl (g) 35.453 121.3 165.2 105.3
Cl2 (g) 70.096 0.0 41.6 0.0
Co2+ (aq) 58.933 -58.2 -113 -54.4
Co3+ (aq) 58.933 92 -305 134
Co (g) 58.933 424.7 179.5 380.3
Co (s) 58.933 0.0 30.0 0.0

204
Khối
ΔHof So ΔGof
Chất lượng mol
(kJ/mol) [J/(mol.K)] (kJ/mol)
(g/mol)
Cr (g) 51.996 396.6 174.5 351.8
Cr (s) 51.996 0.0 30.0 0.0
Cs+ (aq) 132.91 -258.3 133.1 -292.0
Cs (g) 132.91 76.5 175.6 49.6
Cs (s) 132.91 0.0 85.2 0.0
Cu+ (aq) 63.546 71.7 40.6 50.0
Cu2+ (aq) 63.546 64.8 -99.6 65.5
Cu (g) 63.546 337.4 166.4 297.7
Cu (s) 63.546 0.0 33.2 0.0
F – (aq) 18.998 -332.6 -13.8 -278.8
F (g) 18.998 79.4 158.8 62.3
F2 (g) 37.997 0.0 202.8 0.0
Fe2+ (aq) 55.845 -89.1 -137.7 -78.9
Fe3+ (aq) 55.845 -48.5 -315.9 -4.7
Fe (g) 55.845 416.3 180.5 370.7
Fe (s) 55.845 0.0 27.3 0.0
H+ (aq) 1.0079 0.0 0.0 0.0
H (g) 1.0079 218.0 114.7 203.3
H2 (g) 2.0158 0.0 130.6 0.0
He (g) 4.0026 0.0 126.2 0.0
Hg22+ (aq) 401.18 172.4 84.5 153.5
Hg2+ (aq) 200.59 171.1 -32.2 164.4
Hg (g) 200.59 61.4 175.0 31.8
Hg (l) 200.59 0.0 126.2 0.0
I – (aq) 126.90 -55.2 111.3 -51.6
I (g) 126.90 106.8 180.8 70.2
I2 (g) 253.81 62.4 260.7 19.3
I2 (s) 253.81 0.0 116.1 0.0
K+ (aq) 39.098 -252.4 102.5 -283.3
K (g) 39.098 89.0 160.3 60.5
K (s) 39.089 0.0 64.7 0.0
Li+ (aq) 6.941 -278.5 13.4 -293.3
Li (g) 6.941 159.3 138.8 126.6
Li+ (g) 6.941 685.7 133.0 648.5
Li (s) 6.941 0.0 29.1 0.0
Mg2+(aq) 24.305 -466.9 -138.1 -454.8
Mg (g) 24.305 147.1 148.6 112.5
Mg (s) 24.305 0.0 32.7 0.0
Mn2+ (aq) 54.938 -220.8 -73.6 -228.1

205
Khối
ΔHof So ΔGof
Chất lượng mol
(kJ/mol) [J/(mol.K)] (kJ/mol)
(g/mol)
Mn (g) 54.938 280.7 173.7 238.5
Mn (s) 54.938 0.0 32.0 0.0
N (g) 14.007 472.7 153.3 455.5
N2 (g) 28.013 0.0 191.5 0.0
Na+ (aq) 22.990 -240.1 59.0 -261.9
Na (g) 22.990 107.5 153.7 77.0
Na+ (g) 22.990 609.3 148.0 574.3
Na (s) 22.990 0.0 51.3 0.0
Ne (g) 20.180 0.0 146.3 0.0
Ni2+ (aq) 58.693 -54.0 -128.9 -45.6
Ni (g) 58.693 429.7 182.2 384.5
Ni (s) 58.693 0.0 29.9 0.0
O (g) 15.999 249.2 161.1 231.7
O2 (g) 31.999 0.0 205.0 0.0
P (g) 30.974 314.6 163.1 278.3
P4 (s, red) 123.895 -17.6 22.8 -12.1
P4 (s, white) 123.895 0.0 41.1 0.0
Pb2+ (aq) 207.2 -1.7 10.5 -24.4
Pb (g) 207.2 195.2 162.2 175.4
Pb (s) 207.2 0.0 64.8 0.0
Rb+ (aq) 85.468 -251.2 121.5 -284.0
Rb (g) 85.468 80.9 170.1 53.1
Rb (s) 85.468 0.0 76.8 0.0
S (g) 32.065 277.2 167.8 236.7
S8 (g) 256.520 102.3 430.2 49.1
S8 (s) 256.520 0.0 32.1 0.0
Sc (g) 44.956 377.8 174.8 336.0
Si (g) 28.086 450.0 168.0 405.5
Si (s) 28.086 0.0 18.8 0.0
Sn (g) 118.71 301.2 168.5 266.2
Sn (s, gray) 118.71 -2.1 44.1 0.1
Sn (s, white) 118.71 0.0 51.2 0.0
Sr2+ (aq) 87.62 -545.8 -32.6 -559.5
Sr (g) 87.62 164.4 164.6 130.9
Sr (s) 87.62 0.0 52.3 0.0
Ti (g) 47.867 473.0 180.3 428.4
Ti (s) 47.867 0.0 30.7 0.0
V (g) 50.942 514.2 182.2 468.5
V (s) 50.942 0.0 28.9 0.0

206
Khối
ΔHof So ΔGof
Chất lượng mol
(kJ/mol) [J/(mol.K)] (kJ/mol)
(g/mol)
W (s) 183.84 0.0 32.6 0.0
Zn2+ (aq) 65.38 -153.9 -112.1 -147.1
Zn (g) 65.38 130.4 161.0 94.8
Zn (s) 65.38 0.0 41.6 0.0
CÁC HỢP CHẤT VÔ CƠ
AgCl (s) 143.32 -127.1 96.2 -109.8
AgI (s) 234.77 -61.8 115.5 -66.2
AgNO3 (s) 169.87 -124.4 140.9 -33.4
Al2O3 (s) 101.961 -1675.7 50.9 -1582.3
B2H6 (g) 27.669 35.0 232.0 86.6
B2O3 (s) 69.622 -1263.6 54.0 -1184.1
BaCO3 (s) 197.34 -1216.3 112.1 -1137.6
BaSO4 (s) 233.39 -1473.2 132.2 -1362.2
CaCO3 (s) 100.087 -1206.9 54.0 -1184.1
CaCl2 (s) 110.984 -795.4 108.4 -748.8
CaF2 (s) 78.075 -1228.0 68.5 -1175.6
CaO (s) 56.077 -634.9 38.1 -603.3
Ca(OH)2 (s) 74.093 -985.2 83.4 -897.5
CaSO4 (s) 136.142 -1434.5 106.5 -1322.0
CO (g) 28.010 -110.5 197.7 -137.2
CO2 (g) 44.010 -393.5 213.8 -394.4
CO2 (aq) 44.010 -412.9 121.3 -386.2
CS2 (g) 76.143 115.3 237.8 65.1
CS2 (l) 76.143 87.9 151.0 63.6
CsCl (s) 168.358 -443.0 101.2 -414.6
CuSO4 (s) 159.610 -771.4 109.2 -662.2
FeCl2 (s) 126.750 -341.8 118.0 -302.3
FeCl3 (s) 162.203 -399.5 142.3 -334.0
FeO (s) 71.844 -271.9 60.8 -255.2
Fe2O3 (s) 159.688 -824.2 87.4 -742.2
HBr (g) 80.912 -36.3 198.7 -53.4
HCl (g) 36.461 -92.3 186.9 -95.3
HF (g) 20.006 -273.3 173.8 -275.4
HI (g) 127.912 26.5 206.6 1.7
HNO3 (g) 63.013 -135.1 266.4 -74.7
HNO3 (l) 63.013 -174.1 155.6 -80.7
HNO3 (aq) 63.013 -206.6 146.0 -110.5
HgCl2 (s) 271.50 -224.3 146.0 -178.6

207
Khối
ΔHof So ΔGof
Chất lượng mol
(kJ/mol) [J/(mol.K)] (kJ/mol)
(g/mol)
Hg2Cl2 (s) 472.09 -265.4 191.6 -210.7
H2O (g) 18.015 -241.8 188.8 -228.6
H2O (l) 18.015 -285.8 69.9 -237.2
H2S (g) 34.082 -20.17 205.6 -33.01
H2O2 (g) 34.015 -136.3 232.7 -105.6
H2O2 (l) 34.015 -187.8 109.6 -120.4
H2SO4 (l) 98.079 -814.0 156.9 -690.0
H2SO4 (aq) 98.079 -909.2 20.1 -744.5
KBr (s) 119.002 -393.8 95.9 -380.7
KCl (s) 74.551 -436.5 82.6 -408.5
KHCO3 (s) 100.115 -963.2 115.5 -.863.6
K2CO3 (s) 138.205 -1151.0 155.5 -1063.5
LiBr (s) 86.845 -351.2 74.3 -342.0
LiCl (s) 42.394 -408.6 59.3 -384.4
Li2CO3 (s) 73.891 -1215.9 90.4 -1132.1
MgCl2 (s) 95.211 -641.3 89.6 591.8
Mg(OH)2 (s) 58.320 -924.5 63.2 -833.5
MgSO4 (s) 120.369 -1284.9 91.6 -1170.6
MnO2 (s) 86.937 -520.0 53.1 -465.1
NaCH3COO (s) 82.034 -708.8 123.0 -607.2
NaBr (s) 102.894 -361.1 86.82 -349.0
NaCl (s) 58.443 -411.2 72.1 -384.2
NaCl (g) 58.443 -181.4 229.8 -201.3
Na2CO3 (s) 105.989 -1130.7 135.0 -1044.4
NaHCO3 (s) 84.007 -950.8 101.7 -851.0
NaNO3 (s) 84.995 -467.9 116.5 -367.0
NaOH (s) 39.997 -425.6 64.5 -379.5
Na2SO4 (s) 142.043 -1387.1 149.6 -1270.2
NF3 (g) 71.002 -132.1 260.8 -90.6
NH3 (aq) 17.031 -80.3 111.3 -26.50
NH3 (g) 17.031 -46.1 192.5 -16.5
NH4Cl (s) 53.491 -314.4 94.6 -203.0
NH4NO3 (s) 80.043 -365.6 151.1 -183.9
N2H4 (g) 32.045 95.35 238.5 159.4
N2H4 (l) 32.045 50.63 121.52 149.3
NiCl2 (s) 129.60 -305.3 97.7 -259.0
NiO (s) 74.60 -239.7 38.0 -211.7
NO (g) 30.006 90.3 210.7 86.6
NO2 (g) 46.006 33.2 240.0 51.3

208
Khối
ΔHof So ΔGof
Chất lượng mol
(kJ/mol) [J/(mol.K)] (kJ/mol)
(g/mol)
N2O (g) 44.013 82.1 219.9 104.2
N2O4 (g) 92.011 9.2 304.2 97.8
NOCl (g) 65.459 51.7 261.7 66.1
O3 (g) 47.998 5.4 210.2 13.4
PCl3 (g) 137.33 -288.07 311.7 -269.6
PCl3 (l) 137.33 -319.6 217 -272.4
PF5 (g) 125.96 -1594.4 300.8 -1520.7
PH3 (g) 33.998 5.4 210.2 13.4
PbCl2 (s) 278.1 -359.4 136.0 -314.1
PbSO4 (s) 303.3 -920.0 148.5 -813.0
SO2 (g) 64.065 -296.8 248.2 -300.1
SO3 (g) 80.064 -395.7 256.8 -371.1
ZnCl2 (s) 136.30 -415.1 111.5 -369.4
ZnO (s) 81.37 -348.0 43.9 -318.2
ZnSO4 (s) 161.45 -982.8 110.5 871.5
CÁC HỢP CHẤT HỮU CƠ
CCl4 (g) 153.823 -102.9 309.7 -60.6
CCl4 (l) 153.823 -135.4 216.4 -65.3
CH4 (g) 16.043 -74.8 186.2 -50.8
CH3COOH (g) 60.053 -432.8 282.5 -374.5
CH3COOH (l) 60.053 -484.5 159.8 -389.9
CH3OH (g) 32.042 -200.7 239.9 -162.0
CH3OH (l) 32.042 -238.7 126.8 -166.4
C2H2 (g) 26.038 226.7 200.8 209.2
C2H4 (g) 28.054 52.4 219.5 68.1
C2H6 (g) 30.070 -84.67 229.5 -32.9
CH3CH2OH (g) 46.069 -235.1 282.6 -168.6
CH3CH2OH (l) 46.069 -277.7 160.7 -174.9
CH3CHO (g) 44.053 -166 266 -133.7
C3H8 (g) 44.097 -103.8 269.9 -23.5
n-CH3(CH2)2CH3 (g) 58.123 -125.6 310.0 -15.7
n-CH3(CH2)2CH3 (l) 58.123 -147.6 231.0 -15.0
CH3COCH3 (l) 58.079 -248.4 199.8
CH3COCH3 (g) 58.079 -217.1 295.3 -152.7
CH3(CH2)2CH2OH (l) 74.122 -327.3 225.8
(CH3CH2)2O (l) 74.122 -252.1 342.7
(CH3)2C=C(CH3)2 (l) 84.161 66.6 362.6 -69.2
(CH3)2NH (l) 45.084 -43.9 182.3

209
Khối
ΔHof So ΔGof
Chất lượng mol
(kJ/mol) [J/(mol.K)] (kJ/mol)
(g/mol)
(C2H5)2NH (l) 73.138 -103.3
(C2H5)2NH (g) 73.138 -71.4
(CH3)3N (l) 59.111 -46.0 208.5
(CH3)3N (g) 59.111 -23.6 287.1
(CH3CH2)2N (l) 101.191 -134.3
(CH3CH2)2N (g) 101.191 -95.8
C6H6 (g) 78.114 82.9 269.2 129.7
C6H6 (l) 78.114 49.0 172.9 124.5
C6H12O6 (s) 180.158 -1274.4 212.1 -901.1
n-C8H18 (l) 114.231 -249.9 361.1 6.4
n-C8H18 (g) 114.231 -208.6 466.7 16.4
C12H22O11 (s) 342.300 -2221.7 360.2 -1543.8
HCOOH (l) 46.026 -424.7 129.0 -361.4

210
Phụ lục 4. HẰNG SỐ ĐIỆN LY CỦA MỘT VÀI ACID Ở
25 oC
---oOo---
Acid Bậc Phương trình điện ly Ka pKa
Acetic 1 CH3COOH(aq) ⇌ 1,76.10−5 4,75
H+(aq) + CH3COO-(aq)
Arsenic 1 H3AsO4(aq) ⇌ H+(aq) + H2AsO4-(aq) 5,5.10−3 2,26
2 H2AsO4-(aq) ⇌ 1,7x10−7 6.77
H+(aq) + HAsO42-(aq)
3 H3AsO4(aq) ⇌ H+(aq) + H2AsO4-(aq) 5,1.10−12 11,29
Ascorbic 1 H2C6H6O6(aq) ⇌ 9,1.10-5 4,04
H+(aq) + HC6H6O6- (aq)
2 HC6H6O6- (aq) ⇌ 5.10-12 11,3
H+(aq) + C6H6O62- (aq)
Benzoic 1 C6H5COOH (aq) ⇌ C6H5COO- (aq) 6,25.10-5 4,20
Boric 1 H3BO3 (aq) ⇌ H+(aq) + H2BO3- (aq) 5,4.10-10 9,27
2 H2BO3- (aq) ⇌ H+(aq) + HBO32- (aq) < 10-14 > 14
Bromoacetic 1 CH2BrCOOH (aq) ⇌ 2,0.10-3 2,70
H+(aq) + CH2BrCOO-
Butanoic 1 CH3CH2CH2COOH (aq) ⇌ 1,5.10-5 4,82
H+(aq) + CH3CH2CH2COO- (aq)
Cacbonic 1 H2CO3 (aq) ⇌ H+(aq) + HCO3- (aq) 4,3.10-7 6,37
2 HCO3- (aq) ⇌ H+(aq) + HCO32- (aq) 4,7.10-11 10,33
Chloric 1 HClO3 (aq) ⇌ H+(aq) + ClO3- (aq) ~1 ~0
Chloroacetic 1 CH2ClCOOH (aq) ⇌ 1,4.10-3 2,85
H+(aq) + CH2ClCOO- (aq)
Chlorous 1 HClO2 (aq) ⇌ H+(aq) + ClO2- (aq) 1,1.10-2 1,96
Citric 1 HOC(CH2)2(COOH)3 (aq) ⇌ 7,4.10-4 3,13
H+(aq) + HOC(CH2)2(COOH)2COO-
(aq)
2 HOC(CH2)2(COOH)2COO- (aq) ⇌ 1,7.10-5 4,77
H+(aq) + HOC(CH2)2(COOH)(COO-
)2 (aq)
3 HOC(CH2)2(COOH)(COO-)2 (aq) ⇌ 4,0.10-7 6,40
H+(aq) + HOC(CH2)2(COO-)3 (aq)
Dichloroacetic 1 CHCl2COOH (aq) ⇌ 5,5.10-2 1,26
H+(aq) + CHCl2COO- (aq)
Ethanol 1 CH3CH2OH (aq) ⇌ 1,3.10-16 15,9
H+(aq) + CH3CH2O- (aq)
Fluoroacetic 1 CH2FCOOH (aq) ⇌ 2,6.10-3 2,59
H+(aq) + CH2FCOO- (aq)

211
Acid Bậc Phương trình điện ly Ka pKa
formic 1 HCOOH (aq) ⇌ 1,77.10-4 3,75
H+(aq) + HCOO- (aq)
Germanic 1 H2GeO3 (aq) ⇌ 9,8.10-10 9,01
H+(aq) + HGeO3- (aq)
2 HGeO3- (aq) ⇌ 5,0.10-13 12,3
H+(aq) + GeO32- (aq)
Hyd(o)azoic 1 HN3 (aq) ⇌ H+(aq) + N3- (aq) ⇌ 1,9.10-5 4,72
H+(aq)
+ -
Hydrobromic 1 HBr (aq) ⇌ H (aq) + Br (aq) >> 1 <0
(mạnh)
Hydrochloric 1 HCl (aq) ⇌ H+ (aq) + Cl - (aq) >> 1 <0
(mạnh)
Hydrocyanic 1 HCN (aq) ⇌ H+ (aq) + CN - (aq) 6,2.10-10 9,21
Hydrofluoric 1 HF (aq) ⇌ H+ (aq) + F - (aq) 6,8.10-4 3,17
Hydr(o)iodic 1 HI (aq) ⇌ H+ (aq) + I - (aq) >> 1 <0
(mạnh)
Hydrosulfuric 1 H2S (aq) ⇌ H+ (aq) + HS - (aq) 8,9.10-8 7,05
2 HS - (aq) ⇌ H+ (aq) + S2- (aq) ~ 10-19 ~19
Hypobromous 1 HBrO (aq) ⇌ H+ (aq) + BrO - (aq) 2,3.10-9 8,64
Hypochlorous 1 HClO (aq) ⇌ H+ (aq) + ClO - (aq) 2,9.10-8 7,54
Hypoiodous 1 HIO (aq) ⇌ H+ (aq) + IO - (aq) 2,3.10-11 10,64
Iodic 1 HIO3 (aq) ⇌ H+ (aq) + IO3 - (aq) 1,7.10-1 0,77
Iodoacetic 1 CH2ICOOH (aq) ⇌ 7,6.10-4 3,12
H+ (aq) + CH2ICOO - (aq)
Lactic 1 CH3CHOHCOOH (aq) ⇌ 1,4.10-4 3,85
H+ (aq) + CH3CHOHCOO - (aq)
Maleic (cis 1 HOOCCH=CHCOOH (aq) ⇌ 1,2.10-2 1,92
butenedioic) H+ (aq) + HOOCCH=CHCOO - (aq)
2 HOOCCH=CHCOO - (aq) ⇌ 4,7.10-7 6,33
H+ (aq) + - OOCCH=CHCOO - (aq)
Malonic 1 HOOCCH2COOH (aq) ⇌ 1,5.10-3 2,82
H+ (aq) + HOOCCH2COO - (aq)
2 HOOCCH2COO - (aq) ⇌ 2,0.10-6 5,70
H+ (aq) + - OOCCH2COO - (aq)
Nitric 1 HNO3 (aq) ⇌ H+ (aq) + NO3 - (aq) >>1 <0
(mạnh)
Nitrous 1 HNO2 (aq) ⇌ H+ (aq) + NO2 - (aq) 4,0.10-4 3,40
Oxalic 1 HOOCCOOH (aq) ⇌ 5,9.10-12 1,23
H+ (aq) + HOOCCOO - (aq)
2 HOOCCOO - (aq) ⇌ 6,4.10-5 4,19
H+ (aq) + - OOCCOO - (aq)

212
Acid Bậc Phương trình điện ly Ka pKa
Perchloric 1 HClO4 (aq) ⇌ >>1 <0
H+ (aq) + ClO4 - (aq) (mạnh)
Periodic 1 HIO4 (aq) ⇌ H+ (aq) + IO4 - (aq) 2,3.10-2 1,64
Phosphoric 1 H3PO4 (aq) ⇌ 6,9.10-3 2,16
H+ (aq) + H2PO4 - (aq)
2 -
H2PO4 (aq) ⇌ 6,4.10-8 7,19
H+ (aq) + HPO4 2- (aq)
3 2-
HPO4 (aq) ⇌ 4,8.10-13 12,32
H+ (aq) + PO4 3- (aq)
Propanoic 1 CH3CH2COOH (aq) ⇌ 1,4.10-5 4,85
H+ (aq) + CH3CH2COO - (aq)
Pyruvic 1 CH3C(O)COOH (aq) ⇌ 2,8.10-3 2,55
H+ (aq) + CH3C(O)COO - (aq)
Sulfuric 1 H2SO4 (aq) ⇌ >>1 <0
H+ (aq) + HSO4 - (aq) (mạnh)
2 -
HSO4 (aq) ) ⇌ 1,2.10-2 1,92
H+ (aq) + SO4 2- (aq)
Sulfurous 1 H2SO3 (aq) ⇌ 1,7.10-2 1,77
H+ (aq) + HSO3 - (aq)
2 HSO3 - (aq) ⇌ H+ (aq) + SO3 2- (aq) 6,2.10-8 7,21
Thiocyanic 1 HSCN (aq) ⇌ >>1 <0
H+ (aq) + SCN - (aq) (mạnh)
Trichloroacetat 1 CCl3COOH ⇌ 2,3.10-1 0,64
H+ (aq) + CCl3COO - (aq)
Trifluoroacetic 1 CF3COOH ⇌ 5,9.10-1 0,23
H+ (aq) + CF3COO - (aq)
Water 1 H2O (aq) ⇌ H+ (aq) + OH - (aq) 1,0.10-14 14,00

213
Phụ lục 5. TÍCH SỐ TAN CỦA MỘT VÀI CHẤT ĐIỆN
LY KHÓ TAN Ở 25 oC
---oOo---
Tên chất Công thức Ksp
Aluminum hydroxyde Al(OH)3 1,3 × 10−33
Aluminum phosphate AIPO4 6,3 × 10−19
Barium carbonate BaCO3 5,1 × 10−9
Barium chromate BaCrO4 1,2 × 10−10
Barium fluoride BaF2 1,0 × 10−6
Barium hydroxyde Ba(OH)2 5,0 × 10−3
Barium sulfate BaSO4 1,1 × 10−10
Barium sulfite BaSO3 8,0 × 10−7
Barium thiosulfate BaS2O3 1,6 × 10−5
Bismuthyl chloride BiOCI 1,8 × 10−31
Bismuthyl hydroxyde BiOOH 4,0 × 10−10
Cadmium carbonate CdCO3 5,2 × 10−12
Cadmium hydroxyde Cd(OH)2 2,5 × 10−14
Cadmium sulfide CdS 8,0 × 10−28
Calcium carbonate CaCO3 2,8 × 10−9
Calcium chromate CaCrO4 7,1 × 10−4
Calcium fluoride CaF2 5,3 × 10−9
Calcium hydrogen phosphate CaHPO4 1,0 × 10−7
Calcium hydroxyde Ca(OH)2 5,5 × 10−6
Calcium phosphate Ca3(PO4)2 2,0 × 10−29
Calcium sulfate CaSO4 9,1 × 10−6
Calcium sulfite CaSO3 6, 8 × 10−8
Calcium oxalate CaC2O4 2,7 × 10−9
Chromium(II) hydroxyde Cr(OH)2 2,0 × 10−16
Chromium(III) hydroxyde Cr(OH)3 6,3 × 10−31
Cobalt(I) carbonate CoCO3 1,4 × 10−13
Cobalt(II) hydroxyde Co(OH)2 1,6 × 10−15

214
Tên chất Công thức Ksp
Cobalt(III) hydroxyde Co(OH)3 1,6 × 10−44
Copper (II) carbonate CuCO3 1,4 × 10-10
Copper (II) chromate CuCrO4 3,6 × 10-6
Copper (II) ferrocyanide Cu2[Fe(CN)6] 1,3 × 10-16
Copper (II) hydroxyde Cu(OH)2 2,2 × 10-20
Copper (II) sulfideb CuS 6,0 × 10-37
Copper(I) chloride CuCl 1,2 × 10−6
Copper(I) cyanide CuCN 3,2 × 10−20
Copper(I) iodide CuI 1,1 × 10−12
Copper(II) arsenate Cu3(AsO4)2 7,6 × 10−36
Iron (II) carbonate FeCO3 3,2 × 10-11
Iron (II) hydroxyde Fe(OH)2 8,0 × 10-16
Iron (II) sulfideb FeS 6,0 × 10-37
Iron (III) arsenate FeAsO4 5,7 × 10-21
Iron (III) ferrocyanide Fe4[Fe(CN)6]3 3,3 × 10-41
Iron (III) hydroxyde Fe(OH)3 4,0 × 10-38
Iron (III) phosphate FePO4 1,3 × 10-22
Lead (II) arsenate Pb3(AsO4)2 4,0 × 10-36
Lead (II) azide Pb(N3)2 2,5 × 10-9
Lead (II) bromide PbBr2 4,0 × 10-5
Lead (II) carbonate PbCO3 7,4 ×10-4
Lead (II) chloride PbCl2 1,6 × 10-5
Lead (II) chromate PbCrO4 2,8 × 10-13
Lead (II) fluoride PbF2 2,7 × 10-8
Lead (II) hydroside Pb(OH)2 1,2 × 10-15
Lead (II) iodide PbI2 7,1 × 10-9
Lead (II) sulfate PbSO4 1,6 × 10-8
Lead (II) sulfideb PbS 3,0 × 10-28
Lead (II) thiosulfate PbS2O3 4,0 × 10-7
Lithium fluoride LiF 3,8 × 10-3
Lithium phosphate Li3PO4 3,2 × 10-9
215
Tên chất Công thức Ksp
Magnesium ammoinum phosphate MgNH4PO4 2,5 × 10-13
Magnesium carbonate MgCO3 3,5 × 10-8
Magnesium fluoride MgF2 3,7 × 10-8
Magnesium hydroxyde Mg(OH)2 1,8 × 10-11
Magnesium phosphate Mg3(PO4)2 1,0 × 10-25
Silver nitrite AgNO2 6,0 × 10-4
Silver sulfate Ag2SO4 1,4 × 10-5
Silver sulfateb Ag2S 6,0 × 10-51
Silver sulfite Ag2SO3 1,5 × 10-14
Silver thiocyanate AgSCN 1,0 × 10-12
Strontium carbonate SrCO3 1,1 × 10-10
Strontium chromate SrCrO4 2,2 × 10-5
Strontium fluoride SrF2 2,5 × 10-9
Strontium sulfate SrSO4 3,2 × 10-7
Thallium(I) brommide TlBr 3,4 × 10-6
Thallium(I) chloride TlCl 1,7 × 10-4
Thallium(I) iodide TlI 6,5 × 10-8
Thallium(III) hydroxyde Tl(OH)3 6,3 × 10-46
Tin(II) hydroxyde Sn(OH)2 1,4 × 10-11
Tin(II) sulfideb SnS 1,0 × 10-26
Zinc carbonate ZnCO3 1,4 × 10-11
Zinc hydroxyde Zn(OH)2 1,2 × 10-17
Zinc oxalate ZnC2O4 2,7 × 10-8
Zinc phosphate Zn3(PO4)2 9,0 × 10-33
Zinc sulfide ZnS 2,0 × 10-25

216
Phụ lục 6. THẾ ĐIỆN CỰC TIÊU CHUẨN CỦA MỘT SỐ
QUÁ TRÌNH ĐIỆN CỰC TRONG DUNG DỊCH NƯỚC
---oOo---
Quá trình điện cực  o, V
F2 (g) + 2e → 2F- (aq) +2,866
OF2 (g) + 2H+ (aq) + 4e → H2O (l) + 2F- (aq) +2,1
O3 (g) + 2H+ (aq) + 2e → O2 (g) + H2O (l) +2,075
S2O82- (aq) + 2e → 2SO42- (aq) +2,01
Ag2+ (aq) + e → Ag+ (aq) +1,98
H2O2 (aq) + 2H+ (aq) + 2e → 2H2O (l) +1,763
MnO4- (aq) + 4H+ (aq) + 3e → MnO2 (s) + 2H2O (l) +1,70
PbO2 (s) + SO42- (aq) + 4H+ (aq) + 2e → PbSO4 (s) + +1,69
2H2O (l)
Au3+ (aq) + 3e → Au (s) +1,52
MnO4- (aq) + 8H+ (aq) + 5e → Mn2+ (aq) + 4H2O (l) +1,51
2BrO3- (aq) + 12H+ (aq) + 10e → Br2 (l) + 6H2O (l) +1,478
PbO2 (s) + 4H+ (aq) + 2e → Pb2+ (aq) + 2H2O (l) +1,455
ClO3- (aq) + 6H+ (aq) + 6e → Cl- (aq) + 3H2O (l) +1,450
Au3+ (aq) + 2e → Au+ (aq) +1,36
Cl2 (g) + 2e → 2Cl- (aq) +1,358
Cr2O72- (aq) + 14H+ (aq) + 6e → 2Cr3+ (aq) + 7H2O (l) +1,33
MnO2 (s) + 4H+ (aq) + 2e → Mn2+ (aq) + 2H2O (l) +1,23
O2 (g) + 4H+ (aq) + 4e → 2H2O (l) +1,229
2IO3- (aq) + 12H+ (aq) + 10e → I2 (s) + 6H2O (l) +1,20
ClO4- (aq) + 2H+ (aq) + 2e → ClO3- (aq) + H2O (l) +1,19
ClO3- (aq) + 2H+ (aq) + e → ClO2 (g) + H2O (l) +1, 175
NO2 (g) + H+ (aq) + e → HNO2 (aq) +1,07
Br2 (l) + 2e → 2Br- (aq) +1,065
NO2 (g) + 2H+ (aq) + 2e → NO (g) + H2O (l) +1,03
[AuCl4]- (aq) + 3e → Au (s) + 4Cl- (aq) +1,002
VO2+ (aq) + 2H+ (aq) + e → VO2+ (aq) + H2O (l) +1,000
NO3- (aq) + 4H+ (aq) + 3e → NO (g) + 2H2O (l) +0,956

217
Quá trình điện cực  o, V
Hg2+ (aq) + 2e → Hg (l) +0,854
Ag+ (aq) + e → Ag (s) +0,800
Fe3+ (aq) + e → Fe2+ (aq) +0,771
O2 (g) + 2H+ (aq) + 2e → H2O2 (aq) +0,695
2HgCl2 (aq) + 2e → Hg2Cl2 (s) + 2Cl- (aq) +0,63
MnO4- (aq) + e → MnO42- (aq) +0,56
I2 (s) + 2e → 2I- (aq) +0,535
Cu+ (aq) + e → Cu (s) +0,520
H2SO3 (aq) + 4H+ (aq) + 4e → S (s) + 3H2O (l) +0,449
C2N2 (g) + 2H+ (aq) + 2e → 2HCN (aq) +0,37
[Fe(CN)6]3- (aq) + e → [Fe(CN)6]4- (aq) +0,361
Cu2+ (aq) + 2e → Cu (s) +0,340
VO2+ (aq) + 2H+ (aq) + e → V3+ (aq) + H2O (l) +0,337
PbO2 (s) + 2H+ (aq) + 2e → PbO (s) + H2O (l) +0,28
Hg2Cl2 (s) + 2e → 2Hg (l) + 2Cl- (aq) +0,2676
HAsO2 (aq) + 3H+ (aq) + 3e → As (s) + 2H2O (l) +0,240
AgCl (s) + e → Ag (s) + Cl- (aq) +0,2223
SO42- (aq) + 4H+ (aq) + 2e → 2H2O (l) + SO2 (g) +0,17
Cu2+ (aq) + e → Cu+ (aq) +0,159
Sn4+ (aq) + 2e → Sn2+ (aq) +0,154
S (s) + 2H+ (aq) + 2e → H2S (g) +0,14
AgBr (s) + e → Ag (s) + Br- (aq) +0,071
2H+ (aq) + 2e → H2 (g) 0
Pb2+ (aq) + 2e → Pb (s) -0,125
Sn2+ (aq) + 2e → Sn (s) -0,137
AgI (s) + e → Ag (s) + I- (aq) -0,152
V3+ (aq) + e → V2+ (aq) -0,255
Ni2+ (aq) + 2e → Ni (s) -0,276
H3PO4 (aq) + 2H+ (aq) + 2e → H3PO3 (aq) + H2O (1) -0,277
Co2+ (aq) + 2e → Co (s) -0,356
PbSO4 (s) + 2e → Pb (s) + SO42- (aq) -0,403

218
Quá trình điện cực  o, V
Cd2+ (aq) + 2e → Cd (s) -0,424
Fe2+ (aq) + 2e → Fe (s) -0,440
Zn2+ (aq) + 2e → Zn (s) -0,763
2CO2 (g) + 2H+ (aq) + 2e → H2C2O4 (aq) -0,49
Cr2+ (aq) + 2e → Cr (s) -0,9
Mn2+ (aq) + 2e → Mn (s) -1,18
Ti2+ (aq) + 2e → Ti (s) -1,63
U3+ (aq) + 3e → U (s) -1,66
Al3+ (aq) + 3e → Al (s) -1,676
Mg2+ (aq) + 2e → Mg (s) -2,356
Na+ (aq) + e → Na (s) -2,713
Ca2+ (aq) + 2e → Ca (s) -2,84
Sr2+ (aq) + 2e → Sr (s) -2,89
Ba2+ (aq) + 2e → Ba (s) -2,92
Cs+ (aq) + e → Cs (s) -2,923
K+ (aq) + e → K (s) -2,924
Rb+ (aq) + e → Rb (s) -2,924
Li+ (aq) + e → Li (s) -3,040

219
Giáo trình Bài tập Hóa đại cương

Huỳnh Nguyễn Anh Tuấn, Đặng Đình Khôi, Võ Thị Thu Như
Nguyễn Vinh Tiến, Hoàng Minh Hảo, Hồ Phương
Võ Thị Ngà, Phan Thị Anh Đào, Trần Thị Nhung
Trường Đại học Sư phạm Kỹ thuật Thành phố Hồ Chí Minh

NHÀ XUẤT BẢN ĐẠI HỌC QUỐC GIA THÀNH PHỐ HỒ CHÍ MINH
Trụ sở:
Phòng 501, Nhà Điều hành ĐHQG-HCM, P. Linh Trung, TP Thủ Đức, TP.HCM.
ĐT: 028 62726361
E-mail: vnuhp@vnuhcm.edu.vn
Website: www.vnuhcmpress.edu.vn

Chịu trách nhiệm xuất bản và nội dung


TS ĐỖ VĂN BIÊN
Biên tập
TRẦN THỊ ĐỨC LINH
Sửa bản in
ÁI NHẬT
Trình bày bìa
TRƯỜNG ĐẠI HỌC SƯ PHẠM KỸ THUẬT THÀNH PHỒ HỒ CHÍ MINH
Đối tác liên kết
TRƯỜNG ĐẠI HỌC SƯ PHẠM KỸ THUẬT THÀNH PHỒ HỒ CHÍ MINH

Xuất bản lần thứ 1. Số lượng in: 250 cuốn, khổ 16 x 24cm. Số
XNĐKXB: 362-2023/CXBIPH/4-05/ĐHQGTPHCM. QĐXB số: 86/QĐ-NXB
cấp ngày 16/5/2023. In tại: Công ty TNHH In & Bao bì Hưng Phú. Địa chỉ:
162A/1, KP1A, phường An Phú, TP Thuận An, tỉnh Bình Dương. Nộp lưu
chiểu: Năm 2023. ISBN: 978-604-73-9650-4.

Bản quyền tác phẩm đã được bảo hộ bởi Luật Xuất bản và Luật Sở hữu
trí tuệ Việt Nam. Nghiêm cấm mọi hình thức xuất bản, sao chụp, phát tán nội
dung khi chưa có sự đồng ý của tác giả và Nhà xuất bản.
ĐỂ CÓ SÁCH HAY, CẦN CHUNG TAY BẢO VỆ TÁC QUYỀN!
ISBN: 978-604-73-9650-4
NXB ĐHQG-HCM

9 786047 396504

You might also like